Quantcast
Viewing all 9991 articles
Browse latest View live

CBSE Sample Papers for Class 12 Economics Delhi – 2008

CBSE Sample Papers for Class 12 Economics Delhi – 2008

Time allowed : 3                                                                                                                                 hours Maximum marks 100

GENERAL INSTRUCTIONS
(i) All questions in both the sections are compulsory.
(ii) Marks for questions are indicated against each.
(iii) Questions No. 1-5 and 17-21 are very short-answer questions carrying 1 nic-k each. They are required to be answered in one sentence each.
(iv) Questions No. 6-10 :dnd 22-26’ar? short-answer questions carrying 3 marks each. Answers to them should normally not exceed 60 words each.
(v) Questions No. 11-13 and 27-29 are also short-answer questions carrying 4 marks each. Answers to them should normally not exceed 70 words each.
(vi) Questions No. 14-16 and 30-32 are long-ansxoer questions carrying 6 marks each. Answers to them should normally not exceed 100 words each.
(vii) Ansivers should be brief and to the point and the above word limit should be adhered to as far as possible.

SET I

SECTION A
Q.l. Give the meaning of opportunity cost.
Ans. Opportunity cost is defined as the value of the benefit that is forgone by choosing one alternative rather than the other.
Or
Opportunity cost of using a given resource is defined as the value of the next best use to which the resource could be put.

Q.2. Define market demand.
Ans. Market demand for a commodity is the sum total of all individual demands for that commodity.

Q.3. What does cost mean in Economics?
Ans. Cost is the expenditure incurred on the production of a commodity. It includes both fixed cost and variable cost.

Q.4. Define revenue.
Ans. Revenue refers to the money receipts of a firm from selling its output.

Q.5. Define market for a good.
Ans. Market is defined as an area where potential buyers and sellers of a commodity come in contact with each other.

Q.6. Explain the central problem of “What to produce”.
Ans. This problem signifies ‘what goods should be produced and in what quantities’. This problem‘arises when due to scarcity of resources we cannot produce each and every tiling that we vVSrif ‘Therefore, a decision is to be taken as to what goods should be produced and in what quantities. In short, we have to see whether we should produce consumer goods or producer goods or defence goods or all the goods in some quantity combination.
So, on the basis of importance of various goods, an economy has to decide which goods should be produced and in what quantities.

Q.7. When price of a good rises from Rs. 5 per unit to Rs. 6 per unit, its demand falls from 20 units td 10 units. Compare expenditures on the good to determine whether demand is elastic or inelastic.
Ans.
Image may be NSFW.
Clik here to view.
cbse-sample-papers-for-class-12-economics-delhi-2008-1

When price rises, the total expenditure qKtiie commodity falls. This inverse relation between pice and total expenditure shows that me demand for the commodity is elastic (i.e., ed > 1).

Q.8. What is the relation between good X and good Y in each case, if with fall in the price of X demand for good Y (i) rises and (ii) falls? Give reason.
Ans. (a) If the fall in the price of commodity X leads to a rise in the demand for good Y, then X and Y are complementary goods which means they are used with each other for the satisfaction of same want, e.g., bread and butter.
(b) In case of fall in the price of commodity X leads to a fall in demand for commodity Y, then the commodities X and Y are substitutes for each other, i.e., they can be used in place of each other for the satisfaction of same want, e.g., tea and coffee.

Q.9. Explain the effect of technical progress on the supply of a good.
Ans. Technical progress leads’ to an increase in the supply of a commodity, i.e., at the same price more quantity is supplied during a given period. In such a case, as shown in Diagram 1, a rightward shift: takes place in the supply curve.
Or
Image may be NSFW.
Clik here to view.
cbse-sample-papers-for-class-12-economics-delhi-2008-2

Explain the effect of rise in input prices on the supply of a good.
Ans. Rise in the price of inputs will lead to a decrease in the supply of a good which means that at the same price lesser quantity will be offered for sale during a period of time.
Graphically, it can be shown as leftward shift in the supply curve, as shown in Diagram 2.

Q.10. State three features of monopoly.
Ans. Three features of monopoly are:
(i) Single seller of a commodity. This irieans the firm and industry are one and the same,
(ii) No close substitutes. The product offered for sale by the monopolist has no competition from other products.
(iii) No entry of firms, i.e., new firms cannot enter this market.

Q.11. Explain the conditions leading to maximisation of profits by a producer. Use total cost and total revenue approach.
Ans. Producer’s equilibrium means that combination of price and quantity of;Output tyhich yields maximum profit. Under total revenue and total cost approach, produces equilibrium refers to the stage of output level where the difference between TR and TCJ is maximum and total profits fall as more units of output are produced.
The two essential conditions are:
(i) The difference between TR and TC is maximum.
(ii) Total profits fall if output increases beyond equilibrium output ,h . TC Producer’s equilibrium can graphically be shown taking the example of perfect competition.TR is a 45° straight line since AR (price) is constant. TC is total cost curve.
Image may be NSFW.
Clik here to view.
cbse-sample-papers-for-class-12-economics-delhi-2008-3

At OQ level of output, profit i.e. the gap between AR and AC is maximum. This is the point of producer’s equilibrium. Output beyond OQ level, will reduce the total profit.

Note: This example relates to perfect competition.

Q.12. Complete the following table:
Image may be NSFW.
Clik here to view.
cbse-sample-papers-for-class-12-economics-delhi-2008-4

Ans.
Image may be NSFW.
Clik here to view.
cbse-sample-papers-for-class-12-economics-delhi-2008-5

Q.13. Complete the following table:
Image may be NSFW.
Clik here to view.
cbse-sample-papers-for-class-12-economics-delhi-2008-6

Ans.
Image may be NSFW.
Clik here to view.
cbse-sample-papers-for-class-12-economics-delhi-2008-7

(or)
Complte the following table?
Image may be NSFW.
Clik here to view.
cbse-sample-papers-for-class-12-economics-delhi-2008-8

Ans.
Image may be NSFW.
Clik here to view.
cbse-sample-papers-for-class-12-economics-delhi-2008-9

Q.14. A consumer cbriShmes dhly two goods. Explain his equilibrium with tire help of utility approach.
Image may be NSFW.
Clik here to view.
cbse-sample-papers-for-class-12-economics-delhi-2008-10

Ans. A consumer is in a state of equilibrium when he spends his given income on purchase of a commodity (or combination of goods) in such a way that gives him maximum satisfaction. In case of two commodities, consumer attains equilibrium when

Here MUj and MUy = Marginal utilities of x and y respectively.
Px and Py are prices of x and y.
MUm is marginal utility of money.
This implies that a consumer attains equilibrium, (rnaximum satisfaction) when the marginal utility obtained from the last rupee spent on both goods becomes equal. This is known as the law of Equi-Marginal Utility.

Q.15. Explain the Law of Variable Proportions through the behaviour of both Total Product and ’Marginal Product. Give reasons.
Ans. The law of variable proportions explains the relationship between inputs and outputs in the short run. In die short run, some factors of production (inputs) are fixed and other factors inputs are variable. The quantity of output can be increased by increasing the use of variable input.
As more and more units of variable input are employed, the proportion between the fixed and variable factors keeps on changing. The output passes through three phases. These three phases are identified with respect to the marginal product.
Phase I: TP increases at an increasing rate. The first phase of production lasts till the marginal product keeps rising and reaches its highest point. This is the phase of Increasing Returns to a factor and during this phase, total product increases at an increasing rate.
Phase II: TP increases at a diminishing rate. In this phase, Marginal Product is declining, but is positive. In this phase total product increases but at a diminishing rate. This phase ends when Marginal Product is zero and Total Product is at its maximum level. A producer always operates in this stages.
Phase III: TP is falling. In this third phase of production, Marginal Product is declining and is negative. Here total product starts falling.
Image may be NSFW.
Clik here to view.
cbse-sample-papers-for-class-12-economics-delhi-2008-11

These phases are shown graphdally in Diagram 4.
Image may be NSFW.
Clik here to view.
cbse-sample-papers-for-class-12-economics-delhi-2008-12

The reasons for the Operation of the Law are:
1. Optimum combination of factors. The phase of increasing marginal product is due to optimum combination of facto* that is required for any given technology, therefore fixed factors get better utilised.
2. However, when more and more units of variable factors are employed to a fixed factor, the fixed factor cannot absorb it and there is overcrowding of variable factors due to which the marginal product falls and becomes negative. This is the phase of diminishing marginal product.

Q.16. Market for a good is in equilibrium. What is the effect on equilibrium price and quantity if both market demand and market supply of the good increase in the same proportion? Use diagram.
Ans. Market for a good will be in equilibrium when quantity demanded and supplied are equal. However, when supply and demand change, changes take place in the equilibrium price and quantity according to the changes in the supply and demand. In the given example, both
Image may be NSFW.
Clik here to view.
cbse-sample-papers-for-class-12-economics-delhi-2008-13

demand and supply change equally. Thus, as shown in the Diagram) both demand and supply curves shift towards right by equal distance. Initially, the point of equilibrium was E and now in the changed situation it is F. The pricti remains die same, i.e., OP, but equilibrium quantity increases from OQ to OQ1.

For Blind Candidates only in lieu of Q. No. 16
Q.16. Market for goods is in equilibrium. What is the effect on equilibrium price and quantity, if both market demand and market supplyrofthe good increase in the same proportion. Use schedule.
Ans.
Image may be NSFW.
Clik here to view.
cbse-sample-papers-for-class-12-economics-delhi-2008-14

The new equilibrium price will be the fiapte,..i.e., Rs.7, but the equilibrium quantity will increase from 16 to 18 units.

SECTIONS
Q.17. Give meaning 6^ uivolu^taiy unemployment
Ans. In an economy wKen all able persons, who are willing to work at the prevailing wage rate, do not get ,w6fk and are unemployed, it is called involuntary unemployment.

Q.18. Define inflktlbnary gap.
Ans. Inflationary gap is a situation when aggregate demand is greater than die aggregate supply at the full ferriployinent level.

Q.19. What is a central bank?
Ans. Central bank’M#te apex institution in the banking structure of a country that manages a state’s currently, money supply, interest rates and oversees the commercial banking system.

Q.20. State any one objective of government budget.
Ans. One objective asf government budget is reallocation of resources for efficient utilisation of government resources and maximising social benefits.

Q.21. Define flexible exchange rate system.
Ans. The foreign exchange rate which is determined by the forces of demand for and supply of the foreign exchange is known as flexible exchange rate.

Q.22. Calculate ‘Value of Output7 from the following data:
Image may be NSFW.
Clik here to view.
cbse-sample-papers-for-class-12-economics-delhi-2008-15

Ans. Value of Output = NVAFC + IC + Depredation + Net Indirect taxes
= (i) + (ii) + (v) + {(iii) – (iv)}
= 100 + 75 + 10 + (20 – 5) = 185 + 15 = Rs. 200 lakhs.

Q.23. “When Exchange Rate of foreign currency rises, its supply rises. How Explain?
Ans. There is direct relationship between foreign exchange rate and its supply This means higher the exchange rate, higher will be the supply of foreign exchange. As sueb, when exchange rate of foreign currency rises, its supply also rises because the Indian rupee (home currency) becomes relatively cheaper in terms of the foreign currency. This encourages exports of Indian goods and ensures more supply of foreign exchange in the market.

Q.24. State components of the current account of Balance of Payments Account. 3
Ans. Components of the Current Account of Balance of Payments Account are:
(i) Export and import of goods (visibles)
(ii) Export and import of services (invisibles)
(iii) Transfer receipts and payments, such-fts gifts and unilateral payments or receipts
(iv) Investment income.

Q.25. What is bank rate policy? How does it work as a method of credit control?
Ans. Bank Rate is the rate of interest which central bank charges from commerdal banks for giving them loans.
There is direct relationship between bank rate and rate of interest, |but the relationship between rate of interest and demand for creditJs inverse.Thus, when volume of credit is to be reduced, bank rate is increased. On the other hand, when volume of credit is to be increased, bank, rate is reduced. In the aforesaid manner, bank rate is used for controlling the volume of credit.
(or)
What are open market operations? How dcrthese work as a method of credit control?
Ans. Open market operations refer to the purchase and sale of government securities in the open market by the central bank.
The functioning of this method is like this: When central bank of the country wants to increase the volume of credit, it starts purchasing securities from the market. These securities are generally bought at a higher price than the market price. As such, banks start selling them, as a result of which their cash reserves increase, i.e., their liquid assets increase. As a result of this, banks now can create more credit. On the contrary, when central bank wants to control the volume of credit, it starts selling securities in the market which are bought by the commercial banks. With the result, their cash reserves are reduced and this adversely affects their power of creating credit.

Q.26. Give meaning of capital receipts and revenue receipts with an examine of each.
Ans. Capital receipts are those receipts which are created either by incurring a liability or by disposing off assets, such as public debt, disinvestment, etc. Revenue Receipts are those receipts which do not create any asset or liability or which do not reduce assets, such as tax revenue.

Q.27. As a result of increase in investment by Rs. 125 crores, National Income increases by Rs. 500 crores. Calculate marginal propensity to consume.
Ans.
Image may be NSFW.
Clik here to view.
cbse-sample-papers-for-class-12-economics-delhi-2008-16

Q.28. Give four agency functions of commercial banks.
Or
Explain the acceptance of deposits function of commercial banks.
Ans.(Out of Syllabus for 2011 examination and onwards)

Q.29. What is fiscal deficit? What are its implications?
Ans. Fiscal deficit refers to the excess of total expenditure over total receipts (excluding borrowings) during the given fiscal year.
Implications of Fiscal Deficit are as follows:
(1) Debt trap: As fiscal deficit indicates, the total borrowing requirements of the government include both, i.e., repayment of the principal amount and also the payment of interest.
It increases the revenue expenditure in the form of interest payment leading to revenue deficit. This ultimately creates a vicious circle of fiscal and revenue deficit forcing the government to take more loans to service and settle the previous loans.
(2) Inflation: For meeting the borrowing requirements, file government may borrow from the Reserve Bank of India. When RBI prints new currency to meet the deficit requirements, it increases the rribney supply in the economy and creates inflationary pressure.
(3) Reduces future growth: The debt and financial burden of interest payments bn the . revenue expenditure causes a reduction on the capital expenditure for growth and development of the economy.
(4) Foreign dependence: In case fire government borrows from the rest of the world, then dependence of the country on other countries increases and it may not augur well for its prestige.

Note: Points (3) and (4) are not included in the present syllabus

Q.30. Calculate ‘Net Domestic Product at Factor Cost and Gross National Disposable Income from the following data:
Image may be NSFW.
Clik here to view.
cbse-sample-papers-for-class-12-economics-delhi-2008-17

Ans.NDP at\( { F }_{ c }\)
Image may be NSFW.
Clik here to view.
cbse-sample-papers-for-class-12-economics-delhi-2008-18

Q.31. Explain determination of equilibrium level of income using ‘consumption plus investment’ approach. Use diagram.
Ans. ‘Consumption plus Investment’ approach determining the equilibrium level of income is the Aggregate Demand and Aggregate Supply Approach. According to this approach, the equilibrium level of income.is determined at a point where aggregate demand and aggregate supply are equal.
Aggregate Demand (AD). It represents total expenditure in an economy. Consumption expenditure and investment expenditure are its main components. Consumption is a function of income but there is autonomous consumption expenditure also, i.e., consumption at zero level of income.
In the table given below, consumption expenditure is shown in Column II. Investment expenditure is autonomous and is the same at all levels of income. In Column III, investment expenditure is shown. Adding Column II and Column III, we get aggregate demand shown in Column IV.
Aggregate Supply is the value of goods and services produced is equal to National Income. It is shown in Column V.
Image may be NSFW.
Clik here to view.
cbse-sample-papers-for-class-12-economics-delhi-2008-19

Image may be NSFW.
Clik here to view.
cbse-sample-papers-for-class-12-economics-delhi-2008-20

The equilibrium level of income will be 300 because at this level AD and AS are equal.
According to Diagram 6, the equilibrium level of income will be OQ, because at this level AD and AS are equal.

Q.32.Explain the determination of equilibrium level of income using ‘saving-investment approach’. Use diagram
Ans. At the equilibrium level of income, aggregate demand is equal to aggregate supply, and saving is equal to investment. Saving is that part of income which is not consumed. By investment we mean physical investment i.e., creation of physical investment like machines, buildings, etc. Savers and investors are different persons. Those who save are not necessarily those who invest. Savings are done in small amounts and investments are made in huge amounts. Therefore, it is possible that the amount which savers save, may not be the same amount which investors want to invest. It means planned saving may not be equal to planned investment but actual saving is always equal to actual investment.
According to the table (on the next page), the equilibrium level of income rate will be 300, because at this level saving and investment are equal. At any other level of income, either saving is more than investment or less than investment.
Image may be NSFW.
Clik here to view.
cbse-sample-papers-for-class-12-economics-delhi-2008-21

Image may be NSFW.
Clik here to view.
cbse-sample-papers-for-class-12-economics-delhi-2008-22

According to Diagram 7, saving and investment curves intersect each other at point E, when the level of income is OQ.

For Blind Candidates only in lieu of
Q. No. 31 Explain determination of equilibrium level of income using consumption plus investment approach. Use schedule.
Or
Explain determination of equilibrium level of income using savings-investment approach. Use schedule.
Ans. Both the questions have been answered above. But blind candidates do not have to draw diagrams.

Q.32. Giving reasons explain how the following are treated while estimating national income:
(i) Payment of fees to a lawyer engaged by firm.
(ii) Rent free house to an employee by an employer.
(iii) Purchases by foreign tourists.
Ans. (i) It is an intermediate expenditure by the firm hiring the services of a lawyer, hence it will not be included in the National Income.
(ii) It is included in the National Income as compensation of employees.
(iii) It is included, as purchases by foreign tourists generate factor income in the domestic market.

SET II

Note : Except for the following questions,’’all the remaining questions have been asked in Set I.

SECTION A
Q.6. When price of a good falls from Rs. 10 per unit to Rs. 9 per unit, its demand rises from 9 units to 10 units. Compare expenditure on the good to find price elasticity of demand.
Ans.
Image may be NSFW.
Clik here to view.
cbse-sample-papers-for-class-12-economics-delhi-2008-23

With the fall in the price, the total expenditure on the commodity remains unchanged. Therefore \( { E }_{ d }\) = 1 (unitary elastic).

Q.9. State three features of monopolistic competition.
Ans. Features of monopolistic competition are:
1. Large number of sellers selling closely related but not homogeneous products.
2. Product differentiation: The products of the firms are closely related but not perfect substitutes of other firms. There is differentiation in the product on the basis of size, colour, composition, brand, etc.
3. Freedom of entry and exit of firms: The firms can freely enter and exit in this form of market.

Q.10. Explain the central problem of ‘How to produce’.
Ans. The central problem of ‘How to produce’ refers to the choice of the economy in the technique of production. This choice is made because of the difference in the availability of resources. If labour is abundant and cheap, labour intensive technology of production would lead to cost effective production.
In case labour is scarce and costly, capital intensive technology would be used.
For example, cloth can be produced by using more labour (labour intensive technique) as in handloom sector, or cloth can be produced in textile mills using capital intensive technology (i.e., using more machines).

Q.15. Market for a good is in equilibrium. What is the effect on equilibrium price and quantity if the proportionate increase in market demand is greater than increase in market supply. Use diagram.
Ans. When there is increase in demand and supply, both the curves shift towards tight as shown in Diagram 8. DD (demand curve) shifts to D1D1 and supply curve (SS) shifts to SJSJ. Since increase in demand is more than the increase in supply, the shift in DD curve is greater than the shift in SS curve. The initial point of intersection is E, where price is OP and equilibrium of quantity is OQ.
Image may be NSFW.
Clik here to view.
cbse-sample-papers-for-class-12-economics-delhi-2008-24

The new point of intersection is F, where price is OPj and equilibrium output is OQr Thus, in the given situation, both price and equilibrium quantity are greater than the initial position.

SECTION B
Q.17. Define Deflationary Gap.
Ans. Deflationary gap is the excess of aggregate supply over aggregate demand at the full employment level.

Q.21. Give die meaning of full employment
Ans. Full employment refers to the situation in an economy when all the able persons, who are willing to work at the prevailing Wage rate, are fully employed.

Q.26. Calculate ‘intermediate consumption’ from the following data:
Image may be NSFW.
Clik here to view.
cbse-sample-papers-for-class-12-economics-delhi-2008-25

Ans. Intermediate conception.

Q.28. As a result of increase in investment national income rises by Rs. 600 crores. If marginal propensity to consume is 0.75, calculate the increase in investment.
Ans. Given: AY = Rs. 600 crores and MPC = 0.75.
Image may be NSFW.
Clik here to view.
cbse-sample-papers-for-class-12-economics-delhi-2008-27

Image may be NSFW.
Clik here to view.
cbse-sample-papers-for-class-12-economics-delhi-2008-28

Q.31. Calculate Gross National Product at market price and Net National Disposable Income from the following data:
Image may be NSFW.
Clik here to view.
cbse-sample-papers-for-class-12-economics-delhi-2008-29

Ans.\( { GNP }_{ MP }\)
Image may be NSFW.
Clik here to view.
cbse-sample-papers-for-class-12-economics-delhi-2008-30

SET III

Note: Except for the following tfuestions, all the remaining questions have been asked in Set I and Set II.

SECTION A
Q.8. State three features of perfect competition.
Ans. Three features of perfect competition are:
1. Large number of buyers and sellers: There are so many buyers and sellers that no individual buyer or seller can influence the price of the commodity in the market.
2. Homogeneous products: Firms in the market supply homogeneous products, i.e., the products are perfect substitutes for each other.
3. Free entry and exit of firms: Firms have the freedom to move in or move out of the industry or the market. There are no barriers to their entry or exit.

Q.9. Explain the central problem of “for whom to produce.”
Ans. The central problem of ‘for whom to produce’ is the problem of distribution of national product among different people eg., rich and poor. This is known as Personal Distribution of the National Product.
‘For whom to produce’ also refers to the functional distribution of National Income to the factors of production -> in lieu of the factor services generated by them, eg., land earns rent, labour earns wages.

Q.10. When price of a good falls from Rs. 8 per unit to Rs. 7 per unit, its demand rises from 12 units to 16 units. Compare expenditures on the good to determine whether demand is elastic or inelastic?
Ans.
Image may be NSFW.
Clik here to view.
cbse-sample-papers-for-class-12-economics-delhi-2008-31

According to total expenditure method when price and total expenditure move in opposite directions, elasticity of demand will be greater than one. In the given example, price and total expenditure are moving in opposite directions, therefore,\( { e }_{ d }\) > 1.

Q.14. Market for a good is in equilibrium. What is the effect on equilibrium price and quantity if increase in market demand is less than increase in market supply? Use diagram. 6
Ans. The market for a good will be in equilibrium when quantity demanded and quantity supplied are equal. In the given question both demand and supply increase.
Image may be NSFW.
Clik here to view.
cbse-sample-papers-for-class-12-economics-delhi-2008-32

With the result, both demand and supply curves will shift to the right. As shown in foe diagram, supply curve will shift from SS to S1S1 and demand curve will shift from DD to DJDJ. Earlier foe point of equilibrium was E and now it is Er Since foe increase in demand is less than foe increase in supply, foe fall in foe price will be less than foe increase in quantity. Thus, as a result of increase in demand and supply both, price will fall , and equilibrium quantity will increase.

SECTION B
Q.20. What is underemployment equilibrium?
Ans. Underemployment equilibrium refers to foe situation in an economy where foe aggregate demand falls short of aggregate supply at full employment.

Q.21. What is consumption function?
Ans. Consumption function refers to -the functional relationship between consumption and National Income.
C =f(Y), where C is Consumption and Y is National Income.

Q.25. Calculate sales from the following data:
Image may be NSFW.
Clik here to view.
cbse-sample-papers-for-class-12-economics-delhi-2008-33

Ans. Gross Value of Output:
Image may be NSFW.
Clik here to view.
cbse-sample-papers-for-class-12-economics-delhi-2008-34

Q.29. If marginal propensity to consume is 0.9, what is the value of multiplier? How much investment is needed to increase national income by Rs. 5000 crores. Calculate.
Ans. MPC = 0.9
Image may be NSFW.
Clik here to view.
cbse-sample-papers-for-class-12-economics-delhi-2008-35

Q.32. Calculate National Income and Net National Disposal Income from the following data:
Image may be NSFW.
Clik here to view.
cbse-sample-papers-for-class-12-economics-delhi-2008-36

Ans.(i)National Income
Image may be NSFW.
Clik here to view.
cbse-sample-papers-for-class-12-economics-delhi-2008-37

(ii) Net National Disposable Income:
NNDI = NNPFC + (Indirext tax – Subsidies) – Net current transfers to abroad = 495 + (30 – 5) —15.= Rs. 505 crores
Image may be NSFW.
Clik here to view.
cbse-sample-papers-for-class-12-economics-delhi-20080-26

The post CBSE Sample Papers for Class 12 Economics Delhi – 2008 appeared first on Learn CBSE.


NCERT Exemplar Problems Class 8 Mathematics Direct and Inverse Proportion

NCERT  Exemplar Problems Class 8 Mathematics Chapter 10 Direct and Inverse Proportion

Multiple Choice Questions
Question. 1 Both u and v vary directly with each other. When u is 10, v is 15, which of the following is not a possible pair of corresponding values of u and v?
(a)2 and 3 (b) 8 and 12 (c) 15 and 20 (d) 25 and 37.5
Solution.
Image may be NSFW.
Clik here to view.
ncert-exemplar-problems-class-8-mathematics-direct-and-inverse-proportion-1

Question. 2 Both x and y vary inversely with each other. When x is 10, y is 6, which of the following is not a possible pair of corresponding values of x and y?
(a) 12 and 5 (b) 15 and 4 (c) 25 and 2.4 (d) 45 and 1.3
Solution.
Image may be NSFW.
Clik here to view.
ncert-exemplar-problems-class-8-mathematics-direct-and-inverse-proportion-2

Question. 3 Assuming land to be uniformly fertile, the area of land and the yield on it vary
(a) directly with each other
(b) inversely with each other
(c) neither directly nor inversely with each other
(d) sometimes directly and sometimes inversely with each other
Solution. (a) If land to be uniformly fertile, then the area of land and the yield on it vary directly with each other.
Hence, option (a) is correct.
Note Two quantities x and y are said to be in direct proportion, if they increase or decrease together in such a manner that the ratio of their corresponding values remains constant.

Question. 4 The number of teeth and the age of a person vary
(a) directly with each other
(b) inversely with each other
(c) neither directly nor inversely with each other
(d) sometimes directly,and sometimes inversely with each other
Solution. (d) The number of teeth and the age of a person vary sometimes directly and sometimes inversely with each other, we cannot predict about the number of teeth with exactly the age of a person. It change with person-to-person.
Hence, option (d) is correct.

Question. 5 A truck needs 54 litres of diesel for covering a distance of 297 km. The diesel required by the truck to cover a distance of 550 km is (a) 100 litres (b) 50 litres (c) 25.16 litres (d) 25 litres
Solution.
Image may be NSFW.
Clik here to view.
ncert-exemplar-problems-class-8-mathematics-direct-and-inverse-proportion-3

Question. 6 By travelling at a speed of 48 km/h, a car can finish a certain journey in 10 hours. To cover the same distance in 8 hours, the speed of the car should be
(a) 60 km/h (b) 80 km/h (c) 30 km/h (d) 40 km/h
Solution.
Image may be NSFW.
Clik here to view.
ncert-exemplar-problems-class-8-mathematics-direct-and-inverse-proportion-4

Question. 7 In which of the following cases, do the quantities vary directly with each other?
Image may be NSFW.
Clik here to view.
ncert-exemplar-problems-class-8-mathematics-direct-and-inverse-proportion-5

Solution. (a) In option (a),
x = 0.5,2, 8, 32 and y = 2, 8, 32,128
If we multiply x with 4, we get the directly required result as same as shown in corresponding y. In this case, as the value of x increases, the value of y also increases. Hence, option (a) is correct.

Question. 8 Which quantities in the previous question vary inversely with each other?
(a) x and y (b) p and q (c) r and s (d) u and v
Solution.
Image may be NSFW.
Clik here to view.
ncert-exemplar-problems-class-8-mathematics-direct-and-inverse-proportion-6

Question. 9 Which of the following vary inversely with each other?
(a) Speed and distance covered (b) Distance covered and taxi fare
(c) Distance travelled and time taken (d) Speed and time taken
Solution. (d) We know that, when we increases the speed, then the time taken by vehicle decreases.
Hence, speed and time taken vary inversely with each other.
So, option (d) is correct.

Question. 10 Both x and y are in direct proportion, then \(\frac { 1 }{ x }\) and \(\frac { 1 }{ y }\) are
(a) in indirect proportion
(b) in inverse proportion
(c) neither in direct nor in inverse proportion
(d) sometimes in direct and sometimes in inverse proportion
Solution. (b) If both x arid y are in directly proportion, then \(\frac { 1 }{ x }\) and \(\frac { 1 }{ y }\) are in inverse proportion.
Hence, option (b) is correct.
Note Two quantities x and y are said to be in inverse proportion, if an increase in x cause a proportional decrease in y and vice-versa.

Question. 11 Meenakshee cycles to her school at an average speed of 12 km/h and takes 20 minutes to reach her school. If she wants to reach her school in 12 minutes, her average speed should be (a) 20/3 km/h (b) 16 km/h (c) 20 km/h. (d) 15 km/h
Solution.
Image may be NSFW.
Clik here to view.
ncert-exemplar-problems-class-8-mathematics-direct-and-inverse-proportion-7

Question. 12 100 persons had food provision for 24 days. If 20 persons left the place, the provision will last for
(a) 30 days (b) 96/5 days (c) 120 days (d) 40 days
Solution.
Image may be NSFW.
Clik here to view.
ncert-exemplar-problems-class-8-mathematics-direct-and-inverse-proportion-8

Question. 13 If two quantities x and y vary directly with each other, then
(a) \(\frac { x }{ y }\) remains constant (b) x – y remains constant
(c) x + y remains constant ‘ (d)ix y remains constant
Solution. (a) If two quantities x and y vary directly with each other, then \(\frac { x }{ y }\) = k = constant.
Since, in direct proportion, both x and y increases or decreases together such a manner that the ratio of their corresponding value remains constant. Hence, option (a) is correct.

Question. 14 If two quantities p and q vary inversely with each other, then
(a) \(\frac { p }{ q }\) remains constant (b) p + q remains constant (c) p x q remains constant (d) p – q remains constant
Solution. (c) If two quantities p and q vary inversely with each other, then p x q remains constant.
Since, in inverse proportion, an increase in p cause a proportional decrease in q and vice-versa.
Hence, option (c) is correct.

Question. 15 If the distance travelled by a rickshaw in one hour is 10 km, then the distance travelled by the same rickshaw with the same speed in one minute is
(a)\(\frac { 250 }{ 9 }\)m (b)\(\frac { 500 }{ 9 }\)m (c)1000m (d)\(\frac { 500 }{ 3 }\)m
Solution.
Image may be NSFW.
Clik here to view.
ncert-exemplar-problems-class-8-mathematics-direct-and-inverse-proportion-9

Question. 16 Both x and y vary directly with each other and when x is 10, y is 14,
which of the following is not a possible pair of corresponding values of x and y?
(a) 25 and 35 (b) 35 and 25
(c) 35 and 49 (d) 15 and 21
Solution.
Image may be NSFW.
Clik here to view.
ncert-exemplar-problems-class-8-mathematics-direct-and-inverse-proportion-10

Fill in the Blanks
In questions 17 to 42, fill in the blanks to make the statements true.
Question. 17 If x = 5y, then x and y vary——— with each other.
Solution.
Image may be NSFW.
Clik here to view.
ncert-exemplar-problems-class-8-mathematics-direct-and-inverse-proportion-11

Question. 18 If xy = 10, then x and y vary————with each other.
Solution.
Image may be NSFW.
Clik here to view.
ncert-exemplar-problems-class-8-mathematics-direct-and-inverse-proportion-12

Question. 19 When two quantities x and y are in——-proportion or vary——-they are written as \(x\propto y\)
Solution.When two quantities x and y are in direct proportion or vary directly, they are written as \(x\propto y\) [see definition of direct proportion]

Question. 20 When two quantities x and y are in——-proportion or vary———-they are written as \(x\propto \frac { 1 }{ y }\)
Solution. When two quantities x and y are in inverse proportion or vary inversely, they are written as \(x\propto \frac { 1 }{ y }\) [see definition of inverse proportion]

Question. 21 Both x and y are said to vary——–with each other, if for some positive number k, xy =k.
Solution. Both x and y are said to vary inversely with each other, if for some positive number k,xy = k. [see condition of inverse proportion]

Question. 22 x and y are said to vary directly with each other, if for, some positive number k,———-= k.
Solution. x and y are said to vary directly wifh’ether, if for some positive number k, \(\frac { x }{ y }\)=k.

Question. 23 Two quantities are said to vary——— with each other, if they increase (decrease) together in such a manner that the ratio of their corresponding values remains constant.
Solution. Two quantities are said to vary directly with each other, if they increase (decrease) together in such a manner that the ratio of their corresponding values remains constant.

Question. 24 Two quantities are said to vary——–with each other, if an increase in one causes a decrease in the other in such a manner that the product of their corresponding values remains constant.
Solution. Two quantities are said to vary inversely with each other, if increase in one cause a decrease in the other in such a manner that the product of their corresponding values remains constant.

Question. 25 If 12 pumps can empty a reservoir in 20 hours, then time required by 45 such pumps to empty the same reservoir in—-hours.
Solution.
Image may be NSFW.
Clik here to view.
ncert-exemplar-problems-class-8-mathematics-direct-and-inverse-proportion-1

Question. 26 If x varies inversely as y then
Image may be NSFW.
Clik here to view.
ncert-exemplar-problems-class-8-mathematics-direct-and-inverse-proportion-2

Solution.
Image may be NSFW.
Clik here to view.
ncert-exemplar-problems-class-8-mathematics-direct-and-inverse-proportion-3

Question. 27 If x varies directly as y, then
Image may be NSFW.
Clik here to view.
ncert-exemplar-problems-class-8-mathematics-direct-and-inverse-proportion-4

Solution.
Image may be NSFW.
Clik here to view.
ncert-exemplar-problems-class-8-mathematics-direct-and-inverse-proportion-5

Question. 28 When the speed remains constant, the distance travelled is——–proportional to the time.
Solution. When the speed remains constant, the distance travelled is directly proportional to the time.
e.g. If 10 km cover in 10 min with uniform speed, then 20 km cover in 20 min with same speed.

Question. 29 On increasing a, b increases in such a manner that \(\frac { a }{ b }\) remains——and positive, then a and b are said to vary directly with each other.
Solution. On increasing a, b increases in such a manner that \(\frac { a }{ b }\) remains constant and positive, then a and b are said to vary directly with each other.

Question. 30 If on increasing a, b decreases in such a manner that—– remains——–and positive, then a and b are said to vary inversely with each other.
Solution. If on increasing a, b decreases in such a manner that ab remains constant and positive, then a and b are said to vary inversely with each other. [see definition of inverse proportion]

Question. 31 If two quantities x and y vary directly with each other, then——— of their corresponding values remains constant.
Solution. If two quantities x and y vary directly with each other, then ratio of their corresponding values remains constant. [see definition of direct proportion]

Question. 32 If two quantities p and q vary inversely with each other, then————- of their corresponding values remains constant.
Solution. If two quantities p and q vary inversely with each other, then product of their corresponding values remains constant.

Question. 33 The perimeter of a circle and its diameter vary—–with each other.
Solution.
Image may be NSFW.
Clik here to view.
ncert-exemplar-problems-class-8-mathematics-direct-and-inverse-proportion-6

Question. 34 A car is travelling 48 km in one hour. The distance travelled by the car in 12 minutes is ———.
Solution.
Image may be NSFW.
Clik here to view.
ncert-exemplar-problems-class-8-mathematics-direct-and-inverse-proportion-7

Question. 35 An auto rickshaw takes 3 hours to cover a distance of 36 km. If its speed is increased by 4 km/h, the time taken by it to cover the same distance is———–.
Solution.
Image may be NSFW.
Clik here to view.
ncert-exemplar-problems-class-8-mathematics-direct-and-inverse-proportion-8

Question. 36 If the thickness of a pile of 12 cardboard sheets is 45 mm, then the thickness of a pile of 240 sheets is——cm.
Solution.
Image may be NSFW.
Clik here to view.
ncert-exemplar-problems-class-8-mathematics-direct-and-inverse-proportion-9

Question. 37 If x varies inversely as y and x = 4 when y = 6, when x = 3, then value of y is—.
Solution.
Image may be NSFW.
Clik here to view.
ncert-exemplar-problems-class-8-mathematics-direct-and-inverse-proportion-10

Question. 38 In direct proportion \( \frac { { a }_{ 1 } }{ { b }_{ 1 } }\) —————- \( \frac { { a }_{ 2 } }{ { b }_{ 2 } }\)
Solution.
Image may be NSFW.
Clik here to view.
ncert-exemplar-problems-class-8-mathematics-direct-and-inverse-proportion-11

Question. 39 In case of inverse proportion,
Image may be NSFW.
Clik here to view.
ncert-exemplar-problems-class-8-mathematics-direct-and-inverse-proportion-12

Solution.
Image may be NSFW.
Clik here to view.
ncert-exemplar-problems-class-8-mathematics-direct-and-inverse-proportion-13

Question. 40 If the area occupied by 15 postal stamps is 60 \(c{ m }^{ 2 }\), then the area occupied by 120 such postal stamps will be———.
Solution.
Image may be NSFW.
Clik here to view.
ncert-exemplar-problems-class-8-mathematics-direct-and-inverse-proportion-14

Question. 41 If 45 persons can complete a work in 20 days, then the time taken by 75 persons will be—–hours.
Solution.
Image may be NSFW.
Clik here to view.
ncert-exemplar-problems-class-8-mathematics-direct-and-inverse-proportion-15

Question. 42 Devangi travels 50 m distance in 75 steps, then the distance travelled in 375 steps is——– km.
Solution.
Image may be NSFW.
Clik here to view.
ncert-exemplar-problems-class-8-mathematics-direct-and-inverse-proportion-16

True/False
In questions from 43 to 59, state whether the statements are True or False.
Question. 43 Two quantities x and y are said to vary directly with each other, if for some rational number k, xy =k.
Solution. False
Two quantities x and y are said to vary directly with each other, if \({ x }_{ y }\) = k (constant)

Question. 44 When the speed is kept fixed, time and distance vary inversely with each other.
Solution. False
When the speed is kept fixed, time and distance vary directly with each other.

Question. 45 When the distance is kept fixed, speed and time vary directly with each other.
Solution. False
When the distance is kept fixed, speed and time vary indirectly/inversely with each other. Since, if we increase speed, then taken time will less and vice-versa.

Question. 46 Length of a side of a square and its area vary directly with each other.
Solution. False
Length of a side of a square and its area does not vary directly with each other, e.g. Let a be length of each side of a square.
So, area of the square = \({ Side }^{ 2 }\) = \({ a }^{ 2 }\)
So, if we increase the length of the side of a square, then their area increases but not directly.

Question. 47 Length of a side of an equilateral triangle and its perimeter vary inversely with each other.
Solution. False
Length of a side of an equilateral triangle and its perimeter vary directly with each other, e.g. Let a be the side of an equilateral triangle. So, perimeter = 3 x (Side) = 3 x a = 3a . So, if we increase the length of side of the equilateral triangle, then their perimeter will also increases.

Question. 48 If d varies directly as \({ t }^{ 2 }\), then we can write d\({ t }^{ 2 }\) = k, where k is some constant.
Solution. False
If d varies inversely as \({ t }^{ 2 }\), then we can write d\({ t }^{ 2 }\) = k, where k is some constant.
Since, two quantities x and y are said to be in Inverse proportion, if an increases in x cause a proportional decreases in y and vice-versa, in such a manner that the product of their corresponding values remains constant.

Question. 49 If a tree 24 m high casts a shadow of 15 m, then the height of a pole that casts a shadow of 6 m under similar conditions is 9.6 m.
Solution.
Image may be NSFW.
Clik here to view.
ncert-exemplar-problems-class-8-mathematics-direct-and-inverse-proportion-17

Question. 50 If x and y are in direct proportion, then (x -1) and (y -1) are also in direct proportion.
Solution.
Image may be NSFW.
Clik here to view.
ncert-exemplar-problems-class-8-mathematics-direct-and-inverse-proportion-18

Question. 51 If x and y are in inverse proportion, then (x +1) and (y +1) are also in inverse proportion.
Solution. False
If x and y are in inverse proportion, then xy = k (constant) e.g. Let x= 2 and y = 3
.-. xy = 2 x 3= 6. Now, x + 1=2 + 1 = 3 and y+ 1 = 3 + 1 = 4
Then, (x + 1)(y+1) = 3 x 4 = 12 [not in inverse proportion]
Hence, (x+ 1)and (y + 1) cannot be in inverse proportion.

Question. 52 If p and q are in inverse proportion, i.e. pq = k (constant), then (p + 2)and (q – 2) are also in inverse proportion.
Solution. False
If p and q are in inverse proportion, then
xy = k (constant)
e.g. Let p = 3andq = 4
Then, pq = 3×4 = 12
Now, p+ 2 = 3+ 2 = 5 and q-2 = 4-2 =2
(p + 2) (q – 2) = 5 x 2 = 10 [not in inverse proportion]
Henc, (p+2) and (q -2)cannot be in inverse proportion.

Question. 53 If one angle of a, triangle is kept fixed, then the measure of the remaining two angles vary inversely with each other.
Solution.
Image may be NSFW.
Clik here to view.
ncert-exemplar-problems-class-8-mathematics-direct-and-inverse-proportion-19

Question. 54 When two quantities are related in such a manner that, if one increases, the other also increases, then they always vary directly.
Solution. True
When two quantities are related in such a manner that if, one increases the other also increases, then they always vary directly.
Above statement is correct for direct proportion. It is a basic properties of direct proportion.

Question. 55 When two quantities are related in such a manner that if one increases and the other decreases, then they always vary inversely.
Solution. True
When, two quantities are related in such a manner that if one increases and the other decreases, then they always vary inversely. Above statement is correct for inverse proportion. It is a basic properties of inverse proportion.

Question. 56 If x varies inversely as y and when x = 6, y = 8, then for x = 8, the value of y is 10.
Solution.
Image may be NSFW.
Clik here to view.
ncert-exemplar-problems-class-8-mathematics-direct-and-inverse-proportion-20

Question. 57 The number of workers and the time to complete a job is a case of direct proportion.
Solution. False
The number of workers and the time to complete a job is a case of indirect proportion, e.g. If 60 workers can complete a work in 10 days.
Then, 120 workers can complete the same work in 5 days.

Question. 58 For fixed time period and rate of interest, the simple interest is directly proportional to the principal.
Solution.
Image may be NSFW.
Clik here to view.
ncert-exemplar-problems-class-8-mathematics-direct-and-inverse-proportion-21

Question. 59 The area of cultivated land and the crop harvested is a case of direct proportion.
Solution. True
The area of cultivated land and the crop harvested is a case of direct proportion.
Since, the quantities of crop harvested is depend upon area of cultivated land.

In questions 60 to 62, which of the following vary directly and which vary inversely with each other and which are neither of the two ?
Question. 60 (i)The time taken by a train to cover a fixed distance and the speed of the train.
(ii) The distance travelled by CNG bus and the amount of CNG used.
(iii) The number of people working and the time to complete a given work.
(iv) Income tax and the income.
(v) Distance travelled by an auto-rickshaw and time taken.
Solution. (i) The time taken by a train to cover a fixed distance and the speed of the train are inversely proportional. ‘
e.g. Let a train cover 100 km in 1 h with speed 100 km/h.
Then, the same train cover 100 km in 30 min with speed 200 km/h.
(ii) The distance travelled by CNG bus and the amount of CNG used are directly proportional.
e.g. Let a CNG bus can travelled 10 km in 1 kg of CNG.
Then, the same CNG bus travelled 20 km in 2 x 1 = 2 kg of CNG.
(iii) The number of people working and the time to complete a given work are inversely proportional to each other.
e.g. Let 20 workers can complete a work in 1day.
Then, 10 workers can complete the same work in 2 days.
(iv) Income tax and the income are directly proportional to each other, e.g. Let Mr X have 4.5 lakh annual income.
Then, he pay 10% income tax on his income.
But if Mr X have 5.5 lakh annual income, then he has to pay 30% income tax on his salary/income.
(v) Distance travelled by an auto rickshaw and time taken are directly proportional to each other.
e.g. Let an auto rickshaw takes 2 h to travel 10 km.
Then, it will take 4 h to travel 20 km.

Question. 61 (i) Number of students in a hostel and consumption of food.
(ii) Area of the walls of a room and the cost of white washing the walls.
(iii) The number of people working and the quantity of work.
(iv) Simple interest on a given sum and the rate of interest.
(v) Compound interest on a given sum and the sum invested.
Solution.
Image may be NSFW.
Clik here to view.
ncert-exemplar-problems-class-8-mathematics-direct-and-inverse-proportion-22

Question. 62 (i) The quantity of rice and its cost.
(ii) The height of a tree and the number of years.
(iii) Increase in cost and number of shirts that can be purchased, if the budget remains the same.
(iv) Area of land and its cost.
(v) Sales tax and the amount of the bill.
Solution.(i) The quantity of rice and its cost are directly proportional to each other, e.g. Let 1 kg of rice price = Rs 40
Then, 2 kg of rice price = Rs 2 x 40 = Rs 80
(ii) The height of a tree and the number of years are neither directly nor inversely proportional to each other.
(iii) Increase in cost and number of shirts that can be purchased, if the budget remains the same are inversely proportional to each other, e.g. Let 2 shirts price = Rs 800 After increasing in price of each shirt,
1 shirt price became Rs 800
where budget = Rs 800
Image may be NSFW.
Clik here to view.
ncert-exemplar-problems-class-8-mathematics-direct-and-inverse-proportion-23

Question. 63 If x varies inversely as y and x = 20 when y = 600, find y when x = 400.
Solution.
Image may be NSFW.
Clik here to view.
ncert-exemplar-problems-class-8-mathematics-direct-and-inverse-proportion-24

Question. 64 The variable x varies directly as y and x – 80 when y is 160. What is y when x is 64?
Solution.
Image may be NSFW.
Clik here to view.
ncert-exemplar-problems-class-8-mathematics-direct-and-inverse-proportion-25

Question. 65 l varies directly as m and l = 5, when m =\(\frac { 2 }{ 3 }\) . Find l when m =\(\frac { 16 }{ 3 }\) .
Solution.
Image may be NSFW.
Clik here to view.
ncert-exemplar-problems-class-8-mathematics-direct-and-inverse-proportion-26

Image may be NSFW.
Clik here to view.
ncert-exemplar-problems-class-8-mathematics-direct-and-inverse-proportion-27

Question. 66 If x varies inversely as y and y =60 when x = 1.5. Find x, when y = 4.5.
Solution.
Image may be NSFW.
Clik here to view.
ncert-exemplar-problems-class-8-mathematics-direct-and-inverse-proportion-28

Question. 67 In a camp, there is enough flour for 300 persons for 42 days. How long will the, flour last, if 20 more persons join the camp?
Solution.
Image may be NSFW.
Clik here to view.
ncert-exemplar-problems-class-8-mathematics-direct-and-inverse-proportion-29

Question. 68 A contractor undertook a contract to complete a part of a stadium in 9 months with a team of 560 persons. Later on, it was required to complete the job in 5 months. How many extra persons should he employ to complete the work?
Solution.
Image may be NSFW.
Clik here to view.
ncert-exemplar-problems-class-8-mathematics-direct-and-inverse-proportion-30

Question. 69 Sobi types 108 words in 6 minutes. How many words would she type in half an hour?
Solution.
Image may be NSFW.
Clik here to view.
ncert-exemplar-problems-class-8-mathematics-direct-and-inverse-proportion-31

Question. 70 A car covers a distance in 40 minutes with an average speed of 60 km/h. What should be the average speed to cover the same distance in 25 minutes?
Solution.
Image may be NSFW.
Clik here to view.
ncert-exemplar-problems-class-8-mathematics-direct-and-inverse-proportion-32

Question. 71 It is.given that l varies directly as m.
(a) Write an equation which relates l and m.
(b) Find the constant of proportion (k), when l is 6, then m is 18.
(c) Find l, when m is 33.
(d) Find m, when l is 8.
Solution.
Image may be NSFW.
Clik here to view.
ncert-exemplar-problems-class-8-mathematics-direct-and-inverse-proportion-33

Question. 72 If a deposit of Rs 2000 earns an interest of Rs 500 in 3 years, how much interest would a deposit of Rs 36000 earn in 3 years with the same rate of simple interest?
Solution.
Image may be NSFW.
Clik here to view.
ncert-exemplar-problems-class-8-mathematics-direct-and-inverse-proportion-34

Question. 73 The mass of an aluminium rod varies directly with its length. If a 16 cm long rod has a mass of 192 g, find the length of the rod whose mass is 105 g.
Solution.
Image may be NSFW.
Clik here to view.
ncert-exemplar-problems-class-8-mathematics-direct-and-inverse-proportion-35

Question. 74 Find the values of x and y, if a and b are in inverse proportion.
(a) 12 x 8 (b) 305 y
Solution.
Image may be NSFW.
Clik here to view.
ncert-exemplar-problems-class-8-mathematics-direct-and-inverse-proportion-36

Question. 75 If Naresh walks 250 steps to cover a distance of 200 metres, find the distance travelled in 350 steps.
Solution.
Image may be NSFW.
Clik here to view.
ncert-exemplar-problems-class-8-mathematics-direct-and-inverse-proportion-37

Question. 76 A car travels a distance of 225 km in 25 litres of petrol. How many litres of petrol will be required to cover a distance of 540 kilometres by this car?
Solution.
Image may be NSFW.
Clik here to view.
ncert-exemplar-problems-class-8-mathematics-direct-and-inverse-proportion-38

Question. 77 From the following table, determine if x and y are in direct proportion or not.
Image may be NSFW.
Clik here to view.
ncert-exemplar-problems-class-8-mathematics-direct-and-inverse-proportion-39

Solution.
Image may be NSFW.
Clik here to view.
ncert-exemplar-problems-class-8-mathematics-direct-and-inverse-proportion-40

Question. 78 If a and b vary inversely to each other, then find the values of p, q, r; x, y, z and l, m, n.
Image may be NSFW.
Clik here to view.
ncert-exemplar-problems-class-8-mathematics-direct-and-inverse-proportion-41

Solution.
Image may be NSFW.
Clik here to view.
ncert-exemplar-problems-class-8-mathematics-direct-and-inverse-proportion-42

Image may be NSFW.
Clik here to view.
ncert-exemplar-problems-class-8-mathematics-direct-and-inverse-proportion-43

Image may be NSFW.
Clik here to view.
ncert-exemplar-problems-class-8-mathematics-direct-and-inverse-proportion-44

Image may be NSFW.
Clik here to view.
ncert-exemplar-problems-class-8-mathematics-direct-and-inverse-proportion-45

Question. 79 If 25 metres of cloth costs Rs 337.50, then
(a) what will be the cost of 40 metres of the same type of cloth? (b) what will be the length of the cloth bought for Rs 810?
Solution.
Image may be NSFW.
Clik here to view.
ncert-exemplar-problems-class-8-mathematics-direct-and-inverse-proportion-46

Image may be NSFW.
Clik here to view.
ncert-exemplar-problems-class-8-mathematics-direct-and-inverse-proportion-47

Question. 80 A swimming pool.can be filled in 4 hours by 8 pumps of the same type. How many such pumps are required, if the pool is to be filled in 2\(\frac { 2 }{ 3 }\) hours?
Solution.
Image may be NSFW.
Clik here to view.
ncert-exemplar-problems-class-8-mathematics-direct-and-inverse-proportion-48

Question. 81 The cost of 27 kg of iron is ? 1080, what will be the cost of 120 kg of iron of the same quality?
Solution.
Image may be NSFW.
Clik here to view.
ncert-exemplar-problems-class-8-mathematics-direct-and-inverse-proportion-1

Question. 82 At a particular time, the length of the shadow of Qutub Minar whose height is 72 m is 80 m. What will be the height of an electric pole, the length of whose shadow at the same time is 1000 cm?
Solution.
Image may be NSFW.
Clik here to view.
ncert-exemplar-problems-class-8-mathematics-direct-and-inverse-proportion-2

Question. 83 In a hostel of 50 girls, there are food provisions for 40 days. If 30 more girls join the hostel, how long will these provisions last?
Solution.
Image may be NSFW.
Clik here to view.
ncert-exemplar-problems-class-8-mathematics-direct-and-inverse-proportion-3

Question. 84 Campus and Welfare Committee of school is planning to develop a blue shade for painting the entire school building. For this purpose, various shades are tried by mixing containers of blue paint and white paint. In each of the following mixtures, decide which is a lighter shade of blue and also find the lightest blue shade among all of them.
Image may be NSFW.
Clik here to view.
ncert-exemplar-problems-class-8-mathematics-direct-and-inverse-proportion-4

If one container has one litre paint and the building requires 105 litres for painting, how many container of each type is required to paint the building by lightest blue shade?
Solution.
Image may be NSFW.
Clik here to view.
ncert-exemplar-problems-class-8-mathematics-direct-and-inverse-proportion-5

Image may be NSFW.
Clik here to view.
ncert-exemplar-problems-class-8-mathematics-direct-and-inverse-proportion-6

Question. 85 Posing a Question Work with a partner to write at least five ratio statement about this quilt, which has white, blue and purple squares.
Image may be NSFW.
Clik here to view.
ncert-exemplar-problems-class-8-mathematics-direct-and-inverse-proportion-7

How many squares of each colour will be there in 12 such quilts?
Solution.
Image may be NSFW.
Clik here to view.
ncert-exemplar-problems-class-8-mathematics-direct-and-inverse-proportion-8

Question. 86 A packet of sweets was distributed among 10 children and each of them received 4 sweets. If it is distributed among 8 children, how many sweets will each child get?
Solution. The total number of children = 10
If each children received 4 sweets, then The total number of sweets = 10 x 4 = 40 sweets
If 40 sweets distributed between 8 children, then each get 40/8 i.e. 5 sweets.

Question. 87 44 cows can graze a field in 9 days. How many less/more cows will graze the same field in 12 days?
Solution.
Image may be NSFW.
Clik here to view.
ncert-exemplar-problems-class-8-mathematics-direct-and-inverse-proportion-9

Question. 88 30 persons can reap a field in 17 days. How many more persons should be engaged to reap the same field in 10 days?
Solution.
Image may be NSFW.
Clik here to view.
ncert-exemplar-problems-class-8-mathematics-direct-and-inverse-proportion-10

Question. 89 Shabnam takes 20 minutes to reach her school, if she goes at a speed of 6 km/h. If she wants to reach school in 24 minutes, what should be her speed?
Solution.
Image may be NSFW.
Clik here to view.
ncert-exemplar-problems-class-8-mathematics-direct-and-inverse-proportion-11

Image may be NSFW.
Clik here to view.
ncert-exemplar-problems-class-8-mathematics-direct-and-inverse-proportion-12

Question. 90 Ravi starts for his school at 8 : 20 am on his bicycle. If he travels at a speed of 10 km/h, then he reaches his school late by 8 minutes but on travelling at 16 km/h, he reaches the school 10 minutes early. At what time does the school start?
Solution.
Image may be NSFW.
Clik here to view.
ncert-exemplar-problems-class-8-mathematics-direct-and-inverse-proportion-13

Image may be NSFW.
Clik here to view.
ncert-exemplar-problems-class-8-mathematics-direct-and-inverse-proportion-14

Question. 91 Match each of the entries in Column I with the appropriate entry in Column II.
Image may be NSFW.
Clik here to view.
ncert-exemplar-problems-class-8-mathematics-direct-and-inverse-proportion-15

Solution.
Image may be NSFW.
Clik here to view.
ncert-exemplar-problems-class-8-mathematics-direct-and-inverse-proportion-16

Question. 92 There are 20 grams of protein in 75 grams of sauted fish. How many grams of protein is in 225 grams of that fish?
Solution.
Image may be NSFW.
Clik here to view.
ncert-exemplar-problems-class-8-mathematics-direct-and-inverse-proportion-17

Question. 93 Ms Anita has to drive from Jhareda to Ganwari. She measures a distance of 3.5 cm between these village on the map. What is the actual distance between the villages, if the map scale is 1 cm = 10 km?
Solution. The distance between Jhareda to Ganwari in the map = 3.5 cm Given scale, 1 cm = 10 km
So, actual distance between the villages = 35 x 10 = 35 km

Question. 94 A water tank casts a shadow 21 m long. A tree of height 9.5 m casts a shadow 8 m long at the same time. The length of the shadows are directly proportional to their heights. Find the height of the tank.
Image may be NSFW.
Clik here to view.
ncert-exemplar-problems-class-8-mathematics-direct-and-inverse-proportion-18

Solution.
Image may be NSFW.
Clik here to view.
ncert-exemplar-problems-class-8-mathematics-direct-and-inverse-proportion-19

Question. 95 The table shows the time four elevators take to travel various distances. Find, which elevator is fastest and which is slowest.
Image may be NSFW.
Clik here to view.
ncert-exemplar-problems-class-8-mathematics-direct-and-inverse-proportion-20

How much distance will be travelled elevators B and C separately in 140 sec? Who travelled more and by how much?
Solution.
Image may be NSFW.
Clik here to view.
ncert-exemplar-problems-class-8-mathematics-direct-and-inverse-proportion-21

Image may be NSFW.
Clik here to view.
ncert-exemplar-problems-class-8-mathematics-direct-and-inverse-proportion-22

Question. 96 A volleyball court is in a rectangular shape and its dimensions are directly proportional to the dimensions of the swimming pool given below. Find the width of the pool.
Image may be NSFW.
Clik here to view.
ncert-exemplar-problems-class-8-mathematics-direct-and-inverse-proportion-23

Solution.
Image may be NSFW.
Clik here to view.
ncert-exemplar-problems-class-8-mathematics-direct-and-inverse-proportion-24

Question. 97 A recipe for a particular type of muffins requires 1 cup of milk and 1.5 cups of chocolates. Riya has 7.5 cups of chocolates.If she is using the recipe as a guide, how many cups.of milk will she need to prepare muffins?
Image may be NSFW.
Clik here to view.
ncert-exemplar-problems-class-8-mathematics-direct-and-inverse-proportion-25

Solution.
Image may be NSFW.
Clik here to view.
ncert-exemplar-problems-class-8-mathematics-direct-and-inverse-proportion-26

Question. 98 Pattern B consists of four tiles like pattern A. Write a proportion involving blue dots and total dots in patterns A and B. Are they in direct proportion? If yes, write the constant of proportion.
Image may be NSFW.
Clik here to view.
ncert-exemplar-problems-class-8-mathematics-direct-and-inverse-proportion-27

Solution.
Image may be NSFW.
Clik here to view.
ncert-exemplar-problems-class-8-mathematics-direct-and-inverse-proportion-28

Question. 99 A Fowler throws a cricket ball at a speed of 120 km/h. How long does this ball take to travel distance of 20 m to each the batsman?
Solution.
Image may be NSFW.
Clik here to view.
ncert-exemplar-problems-class-8-mathematics-direct-and-inverse-proportion-29

Question. 100 The variable x is inversely proportional to y. If x increases by p%, then by what per cent will y decwsose?
Solution. The variable x is inversely proportional to y. xy = k (constant)
Since, we know that two quantities x and y are said to be in inverse proportion, if an increase in * cause a proportional decrease in y and vice-versa.So, we can say y decrease by p%.

Question. 101 Here is a keyboard of a harmonium.
(a) Find the ratio of white keys to black keys on the keyboard.
Image may be NSFW.
Clik here to view.
ncert-exemplar-problems-class-8-mathematics-direct-and-inverse-proportion-30

(b) What is the ratio of black keys to all keys on the given keyboard?
(c) This pattern of keys is repeated on larger keyboard. How many black keys would you expect to find on a keyboard with 14 such patterns?
Solution.
Image may be NSFW.
Clik here to view.
ncert-exemplar-problems-class-8-mathematics-direct-and-inverse-proportion-31

Question. 102 The following table shows the distance travelled by one of the new eco-friendly energy-efficient car travelled on gas.
Image may be NSFW.
Clik here to view.
ncert-exemplar-problems-class-8-mathematics-direct-and-inverse-proportion-32

Which type of properties are indicated by the table? How much distance will be covered by the car in 8 litres of gas?
Solution.On the basis of given table, the distance travelled by one of the new eco-friendly energy-efficient earns travelled on gas.
The car travelled 15 km In 1 L of gas.
The car travelled 7.5 km in 0.5 L of gas.
The car travelled 30 km in 2 L of gas.
This rate shows direct proportion between litres of gas and the distance cover.
The car can cover the distance in 8 L of gas = 8 x 15 = 120 km

Question. 103 Kritika is following this recipe for bread. She realises her sister used most of sugar syrup for her breakfast. Kritika has only \(\frac { 1 }{ 6 } \) cup of syrup,so she decides to make a small size of bread. How much of each ingredient shall she use?
Bread recipe
1 cup quick cooking oats                                                        2 cups bread flour
\(\frac { 1 }{ 3 } \)cup sugar syrup               1 tablespoon cooking oil
1 \(\frac { 1 }{ 3 } \) cups water                    3 tablespoons yeast
1 tablespoon salt
Solution.
Image may be NSFW.
Clik here to view.
ncert-exemplar-problems-class-8-mathematics-direct-and-inverse-proportion-33

Question. 104 Many schools have a recommended students-teachers ratio as 35:1. Next year, school expects an increase in enrollment by 280 students. How many new teachers will they to appoint to maintain the students-teachers ratio?
Solution.
Image may be NSFW.
Clik here to view.
ncert-exemplar-problems-class-8-mathematics-direct-and-inverse-proportion-34

Question. 105 Kusum always forgets how to convert miles to kilometres and back again. However, she remembers that her car’s speedometer shows both miles and kilometres. She knows that travelling 50 miles per hour is same as travelling 80 kilometres per hour. To cover a distance of 200 km, how many miles Kusum would have to go?
Solution.
Image may be NSFW.
Clik here to view.
ncert-exemplar-problems-class-8-mathematics-direct-and-inverse-proportion-35

Question. 106 The student of Anju’s class sold posters to raise money. Anju wanted to create a ratio for finding the amount of money, her class would make for different numbers of posters sold. She knew, they could raise Rs 250 for every 60 posters sold.
(a) How much money would Anju’s class make for selling 102 posters?
(b) Could Anju’s class raise exactly Rs 2000? If so, how many posters would they need to sell? If not, why?
Solution.
Image may be NSFW.
Clik here to view.
ncert-exemplar-problems-class-8-mathematics-direct-and-inverse-proportion-36

The post NCERT Exemplar Problems Class 8 Mathematics Direct and Inverse Proportion appeared first on Learn CBSE.

NCERT Exemplar Problems Class 8 Mathematics Comparing Quantities

NCERT Exemplar Problems Class 8 Mathematics Chapter 9 Comparing Quantities

Multiple Choice Questions
Question. 1 Suppose for the principal P, rate R% and time T, the simple interest is S and compound interest is C. Consider the possibilities
(i) C>S
(ii) C = S
(iii) C <S Then,
(a) only (i) is correct (b) either (i) or (ii) is correct
(c) either (ii) or (iii) is correct (d) only (iii) is correct
Solution. (a) Let the principal (P) = Rs. 100
Rate of interest (R) = 100% and time period (T) = 2 yr
Image may be NSFW.
Clik here to view.
ncert-exemplar-problems-class-8-mathematics-1

Question. 2 Suppose a certain sum doubles in 2 yr at r% rate of simple interest per annum and R°/o rate of interest per annum compounded annually. Then,
(a) r < R (b) R<r
(c)R = r (d) Cannot be determined
Solution. (b) If the total amount received after 2 yr is same for both simple interest and compound interest on same principal, then the rate of simple interest is greater than the rate of compound interest.
i.e.R<r
Hence, option (b) is correct.

Question. 3 The compound interest on Rs. 50000 at 4% per annum for 2 yr compounded annually is (a) Rs. 4000 (b) Rs. 4080 (c) Rs. 4280 (d) Rs. 4050
Solution. (b) Given, principal (P) = Rs. 50000
Rate of interest (R%) = 4% per annum
Time period (T) = 2 yr We know that,
Image may be NSFW.
Clik here to view.
ncert-exemplar-problems-class-8-mathematics-2

Question. 4 If marked price of an article is Rs. 1200 and the discount is 12%, then the selling price of the article is (a)Rs. 1056 (b) Rs. 1344 (c) Rs. 1212 (d)Rs.1188
Solution. (a) Given, marked price of an article = Rs. 1200 Discount % = 12%
Discount = Discount % on marked price
= A x 1200=12 x 12 = Rs.144
Selling price = Marked price – Discount Selling price = Rs.11200 – Rs. 144 = Rs.1056 Hence, option (a) is correct.

Question. 5 If 90% of x is 315 km, then the value of x is
(a) 325 km (b) 350 km (c) 350 m
Solution. (b) We have, 90% of x = 315 km
Image may be NSFW.
Clik here to view.
ncert-exemplar-problems-class-8-mathematics-3

Question. 6 To gain 25% after allowing a discount of 10%, the shopkeeper must mark the price of the article which cost him Rs. 360 as
(a) Rs. 500 (b) Rs. 450 (c) Rs. 460 (d) Rs. 486
Solution. (a) Let the marked price of the article be Rs.x.
Cost price of the article = Rs. 360 According to the question,
Image may be NSFW.
Clik here to view.
ncert-exemplar-problems-class-8-mathematics-4

Question. 7 If a% is the discount per cent on marked price x, then discount is
(a)\(\frac { x }{ a } \)x 100 (b) \(\frac { a }{ x } \) x 100
(c) x x \(\frac { a }{ 100 } \) (d)\(\frac { 100 }{ xa } \)
Solution. (c) Since, discount can be calculated always on marked price, when discount percentage is given. Discount = Discount % on marked price = \(\frac { a }{ 100 } \)x x
Hence, option (c) is correct.

Question. 8 Ashima took a loan of Rs. 100000 at 12% per annum compounded half-yearly. She paid Rs. 112360. If (1.06)2 is equal to 1.1236, then the period for which she took the loan is
(a) 2 yr (b) 1 yr
(c) 6 months (d) 1 \(\frac { 1 }{ 2 } \) yr
Solution. (b) Given, principal (P) = Rs.100000
Rate of interest (R%) = 12% per annum compounded half-yearly Let m be the time period, v Amount paid = Rs.112360
Image may be NSFW.
Clik here to view.
ncert-exemplar-problems-class-8-mathematics-5

Question. 9 For calculation of interest compounded half-yearly, keeping the principal same, which one of the following is true?
(a) Double the given annual rate and half the given number of years
(b) Double the given annual rate as well as the given number of years
(c) Half the given annual rate as well as the given number of years
(d) Half the given annual rate and double the given number of years
Solution. (d)If interest is compounded half-yearly, then R=\(\frac { R }{ 2 } \) and T=2T=2n
Image may be NSFW.
Clik here to view.
ncert-exemplar-problems-class-8-mathematics-6

So, half the given annual rate and double the given number of years. Hence, option (d) is correct.

Question. 10 Shyama purchases a scooter costing Rs. 36450 and the rate of sales tax is 9%, then the total amount paid by her is
(a) Rs. 36490.50 (b) Rs. 39730.50
(c) Rs. 36454.50 (d) Rs. 33169.50
Solution.  (b) Since, sales tax is charged on the sale of an item by the government and is added to the bill amount. Shyama purchase a scooter of costing = Rs. 36450 Sales tax paid =9%
So, total amount paid by her
= Rs. 36450 of 9%+ Rs. 36450
= \(\frac { 9 }{ 100 } \)x36450 + 36450
= 9 x 364.5 + 36450 = 3280.5 + 36450 = Rs. 39730.5
Hence, option (b) is correct.

Question. 11 The marked price of an article is Rs. 80 and it is sold at Rs. 76, then the discount rate is
(a) 5% (b) 95%
(c) 10% (d) approx 11 %
Solution. (a) The marked price of an article = Rs. 80 Sold price of the article = Rs. 76 We know that,
Selling price = Marked price – Discount Discount = Marked price – Selling price
=> Discount = Rs.80-Rs.76 = Rs.4
Discount % = \(\frac { 4 }{ 80 } \) x 100= \(\frac { 40 }{ 80 } \) = 5%
Hence, option (a) is correct.

Question. 12 A bought a tape recorder for Rs. 8000 and sold it to B. B in turn sold it to C, each earning a profit of 20%. Which of the following is true?
(a) A and B earn the same profit (b) A earns more profit than B
(c) A earns less profit than<5 (d) Cannot be determined
Solution. (c) Cost price of tape recorder for/4 = Rs.8000
Cost price of tape recorder for B =20% profit on cost price for A
=\(\frac { 20 }{ 100 } \) x 8000 + 8000
=20 x 80 + 8000
=1600 + 8000=Rs.9600
Cost price of tape recorder for C=20% profit on cost price for B
=\(\frac { 20 }{ 100 } \) x 9600 + 9600
=1929 + 9600=Rs.11520
Here, profit for A= Rs.1600 Profit for B = Rs.1920
So, A earns less profit than B.
Hence, option (c) is correct.

Question. 13 Latika bought a teapot for Rs. 120 and a set of cups for Rs. 400. She sold teapot at a profit of 5% and cups at a loss of 5%. The amount received by her is
(a) Rs. 494 (b) 7 546
(c) Rs. 506 (d) Rs. 534
Solution. (c) Latika bought a teapot = Rs. 120 and a set of cups = Rs. 400 She sold teapot at a profit of 5%,  So, selling price of teapot =\(\frac { 5 }{ 100 } \) x 120 + 120
= \(\frac { 120 }{ 20 } \) +120
= 6 + 120 = Rs.126 Also, she sold cups at a loss of 5%.
So, selling price of cups = 400 –\(\frac { 5 }{ 100 } \) x 400
= 400 – 20 = Rs. 380
Then, the total amount received by her = Rs. 126 + Rs. 380 = Rs. 506 Hence, option (c) is correct.

Question. 14 A jacket was sold for Rs. 1120 after allowing a discount of 20%. The marked price of the jacket is
(a) Rs. 1440 (b) Rs. 1400 (c) Rs. 960 (d) Rs. 866.66
Solution. (b) Let the marked price of the jacket be Rs. x.
Discount % on marked price = 20%
Image may be NSFW.
Clik here to view.
ncert-exemplar-problems-class-8-mathematics-7

Question. 15 A sum is taken for two years at 16% per annum. If interest is .compounded after every three months, the number of times for which interest is charged in 2 yrs is
(a) 8 (b) 4 (c) 6 (d) 9
Solution. (a) Since, rate of interest is calculated after every three months. Similarly, the time period for amount in a year will 4 times.
If amount is taken for 2 yr, means 4×2 = 8 times charged in 2 yr.
Hertee, option (a) is correct.

Question. 16 The original price of a washing machine which was bought for Rs. 13500 including of 8% VAT, is
(a) Rs. 12420 (b) Rs. 14580 (c) Rs.12500 (d) Rs. 13492
Solution. (a) The price of the washing machine = Rs.13500
VAT [Value Added Tax] is included in selling price, which is 8%.
The original price of the washing machine including of 8% VAT
= 13500-13500 x \(\frac { 8 }{ 100 } \)
= 13500-135 x 8 = 13500-1080= Rs.12420 Hence, option (a) is correct.

Question. 17 Avinash bought an electric iron for Rs. 900 and sold it at a gain of 10%. He sold another electric iron at 5% loss which was bought
Rs. 1200. On the transaction, he has a
(a) profit of Rs. 75 (b) loss of Rs. 75 (c) profit of Rs. 30 (d) loss of Rs. 30
Solution. (c) Avinash bought an electric iron = Rs. 900 He sold it, at 10% profit.
So, selling price of the electric iron = \(\frac { 10 }{ 100 } \)x 900 + 900
= 90+ 900 = Rs. 990
He also sold another electric iron at 5% loss.
Cost price of another electric iron = Rs.1200
So, selling price of the electric iron = 1200\(\frac { 5 }{ 100 } \) x 1200
= 1200-60 = Rs.1140
Total amount paid by Avinash for purchasing electric irons = Rs. 900 + Rs.1200 = Rs.2100 Total received amount = Rs. 990 + Rs. 1140
= Rs. 2130 So, his profit = Rs.2130- Rs.2100 = Rs.30in transaction.
Hence, option (c) is correct.

Question. 18 A TV set was bought for t 26250 including 5% VAT. The original price of the TV set is
(a) Rs. 27562.50 (b) Rs. 25000 (c) Rs. 24937.50 (d) Rs. 26245
Solution. (c) Cost price of TV set = Rs. 26250.
VAT including = 5%
Original price = Cost price of article including VAT = 26250 \(\frac { 5 }{ 100 } \) x 26250
= 26250-\(\frac { 26250 }{ 20 } \)
= 26250-1312.5
So, original price of the TV set = Rs. 24937.5 Hence, option (c) is correct.

Question. 19 40% of [100 – 20% of 300] is equal to
(a) 20 (b) 16 (c) 140 (d) 64
Solution. (b) 40% of [100-20% of 300]
Image may be NSFW.
Clik here to view.
ncert-exemplar-problems-class-8-mathematics-8

Hence, option (b) is correct.

Question. 20 Radhika bought a car for Rs. 250000. Next year, its price decreased by 10% and further next year, it decreased by 12%. In the two years, overall decrease per cent in the price of the car is
(a) 3.2% (b) 22% (c) 20.8% (d) 8%
Solution. (c) Radhika bought a car for Rs. 250000.
Cost price of a car = Rs.250000
Its price decreased next year for 10%.
Image may be NSFW.
Clik here to view.
ncert-exemplar-problems-class-8-mathematics-9

Hence, option (c) is correct.

Fill in the Blanks
In questions 21 to 45, fill in the blanks to make the statements true.
Question. 21 ……………. is a reduction on the marked price of the article.
Solution.  Discount is a reduction on the marked price of the article.

Question. 22 Increase of a number from 150 to 162 is equal to increase of ………………..per cent.
Solution. Initial number = 150
Final number = 162
Increased number = 162 -150 = 12
Per cent of increased number = \(\frac { 12 }{ 150 } \) x 100 =\(\frac { 120 }{ 15 } \) = 8%

Question. 23 15% increase in price of an article, which is Rs. 1620, is the increase of ?
Solution. Let the price of the article be Rs. x.
Image may be NSFW.
Clik here to view.
ncert-exemplar-problems-class-8-mathematics-10

Question. 24 Discount =…………….
Solution. Discount = MP – SP
Here, MP = Marked price, and SP = Selling price

Question. 25 Discount = Discount % of ……………
Solution. Discount = Discount % of marked price
[ discount is a reduction given on marked price]

Question. 26 ………………….is charged on the sale of an item by the government and is added to the bill amount.
Solution.  Sales tax is charged on the sale of an item by the government and is added to the bill amount.
Sales tax = Tax % of bill amount

Question. 27 Amount when interest is compounded annually, is given by the formula……………….
Solution.  Amount when interest is compounded annually, is given by the formula
Image may be NSFW.
Clik here to view.
ncert-exemplar-problems-class-8-mathematics-11

where, P = principal, R = rate per annum and T = time

Question. 28 Sales tax = Tax% of……………….
Solution. Sales tax=Tax % of bill amount

Question. 29 The time period after which the interest is added each time to form a new principal, is called the ……………
Solution. The time period after which the interest is added each time to form a new principal, is called the conversion period.

Question. 30 ……………… expenses are the additional expenses incurred by a buyer for an item over and above its cost of purchase.
Solution. Overhead expenses are the additional expenses incurred by a buyer for an item over and above its cost of purchase.

Question. 31 The discount on an item for sale is calculated on the ……………..
Solution. The discount on an item for sale is calculated on the marked price.

Question. 32 When principal P is compounded semi-annually at r% per annum for t years, then amount = ……………..
Solution. When principal P is compounded semi-annually at r% per annum for t years.
Image may be NSFW.
Clik here to view.
ncert-exemplar-problems-class-8-mathematics-12

Question. 33 Percentages are to fractions with ………………. equal to 100.
Solution.  Percentages are equal to fractions with denominator equal to 100.
e.g. 8% means \(\frac { 8 }{ 100 } \)

Question. 34 The marked price of an article when it is sold for ? 880 after a discount of 12%, is ……………..
Solution. Selling price of an article = Rs.880 Discount % = 12%
We know that, discount is calculated oh marked price. Let the marked price be Rs. x.
Image may be NSFW.
Clik here to view.
ncert-exemplar-problems-class-8-mathematics-13

Question. 35 The compound interest on Rs. 8000 for one year at 16% per annum compounded half-yearly is …………… , given that \({ 1.08 }^{ 2 }\) = 1.1664.
Solution. Given, principal(P) = Rs.8000
Time period (7) = 1 yr
Rate (ft) = 16% per annum compounded half-yearly

Question. 36 In the first year on an investment of Rs. 600000, the loss is 5% and in the second year, the gain is 10%, the net result is ……………….
Solution. Investment amount = 7600000
In 1st year, the loss in 1st year = 5%.
So, investment in 1st year = 600000 – \(\frac { 5 }{ 100 } \) x 600000= 600000 – 30000 = 7 570000
In llnd year, the gain is 10%.
So, net investment = 570000 +\(\frac { 10 }{ 100 } \) x 570000= 570000 + 57000 = 7627000

Question. 37 If amount on the principal of Rs. 6000 is written as 6000 \({ \left[ 1+\frac { 5 }{ 100 } \right] }^{ 3 }\)and compound interest payable half-yearly, then rate of interest per annum is…………….. and……………… time (in years) is
Solution. If amount on the principal of 7 6000 is written as 6000\({ \left[ 1+\frac { 5 }{ 100 } \right] }^{ 3 }\) of and compound interest
payable half-yearly, then rate of interest per annum is 10% and time (in years) is 1\(\frac { 1 }{ 2 } \) yr.

Question. 38 By selling an article for Rs. 112000, a girl gains 40%. The cost price of the article was ……………..
Solution. Selling price of an article = 7 112000
Gain% = 40% .
Image may be NSFW.
Clik here to view.
ncert-exemplar-problems-class-8-mathematics-15

Question. 39 The loss per cent on selling 140 geometry boxes at the loss of SP of 10 geometry boxes is equal to ………………..
Solution. Let the selling price of 1 geometry box be Rs. 1.
So, the selling price for 140 geometry boxes = Rs.1 x 140 = Rs.140
Similarly, selling price of 10 geometry boxes = Rs. 1 x 10 = Rs. 10 v
Loss = Selling price of 10 geometry boxes = Rs.10
Image may be NSFW.
Clik here to view.
ncert-exemplar-problems-class-8-mathematics-16

Question. 40 The cost price of 10 tables is equal to the sale price of 5 tables. The profit per cent in this transaction is ……………..
Solution. Let the cost price of 1 table be Rs. 1.
The cost price of 10 tables = Sale price of 5 tables Profit = Cost price of 5 tables = Rs. 5
Profit percentage =\(\frac { profit }{ CP } \) x 100 = \(\frac { 5 }{ 5 } \)x 100 = 100%

Question. 41 Abida bought 100 pens at the rate of Rs. 3.50 per pen and pays a sales tax of 4%. The total amount paid by Abida is .
Solution. Number of pens bought by Abida = 100 Rate of per pen = Rs.3.50 So, cost of 100 pens = 100 x 3.50 = 1350 Abida also paid 4% sales tax on Rs. 350.
So, the total amount paid by Abida = 350x \(\frac { 4 }{ 100 } \) + 350= 350 x \(\frac { 1 }{ 25 } \) + 350= 14+ 350
= Rs. 364 .

Question. 42 The cost of a tape recorder is Rs. 10800 inclusive of sales tax charged at 8%. The price of the tape recorder before sales tax was charged, is ……………….
Solution. The cost of tape recorder, inclusive of 8% sales tax = Rs.10800 Let the price of the tape recorder before sales tax be Rs. x.
Image may be NSFW.
Clik here to view.
ncert-exemplar-problems-class-8-mathematics-17

Question. 43 2500 is greater than 500 by …………..%.
Solution. Difference between 2500 and 500 = 2500 – 500 = 2000
Hence, \(\frac { 2000 }{ 500 } \) x 100 = \(\frac { 2000 }{ 5 } \) = 400%

Question. 44 Four times a number is a …………….% increase in the number.
Solution. Let x be the number.
So, four times of x is Ax.
Hence, Ax is greater than x by 4x – x = 3x
Percentage increase in x = \(\frac { 3x }{ x } \) x 100 = 300%

Question. 45 5% sales tax is charged on an article marked X 200 after allowing a discount of 5%, then the amount payable is …………….
Solution. The marked price of the article = 1200 Discount = 5%
Selling price of the article = 200 – \(\frac { 5 }{ 100 } \) x 200
=200 -10 = Rs.190
Selling price including 5% sales tax = 190 + \(\frac { 5 }{ 100 } \) x 190
= 190+ 9.5 = Rs.199.5
Payable amount = Rs. 199.50

True/False
Inquestions 46 to 65, state whether the statements are True or False.
Question. 46.Tp calculate the growth of a bacteria if the rate of growth is known, the formula for calculatiorfof amount in compound interest can be used.
Solution. True
For calculating the growth of a bacteria4f the rate of growth is known, then we can use the formula for calculation of amount in compound interest.where, A = growth after nyears, P = initial number of bacteria and R = rate of growth

Question. 47 Additional expenses made after buying an article are included in the cost price and are known as Value Added Tax.
Solution. False
In the selling price (known as MRP) include the tax known as VAT (Value Added Tax).
Hence, VAT is always included in selling price.

Question. 48 Discount is a reduction given on cost price of an article.
Solution. False
Discount is a reduction given on marked price not on cost price.

Question. 49 Compound interest is the interest calculated on the previous year’s amount.
Solution. True
v Compound interest, Cl = A – P
where,
\({ A=P\left[ 1+\frac { R }{ 100 } \right] }^{ n }\)
Here, P = Principal on previous year’s amount and A = Present year’s amount R = Rate of interest and n = Time

Question. 50 CP = MP – Discount
Solution. False
The relation between marked price and discount is given by Selling price = Marked price – Discount

Question. 51 A man purchased a bicycle for Rs. 1040 and sold it for Rs. 800. His loss per cent is 30%.
Solution. False
Cost pride of the bicycle = Rs. 1040 Selling price of the bicycle =Rs. 800
Loss = Cost price – Selling price = Rs.1040 -Rs.800 = Rs.240
Image may be NSFW.
Clik here to view.
ncert-exemplar-problems-class-8-mathematics-comparing-quantities-1

Question. 52 Three times a number is 200% increase in the number, then one-third of the same number is 200% decrease in the number.
Solution. False
Let x be the number.
So, three times of x = 3x
Difference between 3x and x=3x-x=2x
Image may be NSFW.
Clik here to view.
ncert-exemplar-problems-class-8-mathematics-comparing-quantities-2

Question. 53 Simple interest on a given amount is always less than or equal to the compound interest on the same amount for the same time period and at the same rate of interest per annum.
Solution. False
For 1 yr, the simple interest and compound interest for same amount on same rate of interest are equal.
But for 2 yr, the simple interest is less than the compound interest for same amount on same rate of interest.

Question. 54 The cost of a sewing machine is Rs. 7000. Its value depreciates at 8% per annum. Then, the value of the machine after 2 yr is Rs. 5924.80.
Solution. True
Principal = Rs.7000
Rate of depreciation = 3% per annum Time period = 2 yr
Image may be NSFW.
Clik here to view.
ncert-exemplar-problems-class-8-mathematics-comparing-quantities-3

Question. 55 If the discount of Rs. y is available on the marked price of Rs. x, then the discount per cent is\(\frac { x }{ y }\) x 100%,
Solution. False
Marked price =Rs.x
Discount amount =Rs.y
Image may be NSFW.
Clik here to view.
ncert-exemplar-problems-class-8-mathematics-comparing-quantities-4

Question. 56 Number of students appearing for class X CBSE examination increases from 91422 in 1999-2000 to 116054 in 2008-09. Increase in the number of students appeared is approximately 27%.
Solution. True
Number of students increase from 116054 in 2008-09 to 91422 in 1999-2000
= 116054 – 91422 = 24632
Percentage of increase in number of students
Image may be NSFW.
Clik here to view.
ncert-exemplar-problems-class-8-mathematics-comparing-quantities-5

Question. 57.Selling price of 9 articles is equals to the cost price of 15 articles .In this transaction the profit of 66\(\frac { 2 }{ 3 }\)%
Solution. True

Image may be NSFW.
Clik here to view.
ncert-exemplar-problems-class-8-mathematics-comparing-quantities-6

Question. 58The compound interest on a sum of Rs. P for T years at R% per annum compounded annually is given by the formula,\(P\left[ 1+\frac { R }{ 100 } \right] \)
Solution. False
Image may be NSFW.
Clik here to view.
ncert-exemplar-problems-class-8-mathematics-comparing-quantities-8

Question. 59 In case of gain, SP =\(\frac { (100+gain%)\times CP }{ 100 } \)
Solution. True
Image may be NSFW.
Clik here to view.
ncert-exemplar-problems-class-8-mathematics-comparing-quantities-10

Question. 60 In case of loss, CP =\(\frac { 100\times SP }{ 100+Loss% } \)
Solution. False
Image may be NSFW.
Clik here to view.
ncert-exemplar-problems-class-8-mathematics-comparing-quantities-11

Question. 61 The value of a car, bought for Rs. 440000 depreciates each year by 10% of its value at the beginning of that year. So, its value becomes Rs. 308000 after three years.
Solution. False
The value of a car i.e. principal = Rs.440000 Rate of depreciation (R%) = 10% per annum Time period (T) = 3 yr
Image may be NSFW.
Clik here to view.
ncert-exemplar-problems-class-8-mathematics-comparing-quantities-12

Question. 62 The cost of a book marked at Rs. 190 after paying a sales tax of 2% is Rs. 192.
Solution. False
Marked price of a book = Rs.190 .
Image may be NSFW.
Clik here to view.
ncert-exemplar-problems-class-8-mathematics-comparing-quantities-13

Question. 63 The buying price of 5 kg of flour with the rate Rs. 20 per kg, when 5% sates tax is added on the purchase, is Rs. 21.
Solution. True
Total flour bought = 5 kg
Rate of one kg flour = Rs.20
Image may be NSFW.
Clik here to view.
ncert-exemplar-problems-class-8-mathematics-comparing-quantities-14

Question. 64 The original price of a shampoo bottle bought for Rs. 324, if 8% VAT is included in the price, is Rs. 300.
Solution. False
The original price of a shampoo bottle = Rs.300
Cost price of shampoo bottle after 8% VAT
Image may be NSFW.
Clik here to view.
ncert-exemplar-problems-class-8-mathematics-comparing-quantities-15

Question. 65 Sales tax is always calculated on the cost price of an item and is added to the value of the bill.
Solution. False
Sales tax is always calculated on the selling price of an item and is added to the value of the bill.

Question. 66 In a factory, women are 35% of all the workers, the rest of the workers being men. The number of men exceeds that of women by 252. Find the total number of workers in the factory.
Solution. Percentage of women in factory = 35%
Percentage of men in factory = 100 – 35 = 65%
Let the number of persons in the factory be x.
According to the question,
Image may be NSFW.
Clik here to view.
ncert-exemplar-problems-class-8-mathematics-comparing-quantities-16

Question. 67 Three bags contain 64.2 kg of sugar. The second bag contains \(\frac { 4 }{ 5 }\) of the contents of the first and the third contains 45 \(\frac { 1 }{ 2 }\) % of what there is in the second bag. How much sugar is there in each bag?
Solution. The total weight of sugar in three bags = 64.2 kg Let the first bag contains x kg sugar.
Image may be NSFW.
Clik here to view.
ncert-exemplar-problems-class-8-mathematics-comparing-quantities-17

Image may be NSFW.
Clik here to view.
ncert-exemplar-problems-class-8-mathematics-comparing-quantities-18

Question. 68 Find the SP, if
(a) MP = Rs. 5450 and discount = 5%
(b) MP = Rs.1300 and discount = 15%
Solution. (a) Marked price = 7 5450 Discount % = 5%
Image may be NSFW.
Clik here to view.
ncert-exemplar-problems-class-8-mathematics-comparing-quantities-1

Question. 69 Find the MP, if
(a) SP = Rs. 495 and discount = 1%
(b) SP = Rs. 9250 and discount = 7\(\frac { 1 }{ 2 }\) %
Solution. (a) Selling price (SP) = 7495 Discount % = 1%
Image may be NSFW.
Clik here to view.
ncert-exemplar-problems-class-8-mathematics-comparing-quantities-2

Image may be NSFW.
Clik here to view.
ncert-exemplar-problems-class-8-mathematics-comparing-quantities-3

Question. 70 Find discount in percent when
(a) MP = Rs.625 and SP = Rs.562.50
(b) MP = Rs.900 and SP = Rs.873
Solution.  (a) Marked price (MP) = Rs.625 Selling price (SP) = Rs. 562.50
Image may be NSFW.
Clik here to view.
ncert-exemplar-problems-class-8-mathematics-comparing-quantities-111

Question. 71 The marked price of an article is Rs. 500. The shopkeeper gives a discount of 5% and still makes a profit of 25%. Find the cost price of the article.
Solution. Given, marked price of an article = Rs.500 Discount % = 5%
But it makes a profit of 25%.
Let the cost price of the article be Rs. x.
Image may be NSFW.
Clik here to view.
ncert-exemplar-problems-class-8-mathematics-comparing-quantities-5

Question. 72 In 2007-08, the number of students appeared for Class X examination was 105332 and in 2008-09 the number was 116054. If 88151 students pass the examination in 2007-08 and 103804 students in 2008-09. What is the increase or decrease in pass percentage in class X result?
Solution.  Number of students appeared in 2007-08 = 105332 Number of students appeared in 2008-09 = 116054 Number of students passed in 2007-08 = 88151 Number of students passed in 2008-09 = 103804
Image may be NSFW.
Clik here to view.
ncert-exemplar-problems-class-8-mathematics-comparing-quantities-6

Question. 73 A watch worth Rs. 5400 x is offered for sale at Rs. 4500. What per cent discount is offered during the sale?
Solution. Marked price of a watch = Rs.5400
Selling price = Rs.4500
Discount = Marked price – Selling price = Rs.5400 – Rs.4500 = Rs.900
Image may be NSFW.
Clik here to view.
ncert-exemplar-problems-class-8-mathematics-comparing-quantities-7

Question. 74 In the year 2001, tbe number of malaria patients admitted in the hospitals of a state was 4375. Every year this number decreases by 8%. Find the number of-patients in 2003.
Solution. The number of malaria patients admitted in a hospital in 2001 = 4375 Rate of decrement of malaria patients = 8%
Time period = 2 yr i.e.2003-2001=2 yr
Let the number of patients in 2003 be A.
Image may be NSFW.
Clik here to view.
ncert-exemplar-problems-class-8-mathematics-comparing-quantities-8

Question. 75 Jyotsana bought a product for Rs. 3155 including 4.5% sales tax. Find the price before tax was added.
Solution. A product bought by Jyotsana for Rs. 3155 including 4,5% sales tax.
Let the price of the product before sales tax be Rs. x.
Image may be NSFW.
Clik here to view.
ncert-exemplar-problems-class-8-mathematics-comparing-quantities-9

Question. 76 An average Urban Indian uses about 150 L of water everyday.
Image may be NSFW.
Clik here to view.
ncert-exemplar-problems-class-8-mathematics-comparing-quantities-10

(a) What per cent of water is used for bathing and sanitation together per day?
(b) How much less per cent of water is used for cooking in comparison to that used for bathing?
(c) What per cent of water- is used fdr drinking, cooking and gardening together?
Solution. (a) On the basis of given details, water used for bathing per day = 20 L
Water used for sanitation = 40 L
Total water used per day = 150 L
Image may be NSFW.
Clik here to view.
ncert-exemplar-problems-class-8-mathematics-comparing-quantities-11

Question. 77 In 1975, the consumption of water for human use was about 3850 cu km/yr. It increased to about 6000 cu km/yr in the year 2000.
Find the per cent increase in the consumption of water from 1975 to 2000. Also, find the annual per cent increase in consumption
(assuming water consumption increases uniformly).
Solution. The consumption of water for human in 1975 = 3850 cu km/yr
The consumption of water for human in 2000 = 6000 cu km/yr
Increase in consumption of wgjer in 1975 to 2000 = 6000 – 3850 = 2150 cu km/yr
Image may be NSFW.
Clik here to view.
ncert-exemplar-problems-class-8-mathematics-comparing-quantities-12

Question.  78 Harshna gave her car for service at service station on 27-05-2009 and was charged as follows
(a)3.10 L engine oil @ Rs. 178.75 per litre and VAT @ 20%.
(b)Rs. 1105.12 for all other services and VAT @ 12.5%.
(c)Rs. 2095.80 as labour charges and service tax @ 10%.
(d)3% cess on service tax.
Find the bill amount.
Solution. (a) The total litres of engine oil used = 3.10 L
Rate of engine oil per litres =Rs. 178.75
The cost of engine oil = 3.10 x 17875= Rs. 554.125
Image may be NSFW.
Clik here to view.
ncert-exemplar-problems-class-8-mathematics-comparing-quantities-13

Question. 79 Given, the principal = Rs.40000, rate of interest = 8% per annum compound annually. Find
(a) Interest if period is one year.
(b) Principal for Ilnd year.
(c) Interest for Ilnd year.
(dj Amount if period is ttoo year.
Solution. Given, principal (P)= Rs.40000
Rate of interest (R) = 8% per annum
Image may be NSFW.
Clik here to view.
ncert-exemplar-problems-class-8-mathematics-comparing-quantities-14

Question. 80 In Delhi University, in the year 2009-10, 49000 seats were available for admission to various courses at graduation level. Out of these 28200 seats were for the students of General Category while 7400 seats were reserved for SC and 3700 seats for ST. Find the percentage of seats available for
(a) students of General Category.
(b) students of SC Category and ST Category taken together.
Solution. The total number of seats available for admission in 2009-10 = 49000
Seats reserved for General Category students = 28200
Seats reserved for SC Category students = 7400
Seats reserved for ST Category students = 3700
Image may be NSFW.
Clik here to view.
ncert-exemplar-problems-class-8-mathematics-comparing-quantities-15

Question. 81 Prachi bought medicines from a medical store as prescribed by her doctor for Rs 36.40 including 4% VAT. Find the price of before VAT was added.
Solution.
Image may be NSFW.
Clik here to view.
ncert-exemplar-problems-class-8-mathematics-comparing-quantities-1

Question. 82 Kritika ordered one pizza and one garlic bread from a pizza store and paid Rs 387 inclusive of taxes of Rs 43. Find the tax%.
Solution.
Image may be NSFW.
Clik here to view.
ncert-exemplar-problems-class-8-mathematics-comparing-quantities-2

Question. 83 Arunima bought household items whose marked price and discount % is as follows
Image may be NSFW.
Clik here to view.
ncert-exemplar-problems-class-8-mathematics-comparing-quantities-3

Find the total amount of the bill she has to pay.
Solution.
Image may be NSFW.
Clik here to view.
ncert-exemplar-problems-class-8-mathematics-comparing-quantities-4

Question. 84 Devangi’s phone subscription charges for the period 17-02-09 to 16-03-09 were as follows
Image may be NSFW.
Clik here to view.
ncert-exemplar-problems-class-8-mathematics-comparing-quantities-5

Find the final bill amount, if 3% education cess was also charged on service tax.
Solution.
Image may be NSFW.
Clik here to view.
ncert-exemplar-problems-class-8-mathematics-comparing-quantities-6

Image may be NSFW.
Clik here to view.
ncert-exemplar-problems-class-8-mathematics-comparing-quantities-7

Question. 85 If principal = Rs 100000, rate of interest = 10% compounded half-yearly. Find.
(a) Interest for 6 months. (b) Amount after 6 months.
(c) Interest for next 6 months, (d) Amount after one year.
Solution.
Image may be NSFW.
Clik here to view.
ncert-exemplar-problems-class-8-mathematics-comparing-quantities-8

Question. 86 Babita bought 160 kg of mangoes at Rs 48 per kg. She sold 70% of the mangoes at Rs 70 per kg and the remaining mangoes at Rs 40 per kg. Find Babita’s gain or loss per cent on the whole dealing.
Solution.
Image may be NSFW.
Clik here to view.
ncert-exemplar-problems-class-8-mathematics-comparing-quantities-9

Question. 87 A shopkeeper was selling all his items at 25% discount. During the off season, he offered 30% discount over and above the existing discount. If Pragya bought a skirt which was marked for Rs 1200, how much did she pay for it?
Solution.
Image may be NSFW.
Clik here to view.
ncert-exemplar-problems-class-8-mathematics-comparing-quantities-10

Question. 88 Ayesha announced a festival discount of 25% on all the items in her mobile phone shop. Ramandeep bought a mobile phone for himself. He got a discount of ? 1960. What was the marked price of the mobile phone?
Solution.
Image may be NSFW.
Clik here to view.
ncert-exemplar-problems-class-8-mathematics-comparing-quantities-11

Image may be NSFW.
Clik here to view.
ncert-exemplar-problems-class-8-mathematics-comparing-quantities-12

Question. 89 Find the difference between compound interest and simple interest on Rs 45000 at 12% per annum for 5 yr.
Solution.
Image may be NSFW.
Clik here to view.
ncert-exemplar-problems-class-8-mathematics-comparing-quantities-13

Question. 90 A new computer costs Rs 100000. The depredation of computers is very high as new models with better technological advantages are coming into the market. The depredation is as high as 50% every year. How much will the cost of computer be after two years?
Solution.
Image may be NSFW.
Clik here to view.
ncert-exemplar-problems-class-8-mathematics-comparing-quantities-14

Question. 91 The population of a town was decreasing every year due to migration, poverty and unemployment. The present population of the town is 631680. Last year the migration was 4% and the year before last, it was 6%. What was the population two years ago?
Solution.
Image may be NSFW.
Clik here to view.
ncert-exemplar-problems-class-8-mathematics-comparing-quantities-15

Question. 92 Lemons were bought at Rs 48 per dozen and sold at the rate of Rs 40 per 10. Find the gain or loss per cent.
Solution.
Image may be NSFW.
Clik here to view.
ncert-exemplar-problems-class-8-mathematics-comparing-quantities-16

Question. 93 If the price of petrol, diesel and LPG is slashed as follows
Image may be NSFW.
Clik here to view.
ncert-exemplar-problems-class-8-mathematics-comparing-quantities-17

Complete the above table.
Solution.
Image may be NSFW.
Clik here to view.
ncert-exemplar-problems-class-8-mathematics-comparing-quantities-18

Image may be NSFW.
Clik here to view.
ncert-exemplar-problems-class-8-mathematics-comparing-quantities-19

Question. 94 What is the percentage increase or decrease in the number of seats won by A, B, C and D in the general elections of 2009 as compared to the results of 2004?
Image may be NSFW.
Clik here to view.
ncert-exemplar-problems-class-8-mathematics-comparing-quantities-20

Solution.
Image may be NSFW.
Clik here to view.
ncert-exemplar-problems-class-8-mathematics-comparing-quantities-21

Question. 95 How much more per cent seats were won by X as compared to Y in assembly election in the state based on the data given below?
Image may be NSFW.
Clik here to view.
ncert-exemplar-problems-class-8-mathematics-comparing-quantities-22

Solution.
Image may be NSFW.
Clik here to view.
ncert-exemplar-problems-class-8-mathematics-comparing-quantities-23

Question. 96 Ashima sold two coolers for ? 3990 each. On selling one cooler she gained 5% and on selling the other she suffered a loss of 5%. Find her overall gain or loss % in whole transaction.
Solution.
Image may be NSFW.
Clik here to view.
ncert-exemplar-problems-class-8-mathematics-comparing-quantities-24

Question. 97 A lady buys some pencils for Rs 3 and an equal number for Rs 6. She sells them for Rs 7. Find her gain or loss %.
Solution.
Image may be NSFW.
Clik here to view.
ncert-exemplar-problems-class-8-mathematics-comparing-quantities-25

Question. 98 On selling a chair of Rs 736, a shopkeeper suffers a loss of 8%. At what price should he sell it, so as to gain 8%?
Solution.
Image may be NSFW.
Clik here to view.
ncert-exemplar-problems-class-8-mathematics-comparing-quantities-26

Question. 99 A dining table is purchased for Rs 3200 and sold at a gain of 6%. If a customer pays sales tax at the rate of 5%. How much does the customer pay in all for the table?
Solution.
Image may be NSFW.
Clik here to view.
ncert-exemplar-problems-class-8-mathematics-comparing-quantities-27

Question. 100 Achal bought a second hand car for Rs 225000 and spend Rs 25000 for repairing. If she sold.it for Rs 325000, what is his profit per cent?
Solution.
Image may be NSFW.
Clik here to view.
ncert-exemplar-problems-class-8-mathematics-comparing-quantities-28

Question. 101 A lady bought an air-conditioner for Rs 15200 and spent Rs 300 and Rs 500 on its transportation and repair, respectively. At what price should she sell it to make a gain of 15%?
Solution.
Image may be NSFW.
Clik here to view.
ncert-exemplar-problems-class-8-mathematics-comparing-quantities-29

Question. 102 What price should a shopkeeper mark on a article that costs him Rs 600 to gain 20%, after allowing a discount of 10%?
Solution.
Image may be NSFW.
Clik here to view.
ncert-exemplar-problems-class-8-mathematics-comparing-quantities-30

Image may be NSFW.
Clik here to view.
ncert-exemplar-problems-class-8-mathematics-comparing-quantities-31

Question. 103 Brinda purchased 18 coats at the rate of Rs 1500 each and sold them at a profit of 6%. If customer is to pay sales tax at the rate of 4%, how much will one coat cost to the customer and what will be the total profit earned by Brinda after selling all coats?
Solution.
Image may be NSFW.
Clik here to view.
ncert-exemplar-problems-class-8-mathematics-comparing-quantities-32

Question. 104 Film borrowed Rs 1024000 from a bank for one year. If the bank charges interest of 5% per annum, compounded half-yearly, what amount will he have to pay after the given time period. Also, find the interest paid by him.
Solution.
Image may be NSFW.
Clik here to view.
ncert-exemplar-problems-class-8-mathematics-comparing-quantities-33

Question. 105 The following items are purchased from showroom T-Shirt worth Rs 1200 Jeans worth Rs 1000 2 skirts worth Rs 1350 each.What will these items cost to Shikha if the sales tax is 7%?
Solution.
Image may be NSFW.
Clik here to view.
ncert-exemplar-problems-class-8-mathematics-comparing-quantities-34

Question. 106 The food labels given below information about 2 types of soup cream of tomato and sweet corn. Use these labels to answer the given questions, (all the servings are based on a 2000 calories diet.)
Image may be NSFW.
Clik here to view.
ncert-exemplar-problems-class-8-mathematics-comparing-quantities-35

(a) Which can be measured more accurately the total amount of fat it cream of tomato soup or the total amount of fat in sweet corn soup? Explain.
(b) On serving of cream of tomato soup contains 29% of the recommended daily value of sodium for a 2000 calories diet. What is the recommended daily value of sodium in milligrams? Express the answer up to 2 decimal places.
(c) Find the increase per cent of sugar consumed, if cream of tomato soup is chosen over sweet corn soup.
(d) Calculate ratio of calories from fat in sweet corn soup to the calories from fat in cream of tomato soup.
Solution.
Image may be NSFW.
Clik here to view.
ncert-exemplar-problems-class-8-mathematics-comparing-quantities-36

Question. 107 Music CD originally priced at Rs 120 is on sale for 25% off. What is the SP? Sonia and Rahul have different ways of calculating the sale price for the items they bought.
Image may be NSFW.
Clik here to view.
ncert-exemplar-problems-class-8-mathematics-comparing-quantities-37

As you work on the next problem, try both of these methods to see which your refer.
Solution.
Image may be NSFW.
Clik here to view.
ncert-exemplar-problems-class-8-mathematics-comparing-quantities-38

Question. 108 Store A and store B both charge Rs 750 for a video game. This week the video game is on sale Rs 600 at store B and for 25% off at store A. At which store is the game less expensive?
Solution.
Image may be NSFW.
Clik here to view.
ncert-exemplar-problems-class-8-mathematics-comparing-quantities-39

Question. 109 At a toy shop price of all the toys is reduced to 66% of the original price.
(a) What is the sale price of a toy that originally costs Rs 90?
(b) How much money would you save on a toy costing Rs 90?
Solution.
Image may be NSFW.
Clik here to view.
ncert-exemplar-problems-class-8-mathematics-comparing-quantities-40

Question. 110 A store is having a 25% discount sale. Sheela has a Rs 50 gift voucher and wants to use it to buy a board game marked for Rs 320. She is not sure how to calculate the concession she will get. The sales clerk has suggested two ways to calculate the amount payable.
Method 1 Subtract Rs 50 from the price and take 25% off the resulting price.
Method 2 Take 25% off the original price and then subtract Rs 50.
(a) Do you think both the methods will give the same result? If not, predict which method will be beneficial for her.
(b) For each method, calculate the amount Sheela would have to pay. Show your work.
(c) Which method do you think stores actually use? Why?
Solution.
Image may be NSFW.
Clik here to view.
ncert-exemplar-problems-class-8-mathematics-comparing-quantities-41

Question. 111 Living on your own Sanjay is looking for one bedroom appartment on rent. At Neelgiri appartments, rent for the first two months is 20% off. The one bedroom rate at Neelgiri is Rs 6000 per month. At Savana appartments, the first month is 50% off. The one bedroom rate at Savana appartments is Rs 7000 per month. Which appartment will be cheaper for the first two months? By how much?
Solution.
Image may be NSFW.
Clik here to view.
ncert-exemplar-problems-class-8-mathematics-comparing-quantities-42

Question. 112 For an amount, explain why, a 20% increase followed by a 20% decrease is less than the original amount.
Solution.
Image may be NSFW.
Clik here to view.
ncert-exemplar-problems-class-8-mathematics-comparing-quantities-43

Question. 113 Sunscreens block harmful ultraviolet (UV) rays produced by the sun. Each sunscreen has a Sun Protection Factor (SPF) that tells you how many minutes you can stay in the sun before you receive one minute of burning UV rays. e.g. If you apply sunscreen with SPF 15, you get one minute of UV rays for every 15 minutes you stay in the sun.
(i) A sunscreen with SPF 15 allows only \(\frac { 1 }{ 15 } \) of the sun’s UV rays. What per cent of UV rays does the sunscreen abort?
(ii) Suppose, a sunscreen allows 25% of the sun’s UV rays.
(a) What fraction of UV rays does this sunscreen block? Give your answer in lowest terms.
(b) Use your answer from part (a) calculate this sun screen’s SPF. Explain how you found your answer?
(iii) A label on a sunscreen with SPF 30 claims that the sunscreen blocks , about 97% of harmful UV rays. Assuming the SPF factor is accurate,is this claim true. Explain.
Solution.
Image may be NSFW.
Clik here to view.
ncert-exemplar-problems-class-8-mathematics-comparing-quantities-44

Question. 114 A real estate agent receives Rs 50000 as commission, which is 4% of the selling price. At what price does the agent sell the property?
Solution.
Image may be NSFW.
Clik here to view.
ncert-exemplar-problems-class-8-mathematics-comparing-quantities-45

Question. 115 With the decrease in prices of tea by 15%, Tonu, the chaiwallah, was able to buy 2 kg jnore of tea with the same Rs 45 that he spent each month on buying tea leaves for his chai shop. What was the reduced price of tea? What was the original price of tea?
Image may be NSFW.
Clik here to view.
ncert-exemplar-problems-class-8-mathematics-comparing-quantities-46

Solution.
Image may be NSFW.
Clik here to view.
ncert-exemplar-problems-class-8-mathematics-comparing-quantities-47

Question. 116 Below is the report card of Vidit Atrey. Vidit’s teacher left the last column blank. Vidit is not able to make out, in which subject he performed better and in which he needs improvement. Complete the table to help Vidit know his comparative performance.Assessment report for- 2009-2010
Image may be NSFW.
Clik here to view.
ncert-exemplar-problems-class-8-mathematics-comparing-quantities-48

Solution.
Image may be NSFW.
Clik here to view.
ncert-exemplar-problems-class-8-mathematics-comparing-quantities-49

Question. 117 Sita’s practicing basket ball. She has managed to score 32 baskets in 35 attempts. What is her success rate in percentage?
Image may be NSFW.
Clik here to view.
ncert-exemplar-problems-class-8-mathematics-comparing-quantities-50

Solution.
Image may be NSFW.
Clik here to view.
ncert-exemplar-problems-class-8-mathematics-comparing-quantities-51

Question. 118 During school hours, Neha finished 73% of her homework and Minakshi completed \(\frac { 5 }{ 8 } \) of her, homework. Who must finish a greater per cent of 8 homework?
Solution.
Image may be NSFW.
Clik here to view.
ncert-exemplar-problems-class-8-mathematics-comparing-quantities-52

Question. 119 Rain forests are home to 90000 of the 250000 identified plant species in the world. What per cent of the world’s identified plant species are found in rain forests?
Solution.
Image may be NSFW.
Clik here to view.
ncert-exemplar-problems-class-8-mathematics-comparing-quantities-53

Question. 120 Madhu’s room measures 6m x 3m. Her carpet covers 8 \({ m }^{ 2 }\). What per t cent of floor is covered by the carpet?
Solution.
Image may be NSFW.
Clik here to view.
ncert-exemplar-problems-class-8-mathematics-comparing-quantities-54

Question. 121 The human body is made up mostly of water. In fact about 67% of a person total body weight is water. If Jyoti weights 56 kg, how much of her weight is water?
Solution.
Image may be NSFW.
Clik here to view.
ncert-exemplar-problems-class-8-mathematics-comparing-quantities-55

Question. 122 The per cent of pure gold in 14 carat gold is about 58.3%. A 14 carat gold ring weights 7.6 grams. How many grams of pure gold are in the ring?
Solution.
Image may be NSFW.
Clik here to view.
ncert-exemplar-problems-class-8-mathematics-comparing-quantities-56

Question. 123 A student used the proportion \({ n }^{ 100 }\)=\({ 5 }^{ 32 }\) to find 5% of 32. What did the student do wrong?
Solution.
Image may be NSFW.
Clik here to view.
ncert-exemplar-problems-class-8-mathematics-comparing-quantities-57

Question. 124 The table shows the cost of sunscreen of two brands with and without sales tax. Which brand has a greater sales tax rate? Give the sales tax rate of each brand?
Image may be NSFW.
Clik here to view.
ncert-exemplar-problems-class-8-mathematics-comparing-quantities-58

Solution.
Image may be NSFW.
Clik here to view.
ncert-exemplar-problems-class-8-mathematics-comparing-quantities-59

 

The post NCERT Exemplar Problems Class 8 Mathematics Comparing Quantities appeared first on Learn CBSE.

CBSE Sample Papers for Class 12 Economics Outside Delhi – 2008

CBSE Sample Papers for Class 12 Economics Outside Delhi – 2008

Time allowed : 3                                                                                                                                 hours Maximum marks 100

GENERAL INSTRUCTIONS
(i) All questions in both the sections are compulsory.
(ii) Marks for questions are indicated against each.
(iii) Questions No. 1-5 and 17-21 are very short-answer questions carrying 1 nic-k each. They are required to be answered in one sentence each.
(iv) Questions No. 6-10 and 22-26 are? short-answer questions carrying 3 marks each. Answers to them should normally not exceed 60 words each.
(v) Questions No. 11-13 and 27-29 are also short-answer questions carrying 4 marks each. Answers to them should normally not exceed 70 words each.
(vi) Questions No. 14-16 and 30-32 are long-ansxoer questions carrying 6 marks each. Answers to them should normally not exceed 100 words each.
(vii) Answers should be brief and to the point and the above word limit should be adhered to as far as possible.

SET I

SECTION A

Question.l. Define ‘Marginal Rate of Transformation’.
Answer. Marginal Rate of Transformation is the rate at which one more unit of any good X is substituted for good Y. Symbolically,
MRT=\(\frac { \Delta Y }{ \Delta X }\)

Question.2. What is a demand schedule?
Answer. It is a two-column table which indicates various quantities demanded corresponding to various market prices.

Question.3. Define ‘production function’.
Answer. Production function explains the relationship between factor inputs and output under given j technology.

Question.4. What is ‘market supply’.
Answer.It is the total supply of a product (commodity) of all the producing firms at a given price at a given time.

Question.5. Define ‘equilibrium price’.
Answer. The price, where quantity demanded and quantity supplied are equal, is equilibrium price.
Any price other than this will lead to a situation of either excess demand or excess supply.

Question.6. Explain the central problem of ‘choice of technique’.
Answer. A number of techniques are available for producing a commodity, such as more of labour and less of capital or less of labour and more of capital. Therefore, a firm has to decide as } to which technique should be used for producing a commodity. This choice is made because
of the difference in the availability of resources. The technique, which is the cheapest, should be used. If labour is abundant and cheap, labour intensive technology would lead to cost effective production.
In case labour is scarce and costly, capital intensive technology would be used.

Question.7. Price elasticity of demand of a good is (-) 1. At a given price the consumer buys 60 units of the good. How many units will the consumer buy if the price falls by 10 per cent?
% change in quantity demanded
Answer.
Image may be NSFW.
Clik here to view.
cbse-sample-papers-for-class-12-economics-outside-delhi-2008-1

Image may be NSFW.
Clik here to view.
cbse-sample-papers-for-class-12-economics-outside-delhi-2008-2

Question.8. Given the market price of a good. How does a consumer decide as to how many units of that good to buy? Explain.
Answer. This is a question which signifies as to how consumers equilibrium is established in the case of a single commodity. It cannot be decided at the point of equality of price and marginal utility of the commodity because these two variables are incomparable. Therefore, the equilibrium will be established at a point where price of the commodity is equal to its marginal utility in money form i.e., \({ MU }_{ x }\) = \( MU }_{ m }\)

Question.9. What is the likely effect on the supply of a good if the prices of the inputs used in production of that good fall? Explain.
Answer. Fall in the prices of inputs decreases the cost of 8 production as a result of which supply will increase. In such a case, as shown in Diagram 1, a shift towards right takes place in the supply curve.
Image may be NSFW.
Clik here to view.
cbse-sample-papers-for-class-12-economics-outside-delhi-2008-3

Question.10. Explain what happens to the profits in the long run if the firms are free to enter the industry.
Answer. In the long run the firm will earruoQly normal profit (AR = AC) when new firms are free to enter the industry. This is so because if existing firms are earning abnormal profit (AR > AC) entry of new firms will increase the supply leading to decline in price. As a result of this excess of AR over AC will disappear and firms will get only normal profits.
(or)
Explain what happens to losses in the long run if the firms are free to leave the industry.
Answer. In the long run, if the firms are free to leave the industry, the existing firms will not sustain losses and earn only normal profits. This is so because in the case of losses few firms will leave the industry leading to decline in supply and increase in price. Ultimately, it will create a situation of normal profit.

Question.11. Explain producer’s equilibrium using a schedule. Use total cost and total revenue approach.
Answer. See Q. 11, 2008 (I Delhi)
Or
Distinguish between (i) fixed cost and variable cost giving examples and (ii) average cost and marginal cost giving an example.
Answer. (i) Fixed Cost: It is the cost which remains fixed and does not change with the change in , the level of output, such as rent of the building.
Variable Cost: It is the cost which varies directly with the changes in the level of output, such as cost of raw material. This cost will be zero when level of output is zero.
(ii) Average Cost: It is the cost which is obtained by dividing the total cost (TC) by the number of units produced. In shorty
\(AC=\frac { TC }{ Number\quad of\quad units\quad produced } \)
Marginal Cost (MC): The change in total cost that takes place by producing an additional unit is marginal cost. In short, MC = TC(n units) – TC(n-1 units).

Question.12. Draw supply curves with price elasticity of supply throughout equal to (i) zero, (ii) one, (iii) infinity and (iv) less than one.
Answer.
Image may be NSFW.
Clik here to view.
cbse-sample-papers-for-class-12-economics-outside-delhi-2008-4

For Blind Candidate only in lieu of

Question. No. 12 Distinguish between (i) elastic and inelastic supply and (ii) perfectly elastic and perfectly inelastic supply.
Answer. (i) When proportionate change in supply is more than the proportionate change in price, the supply is elastic or >1.
On the other hand, when proportionate change in supply is less than the proportionate .change in price, the supply is inelastic or <1.
(ii) When supply changes to any extent without changes in price, the supply is perfectly elastic. On tire other hand, if supply does not change and remains fixed when price changes, supply is said to be perfectly inelastic, i.e., es = 0.

Question.13. Complete the following table:
Image may be NSFW.
Clik here to view.
cbse-sample-papers-for-class-12-economics-outside-delhi-2008-5

Answer.
Image may be NSFW.
Clik here to view.
cbse-sample-papers-for-class-12-economics-outside-delhi-2008-6

Question.14. Explain the effect of the following on demand for a good:
(i) Rise in income
(ii) Rise in prices of related goods
Answer. (i) Effect of rise in income on the demand for a good. Due to the direct relationship between income and demand, the income demand curve rises upwards to the right. However, this is not true in the case of ail the commodities. When income rises the demand for normal and superior goods rises and the demand for inferior goods declines. Thus, as shown in Diagram 3, the income demand curve of normal goods rises upwards to the right and demand curve for inferior goods slopes downwards to the right .
Image may be NSFW.
Clik here to view.
cbse-sample-papers-for-class-12-economics-outside-delhi-2008-7

Image may be NSFW.
Clik here to view.
cbse-sample-papers-for-class-12-economics-outside-delhi-2008-8

(ii) Effect of rise in the prices of related goods. Related goods can be of two types (i) substitute goods and (ii) complementary goods. Substitute goods are those which can be used in place of one another such as tea and coffee. Complementary goods are those which cannot be used without one another, such as car and petrol.
When the price of a a commodity changes, it ‘ affects the demand far its substitute goods in the opposite manner. For example, if the price of tea increases, its demand will fall but the demand for coffee will increase because’it becomes cheaper in comparison to tea. Thus, as shown in Diagram 4, a shift in demand curve for coffee will lake place towards right when price of tea rises.
If the. price of a good rises (say pen), its demand falls. Thus rise in the price of pen will affect the demand for ink in the same maimer because it is a complemehfary good. Thus, when the price of a good increases, its demand falls and the demand of its complementary good also falls This iS being shown in Diagram 5.

Question.15. Explain, with the help of numerical examples, the effect on total output of a good when all the inputs used in production of that good are increased simultaneously and in the same proportion.
Answer. (Out of syllabus, for 2012 examination and orifflards)

Question.16. Given market equilibrium of a good, what are the effects of simultaneous increase in both demand and supply of that good on its equilibrium price and quantity.
Answer. Market equilibrium of a good is established at a point where quantity demanded is equal to quantity supplied. However, if changes take place in demand and supply,’equilibrium price and quantity are affected in accordance with changes in demand and supply* If demand and supply both increase, their effect on price and quantity can be examined under three situations as given below:
(i) When increase in demand is more than the increase in supply: As shown in diagram 6, price and quantity both will increase. Price will increase from OP to OP, and quantity will increase from OQ to OQ1.
(ii) When increase in demand and supply is equal: As shown in diagram 7, price will in Diagram 8, price will fall and quantity will increase. Price will decline from OP to OP, and quantity will increase from OQ to OQ.
Image may be NSFW.
Clik here to view.
cbse-sample-papers-for-class-12-economics-outside-delhi-2008-9

Image may be NSFW.
Clik here to view.
cbse-sample-papers-for-class-12-economics-outside-delhi-2008-10

(iii) When increase in demand is less than the
increase in supply: In this case, as price will in Diagram 8, price will fall and quantity will increase. Price will decline from OP to OP, and quantity will increase from OQ to OQ.
(or)
Explain the implications of the following:
(i) The feature ‘differentiated products’ under monopolistic competition.
(ii) The feature Targe number of sellers’ undeir perfect competition.
Answer. (i) Under monopolistic competition, products are differentiated on the basis of their size, packing, colour, smell, brand name, etc., such as different kinds of tooth pastes. These are artificial differences created with a view to make a consumer habituated by using a particular brand of a commodity. With the result, even when the price of the brand commodity that the customer is used to, goes up, he does not reduce his demand for that. As such, because of product differentiation the firms have some price control. (ii) Under perfect competition, the number of buyers and sellers of the commodity is very large, it is so large that no individual buyer or seller can influence the price of commodity. The price is fixed by the supply and demand of the whole industry and the, s^, .determined price is accepted by all the firms. As a result of this aspect of perfect competition, a firm gets only normal profit in the long run. In the event of price (AR) being higher than cost (AC), few firms will join the cornpetition, On tiie other hand, in the event of loss, few firms will leave the competition. In both the cases, profits will be normal.

SECTIONS
Question.17. Define ‘aggregate supply’.
Answer. It is the total value of goods and services produced in an economy at a given time.

Question.18. Give meaning of deficient demand.
Answer. It signifies deficiency in aggregate demand in comparison to aggregate supply at the full employment level.

Question.19. What is a commercial bank?
Answer. It is an institution which deals in money and credit.

Question.20. Define government budget.
Answer. A government budget is a statement of estimated receipts and expenditure prepared annually for a fiscal year.

Question.21. What is fixed exchange rate system?
Answer. It is a system under which exchange is officially fixed and declared. This means exchange rate is fixed and does not vary with the changes in the demand for and supply of foreign currency. Only the government has the power to change it.

Question.22. Calculate Net Value Added at factor cost from the following data:
Image may be NSFW.
Clik here to view.
cbse-sample-papers-for-class-12-economics-outside-delhi-2008-11

Answer. Net Value Added at FC
= Sales + Change in stock + Exports – Import of raw. material + Subsidy – Intermediate cost – Depreciation
= (iv) + (vi) – (v) – (vii) + (iii) – (ii) – (i)
= 140 + (-10) + 7-3 + 5 – 90 – 20
= 152 – 123 = Rs. 29 lakhs.

Question.23. When exchange rate of foreign currency falls, its demand rises. Explain, how.
Answer. There is an inverse relationship between the foreign exchange rate and the .demand for foreign exchange. This means higher the foreign exchange rate, lower is the demand for foreign exchange and vice-versa. If the foreign exchange rate falls, this means that in
exchange of the given amount of horde currency more foreign exchange is available. If foreign exchange becomes cheaper, its demand will increase. For example, if external value of dollar in Indian rupee falls from Rs. 50 to Rs. 40, more goods can be obtained with the given amount of home currency, therefore demand for foreign exchange will increase.

Question.24. Distinguish between balance of trade and balance on current account.
Answer. Balance of trade relates to the payments made and received on account of import and export of goods only, i.e., visible items. However, balance on current account, besides including balance of trade, also includes payments relating to invisible items Mid hrdlateral transfers. Thus, balance on current account is a broader term in comparison to balance of trade.

Question.25. Explain the’medium of exchange’function of money.
Answer. An important function of money is that it functions as a medium of exchange. It facilitates transactions of goods and services. Producers sell their goods to the Wholesalers in exchange of money. Wholesalers sell the same goods to the consumers in exchange of money. In the same way, all sections of society sell their services in exchange of money and with that money buy goods and services which they need. Money, working as medium of exchange, has eliminated inconvenience which was faced in barter transactions. However, bank money can operate as medium of exchange’only when it is generally accepted in that role. Bank money can be treated as money simply on the basis of its general acceptability for it is highly useful.
Or
Explain the evolution of money.
Answer. (Out of syllabus, for 2011 examination and onwards)

Question.26. Give meaning of capital expenditure and revenue expenditure in a government budget.
Answer. Capital expenditure. An expenditure which results in the creation of assets or reduction in liabilities is treated as capital expenditure, such as expenditure incurred on construction of buildings, roads, bridges, etc.
Revenue expenditure. An expenditure which does not result in creation of assets or reduction of liabilities, is called revenue expenditure such as payments of salaries, pensions, etc.

Question.27. In an economy an increase in investment leads to increase in National Income which is three times the increase in investment. Calculate marginal propensity to consume.
Answer.
Image may be NSFW.
Clik here to view.
cbse-sample-papers-for-class-12-economics-outside-delhi-2008-12

Question.28. Explain the lending function of commercial banks.
Answer. (Out of syllabus, for 2012 examination and onwards)
(or)
Explain ‘banker to the government’ function of central bank.
Answer. Banker to the government. Central bank is a banker, agent and financial advisor to the government. As a banker to the government, it carries out all banking business of the government. As an agent to the government, it buys and sells securities on behalf of the government. As a financial advisor, the central bank advises the government from time to time on economic, financial and monetary matters.

Question.29. What is revenue deficit? What are its implications?
Answer. The excess of government total revenue expenditure over its revenue receipts is revenue deficit.
Image may be NSFW.
Clik here to view.
cbse-sample-papers-for-class-12-economics-outside-delhi-2008-13

If revenue deficit occurs or increases, it is a warning signal for the government. In such a situation the government either should jreduce the revenue expenditure or should increase revenue receipts. Since most of the States1 are Welfare States, both these measures are difficult to be adopted. However, the deficit is to be met somehow. Therefore, in such a situation the only way available to the government is to increase capital receipts, such as borrowing, , disinvestment, etc.

Question.30. Calculate ‘National Income’ and ‘Private Income’ from the following data:
Image may be NSFW.
Clik here to view.
cbse-sample-papers-for-class-12-economics-outside-delhi-2008-14

Answer. National Income:
= Private final consumption expenditure + Government final consumption expenditure + Net domestic capital formation + Net exports – Net indirect tax + Net factor income from abroad
= (ii) + (vii) + (ix) + (iv) – (viii) + (x)
= 600 + 100 + 70 + (-20) -30 + 10 = 780 – 50 = Rs. 730 crores Private Income
= National Income + National debt interest + Current transfers from government – Net domestic product at factor cost accruing to government – Net current transfers to the rest of the world
= 730 + (iii) + (v) – (vi) – (i)
= 730 + 15 + 5 – 25- 10 = 750 – 35 =Rs.715 crores

Question.31. Explain ‘consumption function’ with the help of a schedule and diagram.
Answer. Consumption expenditure depends uponirtebtne, i.e. “Consumption is the function of income. Therefore, consumption function.explains the Relationship between consumption expenditure and income.” In other-words, consumption function explains as to what kind of changes will take place in consumption expenditure when income changes. The Income-Consumption Schedule, given in the table (on the next page) indicates the relationship between consumption expenditure and income.
The consumption function may be represented by the following equation:
C = [/latex]\bar { C }[/latex] +BY
Where, c = Autonomous amount. It is the minimum amount which
consumers have to spend irrespective of the level of income b – Marginal propensity to consume (MPC)
Y = Level of income
C= Consumption expenditure
Thus, if Y = 500, b = 0.8 and [/latex]\bar { C }[/latex] = 50,
then;CL= [/latex]\bar { C }[/latex] + bY = 50 + 0.8 x 500
= 50 + 400 = 450
In this way, corresponding to any level of Y (income), nature of C (consumption) can be ascertained. The concept of consumption function can be better comprehended looking at the schedule and Diagram 9 given on the next page.
Image may be NSFW.
Clik here to view.
cbse-sample-papers-for-class-12-economics-outside-delhi-2008-15

Since there is autonomous consumption (c), therefore CC curve does not start from the point of origin. Upto OY level of income, consumption is greater than income and beyond that it is less than income because with the increase in income savings are also being made.
Image may be NSFW.
Clik here to view.
cbse-sample-papers-for-class-12-economics-outside-delhi-2008-16

(or)
Explain ‘saving function’ with the help of a schedule and diagram.
“Saving is that part of income which is not spent on current consumption.” In other words, saving means the difference in income and cqpsumption expenditure. In the form of an equation: Saving (S) = Income (Y) – Consumption (C) or S = Y-C
Saving function or propensity to save explains the relationship between saving and income. Like consumption function saving function is also a schedule of saving and income which expresses the amounts of saving at various levels of income.
Consumption expenditure is never zero,_not even when income is zero. In the beginning, when income increases, consumption also increases but consumption is more than income. Therefore, till the point where consumption, is more than income, saving is negative. This.is so because consumption being more than income, past savings are used.
With the increase in the level of income consumption and income become equal at some level and at this level saving is zero.
In the table given on page 24, when income is Rs. 100 saving is zero, but after this level saving starts becoming positive. The reason is very simple. After the level of income of Rs. 100, consumption is less than income and the difference between the two is saving. Therefore, when the level of income increases to Rs. 200, consumption expenditure increases to Rs. 150 and saving becomes Rs. 50. In the same way, when income is Rs. 300, consumption increases to Rs. 200 and saving increases to Rs.100.
Image may be NSFW.
Clik here to view.
cbse-sample-papers-for-class-12-economics-outside-delhi-2008-17

Image may be NSFW.
Clik here to view.
cbse-sample-papers-for-class-12-economics-outside-delhi-2008-18

Saving function can be explained with thehelpof diagram 10 on page 23. In Diagram 10, OE is 45°, i.e., line of equality and CC is consumption curve. Till OY level of income, consumption is greater than income and after that it is less than income, therefore, till OY level of income, saving is negative and after that it becomes positive. Because of that, saving curve \({ S }_{ 1 }\)S starts not from the point of origin but from below that.

For Blind Candidates only in lieu of
Question. No. 31 Explain ‘consumption function’ with the help of a schedule. Also record marginal propensity to consume in the schedule.
Answer. See Q. 31 above for regular students.
Or
Explain ‘savingftinction’ with the help of a schedule. Also record marginal propensity to save in the schedule.
Ans. See Q. 31 (Or) above for regular students.

Question.32. Giving reasons, explain how the following are treated in estimating National Income:
(i) Wheat grown By a farmer but used entirely for family’s consumption.
(ii) Earnings of the shareholders from the sale of shares.
(iii) Expenditure by government on providing free education.
Answer. (i) It is production for self consumption, hence it will be included in National Income. . Production for self consumption is part of National Income.
(ii) These are not included, because these are financial transactions and do not relate to flow of goods and services.
(iii) It will be included in National Income because it is a part of government’s final consumption expenditure.

SET II

Note : Except for the following questions, all the remaining questions have been asked in Set l.

SECTION A
Question.6. Price elasticity of demand for a good is (-) 2. The consumer buys a certain quantity of this good at a price of Rs. 8 per unit. When the juice falls he buys 50 per cent more quantity. What is the new price?
Answer.
Image may be NSFW.
Clik here to view.
cbse-sample-papers-for-class-12-economics-outside-delhi-2008-19

Question.8. What is the likely effect on supply of a good if a unit tax is imposed on that good? Explain.
Answer. A unit tax is a tax which the government levies per unit sold or produced. This tax naturally increases the cost of production of the firm for every unit of output produced. As a result of this, marginal cost of every unit increases. This leads to a shift in supply curve towards left, as shown in Diagram 11.
Image may be NSFW.
Clik here to view.
cbse-sample-papers-for-class-12-economics-outside-delhi-2008-20

Question.10. Explain the central problem of the choice of products to be produced.
Answer. Because of scarcity of resources each and every commodity that we want cannot be produced. Therefore, it has to be decided as to which commodities should be produced and in what quantity. Should we produce goods for consumption or for military purposes. Similarly, it may be decided that we should produce either luxury goods or goods meant for general public. Actually speaking, this problem is resolved by the introduction of the forces of supply and demand of the commodity/commodities and price mechanism.

Question.11. Complete the following table:
Image may be NSFW.
Clik here to view.
cbse-sample-papers-for-class-12-economics-outside-delhi-2008-111

Answer.
Image may be NSFW.
Clik here to view.
cbse-sample-papers-for-class-12-economics-outside-delhi-2008-22

SECTION B
Question.27. What is fiscal deficit? What are its implications?
Answer. See Q.29, 2008 (I Delhi).

Q.28. If marginal propensity to save is 0.2, how much new investment is required to make the National Income rise by Rs. 600 crores? Calculate.
Answer. MPS = 0.2
Image may be NSFW.
Clik here to view.
cbse-sample-papers-for-class-12-economics-outside-delhi-2008-23

Question.31. Calculate ‘Gross National Product at Market Price’ and ‘Personal Income’ bom the following data:
Image may be NSFW.
Clik here to view.
cbse-sample-papers-for-class-12-economics-outside-delhi-2008-24

Answer. \({ GNP }_{ MP }\)
= Wages and salaries + Social security contributions by employers + Operating surplus + Net factor income earned from abroad + Consumption of fixed capital + Net indirect
tax
= (ii) + (viii) + (iv) + (vi) + (vii) + (ix)
= 200 + 30 + 400 + (-10) + 20 + 40 = 690 – 10 = Rs. 680 crores Personal Income
= Net domestic product at factor cost accruing to the private sector + National debt interest + Net current transfers from abroad + Current transfers from government – Corporation tax + Net factor income from abroad = (x) + (iii) + (v) + (xi) – (i) + (vi)
= 500 + 25 + 15 + 5 – 35 + (-10) = 545 – 45 = Rs. 500 crores.

SET III
Note: Except for the following questions, all the remaining questions have been asked in Set I and Set II.

SECTION A
Question.7. What is the likely effect of technological progress on the supply of a good? Explain,
Answer. See Q. 9, 2008 (I Delhi)

Question.9. Explain the central problem of distribution of income.
Answer. Due to scarcity of resources, goods cannot be produced for everybody. Therefore, it has to be decided as to for whom goods should be produced.
This problem relates to how goods and services produced in the economy should be distributed. The distribution 6f the product is according to the ability to buy, i.e., income of the person or his purchasing power.
The question ‘for whom to produce’ actually relates to the distribution of income generated from production of output. Incomes are distributed in the form of factor earnings, i.e., wages, : rent, interest and profit. Hence it is also called the problem of functional distribution of income.
In a capitalist economy, it is observed that price mechanism facilitates more production of luxuries meant for rich people and less production of goods of mass consumption meant for poor people.

Question.10. Price elasticity of demand of a good is (-) 3. If the price rises from Rs. 10 per unit to Rs. 12 per unit, what is the percentage change in demand?
Answer.
Image may be NSFW.
Clik here to view.
cbse-sample-papers-for-class-12-economics-outside-delhi-2008-25

Question.12. Complete the following table:
Image may be NSFW.
Clik here to view.
cbse-sample-papers-for-class-12-economics-outside-delhi-2008-26

Answer.
Image may be NSFW.
Clik here to view.
cbse-sample-papers-for-class-12-economics-outside-delhi-2008-27

SECTION B
Question.25. Calculate ‘Gross Value jf^dded at Factor Cost’ from the following data:
Image may be NSFW.
Clik here to view.
cbse-sample-papers-for-class-12-economics-outside-delhi-2008-28

Answer. Gross Value Added at Factor Cost (GVAPC)
= Sales + Change in stock Sales tax – Excise duty – Purchase of raw material
= (ii) + (v) – (i) – (iv) – (iii)
= 400 + (-40) – 20 – 30 – 250 = 400 – 340 = Rs. 60 lakhs

Question.28. Distinguish between:
(i) balanced budget and surplus budget
(ii) developmental expenditure and non-developmental expenditure
Ans. (i) Balanced budget and surplus budget
Balanced budget signifies a situation when estimated receipts and expenditure of the government are equal. On the other hand, surplus budget signifies a situation when estimated receipts are more than the estimated expenditure of the government. In real world, we find neither the situation of balanced budget nor of surplus budget. What we find is a situation of deficit budget where proposed receipts are less than the proposed expenditure and the deficit is financed through borrowings. The main cause of such a situation is the nature of modem States which are Welfare States, where government cannot think of spending less, rather expenditure keeps on increasing, (ii) Developmental expenditure and non-developmental expenditure
The expenditure which is incurred on activities directly related to economic development is called developmental expenditure. Hence expenditure incurred on education, health care, scientific research, infrastructure (such as construction of roads, bridges, dams, etc.) is developmental expenditure.
Expenditure incurred on general essential services required for normal running of the government is regarded as non-developmental expenditure. Expenditure incurred on services related to general administration, police, defence, judiciary etc. come under non-developmental expenditure.

Question.29. Given the marginal propensity to save is equal to 0.25. What will be the increase in national income investment increases by Rs. 125 crores? Calculate.
Answer. MPS = 0.25
Image may be NSFW.
Clik here to view.
cbse-sample-papers-for-class-12-economics-outside-delhi-2008-29

Question.32. Calculate ‘Net National Product at Market Price’ and ‘Private Income’ from the following, data:
Image may be NSFW.
Clik here to view.
cbse-sample-papers-for-class-12-economics-outside-delhi-2008-30

Answer. Net National Product
Private final consumption expenditure + Government final consumption expenditure + Gross domestic capital formation + Net exports + Net factor income from abroad – Gross national disposable income + Net national disposable income = (ii) + (v) + (vii) + (ix) + (i) – (iv) + (xi)
= 100 + 20 + 30 + (-10) + (-5) – 170 + 145 = 295 – 185 = Rs. 110 crores Private Income
= Personal disposable income + Personal tax + Corportion tax + Saving of private
corporate sector
= (viii) + (iii) + (vi) + (x)
= 70 + 20 + 15 + 5 = Rs. 110 crores

 

The post CBSE Sample Papers for Class 12 Economics Outside Delhi – 2008 appeared first on Learn CBSE.

NCERT Exemplar Problems Class 8 Mathematics Playing with Numbers

NCERT  Exemplar Problems Class 8 Mathematics Chapter 13 Playing with Numbers

Multiple Choice Questions
Question. 1 Generalised form of a four-digit number abdc is
(a) 1000a + 100b + 10c + d (b) 1000a + 100c + 10b + d
(c) 1000a + 100b + 10d + c (d) a x b x c x d
Solution. (c) In generalised form, we express a number as the sum of the products of its digits with their respective place values.
abdc is written in generalised form as 1000a + 100b + 10d + c.
i.e. abdc = 1000a + 100b + 10d + c

Question. 2 Generalised form of a two-digit number xy is
(a)x + y (b)10x + y (c)10x-y (d)10y+x
Solution. (b) In generalised form, xy can be written as the sum of the products of its digits with their respective place values, i.e.xy = 10x+ y

Question. 3 The usual form of 1000a + 10b + c is
(a) abc (b) abc0 (c) a0be (d) ab0c
Solution. (c) Given expanded (or generalised) form of a number is 1000a + 10b + c. Then, we have to find its usual form.
We can write it as 1000 x a + 100 x 0 + 10 x b + c
i.e. a0bc, which is the usual form.

Question. 4 Let abc be a three-digit number. Then, abc – cba is not divisible by
(a) 9 (b) 11 (c) 18 (d) 33
Solution. (c) Given, abc is a three-digit number.
Then, abc = 100a + 10b + c
and cba = 100c + 10b + a
abc -eba = (100a + 10b + c)- (100c + 10b + a)
= 100a – a + 10b – 10b + c -100=
= 99a – 99c = 99 (a -c)
= abc-cba is divisible by 99.
=> abc – cba is divisible by 9,11,33, but it is not divisible by 18.

Question. 5 The sum of all the numbers formed by the digits x, y and z of the number xyz is divisible by (a) 11 (b) 33 (c) 37 (d) 74
Solution. (c) We have, xyz + yzx + zxy
= (100x + 10y + z) + (100y + 10 z+ x) + (100z+ 10x + y) …(i)
= 100x + 10x + x + 10y + 100y + y + z+ 100z+ 10Z
= 111x + 111y + 111z = 111 (x + y + z)
= 3 x 37 x (x + y + z)
Hence, Eq. (i) is divisible by 37, but not divisible by 11,33 and 74.

Question. 6 A four-digit number aabb is divisible by 55. Then, possible value(s) of b is/are
(a) 0 and 2 (b) 2 and 5 (c) 0 and 5 (d) 7
Solution. (c) It is given that, aabb is divisible by 55. Then, it is also divisible by 5.
Now, if a number is divisible by 5, then its unit digit is either 0 or 5.
Hence, the possible values of b are 0 and 5.

Question. 7 Let abc be a three-digit number. Then, abc + bca + cab is not divisible by
(a) a + b + c (b) 3 (c) 37 (d) 9
Solution. (d) We know that, the sum of three-digit numbers taken in cyclic order can be written as 111 (a + b + c).
i.e. abc + pea + cab = 3 x 37 x (a + b + c)
Hence, the sum is divisible by 3, 37 and (a + b + c) but not divisible by 9.

Question. 8 A four-digit number 4ob5 is divisible by 55. Then, the value of b-a is
(a) 0 (b) 1 (0 4 (d) 5
Solution. (b) Given, a four-digit number 4ab5 is divisible by 55. Then, it is also divisible by 11.
The difference of sum of its digits in odd places and sum of its digits in even places is either 0 or multiple of 11.
i.e. (4 + b) – (a + 5) is 0 or a multiple of 11, if 4 + b — a — 5 = 0 => b-a = 1

Question. 9 If abc is a three-digit number, then number abc -a-b-c is divisible by
(a) 9 (b) 90 (c)10 (d) 11
Solution. (d) We have, abc= 100a + 10b+c
.-. abc – a – b-c = (100a + 10b + c)-a – b-c = 100a – a + 10b – b = 99a + 9b = 9(11a + b)
Hence, the given number abc – a – b-c is divisible by 9.

Question. 10 A six-digit number is formed by repeating a three-digit number. For example, 256256, 678678 etc. Any number of this form is divisible by (a) 7 only (b) 11 only (c) 13 only (d) 1001
Solution.
Image may be NSFW.
Clik here to view.
ncert-exemplar-problems-class-8-mathematics-playing-with-numbers-1

Question. 11 If the sum of digits of a number is divisible by three, then the number is always divisible by (a) 2 (b) 3 (c) 6 (d) 9
Solution. (b) We know that, if sum of digits of a number is divisible by three, then the number must be divisible by 3, i.e. the remainder obtained by dividing the number by 3 is same as the remainder obtained by dividing the sum of its digits by 3.

Question. 12 If x +y + z = 6 and z is an odd digit, then the three-digit number xyz is
(a) an odd multiple of 3 (b) an odd multiple of 6
(c) an even multiple of 3 (d) an even multiple of 9
Solution. (a) We have, x + y+ z= 6 and z is an odd digit. Since, sum of the digits is divisible by 3, it will also be divisible by 2 and 3 but unit digit is odd, so it is divisible by 3 only.Hence, the number is an odd multiple of 3.

Question. 13 If 5A + 53 = 65, then the values of A and B is
(a) A = 2,8 = 3 (b) A = 3,8 = 2 (c)A = 2,8 = 1 (d)A=1,8 = 2
Solution.
Image may be NSFW.
Clik here to view.
ncert-exemplar-problems-class-8-mathematics-playing-with-numbers-2

Question. 14 If A3 + 8B = 150, then the value of A + B is
(a) 13 (b) 12 (c) 17 (d) 15
Solution. (a) We have, A3+ 8B = 150
Here, 3 + B = 0, so 3 + B is a two-digit number whose unit’s digit is zero.
.-. 3+B = 10=>B = 7
: Now, considering ten’s column, A+ 8 + 1 = 15
= A + 9 = 15
=> A = 6
Hence, A+B=6+7 = 13

Question. 15 If 5A x A = 399, then the value of A is
(a) 3 (b) 6 (c) 7 (d) 9
Solution. (c) We have, 5A x A = 399
Here, A x A= 9 i.e. A x A is the number 9 or a number whose unit’s digit is 9. Thus, the number whose product with itself produces a two-digit number having its unit’s digit as 9 is 7.
i.e. A x A = 49 => A=7
Now, 5 x A + 4 = 39
=> 5 x 7+4 = 39
So, A satisfies the product.
Hence, the value of A is 7.

Question. 16 If 6A x B = > 488, then the value of A-B is
(a)-2 (b) 2 (c) -3 (d) 3
Solution. (c) Given, 6A x B = A86
Let us assume, A = 1 and S = 3 Then, LHS = 61 x 3 = 183 and RHS = 183 Thus, our assumption is true.
A-6 = 1-3=-2

Question. 17 Which of the following numbers is divisible by 99?
(a) 913462 (b) 114345 (c) 135792 (d) 3572406
Solution. (b) Given a number is divisible by 99.
Now, going through the options, we observe that the number (b) is divisible by 9 and 11 both as the sum of digits of the number is divisible by 9 and sum of digits at odd places = sum of digits at even places.

Fill in the Blanks
In questions 18 to 33, fill in the blanks to make the statements true.
Question. 18 3134673 is divisible by 3 and————-.
Solution. 9
3134673 is divisible by 3 and 9 as sum of the digits, 3+1+3+4+6+7 + 3 = 27 is divisible by both 3 and 9.

Question. 19 20 x 3 is a multiple of 3, if the digit x is——–or——— or————.
Solution. 1,4,7
We know that, if a number is a multiple of 3, then the sum of its digits is again a multiple of 3, i.e. 2+0+x+3 is a multiple of 3.
x + 5 = 0, 3, 6, 9, 12, 15 But, x is a digit of the number 20 x 3.
x can take values 0, 1,2, 3,……..9.
=> x + 5 = 6 or 9 or 12
Hence, x = 1 or4 or 7

Question. 20 3 x 5 is divisible by 9, if the digit x is————.
Solution. 1
Since, the number 3 x 5 is divisible by 9, then the sum of its digits is also divisible by 9. i.e. 3 + x + 5 is divisible by 9.
=>x + 8 can take values 9,18, 27,…
But x is a digit of the number 3 x 5, so x = 1.

Question. 21 The sum of a two-digit number and the number obtained by reversing the digits is always divisible by———–.
Solution. 11
Let ab be any two-digit number, then the number obtained by reversing its digits is ba.
Now, ab + ba = (10a + b) + (10b + a) = 11a + 11b = 11(a + b)
Hence, ab + ba is always divisible by 11 and (a + b).

Question. 22 The difference of two-digit number and the number obtained by reversing its digits is always divisible by ———-.
Solution. 9
Let ab be any two-digit number, then we have
ab – ba = (10a + b)- (10b + a)
= 9a – 9b = 9(a – b)
Hence, ab – ba is always divisible by 9 and (a – b).

Question. 23 The difference of three-digit number and the number obtained by putting the digits in reverse order is always divisible by 9 and——-.
Solution. 11
Let abc be a three-digit number, then we have
abc -cba = (100a + 10b + c)- (100c + 10b+ a) ; = (100a – a) + (c – 10Cc)
= 99a – 99c = 99(a -c)
= 9 x 11 x (a – c)
Hence, abc – cba is always divisible by 9,11 and (a – c).

Question. 24
Image may be NSFW.
Clik here to view.
ncert-exemplar-problems-class-8-mathematics-playing-with-numbers-3

Solution.
Image may be NSFW.
Clik here to view.
ncert-exemplar-problems-class-8-mathematics-playing-with-numbers-4

Question. 25
Image may be NSFW.
Clik here to view.
ncert-exemplar-problems-class-8-mathematics-playing-with-numbers-5

Solution.
Image may be NSFW.
Clik here to view.
ncert-exemplar-problems-class-8-mathematics-playing-with-numbers-6

Question. 26
Image may be NSFW.
Clik here to view.
ncert-exemplar-problems-class-8-mathematics-playing-with-numbers-7

Solution.
Image may be NSFW.
Clik here to view.
ncert-exemplar-problems-class-8-mathematics-playing-with-numbers-8

Question. 27 1 x 35 is divisible by 9, if x =———.
Solution.
Image may be NSFW.
Clik here to view.
ncert-exemplar-problems-class-8-mathematics-playing-with-numbers-9

Question. 28 A four-digit number abed is divisible by 11, if d + b =———–or————-.
Solution. a + c,12(a + c)
We know that, a number is divisible by 11, if the difference between the sum of digits at odd places and the sum of its digits at even places is either 0 or a multiple of 11.
Hence, abcd is divisible by 11, if (d + b)- (a + c) = 0,11,22, 33,…
=> d + b = a + c or d + b = 12(a + c)

Question. 29 A number is divisible by 11, if the differences between the sum of digits at its odd places and that of digits at the even places is either 0 or divisible by ————-.
Solution. 11
By test of divisibility by 1,1, we know that, a number is divisible by 11, if the sum of digits at odd places and even places are equal or differ by a number, which is divisible by 11.

Question. 30 If o three-digit number abc is divisible by 11, then——–is either 0 or multiple of 11.
Solution. (a+c)-b
Since, abc is divisible by 11, the difference of sum of its digits at odd places and that of even places is either zero or multiple of 11, i.e. (a + c) – b is either zero or multiple of 11.

Question. 31 If A x 3 = lA then A =————.
Solution.
Image may be NSFW.
Clik here to view.
ncert-exemplar-problems-class-8-mathematics-playing-with-numbers-10

Question. 32 If B x B = AB, then either A = 2, B = 5 or A =———- B =———-.
Solution.
Image may be NSFW.
Clik here to view.
ncert-exemplar-problems-class-8-mathematics-playing-with-numbers-11

Question. 33 If the digit 1 is placed after a two-digit number whose ten’s is t and one’s digit is u, the new number is———–.
Solution. tu1
Given, a two-digit number whose ones digit isu in and tens digit isu. If the digit 1 is placed after this number, then the next number will be tu1.

True/False
In questions 34 to 44, state whether the given statements are True or False.
Question. 34 A two-digit number ab is always divisible by 2, if b is an even number.
Solution. True
By the test of divisibility by 2, we know that a number is divisible by 2, if its unit’s digit is even.

Question. 35 A three-digit number abc is divisible by 5, if c is an even number.
Solution. False
By the test of divisibility by 5, we know that if a number is divisible by 5, then its one’s digit will be either 0 or 5, i.e. the numbers ending with the digits 0 or 5 are divisible by 5.

Question. 36 A four-digit mmbeFabcd is divisible by 4, if ab is divisible by 4.
Solution. False
As we know that, if a number is divisible by 4, then the number formed by its digits in unit’s and ten’s place is divisible by 4.

Question. 37 A three-digit number abc is divisible by 6, if c is an even number and a + b + c is a multiple of 3.
Solution. True
If a number is divisible by 6, then it is divisible by both 2 and 3. Since, abc is divisible by 6, it is also divisible by 2 and 3. Therefore, c is an even number and the sum of digits is divisible by 3, i.e. multiple of 3.

Question. 38 Number of the form 3N + 2 will leave remainder 2 when divided by 3.
Solution. True
Let x = 3N + 2. Then, it can be written as.
x = (a multiple of 3) + 2
i.e. x is a number which is 2 more than a multiple of 3
i.e. x is a number, which when divided by 3, leaves the remainder 2.

Question. 39 Number 7N+1 will leave remainder 1 when divided by 7.
Solution. True
Given, a number of the form 7N + 1 = x (say)
Here, we observe that * is a number which is one more than a multiple of 7. i.e. when x is divided by 7, it leaves the remainder 1.

Question. 40 If a number a is divisible by b, then it must be divisible by each factor of b.
Solution. True
Given, a is divisible by b.
Let b = p1•p2, where p1 and p2 are primes.
Since, a is divisible by b, a is a multiple of b
i.e. a = mb
=> a = m.p1.p2
or a=cp2=dp1, where c = mp1, d = mp2
=>a is a multiple of p1 as well as p2.
Hence, a is divisible by each factor b.

Question. 41 If AB x 4 = 192, then A + 6 = 7.
Solution.
Image may be NSFW.
Clik here to view.
ncert-exemplar-problems-class-8-mathematics-playing-with-numbers-12

Question. 42 If AB + 7C = 102, where \(B\neq 0\), \(C\neq 0\), then A + B + C = 14.
Solution.
Image may be NSFW.
Clik here to view.
ncert-exemplar-problems-class-8-mathematics-playing-with-numbers-13

If B= C = 1, lf8 = 5,C=7, A = 2 and A +B+C=2 + 5+ 7 = 14

Question. 43 If 213 x 27 is divisible by 9, then the value of x is 0.
Solution. False
Given, 213 x 27 is divisible by 9, so sum of its digits is also divisible by 9.
i.e. 2 1 + 3 + x + 2 + 7 — 0, 9,18, 27, 36,…
=> x+ 15= 0,9,18,27,36,…
=> x + 15 = 18 [x is a digit of a number]
=> x= 3

Question. 44 In N + 5 leaves remainder 3 and \(N\div 2\) leaves remainder 0, then \(N\div 10\) leaves remainder 4.
Solution.
Image may be NSFW.
Clik here to view.
ncert-exemplar-problems-class-8-mathematics-playing-with-numbers-14

Image may be NSFW.
Clik here to view.
ncert-exemplar-problems-class-8-mathematics-playing-with-numbers-15

Question. 45 Find the least value that must be given to number a, so that the number 91876a2 is divisible by 8.
Solution. Given, 91876a2 is divisible by 8.
Since, we know that, if a number is divisible by 8, then the number formed by last 3 digits is divisible by 8.
So, 6a2 is divisible by 8.
Here, a can take values from 0 to 9.
For a = 0, 602 is not divisible by 8.
For a = 1, 612, which is not divisible by 8.
For a = 3, 632 is divisible by 8.
Hence, the minimum value of a is 3 to make 91876a2 divisible by 8.

Question. 46
Image may be NSFW.
Clik here to view.
ncert-exemplar-problems-class-8-mathematics-playing-with-numbers-16

Solution.
Image may be NSFW.
Clik here to view.
ncert-exemplar-problems-class-8-mathematics-playing-with-numbers-17

Question. 47 If 1AB + CCA = 697 and there is no carry-over in addition, find the value of A + B + C.
Solution.
Image may be NSFW.
Clik here to view.
ncert-exemplar-problems-class-8-mathematics-playing-with-numbers-18

Image may be NSFW.
Clik here to view.
ncert-exemplar-problems-class-8-mathematics-playing-with-numbers-19

Question. 48 A five-digit number AABAA is divisible by 33. Write all the numbers of this form.
Solution. Given, a number of the form AABAA is divisible by 33. Then, it is also divisible by 3 and
11, as if a number a is divisible by b, then it is also divisible by each factor of b.
Since, AABAA is divisible by 3, sum its digits is also divisible by 3. i.e. 4 + 4 + 8 + A + .4 = 0,3, 6,9…
or 4/4 + 8 = 0, 3, 6 9,… …(i)
From Eq. (i), we have
Further, the given number is also divisible by 11, therefore (2/4 + 8) – 2A = 0,11,22,…
B=Q 11,22,…
8 = 0 [v8 is a digit of the given number]
4/4 = 12or 24 or 36 A= 3, 6 9
Hence, the required numbers are 33033, 66066 and 99099.

In questions 49 to 60, find the value of the letters in each of the following questions.
Question. 49
Image may be NSFW.
Clik here to view.
ncert-exemplar-problems-class-8-mathematics-playing-with-numbers-20

Solution.
Image may be NSFW.
Clik here to view.
ncert-exemplar-problems-class-8-mathematics-playing-with-numbers-21

Question. 50
Image may be NSFW.
Clik here to view.
ncert-exemplar-problems-class-8-mathematics-playing-with-numbers-22

Solution.
Image may be NSFW.
Clik here to view.
ncert-exemplar-problems-class-8-mathematics-playing-with-numbers-23

Question. 51
Image may be NSFW.
Clik here to view.
ncert-exemplar-problems-class-8-mathematics-playing-with-numbers-24

Solution.
Image may be NSFW.
Clik here to view.
ncert-exemplar-problems-class-8-mathematics-playing-with-numbers-25

Question. 52
Image may be NSFW.
Clik here to view.
ncert-exemplar-problems-class-8-mathematics-playing-with-numbers-26

Solution.
Image may be NSFW.
Clik here to view.
ncert-exemplar-problems-class-8-mathematics-playing-with-numbers-27

Question. 53
Image may be NSFW.
Clik here to view.
ncert-exemplar-problems-class-8-mathematics-playing-with-numbers-28

Solution.
Image may be NSFW.
Clik here to view.
ncert-exemplar-problems-class-8-mathematics-playing-with-numbers-29

Image may be NSFW.
Clik here to view.
ncert-exemplar-problems-class-8-mathematics-playing-with-numbers-30

Question. 54
Image may be NSFW.
Clik here to view.
ncert-exemplar-problems-class-8-mathematics-playing-with-numbers-31

Solution.
Image may be NSFW.
Clik here to view.
ncert-exemplar-problems-class-8-mathematics-playing-with-numbers-32

Question. 55
Image may be NSFW.
Clik here to view.
ncert-exemplar-problems-class-8-mathematics-playing-with-numbers-33

Solution.
Image may be NSFW.
Clik here to view.
ncert-exemplar-problems-class-8-mathematics-playing-with-numbers-34

Question. 56
Image may be NSFW.
Clik here to view.
ncert-exemplar-problems-class-8-mathematics-playing-with-numbers-35

Solution.
Image may be NSFW.
Clik here to view.
ncert-exemplar-problems-class-8-mathematics-playing-with-numbers-36

Image may be NSFW.
Clik here to view.
ncert-exemplar-problems-class-8-mathematics-playing-with-numbers-37

Question. 57
Image may be NSFW.
Clik here to view.
ncert-exemplar-problems-class-8-mathematics-playing-with-numbers-38

Solution.
Image may be NSFW.
Clik here to view.
ncert-exemplar-problems-class-8-mathematics-playing-with-numbers-39

Question. 58
Image may be NSFW.
Clik here to view.
ncert-exemplar-problems-class-8-mathematics-playing-with-numbers-40

Solution.
Image may be NSFW.
Clik here to view.
ncert-exemplar-problems-class-8-mathematics-playing-with-numbers-41

Question. 59
Image may be NSFW.
Clik here to view.
ncert-exemplar-problems-class-8-mathematics-playing-with-numbers-42

Solution.
Image may be NSFW.
Clik here to view.
ncert-exemplar-problems-class-8-mathematics-playing-with-numbers-43

Question. 60
Image may be NSFW.
Clik here to view.
ncert-exemplar-problems-class-8-mathematics-playing-with-numbers-45

Solution.
Image may be NSFW.
Clik here to view.
ncert-exemplar-problems-class-8-mathematics-playing-with-numbers-46

Question. 61 If \(27\div A\)= 33, then find the value of A
Solution. We observe that, 4 x 3 can never be a single digit number 2, so 4 x 3 must be a two-digit number, whose ten’s digit is 2 and unit’s digit is the number less than or equal to 4. Therefore, the value of 4 can be 9, as the values of 4 from 1 to 8 do not fit.

Question. 62 212 x 5 is a multiple of 3 and 11. Find the value of x.
Solution. Since, 212 x 5 is a multiple of 3,
2 +1 + 2 +x+5 = 0, 3, 6,9,12,15,18,
=> 10 + x = 0, 3, 6,………..
=> x =2, 5, 8 …(i)
Again, 212×5 is a multiple of 11, (2 + 2 + 5) — (1 + x) = 0,11,22, 33
=> 8 – x = 0,11,22,…
=> x = 8 …(ii)
From Eqs. (i) and (ii), we have ‘
x = 8

Question. 63 Find the value of k, where 31K2 is divisible by 6.
Solution. Given, 31k2 is divisible by 6. Then, it is also divisible by 2 and 3 both.
Now, 31K2 is divisible by 3, sum of its digits is a multiple of 3.
i.e. 3+ 1 + k + 2 = 0, 3, 6, 9,12,…
=> k+ 6 = 0, 3,6, 9,12
=> k = 0 or 3, 6, 9

Question. 64 1y3y6is divisible by 11. Find the value of y.
Solution. It is given that, 1y3y6 is divisible by 11.
Then, we have (1 + 3 + 6) – (y + y) = 0,11,22,…
=> 10-2y= 0,11,22,…
=> 10-2y = 0 [other values give a negative number]
=> 2y = 10
=> y= 5

Question. 65 756x is a multiple of 11, find the value of x.
Solution. We are given that, 756x is a multiple of 11. Then, we have to find the value of x.
Since, 756x is divisible by 11, then (7 + 6) – (5 + x) is a multiple of 11,
i.e. 8-x = 0,11,22,…
=> 8- x = 0
=> x = 8

Question. 66 A three-digits number 203 is added to the number 326 to give a three-digits number 5b9 Which is divisible by 9. Find the value of b – a.
Solution.
Image may be NSFW.
Clik here to view.
ncert-exemplar-problems-class-8-mathematics-playing-with-numbers-47

Question. 67 Let E = 3, B = 7 and A = 4. Find the other digits in the sum
Image may be NSFW.
Clik here to view.
ncert-exemplar-problems-class-8-mathematics-playing-with-numbers-48

Solution.
Image may be NSFW.
Clik here to view.
ncert-exemplar-problems-class-8-mathematics-playing-with-numbers-49

Image may be NSFW.
Clik here to view.
ncert-exemplar-problems-class-8-mathematics-playing-with-numbers-50

Question. 68 Let D = 3, L = 7 and A = 8. Find the other digits in the sum
Image may be NSFW.
Clik here to view.
ncert-exemplar-problems-class-8-mathematics-playing-with-numbers-51

Solution. In the first column, we have
3 + S + 8, which is definitely a two digits number whose unit’s digit is 7.
S must be 6.
Now, in second column, 2A +1 = 16+1 = 7 [1 is carry forward]
In third column, M + 1 is a 2 digit number, therefore M must be 9.
Then, M + 1 = 9+ 1 = 10 6 = 1, U = 0
Hence, S = 6, M = 9, 6 = 1 and U = 0

Question. 69 If from a two-digit number, we subtract the number formed by reversing its digits then the result so obtained is a perfect cube. How many such numbers are possible? Write all of them.
Solution. Let ab be any two-digit number. Then, the digit formed by reversing it digits is ba.
Now, ab-ba = (10a+b)-(10b +a)
=(10a-a)+(b-10b)
= 9a – 9b = 9(a – b)
Further, since ab-ba is a perfect cube and is a multiple of 9.
.-.The possible value of a – b is 3.
i.e. a = b + 3
Here, b can take value from 0 to 6.
Hence, possible numbers are as follow.
For b = 0, a = 3, i.e. 30
For b = 1, a = 4, i.e. 41
For b =2, a = 5, i.e. 52
For b = 3, a = 6, i.e. 63
For b = 4, a = 7, i.e. 74
For b = 5,a = 8, i.e. 85
For b = 6, a = 9, i.e. 96

Question. 70 Work out the following multiplication.
Image may be NSFW.
Clik here to view.
ncert-exemplar-problems-class-8-mathematics-playing-with-numbers-52

Use the result to answer the following questions.
(a) What will be 12345679 x 45?
(b) What will be 12345679 x 63?
(c) By what number should 12345679 be multiplied to get 888888888?
(d) By what number should 12345679 be multiplied to get 999999999?
Solution.
Image may be NSFW.
Clik here to view.
ncert-exemplar-problems-class-8-mathematics-playing-with-numbers-53

Question. 71 Find the value of the letters in each of the following.
Image may be NSFW.
Clik here to view.
ncert-exemplar-problems-class-8-mathematics-playing-with-numbers-54

Solution.
Image may be NSFW.
Clik here to view.
ncert-exemplar-problems-class-8-mathematics-playing-with-numbers-55

Question. 72 If 148101B095 is divisible by 33, find the value of B.
Solution. Given that the number 148101S095 is divisible by 33, therefore it is also divisible by 3 and 11 both.
Now, the number is divisible by 3, its sum of digits is a multiple of 3. i.e. 1 + 4+ 8+1 + 0+1 + B+ 0+ 9+ 5 is a multiple of 3.
29 + B = 0, 3, 6,9,…
=> B=1,4,7 …(i)
Also, given number is divisible by 11, therefore
(1+ 8 + 0+ B + 9)-(4 + 1+ 1+ 0 + 5)=0, 11,22, …
=> (18 + B) -11 = 0,11,22
B+ 7 = 0,11,22
=> B+7 = 11 => B = 4 …(ii)
From Eqs. (i) and (ii), we have B = 4

Question. 73 If 123123A4 is divisible by 11, find the value of A.
Solution. Given, 12312344 is divisible by 11, then we have (1 + 3 + 2 + 4) – (2 + 1 + 3 + 4) is a multiple of 11.
i.e. (6+ 4)-10 =0,11,22,,..
=> A-4 = 0,11,22,…
=> A-4 = 0 [A is a digit of the given number]
=> A = 4

Question. 74 If 56 x 32y is divisible by 18, find the least value of y.
Solution. It is given that, the number 56 x 32y is divisible by 18. Then, it is also divisible by each factor of 18.
Thus, it is divisible by 2 as well as 3.
Now, the number is divisible by-2, its unit’s digit must be an even number that is 0, 2,4, 6, Therefore, the least value of y is 0.
Again, the number is divisible by 3 also, sum of its digits is a multiple of 3. i.e. 5 + 6 + x + 3 + 2 + y is a multiple of 3
=> 16 + x + y = 0, 3, 6, 9,…
=> 16+ x = 18
=>x = 2, which is the least value of x.

The post NCERT Exemplar Problems Class 8 Mathematics Playing with Numbers appeared first on Learn CBSE.

NCERT Exemplar Problems Class 8 Mathematics Mensuration

NCERT Exemplar Problems Class 8 Mathematics  Chapter 11 Mensuration

Multiple Choice Questions 
Question. 1 A cube of side 5 cm is painted on all its faces. If it is sliced into 1 cubic centimetre cubes, then how many 1 cubic centimetre cubes will have exactly one of their faces painted?
(a) 27     (b) 42       (c) 54     (d) 142
Solution.
Image may be NSFW.
Clik here to view.
ncert-exemplar-problems-class-8-mathematics-mensuration-1

Question. 2 A cube of side 4 cm is cut into 1 cm cubes. What is the ratio of the surface areas of the original cube and cut-out cubes?
(a) 1 : 2 (b) 1 : 3 (c) 1 : 4 (d) 1 : 6
Solution.
Image may be NSFW.
Clik here to view.
ncert-exemplar-problems-class-8-mathematics-mensuration-2

Question. 3 A circle of maximum possible size is cut from a square sheet of board. Subsequently, a square of maximum possible size is cut from the resultant circle. What will be the area of the final square?
(a) 3/4 of original square (b) 1/2 of original square
(c) 1/4 of original square (d) 2/3 of original square
Solution.
(b) Let a be the side of a square sheet.
Image may be NSFW.
Clik here to view.
ncert-exemplar-problems-class-8-mathematics-mensuration-3

Question.4 What is the area (in sq units) of the largest triangle, that can be fitted into a rectangle of length l units and width w units?
(a)lw/2 (b) lw/3 (c) lw/6 (d) lw/4
Solution.
Image may be NSFW.
Clik here to view.
ncert-exemplar-problems-class-8-mathematics-mensuration-4

Image may be NSFW.
Clik here to view.
ncert-exemplar-problems-class-8-mathematics-mensuration-5

Question. 5 If the height of a cylinder becomes1/4 of the original height and the radius
is doubled, then which of the following will be true?
(a) Volume of the cylinder will be doubled
(b) Volume of the cylinder will remain unchanged
(c) Volume of the cylinder will be halved
(d) Volume of the cylinder will be 1/4 of the original volume
Solution.
Image may be NSFW.
Clik here to view.
ncert-exemplar-problems-class-8-mathematics-mensuration-6

Question. 6 If the height of a cylinder becomes 1/4 of the original height and the radius
is doubled, then which of the following will be true?
(a) Curved surface area of the cylinder will be doubled
(b) Curved surface area of the cylinder will remain unchanged
(c) Curved surface area of the cylinder will be halved
(d) Curved surface area will be 1/4 of the original curved surface
Solution.
Image may be NSFW.
Clik here to view.
ncert-exemplar-problems-class-8-mathematics-mensuration-7

Question. 7 If the height of a cylinder becomes 1/4 of the original height and the radius
is doubled, then which of the following will be true?
(a) Total surface area of the cylinder will be doubled
(b) Total surface area of the cylinder will remain unchanged
(c) Total surface area of the cylinder will be halved
(d) None of the above
Solution.
Image may be NSFW.
Clik here to view.
ncert-exemplar-problems-class-8-mathematics-mensuration-8

Question. 8 The surface area of the three coterminus faces of a cuboid are 6, 15 and 10 cm2, respectively. The volume of the cuboid is
(a) 30 Cm3 (b) 40 Cm3 (c) 20 Cm3 (d) 35 Cm3
Solution.
Image may be NSFW.
Clik here to view.
ncert-exemplar-problems-class-8-mathematics-mensuration-9

Question. 9 A regular hexagon is inscribed in a Circle of radius r. The perimeter of the regular hexagon is
(a) 3r                       (b) 6r                   (c)9r                          (d)12r
Solution.
(b) A regular hexagon comprises 6 equilateral triangles, each of them having one of their vertices at the centre of the hexagon.
The sides of the equilateral triangle are equal to the radius of the smallest circle inscribing the hexagon.
Hence, each side of the hexagon is equal to the radius of the hexagon and the perimeter of the hexagon is 6 r.

Question. 10 The dimensions of a godown are 40 m, 25 m and 10 m. If it is filled with . cuboidal boxes each of dimensions 2 m x 1.25 m x 1 m, then the number of boxes will be .
(a) 1800 (b) 2000 (c) 4000 (d) 8000
Solution.
Image may be NSFW.
Clik here to view.
ncert-exemplar-problems-class-8-mathematics-mensuration-10

Image may be NSFW.
Clik here to view.
ncert-exemplar-problems-class-8-mathematics-mensuration-11

Question. 11 The volume of a cube is 64 cm3. Its surface area is
(a) 16 cm2 (b) 64 cm2
(c) 96 cm2 (d) 128 cm2
Solution.
(c) Let the side of the cube be a. Then,
Volume of cube = a3 = 64 [given]
⇒                            a = 4
Now, surface area of the cube = a2 = 6 x 42 = 96 cm2

Question. 12 If the radius of a cylinder is tripled but its curved surface area is unchanged, then its height will be
(a) tripled , (b) constant
(c) one-sixth (d) one-third
Solution.
Image may be NSFW.
Clik here to view.
ncert-exemplar-problems-class-8-mathematics-mensuration-12

Question. 13 How many small cubes with edge of 20 cm each can be just accommodated in a cubical box of 2 m edge?
(a) 10 (b) 100 (c) 1000 (d) 10000
Solution.
Image may be NSFW.
Clik here to view.
ncert-exemplar-problems-class-8-mathematics-mensuration-13

Question. 14 The volume of a cylinder whose radius r is equal to its height, is
Image may be NSFW.
Clik here to view.
ncert-exemplar-problems-class-8-mathematics-mensuration-140

Solution.
(c) Given, r = h
Then, volume of cylinder = πr2h = πr2r=πr3

Question. 15 The volume of a cube whose edge is 3x, is
(a) 27x3    (b) 9x3     (c)6x3    (d) 3x3
Solution.
Image may be NSFW.
Clik here to view.
ncert-exemplar-problems-class-8-mathematics-mensuration-14

Question. 16 The figure ABCD is a quadrilateral, in which AB area is CD and BC = AD. Its area is
Image may be NSFW.
Clik here to view.
ncert-exemplar-problems-class-8-mathematics-mensuration-15

Solution.

Image may be NSFW.
Clik here to view.
ncert-exemplar-problems-class-8-mathematics-mensuration-16

Question. 17 What is the area of the rhombus ABCD below, if AC = 6 cm and BE = 4 cm?
Image may be NSFW.
Clik here to view.
ncert-exemplar-problems-class-8-mathematics-mensuration-17

Solution.
Image may be NSFW.
Clik here to view.
ncert-exemplar-problems-class-8-mathematics-mensuration-18

Question. 18 The area of a parallelogram is 60 cm2 and one of its altitude is 5 cm. The length of its corresponding side is 
(a) 12 cm    (b)6 cm     (c) 4 cm     (d) 2 cm
Solution.
Image may be NSFW.
Clik here to view.
ncert-exemplar-problems-class-8-mathematics-mensuration-19

Question. 19 The perimeter of a trapezium is 52 cm and its each non-parallel side is equal to 10 cm with its height 8 cm. Its area is
(a) 124 cm2 (b) 118 cm2 (c) 128 cm2 (d) 112 cm2
Solution.
Image may be NSFW.
Clik here to view.
ncert-exemplar-problems-class-8-mathematics-mensuration-20

Question. 20 Area of a quadrilateral ABCD is 20 cm2 and perpendiculars on BD from opposite vertices are 1 cm and 1.5 cm. The length of BD is
(a) 4 cm          (b) 15 cm                   (c) 16 cm                (d) 18 cm
Solution.
Image may be NSFW.
Clik here to view.
ncert-exemplar-problems-class-8-mathematics-mensuration-21

Question. 21 A metal sheet 27 cm long, 8 cm broad and 1 cm thick is melted into a cube. The side of the cube is
(a) 6 cm (b) 8 cm (c) 12 cm (d) 24 cm
Solution.
(a) Given, a metal sheet 27 cm long, 8 cm broad and 1 cm thick.
Then, volume of the sheet (cubiodal) = l x b x h
=27 x 8 x 1 = 216 cm3
Now, since this sheet is melted to form a cube of edge length a (say).
Then, volume of the cube = Volume of the metal sheet
⇒    a3 =216 cm3
⇒   a = 6 cm
Hence, the side of the cube is 6 cm.

Question. 22 Three cubes of metal whose edges are 6 cm, 8 cm and 10 cm respectively, are melted to form a single cube. The edge of the new cube is
(a) 12 cm (b) 24 cm
(c) 18 cm (d) 20 cm
Solution.
Image may be NSFW.
Clik here to view.
ncert-exemplar-problems-class-8-mathematics-mensuration-22

Question. 23 A covered wooden box has the inner measures as 115 cm, 75 cm and 35 cm and thickness of wood as 2.5 cm. The volume of the wood is
(a) 85000 cm3 (b) 80000 cm3
(c) 82125 cm3 (d) 84000 cm3
Solution.
Image may be NSFW.
Clik here to view.
ncert-exemplar-problems-class-8-mathematics-mensuration-23

Question.24 The ratio of radii of two cylinders is 1: 2 and heights are in the ratio 2 : 3. The ratio of their volumes is
(a) 1 : 6 (b) 1 : 9 (c) 1 : 3 (d) 2 : 9
Solution.
Image may be NSFW.
Clik here to view.
ncert-exemplar-problems-class-8-mathematics-mensuration-24

Question. 25 Two cubes have volumes in the ratio 1 : 64. The ratio of the areas of a face of first cube to that of the other is
(a) 1 : 4 (b) 1 : 8
(c) 1 : 16 (d) 1 : 32
Solution.
Image may be NSFW.
Clik here to view.
ncert-exemplar-problems-class-8-mathematics-mensuration-25

Question. 26 The surface areas of the six faces of a rectangular solid are 16, 16, 32, 32, 72 and 72 sq cm. The volume of the solid (in cu cm) is (a) 192 (b) 384 (c) 480 (d) 2592
Solution.
Image may be NSFW.
Clik here to view.
ncert-exemplar-problems-class-8-mathematics-mensuration-26

Question. 27 Ramesh has three containers.
(i) Cylindrical container A having radius r and height h.
(ii)Cylindrical container B having radius 2r and height 1/2 h.
(iii)Cuboidal container C having dimensions r x r x h.
The arrangement of the containers in the increasing order of their volumes is
(a)A,B,C             (b) B, C, A
(c) C, A, B          (d) Cannot be arranged
Solution.
(c) (i) The volume of the cylindrical container having radius r and height h
= πrh
(ii) The volume of the cylindrical container with radius 2 r and height 1/2
= π (2 r)2 x 1/2 h = π x 4 r2 x 1/2 h
‘ =2 πr2h
(iii) The volume of the cuboidal container having dimensions rxrxh
= rh
From parts (i), (ii) and (iii), we have the following order C, A, B.

Question. 28 If R is the radius of the base of the hat, then the total outer surface area of the hat is
Image may be NSFW.
Clik here to view.
ncert-exemplar-problems-class-8-mathematics-mensuration-27

Solution.
Image may be NSFW.
Clik here to view.
ncert-exemplar-problems-class-8-mathematics-mensuration-28

Fill in the Blanks
In questions 29 to 52, fill in the blanks to make the statements are true.
Question. 29 A cube of side 4 cm is painted on all its sides. If it is sliced in 1 cu cm cubes, then number of such cubes that will have exactly two of their faces painted, is_______.
Solution. 24
The volume of a cube of side 4 cm = 4 x 4 x 4 = 64cm3 When it is sliced into 1 cm3 cubes, we will get 64 small cubes.
In each side of the larger cube, the smaller cubes in the edges will have more than one face painted.
The cubes which are situated at the corners of the big cube, have three faces painted.
So, to each edge two small cubes are left which have two faces painted. As, the total number of edges in a cube are 12.
Hence, the number of small cubes with two faces painted = 12 x 2 = 24

Question. 30 A cube of side 5 cm is cut into 1 cm cubes. The percentage increase in volume after such cutting is_______.
Solution.
Image may be NSFW.
Clik here to view.
ncert-exemplar-problems-class-8-mathematics-mensuration-29

Question. 31 The surface area of a cuboid formed by joining two cubes of side a face-to-face, is_______.
Solution. 10a2
We have, two cubes of side a.
These two cubes are joined face-to-face, then the resultant solid figure is a cuboid which has same breadth and height as the joined cubes has length twice of the length of a cube, i.e. l = 2a,b= aandh = a
Thus, the total surface area of the cuboid = 2 (lb + bh + hi)
= 2 (2a x a + a x a + a x 2a)
= 2 (2 a2 + a2 + 2 a2)=2 x 5 a2 = 10a2

Question. 32 If the diagonals of a rhombus get doubled, then the area of the rhombus becomes_______its original area.
Solution.
Image may be NSFW.
Clik here to view.
ncert-exemplar-problems-class-8-mathematics-mensuration-30

Question. 33 If a cube fits exactly in a cylinder with height h, then the volume of the cube is_______and_______surface area of the cube is .
Solution. h3 ,6h2
Since, the cube fits exactly in the cylinder with height h, therefore each side of the cube = h
Now, volume of the cube = (Side)3 =  h3
and surface area of the cube = 6 x (Side)2
=6 x h2

Question. 34 The volume of a cylinder becomes_______the original volume, if its
radius becomes half of the original radius.
Solution.

Image may be NSFW.
Clik here to view.
ncert-exemplar-problems-class-8-mathematics-mensuration-31

Question. 35 The curved surface area of a cylinder is reduced by_______per cent, if
the height is half of the original height.
Solution.
Image may be NSFW.
Clik here to view.
ncert-exemplar-problems-class-8-mathematics-mensuration-32

Question. 36 The volume of a cylinder which exactly fits in a cube of side a, is _______.
Solution.
Image may be NSFW.
Clik here to view.
ncert-exemplar-problems-class-8-mathematics-mensuration-33

Question. 37 The curved surface area of a cylinder which exactly fits in a cube of side b, is _______.
Solution.
Image may be NSFW.
Clik here to view.
ncert-exemplar-problems-class-8-mathematics-mensuration-34

Question. 38 If the diagonal d of a quadrilateral is doubled and the heights h1 and h2
falling on d are halved, then the area of quadrilateral is _______.
Solution.
Image may be NSFW.
Clik here to view.
ncert-exemplar-problems-class-8-mathematics-mensuration-35

Question. 39 The perimeter of-a-rectangle becomes_______times its original
perimeter, if its length and breadth are doubled.
Solution. 2 times
Perimeter of a rectangle with length l and breadth b = 2(l + b)
If the length and the breadth are doubled, then the new perimeter
= 2(2l + 2b)
= 2[2(l + b)]

Question. 40 A trapezium with 3 equal sides and one side double the equal side, can be divided into_______ equilateral triangles of_______area.
Solution.
Image may be NSFW.
Clik here to view.
ncert-exemplar-problems-class-8-mathematics-mensuration-36

Question. 41 All six faces of a cuboid are_______in shape and of_______area.
Solution. rectangular, different
We know that, a cuboid is made of 6 rectangular plane regions, i.e. 6 rectangular faces, which have different lengths and breadths Therefore the area of the rectangular faces are different. .

Question.42 Opposite faces of a cuboid are_______in area.
Solution. equal
We know that, a cuboid has 6 rectangular races, of which opposite faces have the same length and breadth. Therefore, area of the opposite faces are equal.

Question.43 Curved surface area of a cylinder of radius h and height r is_______.
Solution. 2πhr  (or) 2πrh
We know that, the curved surface area of a cylinder of radius h and height r
= 2π x Radius x Height .
= 2π x h x r=2πhr
= 2πrh

Question.44 Total surface area of a cylinder of radius h and height r is_______ .
Solution. 2πh(r+ h)
Given, radius of cylinder = h and height of cylinder = r
...Total surface area of a cylinder = Curved surface area + Area of top surface + Area of base
= 2 x π x Radius x Height + π (Radius)2+ π (Radius)2
= 2πhr+ πh2 + πh2
= 2πrh + 2πh2
= 2πh(r + h)

Question.45 Volume of a cylinder with radiusT? and height r is_______.
Solution. πh2r
Given, radius of cylinder = h and height of cylinder = r.
Now, volume of a cylinder
= π x (Radius)x Height =π x h2 x r = πh2r

Question.46 Area of a rhombus = 1/2 product of_______.
Solution. diagonals
We know that, the area of a rhombus = Half of the product of its diagonals
=1/2 [Product of diagonals]

Question. 47 Two cylinders A and B are formed by folding a rectangular sheet of dimensions 20 cm x 10 cm along its length and also along its breadth, respectively. Then, volume of A is_______of volume of B.
Solution.
Image may be NSFW.
Clik here to view.
ncert-exemplar-problems-class-8-mathematics-mensuration-37

Question. 48 In the above question, curved surface area of A is_______surface area of B.
Solution.
Image may be NSFW.
Clik here to view.
ncert-exemplar-problems-class-8-mathematics-mensuration-38

Question. 49_______of a solid is the measurement of the space occupied by it.
Solution. Volume
We know that, a solid always occupies some space and magnitude of this space region is known as the volume of the solid.

Question. 50_______surface area of room = Area of 4 walls.
Solution. Lateral
We know that, a room is in the shape of a cuboid. Its 4 walls are treated as lateral faces of the cuboid.
... Lateral surface area of room = Area of 4 walls

Question. 51 Two cylinders of equal volume have heights in the ratio 1: 9. The ratio of their radii is_______.
Solution.
Image may be NSFW.
Clik here to view.
ncert-exemplar-problems-class-8-mathematics-mensuration-39

Question. 52 Two cylinders of same volume have their radii in the ratio 1 : 6. Then, ratio of their heights is_______.
Solution.
Image may be NSFW.
Clik here to view.
ncert-exemplar-problems-class-8-mathematics-mensuration-40

True/False
In questions 53 to 61, state whether the statements are True or False.

Question. 53 The areas of any two faces of a cube are equal.
Solution. True
Since, all the faces of a cube are squares of same side length, therefore the areas of any two faces of a cube are equal.

Question. 54 The areas of any two faces of a cuboid are equefl.
Solution. False
A cuboid has rectangular faces with different lengths and breadths. Only opposite faces of cuboid have the same length and breadth.
Therefore, areas of only opposite faces of a cuboid are equal.

Question. 55 The surface area of a cuboid formed by joining face-to-face 3 cubes of side x is 3 times the surface area of a cube of side x.
Solution. False
Three cubes having side x are joined face-to-face, then the cuboid so formed has the same height and breadth as the cubes but its length will be thrice that of the cubes.
Hence, the length, breadth and height of the cuboid so formed are 3x, x and x, respectively. Then, its surface area = 2 (lb + bh + hl)
= 2(3x x x + x x x + x x 3 x )=2(3xx2 + 3x2)
= 2 x 7x2 = 14x2
Now, the surface area of the cube of side x = 6 (Side)2 = 6x2 Hence, the statement is false.

Question. 56 Two cuboids with equal volume will always have equal surface area.
Solution.
Image may be NSFW.
Clik here to view.
ncert-exemplar-problems-class-8-mathematics-mensuration-41

Question. 57 The area of a trapezium becomes 4 times, if its height gets doubled.
Solution. False
We know that,
Area of a trapezium = 1/2 (a + b) x h
where, a and b are the lengths of parallel sides and h is the altitude (height).
Now, if the height gets doubled, then
Area of trapezium = 1/2(a + b) x 2h = 2(1/2(a + b) x h)
Hence, the area is doubled.
So, the statement is false.

Question. 58 A cube of side 3 cm painted on all its faces, when sliced into 1 cu cm cubes, will have exactly 1 cube with none of its faces painted.
Solution. True
Given, a cube of side 3 cm is painted on all its faces. Now, it is sliced into 1 cu cm cubes. Then, there will be 8 corner cubes that have 3 sides painted, 6 centre cubes with only one side painted and only 1 cube in the middle that has no side painted.

Question. 59 Two cylinders with equal volume will always have equal surface area.
Solution. False
Image may be NSFW.
Clik here to view.
ncert-exemplar-problems-class-8-mathematics-mensuration-141

Question. 60 The surface area of a cube formed by cutting a cuboid of dimensions 2 x 1 x 1 in 2 equal parts, is 2 sq units.
Solution. False
The dimensions of the given cuboid are 2 x 1 x 1. It is sliced into two equal parts, which are cubes.
Then, the dimensions of the cube, so formed are 1 x 1 x 1.
...The surface area of the cube so formed = 6 (Side)2 = 6 x (1)2 = 6sq units
Hence, the surface area of the sliced cube is 6 sq units.

Question.61 Ratio of area of a circle to the area of a square whose side equals radius of circle, is 1 : TC.
Solution. False
Given, side of a square equals radius of a circle.
Then, area of the square = r2
and area of the circle =πr2
where r is a radius of the circle.
Now, the ratio of area of the circle to area of the square = πr2: r2  = π : 1.

Question. 62 The area of a rectangular field is 48 m2 and one of its sides is 6m. How long will a lady take to cross the field diagonally at the rate of 20 m/min?
Solution.
Image may be NSFW.
Clik here to view.
ncert-exemplar-problems-class-8-mathematics-mensuration-42

Question. 63 The circumference of the front wheel of a cart is 3 m long and that of the back wheel is 4 m long. What is the distance travelled by the cart, when the front wheel makes five more revolutions than the rear wheel?
Solution.
Given, circumference of front wheel = 3 m
Now, distance covered by front wheel of the cart in 1 revolution
= Circumference of front wheel .
... Distance covered by front wheel in 5 revolutions = 3 x 5 = 15 m
Hence, the distance covered by the cart is 15 m.

Question. 64 Four horses are tethered with equal ropes at 4 corners of a square field of side 70 m, so that they just can reach one-another. Find the area left ungrazed by the horses.
solution.
Given, side of a square = 70 m
Also, four horses are tethered with equal ropes at 4 corners of the square field. Hence, each horse can graze upto 35 m of distance along the side.
Image may be NSFW.
Clik here to view.
ncert-exemplar-problems-class-8-mathematics-mensuration-43

Question. 65 The walls and ceiling of a room are to be plastered. The length, breadth and height of the room are 4.5 m, 3m and 350 cm, respectively. Find the cost of plastering at the rate of ? 8 per m2.
solution.
Image may be NSFW.
Clik here to view.
ncert-exemplar-problems-class-8-mathematics-mensuration-44

Question. 66 Most of the sailboats have two sails, the jib and the mainsail. Assume that the sails are triangles. Find the total area of each sail of the sailboats to the nearest tenth.
Image may be NSFW.
Clik here to view.
ncert-exemplar-problems-class-8-mathematics-mensuration-45

solution.
Image may be NSFW.
Clik here to view.
ncert-exemplar-problems-class-8-mathematics-mensuration-46

Image may be NSFW.
Clik here to view.
ncert-exemplar-problems-class-8-mathematics-mensuration-47

Image may be NSFW.
Clik here to view.
ncert-exemplar-problems-class-8-mathematics-mensuration-48

Question. 67 The area of a trapezium with equal non-parallel sides is 168 m2. If the lengths of the parallel sides are 36 m and 20 m, then find the length of the non-parallel sides.
solution.
Image may be NSFW.
Clik here to view.
ncert-exemplar-problems-class-8-mathematics-mensuration-49

Question. 68 Mukesh walks around a circular track of radius 14 m with a speed of 4 km/h. If he takes 20 rounds of the track, for how lontj does he walk?
solution.
Image may be NSFW.
Clik here to view.
ncert-exemplar-problems-class-8-mathematics-mensuration-50

Question. 69 The areas of two circles are in the ratio 49 : 64. Find the ratio of their circumferences.
solution.
Image may be NSFW.
Clik here to view.
ncert-exemplar-problems-class-8-mathematics-mensuration-51

Question. 70 There is a circular pond and a footpath runs along its boundary. A person walks around it, exactly once keeping close to the edge. If his step is 66 cm long and he takes exactly 400 steps to go around the pond, then find the diameter of the pond.
solution.
Image may be NSFW.
Clik here to view.
ncert-exemplar-problems-class-8-mathematics-mensuration-52

Image may be NSFW.
Clik here to view.
ncert-exemplar-problems-class-8-mathematics-mensuration-53

Question. 71 A running track has 2 semi-circular ends of radius 63 m and two straight lengths. The perimeter of the track is 1000 m. Find each straight length.
solution.
Image may be NSFW.
Clik here to view.
ncert-exemplar-problems-class-8-mathematics-mensuration-142

Question. 72 Find the perimeter of the given figure.
Image may be NSFW.
Clik here to view.
ncert-exemplar-problems-class-8-mathematics-mensuration-54

solution.
Image may be NSFW.
Clik here to view.
ncert-exemplar-problems-class-8-mathematics-mensuration-55

Question. 73 A bicycle wheel makes 500 revolutions in moving 1 km. Find the diameter of the wheel.
solution.
Image may be NSFW.
Clik here to view.
ncert-exemplar-problems-class-8-mathematics-mensuration-56

Image may be NSFW.
Clik here to view.
ncert-exemplar-problems-class-8-mathematics-mensuration-57

Question. 74 A boy is cycling such that the wheels of the cycle are making 140 revolutions per hour. If the diameter of the wheel is 60 cm, then calculate the speed (in km/h) with which the boy is cycling.
solution.
Image may be NSFW.
Clik here to view.
ncert-exemplar-problems-class-8-mathematics-mensuration-58

Question. 75 Find the length of the largest pole, that can be placed in a room of dimensions 12 mx 4 m x 43 m.
solution.
Image may be NSFW.
Clik here to view.
ncert-exemplar-problems-class-8-mathematics-mensuration-59

Image may be NSFW.
Clik here to view.
ncert-exemplar-problems-class-8-mathematics-mensuration-60

Question. 76 Find the area of the following fields. All dimensions are in metres.
Image may be NSFW.
Clik here to view.
ncert-exemplar-problems-class-8-mathematics-mensuration-61

solution.
Image may be NSFW.
Clik here to view.
ncert-exemplar-problems-class-8-mathematics-mensuration-62

Image may be NSFW.
Clik here to view.
ncert-exemplar-problems-class-8-mathematics-mensuration-63

Question. 77 Find the area of the following fields. All dimensions are in metres.
Image may be NSFW.
Clik here to view.
ncert-exemplar-problems-class-8-mathematics-mensuration-64

solution.
Image may be NSFW.
Clik here to view.
ncert-exemplar-problems-class-8-mathematics-mensuration-65

Image may be NSFW.
Clik here to view.
ncert-exemplar-problems-class-8-mathematics-mensuration-66

In questions from 78 to 85, find the area of the shaded portion in the following figures.

Question. 78
Image may be NSFW.
Clik here to view.
ncert-exemplar-problems-class-8-mathematics-mensuration-67

solution.
Image may be NSFW.
Clik here to view.
ncert-exemplar-problems-class-8-mathematics-mensuration-68

Question. 79
Image may be NSFW.
Clik here to view.
ncert-exemplar-problems-class-8-mathematics-mensuration-69

solution.
Image may be NSFW.
Clik here to view.
ncert-exemplar-problems-class-8-mathematics-mensuration-70

Question.80
Image may be NSFW.
Clik here to view.
ncert-exemplar-problems-class-8-mathematics-mensuration-71

solution.
Image may be NSFW.
Clik here to view.
ncert-exemplar-problems-class-8-mathematics-mensuration-72

Question.81
Image may be NSFW.
Clik here to view.
ncert-exemplar-problems-class-8-mathematics-mensuration-73

solution.
Image may be NSFW.
Clik here to view.
ncert-exemplar-problems-class-8-mathematics-mensuration-74

Question.82
Image may be NSFW.
Clik here to view.
ncert-exemplar-problems-class-8-mathematics-mensuration-76

solution.
Image may be NSFW.
Clik here to view.
ncert-exemplar-problems-class-8-mathematics-mensuration-77

Image may be NSFW.
Clik here to view.
ncert-exemplar-problems-class-8-mathematics-mensuration-78

Question.83
Image may be NSFW.
Clik here to view.
ncert-exemplar-problems-class-8-mathematics-mensuration-79

solution.
Image may be NSFW.
Clik here to view.
ncert-exemplar-problems-class-8-mathematics-mensuration-80

Question.84
Image may be NSFW.
Clik here to view.
ncert-exemplar-problems-class-8-mathematics-mensuration-81

solution.
Image may be NSFW.
Clik here to view.
ncert-exemplar-problems-class-8-mathematics-mensuration-82

Question.85
Image may be NSFW.
Clik here to view.
ncert-exemplar-problems-class-8-mathematics-mensuration-83

solution.
Image may be NSFW.
Clik here to view.
ncert-exemplar-problems-class-8-mathematics-mensuration-84

Question.86 Find the volume of each of the given figure, if Volume = Base area x Height
Image may be NSFW.
Clik here to view.
ncert-exemplar-problems-class-8-mathematics-mensuration-85

solution.
Image may be NSFW.
Clik here to view.
ncert-exemplar-problems-class-8-mathematics-mensuration-86

Image may be NSFW.
Clik here to view.
ncert-exemplar-problems-class-8-mathematics-mensuration-87

Question.87 A cube of side 5 cm is cut into as many 1 cm cubes as possible. What is the ratio of the surface areas of the original cube to that of the sum of the surface areas of the smaller cubes?
solution.
Surface area of a cube = 6 a2, where a is side of a cube.
 ... Side of cube = 5cm
 ... Surface area of the cube = 6 x (5)2 = 6 x 25
= 150cm2
Now, surface area of the cube with side 1 cm = 6 x (1)2= 6 cm2
 ... Surface area of 5 cubes with side 1 cm = 5 x 6 = 30 cm2
Ratio of the surface area of the original cube to that of the sum of the surface area of the smaller cubes
=30/150=3/15=1:5

Question. 88 A square sheet of paper is converted into a cylinder by rolling it along its side. What is the ratio of the base radius to the side of the square?
solution.
Image may be NSFW.
Clik here to view.
ncert-exemplar-problems-class-8-mathematics-mensuration-88

Question. 89 How many cubic metres of Earth must be dug to construct a well 7 m deep and of diameter 2.8 m?
solution.
Image may be NSFW.
Clik here to view.
ncert-exemplar-problems-class-8-mathematics-mensuration-89

Question. 90 The radius and height of a cylinder are in the ratio 3 : 2 and its volume is 19404 cm3. Find its radius and height.
solution.
Image may be NSFW.
Clik here to view.
ncert-exemplar-problems-class-8-mathematics-mensuration-90

Question. 91 The thickness of a hollow metallic cylinder is 2 cm. It is 70 cm long with outer radius of 14 cm. Find the volume of the metal used in making the cylinder, assuming that it is open at both the ends. Also, find its weight if the metal weighs 8 g per cm3.
solution.
Image may be NSFW.
Clik here to view.
ncert-exemplar-problems-class-8-mathematics-mensuration-91

Image may be NSFW.
Clik here to view.
ncert-exemplar-problems-class-8-mathematics-mensuration-92

Question. 92 Radius of a cylinder is r and the height is h. Find the change in the volume if the
(i) height is doubled.
(ii)height is doubled and the radius is halved.
(iii)height remains
solution.
Image may be NSFW.
Clik here to view.
ncert-exemplar-problems-class-8-mathematics-mensuration-93

Question. 93 If the length of each edge of a cube is tripled, what will be the change in its volume?
solution.
Let the edge of a cube be a.
If edge of the cube became tripled i.e. a = 3 x a = 3a
... Volume of the cube = a3
 ... Volume of the cube with edge tripled = (3a)3 = 27 a3
Hence, volume is 27 times of the original volume.

Question. 94 A carpenter makes a box which has a volume of 13400 cm3 . The base has an area of 670 cm2 . What is the height of the box?
solution.
Image may be NSFW.
Clik here to view.
ncert-exemplar-problems-class-8-mathematics-mensuration-94

Question. 95 A cuboidal tin’ box opened at the top has dimensions 20 cm x 16 cm x 14 cm. What is the total area of metal sheet required to make 10 such boxes?
solution.
Dimensions of cuboidal tin box are20 cm x 16cm x 14 cm,
...Area of metal sheet for 1 box = Surface area of cuboid
= 2(lb + bh+hl)
= 2(20 x 16+16 x 14+ 14 x 20)
= 2(320 + 224 + 280)
= 2(824)
= 1648 c m2
... Area of metal sheet required to make 10 such boxes = 10 x 1648= 16460c m2

Question. 96 Find the capacity of water tank, in litres, whose dimensions are 4.2 m, 3 m and 1.8 m?
solution.
Image may be NSFW.
Clik here to view.
ncert-exemplar-problems-class-8-mathematics-mensuration-95

Question. 97 How many cubes each of side 0.5 cm are required to build a cube of volume 8 cm3?
solution.
Image may be NSFW.
Clik here to view.
ncert-exemplar-problems-class-8-mathematics-mensuration-96

Question. 98 A wooden box (including the lid) has external dimensions
40 cm x 34 cm x 30 cm. If the wood is 1 cm thick, how many cm3 of wood is used in it?
solution.
Image may be NSFW.
Clik here to view.
ncert-exemplar-problems-class-8-mathematics-mensuration-97

Question.99 A river 2 m deep and 45 m wide is flowing at the rate of 3 km per hour. Find the amount of water in cubic metres that runs into the sea per minute.
solution.
Image may be NSFW.
Clik here to view.
ncert-exemplar-problems-class-8-mathematics-mensuration-98

Question.100 Find the area to be painted in the following block with cylindrical hole. Given that, length is 15cm, width 12cm, height 20cfn and radius of the hole 2.8 Cm.
Image may be NSFW.
Clik here to view.
ncert-exemplar-problems-class-8-mathematics-mensuration-99

solution.
Image may be NSFW.
Clik here to view.
ncert-exemplar-problems-class-8-mathematics-mensuration-100

Question. 101 A truck carrying 7.8 m3 concrete arrives at a job site. A platform of width 5 m and height 2 m is being constructed at the site. Find the length of the platform, constructed from the amount of concrete on the truck?
solution.
Image may be NSFW.
Clik here to view.
ncert-exemplar-problems-class-8-mathematics-mensuration-101

Question. 102 A hollow garden roller of 42 cm diameter and length 152 cm is made of cast iron 2 cm thick. Find the volume of iron used in the roller.
solution.
Image may be NSFW.
Clik here to view.
ncert-exemplar-problems-class-8-mathematics-mensuration-102

Image may be NSFW.
Clik here to view.
ncert-exemplar-problems-class-8-mathematics-mensuration-103

Question. 103 Three cubes each of side 10 cm are joined end to end. Find the surface area of the resultant figure.
solution.
Image may be NSFW.
Clik here to view.
ncert-exemplar-problems-class-8-mathematics-mensuration-104

Question. 104 Below are the drawings of cross sections of two different pipes used to fill swimming pools. Figure A is a combination of 2 pipes each having a radius of 8 cm. Figure B is a pipe having a radius of 15 cm. If the force of the flow of water coming out of the pipes is the same in both the cases, which will fill the swimming pool faster?
Image may be NSFW.
Clik here to view.
ncert-exemplar-problems-class-8-mathematics-mensuration-105

solution.
Image may be NSFW.
Clik here to view.
ncert-exemplar-problems-class-8-mathematics-mensuration-106

Question. 105 A swimming pool is 200 m x 50 m and has an average depth of 2 m. By the end of a summer day, the water level drops by 2 cm. How many cubic metres of water is lost on the day?
solution.
Image may be NSFW.
Clik here to view.
ncert-exemplar-problems-class-8-mathematics-mensuration-107

Question. 106 A housing society consisting of 5500 peoples needs 100 L of water per person per day. The cylindrical supply tank is 7 m high and has a diameter 10 m. For how many days will the water in the tank last for
the society?
solution.
Image may be NSFW.
Clik here to view.
ncert-exemplar-problems-class-8-mathematics-mensuration-108

Question. 107 Metallic discs of radius 0.75 cm and thickness 0.2 cm are melted to
obtain 508.68 cm3 of metal. Find the number of disc melted (Use π = 3.14)
Solution.
Image may be NSFW.
Clik here to view.
ncert-exemplar-problems-class-8-mathematics-mensuration-109

Image may be NSFW.
Clik here to view.
ncert-exemplar-problems-class-8-mathematics-mensuration-110

Question. 108 The ratio of the radius and height of a cylinder is 2:3. If its volume is 12936 cm3 find the the total surface area of the cylinder.
solution.
Image may be NSFW.
Clik here to view.
ncert-exemplar-problems-class-8-mathematics-mensuration-111

Question. 109 External dimensions of a closed wooden box are in the ratio 5:4:3. If the cost of painting its outer surface at the rate of Rs 5 per dm2is Rs 11750, find the dimensions of the box.
solution.
Image may be NSFW.
Clik here to view.
ncert-exemplar-problems-class-8-mathematics-mensuration-112

Image may be NSFW.
Clik here to view.
ncert-exemplar-problems-class-8-mathematics-mensuration-113

Question. 110 The capacity of a closed cylindrical vessel of height 1 m is 15.4 L. How many square metres of metal sheet would be needed to make it?
solution.
Image may be NSFW.
Clik here to view.
ncert-exemplar-problems-class-8-mathematics-mensuration-114

Question. 111. What will happen to the volume of the cube, if its edge
(a) tripled (b) reduced to one-fourth?
solution.
Image may be NSFW.
Clik here to view.
ncert-exemplar-problems-class-8-mathematics-mensuration-115

Question. 112 A rectangular sheet of dimensions 25 cm x 7 cm is rotated about its . longer side. Find the volume and the whole surface area of the solid thus generated.
solution.
Image may be NSFW.
Clik here to view.
ncert-exemplar-problems-class-8-mathematics-mensuration-116

Image may be NSFW.
Clik here to view.
ncert-exemplar-problems-class-8-mathematics-mensuration-117

Question. 113 From a pipe of inner radius 0.75 cm, water flows at the rate of 7 m per second. Find the volume in litres of water delivered by the pipe in 1 h.
solution.
Image may be NSFW.
Clik here to view.
ncert-exemplar-problems-class-8-mathematics-mensuration-118

Question. 114 Four times the area of the curved surface of a cylinder is equal to 6 times the sum of the areas of its bases. If its height is 12 cm, find its curved surface area.
solution.
Let the radius and height of the cylinder be r and h, respectively.
Curved surface area of cylinder = 2 πrh
Area of base = πr2
Sum of areas of bases = 2 πr2
Image may be NSFW.
Clik here to view.
ncert-exemplar-problems-class-8-mathematics-mensuration-119

Question. 115 A cylindrical tank lias a radius of 154 cm. It is filled with water to a height of 3 m. If water to a height of 4.5 m is poured into it, what will be the increase in the volume of water in kL?
solution.
Image may be NSFW.
Clik here to view.
ncert-exemplar-problems-class-8-mathematics-mensuration-120

Question. 116 The length, breadth and height of a cuboidal reservoir is 7 m, 6 m and 15 m respectively. 8400 L of water is pumped out from the reservoir. Find the fall in the water level in the reservoir.
solution.
Image may be NSFW.
Clik here to view.
ncert-exemplar-problems-class-8-mathematics-mensuration-121

Question. 117 How many bricks of size 22 cm x 10 cm x 7 cm are required to
construct a wall 11 m long, 3.5 m high and 40 cm thick, if the cement and sand used in the construction occupy (1/10)th part of
the wall?
solution.
Image may be NSFW.
Clik here to view.
ncert-exemplar-problems-class-8-mathematics-mensuration-122

Question. 118 A rectangular examination hall having seats for 500 candidates has to be built so, as to allow 4 cubic metres of air and 0.5 square metres of floor area per candidate. If the length of hall be 25 m, find the height and breadth of the hall.
solution.
Image may be NSFW.
Clik here to view.
ncert-exemplar-problems-class-8-mathematics-mensuration-123

Question. 119 The ratio between the curved surface area and the total surface area of ‘ a right circular cylinder is 1: 2. Find the ratio between the height and
radius of the cylinder.
solution.
Image may be NSFW.
Clik here to view.
ncert-exemplar-problems-class-8-mathematics-mensuration-124

Question. 120 A birthday cake has two tiers as shown in the figure below. Find the volume of the cake.
Image may be NSFW.
Clik here to view.
ncert-exemplar-problems-class-8-mathematics-mensuration-125

solution.
Image may be NSFW.
Clik here to view.
ncert-exemplar-problems-class-8-mathematics-mensuration-126

Question. 121
Image may be NSFW.
Clik here to view.
ncert-exemplar-problems-class-8-mathematics-mensuration-127

Solution.
Image may be NSFW.
Clik here to view.
ncert-exemplar-problems-class-8-mathematics-mensuration-128

Question. 122
Image may be NSFW.
Clik here to view.
ncert-exemplar-problems-class-8-mathematics-mensuration-129

Solution.
Image may be NSFW.
Clik here to view.
ncert-exemplar-problems-class-8-mathematics-mensuration-130

Question. 123
Image may be NSFW.
Clik here to view.
ncert-exemplar-problems-class-8-mathematics-mensuration-131

Solution.
Image may be NSFW.
Clik here to view.
ncert-exemplar-problems-class-8-mathematics-mensuration-132

Image may be NSFW.
Clik here to view.
ncert-exemplar-problems-class-8-mathematics-mensuration-133

Question. 124
Image may be NSFW.
Clik here to view.
ncert-exemplar-problems-class-8-mathematics-mensuration-134

Solution.
Image may be NSFW.
Clik here to view.
ncert-exemplar-problems-class-8-mathematics-mensuration-135

Question. 125 Water flows from a tank with a rectangular base measuring 80 cm x 70 cm into another tank with a square base of side 60 cm. If the water in the first tank is 45 cm deep, how deep will it be in the second tank?
solution.
Image may be NSFW.
Clik here to view.
ncert-exemplar-problems-class-8-mathematics-mensuration-136

Question. 126 A rectangular sheet of paper is rolled in two different ways to form two different cylinders. Find the volume of cylinders in each case if the sheet measures 44 cm x 33 cm.
solution.
Image may be NSFW.
Clik here to view.
ncert-exemplar-problems-class-8-mathematics-mensuration-137

Image may be NSFW.
Clik here to view.
ncert-exemplar-problems-class-8-mathematics-mensuration-138

 

The post NCERT Exemplar Problems Class 8 Mathematics Mensuration appeared first on Learn CBSE.

CBSE Sample Papers for Class 12 Economics Delhi – 2009

CBSE Sample Papers for Class 12 Economics Delhi – 2009

Time allowed : 3  hours                                                                                         Maximum marks 100

GENERAL INSTRUCTIONS
(i) All questions in both the sections are compulsory.
(ii) Marks for questions are indicated against each.
(iii) Questions No. 1-5 and 17-21 are very short-answer questions carrying 1 nick each. They are required to be answered in one sentence each.
(iv) Questions No. 6-10 and 22-26 are? short-answer questions carrying 3 marks each. Answers to them should normally not exceed 60 words each.
(v) Questions No. 11-13 and 27-29 are also short-answer questions carrying 4 marks each. Answers to them should normally not exceed 70 words each.
(vi) Questions No. 14-16 and 30-32 are long-answers questions carrying 6 marks each. Answers to them should normally not exceed 100 words each.
(vii) Answers should be brief and to the point and the above word limit should be adhered to as far as possible.

SET I

SECTION A

Question.1. Give two examples of Microeconomic studies.
Answer. The two examples of microeconomic studies are (i) study of relation between price and demand of goods and (ii) price determination.

Question.2. When is the demand of a commodity said to be inelastic?
Answer. Demand of a commodity is said to be inelastic when percentage change in quantity demanded is less than the percentage change in price i.e., e < 1.

Question.3. Define fixed cost.
Answer. Fixed costs are those which do not change with the change in the level of output.
TFC – TC – TVC.

Question.4. What causes a downward movement along a supply curve?
Answer. A downward movement along a supply curve, indicating contraction of supply, is caused due to fall in the price of a commodity other factors affecting the, supply remaining same/unchanged. See diagram.
Image may be NSFW.
Clik here to view.
cbse-sample-papers-class-12-economics-delhi-2009-1

Question.5. Define monopoly.
Answer. Monopoly refers to a market situation where there is a single seller selling a product which has no close substitutes

Question.6. Why does an economic problem arise? Explain.
Or
Explain the problem of ‘What to produce’.
Answer. Scarcity of resources having alternative uses in relation to unlimited wants gives rise to an economic problem.
There are three causes for an economic problem:
(i) Unlimited wants. Human wants are unlimited and differ in intensity.
(ii) Limited resources. Resources, generally Called factors of production, are scarce in relation to their need.
(iii) Alternative uses of resources. A resource can be used for many purposes. But it can be used for only one purpose at a time.
What to produce? This problem signifies what goods should be produced and in what quantities. This problem arises when due to scarcity of resources we cannot produce each and every thing which we want. Therefore, a decision has to be taken as to what goods should be produced and in what quantities. In short, we have to see whether we should produce consumer goods or producer goods or defence goods or all the goods in some quantity combination.
So, on the basis of importance of various goods, an economy has to decide which goods should be produced and in what quantities.

Question.7. Distinguish between a normal good and an inferior good. Give example in each case. 3
Answer. Normal Good
1. A normal good is one whose demand increases with an increase in the money income of the consumer.
2. Normal goods have positive income effect, e.g. if a consumer buys more of milk for his family as his income rises, then milk will be called a normal good as shown in Diagram 1.
Image may be NSFW.
Clik here to view.
cbse-sample-papers-class-12-economics-delhi-2009-2

Inferior Good
1. An inferior good is one whose demand falls with a rise in income of the consumer because he can now afford to buy a normal (superior) good as shown in Diagram 2.
2. Inferior goods have negative income effect; e.g. if a consumer reduces the consumption of toned milk when his income rises, then toned milk is an inferior good for that consumer, as shown in Diagram2.

Question.8. How is the price elasticity of demand of a commodity affected by the number of its substitutes? Explain.
Answer. When a commodity has a large number of close substitutes, its demand is usually very elastic. This is because a consumer can easily shift from one substitute to another in case of a change in price. If there are no substitutes of a good, the consumer has no option but to buy the given good, e.g. demand for cold drinks like Coca-Cola, Pepsi, etc. is highly elastic because of availability of close substitutes.

Question.9. Explain the meaning of ‘increase in supply’ and ‘increase in quantity supplied’ with the help of a schedule.
Answer. Increase in supply occurs when quantity supplied increases at the existing price due to factors other than the price of the commodity. It can be explained with the help of the following .schedule and Diagram 3.
Image may be NSFW.
Clik here to view.
cbse-sample-papers-class-12-economics-delhi-2009-3

Image may be NSFW.
Clik here to view.
cbse-sample-papers-class-12-economics-delhi-2009-4

As per the above schedule, initially the quantity supplied is 10 units at a price of Rs. 5 per unit. Due to other factors, like technological improvement or reduction in input prices, quantity supplied increases to 20 units at the same price. This is known as increase in supply. Increase in quantity supplied occurs when quantity supplied increases due to increase in the commodity’s own price. This can be explained with the help of the following supply schedule and Diagram 4.
Image may be NSFW.
Clik here to view.
cbse-sample-papers-class-12-economics-delhi-2009-5

Image may be NSFW.
Clik here to view.
cbse-sample-papers-class-12-economics-delhi-2009-6

According to the above schedule, 20 units of the commodity are supplied at a price of Rs. 5 per unit But the quantity supplied increases to 30 units when price of the commodity increases to ? 10 per unit. This extension of supply occurs when price increases from Rs. 5 to Rs. 10 per unit as shown in Diagram 4.

Question.10. Why is. a firm under Perfect Competition a price-taker? Explain.
Answer. Perfect Competition is a market situation in which a large number of firms produce homogeneous goods and sell them at a uniform price.
The share of each seller in the total market supply is so small that no single seller can influence the price. Hence it has no option but to sell the product at the price determined by the industry (determined by interaction of market demand and supply). It is because of this that each firm is said to be a price taker in perfect competition.

Question.11. Complete the following table:
Image may be NSFW.
Clik here to view.
cbse-sample-papers-class-12-economics-delhi-2009-7

Answer.
Image may be NSFW.
Clik here to view.
cbse-sample-papers-class-12-economics-delhi-2009-8

(or)
Complete the following table:
Image may be NSFW.
Clik here to view.
cbse-sample-papers-class-12-economics-delhi-2009-9

Answer.
Image may be NSFW.
Clik here to view.
cbse-sample-papers-class-12-economics-delhi-2009-10

Question.12. Commodities X and Y have eqval price elasticity of supply. The supply of X rises from 400 units to 500 units due to a 20 peir cent rise in its price. Calculate the percentage fall supply of Y if its price falls by 8 percent.
Answer.
Image may be NSFW.
Clik here to view.
cbse-sample-papers-class-12-economics-delhi-2009-11

Question.13. From the following schedule find out the level of output at which the producer is equilibrium. Give reasons for your answer.
Image may be NSFW.
Clik here to view.
cbse-sample-papers-class-12-economics-delhi-2009-12

Answer.
Image may be NSFW.
Clik here to view.
cbse-sample-papers-class-12-economics-delhi-2009-13

The producer is in equilibrium at 6th unit of output. This is because at this level of output his profit is maximum. Also at this level his
MR = MC – 24.

Question.14. Explain the causes of a rightward shift in demand curve of a commodity of an individual consumer .
Answer. The main reasons of rightward shift in demand curve of a commodity are:
1. Increase in income of buyers of normal good. With increase in income of the consumer, the demand curve for normal good shifts to the right.
2. Fall in iricome in case of inferior good. Fall in income of a consumer may shift the demand curve of an inferior good to the right.
3. Rise in price of substitute goods. In case of substitute goods, demand tor a commodity rises with rise in price of its substitute commodity.
4. Fall in price of a complementary good. In case of complementary goods, demand for the commodity rises with a fall in the price of the complementary commodity.
5. Favourable taste and preference for a commodity. If consumer’s tastes and preferences change in favour of a commodity, the demand for the commodity rises shifting the demand curve towards right.
(or)
Explain the conditions of consumer’s equilibrium in case of (i) single commodity and (ii) two commodities. Use utility approach.
AM. A consumer is in a state of? equilibrium when he spends his given income on tire purchase of a commodity (or combination of goods) in such a way that gives him maximum satisfaction.
(i) Consumer’s equilibrium in case of a single commodity is attained when MU= Price
This implies that a consumer makes purchases only upto the point where utility of the last unit is equal to the price of that unit. This is the stage when he maximises his gains. If he moves beyond this stage and purchases more units of the commodity/ then MU will fall (Law of diminishing marginal utility) and will become less than the price paid for it. Here marginal utility means marginal utility of ?
(ii) In case of two commodities, consumer attains equilibrium when:
Image may be NSFW.
Clik here to view.
cbse-sample-papers-class-12-economics-delhi-2009-14

This implies that a consumer attains equilibrium (maximum satisfaction) when the marginal utility obtained from the last rupee spent on both the goods becomes equal. This is also known as the Law of Equi-Marginal Utility.
Suppose the MU obtained from the last rupee spent on X is greater than the MU of the last rupee spent on Y. This induces the consumer to increase consumption of commodity X. Due to operation of Law of Diminishing Marginal Utility \({ MU }_{ x }\) will fall ,
and \({ MU }_{ x }\) will rise till finally \( \frac { { MU }_{ x } }{ { P }_{ x } }\) will become equal to \( \frac { { MU }_{ y } }{ { P }_{ y } }\) . When they are equal, the consumer gets the same MU per rupee and has no reason to further re-allocate his income on the two goods.

Question.15. Giving reasons, state whether the following statements are true or false:
(i) When there are diminishing returns to a factor, total product always decreases.
(ii) Total product will increase only when marginal product increases.
(iii) When marginal revenue is zero, average revenue will be constant.
Answer. (i) False. Because diminishing returns to a factor implies a situation where marginal product is falling. A fall in MP implies that TP should be increasing though at a diminishing rate. Total Product will increase till Marginal Product remains positive.
(ii) False. Total Product will also increase when Marginal Product decreases though at a diminishing rate.
(iii) False. Marginal Revenue (MR) will be zero only when Total Revenue (TR) is constant- This implies that Average Revenue (AR) will fall as output increases.

Question.16. With the help of a diagram, explain the effect of “decrease”’ in demand of a commodity on its equilibrium price and quantity.
Answer. Equilibrium price of a good is the price at which the market supply of the good equals the market deihand for that good.
The effect of decrease in demand for a commodity on its equilibrium price and quantity is explained with the help of the following Diagram:
Image may be NSFW.
Clik here to view.
cbse-sample-papers-class-12-economics-delhi-2009-15

Decrease in demand shifts the demand curve from DD to \({ D }_{ 1 }\) \({ D }_{ 1 }\) towards left. E is the original equilibrium point where OP is the equilibrium price and OQ is the equilibrium quantity. After the shift in the demand curve, the new demand curve (\({ D }_{ 1 }\)\({ D }_{ 1 }\)) intersects tire original supply curve (SS) at -point Er which is the new equilibrium. At \({ E }_{ 1 }\), the -equilibriumprice is OPa and equilibrium quantity is \({ OQ }_{ 1 }\) Thus, both equilibrium price and equilibrium quantity will fall when there is a decrease in demand.

For Blind Candidates only in lieu of Q. No. 16

Define equilibrium price. How is it affected by a “decrease” in demand of a commodity?
Answer. Same as for Q. 16.

Section B
Question.17. Why is repayment of loan a capital expenditure?
Answer. Loan is a liability. Its repayment is a capital expenditure because it reduces the liability of the : government. .

Question.18. What is meant by excess demand in Macroeconomics?
Answer. Excess Demand refers to a situation when Aggregate Demand (AD) is more than Aggregate Supply (AS) at the full employment level in an economy.

Question.19. What can be the minimum value of investment multiplier?
Answer. The minimum value of investment multiplier can be one.

Question.20. Define bank rate.
Answer. It is the rate at which the Central Bank gives credit to commercial banks.

Question.21. Define involuntary unemployment.
Answer. It is a situation in which people are able and willing to work at the existing wage rate but are unable to find work.

Question.22. Complete the following table:
Image may be NSFW.
Clik here to view.
cbse-sample-papers-class-12-economics-delhi-2009-16

Answer.
Image may be NSFW.
Clik here to view.
cbse-sample-papers-class-12-economics-delhi-2009-17

Question.23. State any three points of distinction between Central Bank and Commercial Banks.
Answer.
Image may be NSFW.
Clik here to view.
cbse-sample-papers-class-12-economics-delhi-2009-18

Question.24. How can a government budget help in reducing inequalities of income? Explain.
Answer. Government can achieve this aim bj^fakifig two fiscal policy steps. It can raise and impose taxes on income of rich and on the goods consumed by them thereby reducing their disposable income. The government can also increase subsidies and free services to the poor like education, medical treatment etc.
In this way, income inequalities can be, reduced.

Question.25. Explain the circular flow of income.
Answer. The flow of production, income generation and expenditure involving different sectors of the economy in the form of wages, rent and dividends, is known as circular flow of income (see diagram).
Image may be NSFW.
Clik here to view.
cbse-sample-papers-class-12-economics-delhi-2009-19

Production gives rise to income (factor incomes), which in turn gives rise to demand for goods and services. This demand leads to expenditure by households on goods and services produced. In this way income generated by production units reaches bade to production units and makes the circular flow complete.
Or
Distinguish between intermediate products and final products. Give examples.
Answer.
Image may be NSFW.
Clik here to view.
cbse-sample-papers-class-12-economics-delhi-2009-20

Question.26. List the items of the current account of balance of payments account. Also define ‘balance of trade’.
Answer. The items which are recorded in the current account are:
(i) Export and import of goods (visible items).
(ii) Export and import of sendees (invisible items).
(iii) Unilateral receipts and payments.
Balance of Trade refers to the difference in the value of imports and exports of visible items only.
BOT = Export of visible items – Import of visible items

Question.27. Explain the meaning and two merits of fixed foreign exchange rate.
Answer. Fixed Foreign Exchange Rate refers to the exchange or conversion rate as decided by die government. It has the following two merits:
(i) Elimination of Risk: There is a certain degree of risk involved for the exporter in the process of trade if there is a time lag between the delivery of goods and services and the receipt of payment. Fixed exchange rate eliminates this element of risk.
(ii) Mobility of Capital: Because of the uncertainty involved in the system of flexible exchange rates, investors often hesitate to invest in foreign countries. A system of fixed exchange rate facilitates movement of capital withiri countries eliminating this uncertainty.
Or
Explain two sources each of demand and supply of foreign exchange.
Answer. Two sources of demand of foreign exchange are:
• for importing goods and services. –
• for making investments abroad in financial and physical assets.
Two sources of supply of foreign exchange are:
• from transfer payments in the form of gifts, donations, cash remittances by family, etc.
• from exporting goods and services.

Question.28. State the four functions of money. Explain any one pf them.
Answer. Four functions of money are as follows:
(i) Money serves as a medium of exchange.
(ii) Money serves as a store of value.
(iii) Money is a standard of deferred payment.
(iv) Money serves as a unit of account.
Money as a store of value: People save a part of their earnings for use in future. Money fulfils this need of the people. People can store surplus purchasing power for future use. This means that one can hold orte’s earnings until the time one wants to spend it. This is facilitated by the fact that money comes in convenient denominations and is easily portable.

Question.29. Distinguish between:
(i) Direct tax and indirect tax
(ii) Revenue deficit and fiscal deficit.
Answer.(i)
Image may be NSFW.
Clik here to view.
cbse-sample-papers-class-12-economics-delhi-2009-21

(ii) Revenue deficit: It refers to the excess of the government’s total revenue expenditure over its total revenue receipts.
Revenue deficit = Total revenue expenditure – Total revenue receipts
Fiscal deficit: Fiscal deficit in a government budget refers to the excess of total
expenditure over the sum of revenue receipts and non-debt capital receipts.
Fiscal deficit = Total expenditure – (Revenue receipts + Capital receipts excluding borrowings)

Question.30. How will you treat the following while estimating domestic factor incotne of India? Give reasons for your answer.
(i) Remittances from non-resident Indians to their families in India.
(ii) Rent paid by the embassy of Japan in India to a resident Indian.
(iii) Profits earned by branches of foreign bank in India.
Answer. (i) These are transfer payments and are not to be included in the estimation of domestic factor income of India.
(ii) Since the Japan Embassy in India is not a part of the Indian domestic territory, therefore, it will not be included in the domestic factor income of India.
(iii) These will be included in the domestic income because the branches of the foreign bank are within the domestic territory of India.

Question.31. Given consumption function C = 100 + 0.75 Y (where C = Consumption expenditure and Y = National Income) and investment expenditure = Rs. 1,000, calculate:
(i) Equilibrium level of National Income.
(ii) Consumption expenditure at equilibrium level of National Income.
Answer. (i) Given: C = 100 + 0.75Y, I = Investment expenditure=Rs. 1,000.
Image may be NSFW.
Clik here to view.
cbse-sample-papers-class-12-economics-delhi-2009-22

Or
What changes will take place to bring an economy in equilibrium, if:
(i) planned savings are greater than planned investment, and
(ii) planned savings are less than planned investment.
Answer. (i) When planned savings are greater than planned investment: It is a situation when aggregate demand is less than aggregate supply, i.e., there is a situation of deficient : demand in the economy.
Since the buyers are buying less than what is expected, the stock of inventories starts increasing.
As a result, the producers cut back on production thereby reducing employment and income levels.
The downward trend continues till AD and AS once again become equal bringing the ; economy back to equilibrium.
(ii) Planned savings are less than planned investment: This is a situation of excess
demand, wherein aggregate demand (AD) is more than aggregate supply (AS).
In this situation the inventories start falling as the investment demand of business firms is more than the planned savings.
To bring the economy back to equilibrium level, producers will have to increase production raising the employment and income level.

Question.32. Calculate “gross national product at factor cost” from the following data by (i) income method, and (ii) expenditure method.
Image may be NSFW.
Clik here to view.
cbse-sample-papers-class-12-economics-delhi-2009-23

Answer.
Image may be NSFW.
Clik here to view.
cbse-sample-papers-class-12-economics-delhi-2009-24

SET II

Note : Except for the following questions, all the remaining questions have been asked in Set I.
SECTION A
Question.1. Give the meaning of opportunity cost
Answer. Opportunity cost is the value of the next best alternative foregone by choosing one alternative rather than the other.

Question.11. The price elasticity of supply of commodity X and Y are equal. The price of X falls from Rs. 10 to Rs. 8 per unit and its quantity supplied falls by 16 per cent The price of Y rises by 10 percent Calculate the percentage increase in its supply.
Answer.
Image may be NSFW.
Clik here to view.
cbse-sample-papers-class-12-economics-delhi-2009-25

Image may be NSFW.
Clik here to view.
cbse-sample-papers-class-12-economics-delhi-2009-26

Question.13. From the following table find out the -level of output at which die producer is in equilibrium. Give reasons for your answer.
Image may be NSFW.
Clik here to view.
cbse-sample-papers-class-12-economics-delhi-2009-27

Answer.
Image may be NSFW.
Clik here to view.
cbse-sample-papers-class-12-economics-delhi-2009-28

The producer will be in equilibrium at the 6* unit of output. This is because he earns the maximum profit (TR – TC); which is Rs. 2 at this unit of output.

Question.15. Giving reasons, state whether the following statements are true or false:
(i)Increase in total pro chi ct always indicates that there are increasing returns to a factor.
(ii) Marginal revenue is always the price at which the last unit of a commodity is sold,
(iii) When there are diminishing returns to a factor, marginal and total product both
always fall.
Answer. (i) False. Total Product (TP) can also increase when there are decreasing returns to a factor (though at a diminishing rate) so long as Marginal Product (MP) is positive.
(ii) False. Marginal Revenue (MR) can be the price of the last unit sold but only in case of perfect competition. However, this is not always the case. In fact MR can work out to zero or negative also while that is never the case with price.
(iii) False. In case of diminishing returns to a factor only Marginal Product (MP) tends to fall. Total Product (TP) can rise so long as Marginal Product (MP) is positive.

SECTION B
Question.18. Define Deflationary Gap.
Answer. The deficiency in the aggregate demand at full employment is deflationary gap.

Question.19. If investment multiplier is 1, what will be the value of marginal propensity to consume?
Answer. Given, investment multiplier, K=1
K=\(\frac { 1 }{ 1-MPC } \) K=1..(given)
1=\(\frac { 1 }{ 1-MPC } \) or 1-MPC=1 or MPC=1-1=0.
MPC will be equal to 0, when (K) investment multiplier is 1.

Question.22. Complete the following table:
Image may be NSFW.
Clik here to view.
cbse-sample-papers-class-12-economics-delhi-2009-29

Answer.
Image may be NSFW.
Clik here to view.
cbse-sample-papers-class-12-economics-delhi-2009-30

Question.28. Explain any two functions of money.
Answer. The two functions of money are:
(i) Medium of exchange: Money as a medium of exchange’means money as a means of payment for exchange of goods and services. This solves the problem of double coincidence of wants inherent in the barter system of trade. This function of money facilitates trade.
(ii) Unit of account: This is also called as the measure of value function. Money as a unit of account means it works as a standard unit for quoting prices. It makes money a medium of comparing prices of goods and services.

Question.30. How will you treat the following while estimating National Income of India? Give reasons for your answer.
(i) Salaries received If Indian residents working in Russian Embassy in India.
(ii) Profits earned by an Indian bank from its branches: abroad.
(in) Entertainment tax received by the government.
Answer. (i) It will be included in the-estimation of National Income because it is factor income from abroad.
(ii) This will also be included in the estimation of National Income because it is a part of factor income from abroad.
(iii) National Income is calculated at factor cost while entertainment tax is an indirect tax. Therefore, it will not be included in the estimation of National Income.

Question.31. Calculate “gross national product at factor cost” from the following data by (i) income method, and (ii) expenditure method:
Image may be NSFW.
Clik here to view.
cbse-sample-papers-class-12-economics-delhi-2009-31

Image may be NSFW.
Clik here to view.
cbse-sample-papers-class-12-economics-delhi-2009-32

Answer. (i) Income Method
GNPFC = Profits + Compensation of employees + Rent + Interest + Consumption of fixed capital + Net factor income from abroad
= (iii) + (iv ) +(v) + (viii) + (vii) + (x)
= 400 + 800 + 250 + 150 + 60 + (-10)
= 1660, – 10 = Rs. 1650 Crores
(ii) Expenditure Method
GNPFC = Private final consumption expenditure + Government final consumption expenditure + Net domestic capital formation + Consumption of fixed capital + Net exports + Net factor incorfie from abroad – Net indirect taxes = (i) + (vi) + (ii) + (vii) + (xi) + (x) – (xii)
= 1,000 + 500 + 200 + 60 + (-20)+ (-10)-80
= 1760 – 110 = Rs. 1650 Crores

SET III

Note: Except for the following questions, all the remaining questions have been asked in Set I and – Set II.
SECTION A

Question.3. Give the meaning of micro-economics.
Answer. Micro-economics is a study of individual economic units Such as firms, households, industries etc. to make economic decisions.

Question.15. Explain the causes of a leftward shift in demand curve of a commodity.
Answer. Main causes of a leftward shift in demand (decrease in demand) are:
(i)Income of the consumer: With decrease-in income of the consumer, the demand curve for a normal good shifts to the left.
(ii) Tastes and preferences: Unfavourable change in the taste and preference of the
commodity decreases its demand and leads to a leftward shift in the demand curve.
(iii) Fall in price of substitute goods: Demand for a commodity falls (and demand curve shifts towards left), when the price of its substitute commodity falls.
(iv) Rise in price of complementary goods: In case of complementary goods, demand for the commodity falls with a rise in the price of its complementary commodity.
Or
Explain consumer’s equilibrium in case of a single commodity with the help of a utility schedule.
Answer. Consumer’s equilibrium in case of a single commodity is attained when Marginal Utility in r money terms is equal to price, i.e., MU (money terms) = Price
\(\frac { MU of a product }{ MU of a rupee }\)= Price of the product
MU of a rupee is the extra utility when an additional rupee is spent on other available goods r in general.
It is assumed that a rational consumer will purchase a good only if the benefit derived in the form of utility is greater than or at least is equal to its price.
The above mentioned condition of consumer’s equilibrium can be explained with the help of an example.
Example: The following schedule shows the MU of a consumer derived from consumption
of successive orange. Suppose the price of an orange is Rs. 1 per piece and the MU of a rupee is 2 utils. Consumer equilibrium in case of a single good:
Image may be NSFW.
Clik here to view.
cbse-sample-papers-class-12-economics-delhi-2009-33

As per the above schedule, the consumer will attain equilibrium at the 4th unit because the purchase of the 4th unit of orange gives him utility worth Rs. 1(2/2) which is equal to the price of an orange (Rs. 1). He will not buy the 5th unit because MU in money terms (0.5) is less than its price (Rs. 1).

Question.16. How is equilibrium price of a commodity determined? Explain with the help of a demand and supply schedule.
Answer. Equilibrium price of a good is the price at which the market supply of the good equals the market demand for that good. By market supply, we mean the sum total of supply by all the producers of a good taken together. Whereas market demand refers to the sum total of demand by all the consumers of that good taken together.
This can be better explained with the help of the following schedule and Diagram.
Image may be NSFW.
Clik here to view.
cbse-sample-papers-class-12-economics-delhi-2009-34

Image may be NSFW.
Clik here to view.
cbse-sample-papers-class-12-economics-delhi-2009-cbse-sample-papers-class-12-economics-delhi-2009-35

In, the above table, equilibrium price is Rs. 3 per unit and equilibrium quantity is 300 units. At any other price, market demand and market supply are not equal.
The equilibrium attained from demand and supply in case of perfect competition is represented in the given Diagram. Equilibrium price is OP which is determined by the intersection of demand and supply curves at point E. At that point equilibrium quantity is OQ.

SECTION B
Question.17. Define a Government Budget.
Answer. Budget is a statement made by the Government showing the estimated receipts and expenditure during the period of the financial year.

Question.19. Give the meaning of autonomous consumption.
Answer. Consumption at zero level of income is called autonomous consumption. Autonomous consumption expenditure is not affected by change in income.

Question.22.Complete the following table:
Image may be NSFW.
Clik here to view.
cbse-sample-papers-class-12-economics-delhi-2009-36

Answer.
Image may be NSFW.
Clik here to view.
cbse-sample-papers-class-12-economics-delhi-2009-37

Question.30. How will you treat the following while estimating National Income of India? Give reasons for your answer.
(i) Salaries paid to Russians working in Indian Embassy in Russia.
(ii) Profits earned by an Indian company from its branch inSingapore.
(iii) Capital gains to Indian residents from sale of shares of a foreign company.
Answer. (i) It is not included while estimating National Income because it is a part of factor income paid abroad.
(ii) It is included while estimating National Income because it is a factor income from abroad.
(iii) It is not included in National Income of India because it is a financial transaction – corresponding to which there is no flow of goods and services in the economy.

Question.31. From the following data, calculate gross national product at factor cost by (i) income method and (ii) expenditure method.
Image may be NSFW.
Clik here to view.
cbse-sample-papers-class-12-economics-delhi-2009-38

Answer.(i)By Income Method
\(\frac { GNP }{ FC }\)= Compensation of employees + Rent + Interest + Profits + Net factor income from abroad + Depreciation (Consumption of fixed capital)
= (ii) + (vi) + (viii) + (x) + (xi) + (iii)
= 1,850 + 400 + 500 + 1,100 + (-50) + 100 = 3,950 – 50 = Rs. 3,900 Crores
(ii) By Expenditure Method
\(\frac { GNP }{ FC }\) = Government final consumption expenditure + Private final consumption expenditure + Net domestic capital formation + Consumption of fixed capital + Net exports + Net factor income from abroad – Net indirect taxes. = (iv) + (v) + (i) + (iii) + (ix) + (xi) – (xii)
= 1,100 + 2,600 + 500 + 100 + (-100) + (-50) – 250 = 4,300 – 400 = Rs. 3,900 Crores

The post CBSE Sample Papers for Class 12 Economics Delhi – 2009 appeared first on Learn CBSE.

CBSE Sample Papers for Class 12 Economics Delhi – 2016

CBSE Sample Papers for Class 12 Economics Delhi – 2016

Time allowed : 3  hours                                                                                         Maximum marks 100

GENERAL INSTRUCTIONS
(i) All questions in both the sections are compulsory.
(ii) Marks for questions are indicated against each.
(iii) Questions No. 1-5 and 17-21 are very short-answer questions carrying 1 nick each. They are required to be answered in one sentence each.
(iv) Questions No. 6-10 and 22-26 are? short-answer questions carrying 3 marks each. Answers to them should normally not exceed 60 words each.
(v) Questions No. 11-13 and 27-29 are also short-answer questions carrying 4 marks each. Answers to them should normally not exceed 70 words each.
(vi) Questions No. 14-16 and 30-32 are long-answers questions carrying 6 marks each. Answers to them should normally not exceed 100 words each.
(vii) Answers should be brief and to the point and the above word limit should be adhered to as far as possible.

SET I

SECTION A
Question. 1 What is the relation between marginal cost and average variable cost when marginal cost is rising and average variable cost is falling?
Answer. When marginal cost is rising and average variable cost is falling, marginal cost lies below the average variable, cost i.e., MC < AVC.

Question.2 Suppose total revenue is rising at a constant rate as more and more units of a. commodity are sold, marginal revenue would be: (choose the correct alternative)
(a) Greater than average revenue
(b) Equal to average revenue
(c) Less than average revenue
(d) Rising 1
Answer. (b) Equal to average revenue.

Question. 3 When does ‘increase’ in demand take place? 
Answer. When demand for a good increases with a change in factors other than the price of the good, increase in demand takes place. The other factors may be increase in price of substitute goods, fall in price of complementary good, increase in income of consumer etc.

Question. 4 ‘Homogeneous products’ is a characteristic of: (choose the correct alternative)
(a) Perfect competition only
(b) Perfect oligopoly only
(c) Both (a) and (b)
(d) None of the above
Answer. (c) Both (a) and (b).

Question. 5 There is inverse relation between price and demand for the product of a firm under: (choose the correct alternative) ,
(a) Monopoly only
(b) Monopolistic competition only
(c) Both under monopoly and monopolistic competition
(d) Perfect competition only
Answer. (c) Both under monopoly and monopolistic competition.

Question. 6 A consumer consumes only two goods X and Y. Marginal utilities of X and Y are 5’and 4 respectively. The prices of X and Y are Rs 4 per unit and Rs 5 per unit respectively. Is the consumer in equilibrium? What will be the further reaction of the consumer? Explain.
Answer.
Image may be NSFW.
Clik here to view.
cbse-sample-papers-class-12-economics-delhi-2016-1

Question. 7 Price elasticity of demand of good X is -2 and of good Y is -3. Which of the two goods is more price elastic and why?
Answer. Due to negative relationship between price and demand, elasticity of demand has a. negative sign. But, for the purpose of comparison of price elasticity of demand of two goods, the absolute value is taken. -2 and -3 are the actual Values of the price elasticities of the two goods. But on comparing the two, -3 is taken to be greater than -2 as we consider the absolute values. Therefore, Good Y is more price elastic as compared to Good X.

Question. 8 What is Maximum Price Ceiling? Explain its implications.
Answer. Maximum price ceiling refers to imposition of upper limit by the government on the price of a good that can be Charged by a seller. The government usually fixes this price at a level lower than the equilibrium price so that it is within the reach of the poorer sections of society. For example, in the diagram, OP is the price ceiling which is less than the equilibrium price OPr At this price, the producers are willing to supply only PA (or OQ1) quantity of the goods while consumers demand PB (or OQ2) quantity.
Image may be NSFW.
Clik here to view.
cbse-sample-papers-class-12-economics-delhi-2016-2

The demand becomes higher than supply and this leads to shortage (equal to AB or Q1Q2) in the market which may further lead to hoarding and black marketing. This situation of black marketing can be solved by introducing the system of rationing.
Or
Explain the chain effects, if the prevailing market price is below the equilibrium price.
Answer. In the given diagram, demand is equal to supply at the equilibrium market price OP. Suppose, price falls to OP1. If the prevailing market price is below the equilibrium price, a situation of excess demand (SJDJ) emerges. In this situation, consumers will riot beadle to buy all what they want. It will lead to competition among consumers. They will start offering a higher price to the suppliers. At a higher price, suppliers will be willing to supply more. The price will start moving upwards along the demand curve (contraction) and supply curve (extension), as indicated by the arrows, till it reaches OP at E (equilibrium level).
Image may be NSFW.
Clik here to view.
cbse-sample-papers-class-12-economics-delhi-2016-3

Question. 9 Explain the effect of change in prices of the related goods on demand for the given good.
Answer. The effect of change in prices of the related goods on demand for the given good can be studied as under:
(a) Substitute Goods. Substitute goods are those goods which can be used in place of one another. For example, tea and coffee. An increase in the price of a substitute good (say tea)causesSn increase in demand for the given good (coffee) and shifts the
demand curve to the right. This is because the product (coffee in this case) becomes cheaper in comparison to its substitute (tea in this case).
Similarly, a decrease in the price of the substitute will decrease the demand of the given good leading to a leftward shift in the demand curve of the given good.

Question.10 Define Production function. Distinguish between short run and long run production functions.
Answer. The functional relationship between physical inputs and physical output of a commodity is called Production function.
Image may be NSFW.
Clik here to view.
cbse-sample-papers-class-12-economics-delhi-2016-4

Or
Define Cost. Distinguish fixed and variable costs. Give one example of each.
Answer. Cost is the total expenditure incurred in producing a commodity. It is the sum-total of explicit costs and implicit costs.
Image may be NSFW.
Clik here to view.
cbse-sample-papers-class-12-economics-delhi-2016-5

Question. 11 A producer supplies 80 units of a good at a price of Rs 10 per unit. Price elasticity of supply is 4. How much will he supply at Rs9 per unit?
Answer.
Image may be NSFW.
Clik here to view.
cbse-sample-papers-class-12-economics-delhi-2016-6

Question. 12 Assuming that no resource is equally efficient in production of all goods, name the ‘ curve which shows production potential of the economy. Explain, giving reasons, its properties.
Answer. Production Possibility Curve (PPC) is the curve showing the production potential or the alternate production possibilities of two goods in an economy with given resources and technique of production assuming full and efficient utilization of these resources. . Properties of PPC are as follows:
(i) PPC slopes downwards. PPC slopes downwards from left to right. This is so
because to produce more quantity of one good the society has to sacrifice some quantity of the other good. The reason being that resources are limited.. ,
(ii) PPC is concave to the point of origin. The PP curve is concave because marginal rate of transformation (MRT) which is the slope of PPC, increases continuously as more and more of one good is produced by reducing the quantity of the other good. It means that to produce more and more units of one good each time, the quantity of the other good is sacrificed at an increasing rate. MRT increases because no resource is equally efficient in production of all goods.

Question. 13 Explain the conditions of consumer’s equilibrium using indifference curve analysis.
Answer. See Q. 14, 2014 (Comptt. I Delhi).

Question. 14 Explain the distinction “change in quantity supplied” and “change in supply”. Use diagram.
Answer. Difference between change in quantity supplied and change in supply
Image may be NSFW.
Clik here to view.
cbse-sample-papers-class-12-economics-delhi-2016-7

Image may be NSFW.
Clik here to view.
cbse-sample-papers-class-12-economics-delhi-2016-8

Image may be NSFW.
Clik here to view.
cbse-sample-papers-class-12-economics-delhi-2016-9

Note: For BLIND CANDIDATES in lieu of Q. No. 14.
Explain the distinction between “change in quantity supplied” and “change in supply”. Use schedule.
Answer. Same as Q. 14 (above).

Question. 15 Explain the implications of the following in a perfectly competitive market:
(a) Large number of buyers (b) Freedom of entry and exit to firms
Answer. Following are the two important features of a perfectly competitive market:
(a) Large number of buyers. See Q. 7, 2015 (I Delhi). [Page 321
(b) Freedom to the firms to enter the industry. Under perfect competition firms have the freedom to enter the ihdtistry. As a result of this, existing firms which are earning abnormal profit will cease to earn that. New firms entering the industry will increase the supply of the product by lowering the price, heading to a situation where a firm earns only normal profit. This means price will be the same for all the firms.
Or
Explain the implications of the following in an oligopoly market:
(a) Inter-dependence between firms (b) Non-price competition
Answer. (a) Inter-dependence between firms. See Q. 8, 2015 (Comptt. II Outside Delhi). [Page 370
(b) Non-price competition. See Q. 8, 2015 (II Outside Delhi).

SECTION B
Question. 16 Define Stocks.
Answer. Stocks are variables whose magnitude is measured at a particular point of time. For example, capital invested as on 31st March, 2015 by a firm.

Question. 17 Depreciation of fixed capital assets refers to: (choose the correct alternative)
(a) Normal wear and tear (b) Foreseen obsolescence
(c) Normal wear and tear and foreseen obsolescence
(d) Unforeseen obsolescence
Answer. (c) Normal wear and tear and foreseen obsolescence.

Question. 18 What is Revenue Expenditure?
Answer. Revenue expenditure of the government is that expenditure which neither creates any asset nor does it lead to any reduction in liability.

Question. 19 Fiscal deficit equals: (choose the correct alternative)
(a) Interest payments (b) Borrowings
(c) Interest payments less borrowing (d) Borrowings less interest payments
Answer. (b) Borrowings

Question. 20 Foreign exchange transactions dependent on other foreign exchange transactions are called (choose the correct alternative)
(a) Current account transactions (b) Capital account transactions
(c) Autonomous transactions (d) Accommodating transactions
Answer. (d) Accommodating transactions.

Question.21 Find Net Value Added at Factor Cost:
Image may be NSFW.
Clik here to view.
cbse-sample-papers-class-12-economics-delhi-2016-10

Answer.
Image may be NSFW.
Clik here to view.
cbse-sample-papers-class-12-economics-delhi-2016-11

Question. 22 Distinguish between marginal propensity to consume and average, propensity to consume. Give a numerical example
Answer. Difference between marginal propensity to consume and average propensity to consume:
Image may be NSFW.
Clik here to view.
cbse-sample-papers-class-12-economics-delhi-2016-12

Or
Explain the role of taxation in reducing excess demand.
Answer. Excess demand refers to the situation when aggregate demand is in excess of aggregate supply i.e., (AD > AS) corresponding to full employment in .the economy.
To reduce excess demand, the government under its fiscal policy measures can increase the tax burden on households and producers. This will reduce the purchasing power in the economy and hence reduce aggregate demand.

Question. 23 In an economy investment is increased by Rs 300 crore. If marginal propensity to consume is 2/3, calculate increase in national income.
Answer.
Image may be NSFW.
Clik here to view.
cbse-sample-papers-class-12-economics-delhi-2016-13

Question. 24 Government incurs expenditure to popularize yoga among the masses. Analyse its impact on gross domestic product and welfare of the people.
Answer. By trying to popularize yoga among the masses, the government is trying to create health awareness. It is also trying to get people to adopt a healthy lifestyle. This, in the long run, will benefit both the society and economy. Adoption of healthy habits will raise the physical and emotional welfare of the people. A healthy and fit workforce will have a positive impact on the gross domestic product (GDP) of the country. Improvement in health and increase in fitness levels will lead to improved productivity arid efficiency of workforce along with greater stamina to work and reduced absenteeism. This in turn will increase the GDP of the country, i.e., higher availability of goods and services per person. This will further increase the welfare, of people.

Question. 25 Explain the ‘Store of Value’ function of money. How has it solved the related problem created by barter?
Answer. Store of Value. Store of value means storing of wealth for future use. Money as a store of value means that an individual can save his earnings until the time he wants to spend the same. Storing wealth has become considerably easy with the introduction of money. This is because money comes in convenient denominations, it is easily portable and can be easily exchanged for goods at’ alHimes. It is also a very economical and convenient way of storing wealth.
In the barter system, no goods served as convenient assets for use in future. For storing purpose the good must be non-perishable, easily portable and readily acceptable for exchange at any time with other goods. All these features were found in money which helped overcome this particular limitation of barter system.
Or
Explain the ‘Unit of Account’ function of money. How has it solved the related problem created by barter?
Answer. Unit of Account. Unit of account means that the value of each good or service is measured in monetary unit.
The main difficulty that arose in the barter system was that at what rate any exchange should be made? In the absence of a common unit of value, the price of each commodity had to be quoted in terms of all the other commodities available in the market. The price list, thus, became extremely complicated.
Introduction of money overcame this problem. Money became a powerful medium of comparing prices of goods and services. It became the standard unit of measurement in terms of which the value of all goods and services are being measured and expressed (known as price). It is treated as the standard unit for borrowing and lending activities. It has also made possible the keeping of business accounts and thus facilitated trade.

Question. 26 Explain how open market operations are helpful in controlling credit creation.
Answer. See Q. 29, 2013 (II Outside Delhi).

Question. 27 What is Government budget? Explain how taxes and subsidies can be used to influence allocation, of resources.
Answer. A government budget is a statement showing item-wise estimated receipts and expenditures of the government under various heads during a given financial year.
The government tries to influence economic life through its budgetary measures like taxes and subsidies. Reallocation of resources is an important objective of government budget.
Allocation of resources. In market economies allocation of resources is determined by the forces of demand and supply. Resources are allocated to the production of those goods which earn high profits ignoring social welfare. The government corrects or reallocates the resources throughTits taxation policy to maintain a balance between social welfare and economic welfare.
(i) Government discourages the production of harmful consumption goods (like liquor, cigarettes etc.) by imposing high taxes.
(ii) Tax concession and subsidies are given to encourage the production of essential goods such as food grains, kerosene etc.
(iii) The concessions and subsidies are also given to encourage productive activities in backward areas.
Or
Define Revenue receipts in a government budget. Explain how government budget can be used to bring in price stability in the economy.
Answer. Revenue receipts of the government may be defined as those money receipts which do not create a liability for thq government and neither do they lead to a reduction in assets of the government. These include tax and non-tax receipts, duties and fines, interest and dividend receipts on government assets and investments.
The government budget is used to prevent economic fluctuations. Economic fluctuations refer to the situations of inflation or deflation. Such fluctuations create uncertainties in the economy. Absence of large scale fluctuation in prices promote economic stability. Government can exercise control over these fluctuations through taxes and expenditure. For example, in inflationary situation, government can discourage spending by increasing taxes and at the same time reduce its own expenditure. In times of depressed conditions, government can encourage spending by giving tax concessions, subsidies etc. and at the same time increase its own expenditure.

Question. 28 Given Consumption Curve, derive saving curve and state the steps taken in the process of derivation. Use diagram.
Answer. Saving function refers to the functional relationship between saving and income. Consumption and Saving curves are complementary curves as they together make up Income. Therefore, the saving curve can easily be derived from the consumption curve. Total Income = Consumption expenditure + Saving. Consumption is the function of Income. However, some consumption is always there even if there is no income, i.e., at zero level of income. This is called autonomous consumption.
Image may be NSFW.
Clik here to view.
cbse-sample-papers-class-12-economics-delhi-2016-14

Note: For BLIND CANDIDATES in lieu of Q. No. 28.
Explain the components of consumption function. Derive saying function from consumption function
Answer. Consumption function refers to the mathematical relation between consumption expenditure and income.
This relationship can be expressed in the form of an equation in the following way:
Image may be NSFW.
Clik here to view.
cbse-sample-papers-class-12-economics-delhi-2016-15

Question. 29 (a) In which sub-account and on which side of balance of payments account will
foreign investments in India be recorded? Give reasons.
(b) What will be the effect of foreign investments in India on exchange rate? Explain.
Answer. (a) Foreign investments in India will be recorded on the credit side of Capital Account of BOP indicate by the positive sign. This is because Borrowings and Investments involving the rest of the world are the two principal components of Capital Account of BOP. Also, inflow of foreign exchange or receipts from the rest of the world are recorded as credit.
(b) Foreign investments in India will increase the supply of foreign currency. This will lead to decrease in the exchange rate which implies that domestic currency appreciates in relation to the foreign currency.

Question. 30 Find national income and private income:
Image may be NSFW.
Clik here to view.
cbse-sample-papers-class-12-economics-delhi-2016-16

Answer.
Image may be NSFW.
Clik here to view.
cbse-sample-papers-class-12-economics-delhi-2016-17

SET II

Note: Except for the following questions, all the remaining questions have been asked in Set-I.
SECTION A
Question. 4 When does ‘increase’ in supply take place?
Answer. Increase in supply takes place due to a favourable change in factors other than own price – of the commodity.

Question. 5 What is the relation between marginal cost and average cost when average cost is constant?
Answer. Average Cost (AC) is constant when it is at its minimum and at this point MC = AC.

Question. 7 A consumer consumes only two goods X and Y. If marginal utilities of X and Y are 4 and 5 respectively, and if price of X is Rs 5 per unit and that of Y is Rs 4 per unit, is the consumer in equilibrium? What will be further reaction of the consumer? Explain.
Answer.
Image may be NSFW.
Clik here to view.
cbse-sample-papers-class-12-economics-delhi-2016-18

Question. 9 Price elasticity of supply of good is 2. A producer supplies 100 units of a good at a price of Rs 20 per unit. At what price will he supply 80 units.
Answer.
Image may be NSFW.
Clik here to view.
cbse-sample-papers-class-12-economics-delhi-2016-19

Question. 10 Explain the effects of change in income on demand for a good
Answer. Demand for a commodity is also affected by change in income of the consumer. The effect of change in income on the demand depends on the type of good.
(i) Normal goods. A consumer buys more of such goods when his income rises leading to a rightward shift in the demand curve of normal goods. With falls in income, the demand for normal goods fall shifting the demand curve towards left.
(ii) Inferior goods. An increase or decrease in income inversely affects the demand of inferior goods. As income of the consumer increases the demand for inferior goods falls resulting in a leftward shift in the demand curve of inferior goods and vice versa:

SECTION B
Question. 21 Suppose marginal propensity to consume is 0.8. How much increase in investment is required to increase national income by Rs 2000 crore? Calculate.
Answer.
Image may be NSFW.
Clik here to view.
cbse-sample-papers-class-12-economics-delhi-2016-20

Question. 22 Find Net Value Added at Market Price:
Image may be NSFW.
Clik here to view.
cbse-sample-papers-class-12-economics-delhi-2016-21

Answer.
Image may be NSFW.
Clik here to view.
cbse-sample-papers-class-12-economics-delhi-2016-22

Question. 24 Explain how ‘bank rate’ is helpful in controlling credit creation?
Answer. Bank rate is the rate of interest at which the RBI gives long-term loans to commercial banks. A rise in bank rate would make borrowings by commercial banks costly. They in turn increase the lending rates to the general public. As borrowings from banks become costly, it leads to a decline in demand for loans from banks which adversely affects credit creation. On the other hand, a fall in bank rate induces the commercial banks to reduce the interest rates on lendings to general public. This increases demand for loans from banks increasing credit creation in the economy.

Question. 27 Find net domestic product at factor cost and personal income:
Image may be NSFW.
Clik here to view.
cbse-sample-papers-class-12-economics-delhi-2016-23

Image may be NSFW.
Clik here to view.
cbse-sample-papers-class-12-economics-delhi-2016-24

Answer.
Image may be NSFW.
Clik here to view.
cbse-sample-papers-class-12-economics-delhi-2016-25

SET III

Note: Except for the following questions, all the remaining questions have been asked in Set I and Set II.
SECTION A
Question. 5 When does ‘decrease’ in supply take place?
Answer. Decrease in supply takes place due to unfavorable changes in factors other than the price of the good like increase in taxes, decrease in subsidies, increase in input prices, ‘ high cost of production due to obsolete technology etc.

Question. 8 A consumer consumes only two goods X and Y. Marginal utility of each is 2. The price per unit of X and Y is Rs 1 and Rs 2 respectively. Is the consumer in equilibrium? What will be the further reaction of the consumer? Explain.
Answer.
Image may be NSFW.
Clik here to view.
cbse-sample-papers-class-12-economics-delhi-2016-26

Question. 10 When price of a good rises from Rs 12 per unit to Rs 15 per unit the producer supplies 50 per cent more output. What is the price elasticity of supply? Calculate.
Answer.
Image may be NSFW.
Clik here to view.
cbse-sample-papers-class-12-economics-delhi-2016-27

Question. 11 Explain the effect of: (a) change in own price and (b) change in price of substitute on demand of a good.
Answer. (a) Change in own price. Other things remaining constant, a change in the price of the commodity leads to a change in the quantity demanded. Graphically, it means movement along the same demand curve. When quantity demanded increases as a result of fall in price of the commodity, it leads to a downward movement (called expansion) along the same demand curve.
On the other hand, an increase in the price of the commodity results in a reduction in the quantity demanded leading to an upward movement on the same demand curve (called contraction).
(b) See Q. 9(a), 2016 (I Delhi).

SECTION B
Question. 20 What is Revenue deficit in government budget?
Answer. Revenue deficit is the excess of government’s revenue expenditure over revenue receipts.
Revenue Deficit- Revenue Expenditure – Revenue Receipts

Question. 22 In an economy an increase in investment by Rs 100 crore led to ‘increase’ in national income by Rs 1000 crore. Find marginal propensity to consume.
Answer.
Image may be NSFW.
Clik here to view.
cbse-sample-papers-class-12-economics-delhi-2016-28

Question. 29 Find net national product at market price and personal disposable income:
Image may be NSFW.
Clik here to view.
cbse-sample-papers-class-12-economics-delhi-2016-29

Answer.
Image may be NSFW.
Clik here to view.
cbse-sample-papers-class-12-economics-delhi-2016-30

The post CBSE Sample Papers for Class 12 Economics Delhi – 2016 appeared first on Learn CBSE.


NCERT Exemplar Problems Class 8 Mathematics Introduction to Graphs

NCERT Exemplar Problems Class 8 Mathematics  Chapter 12 Introduction to Graphs

Multiple Choice Questions
Question. 1 Comparison of parts of a whole may be done by a
(a) bar graph (b) pie chart
(c) linear graph (c) line graph
Solution.
(b) There are various ways to represent and compare the data. One of them is pie chart.
Pie chart is a pictorial representation of the data in which the whole is represented by a circle and the parts, by non-intersecting adjacent sectors.
Hence, comparison of parts of a whole may be done by a pie chart.

Question. 2 A graph that displays data that changes continuously over periods of time is 451
(a) bar graph (b) pie chart
(c) histogram (d) line graph
Solution.
(d) Line graph is an important way to represent and compare the data which varies continuously. •
A line graph displays the relation between two varying quantities.
In a line graph, we connect all the points by a line segment while in bar graph and histogram, we use rectangles of uniform width.

Question. 3 In the given graph the coordinates of point P are
Image may be NSFW.
Clik here to view.
ncert-exemplar-problems-class-8-mathematics-introduction-to-graphs-2-1

Solution.
(c) The foot of the perpendicular drawn from the indicated point on X-axis is at a distance of 3 units from the origin.
The x-coordinate of the point is 3.
Again, the perpendicular drawn from the given point on Y-axis meets the Y-axis at a point which is at a distance of 2 units from origin.
The y-coordinate of the point is 2.
Hence, the coordinates of the point are (3,2).
Image may be NSFW.
Clik here to view.
ncert-exemplar-problems-class-8-mathematics-introduction-to-graphs-2-2

Question. 4 In the given graph the letter that indicates the point (0,3) is
Image may be NSFW.
Clik here to view.
ncert-exemplar-problems-class-8-mathematics-introduction-to-graphs-2-3

Solution.
(c) The letter that indicates the point (0,3) is R as it lies on the Y-axis at a distance of 3 units from the origin.
The coordinates of the point P and Sare (3,0) and (3,3), respectively.

Question. 5 The point (3,4) is at a distance of
(a) 3 from both the axes
(b) 4 from both the axes
(c) 4 from the X-axis and 3 from Y-axis
(d) 3 from X-axis and from Y-axis
Solution.
(c) We know that, the x-coordinate is the distance of the point from Y-axis and that of y-coordinate is the distance from X-axis. .
Hence, the point (3,4) is at a distance of 4 from the X-axis and 3 from Y-axis.

Question. 6 A point which lies on both the axes is
(a) (0,0) (b) (0,1) (0(1,0) (d) (1,1)
Solution.
(a) We know that, the axes are two mutually perpendicular lines intersecting each other at the point (0,0) also known as the origin.
Hence, the point which lies on both the axes is (0,0).

Question. 7 The coordinates of a point at a distance of 3 units from the X-axis and 6 units from the y-axis are
(a) (0,3) (b) (6,0) (c) (3,6) (d) (6,3)
Solution.
(d) We know that, the x-coordinate is the distance of the point from /-axis and the y-coordinate is the distance of the point from X-axis.
Hence, the coordinates of the required point are (6,3).

Question. 8 In the given figure, the position of the book on the table may be given by
Image may be NSFW.
Clik here to view.
ncert-exemplar-problems-class-8-mathematics-introduction-to-graphs-2-5

Solution.
(b) The book is at a distance of 3 units from Y-axis and 7 units from X-axis.
Hence, the position of the book on the table is given by (3,7).

Question. 9 Data was collected on a student’s typing rate and graph was drawn as shown below. Approximately how many words had this student typed in 30 s?
Image may be NSFW.
Clik here to view.
ncert-exemplar-problems-class-8-mathematics-introduction-to-graphs-2-6

Solution.
(c) Observing the graph, we see that the graph intersects the line x = 30 just below the point (30,30), which is the intersection of the lines* = 30 and y- 30.
Since, the X-axis represents the time (in seconds) and the Y-axis represents the number of words typed, therefore we conclude that the students had typed approximately 28 words In 30 s.

Question. 10 Which graph of the following represents the table below?
Image may be NSFW.
Clik here to view.
ncert-exemplar-problems-class-8-mathematics-introduction-to-graphs-2-7

Solution.
(d) Here, the lengths of sides of squares are represented on X-axis and their perimeters are represented on Y-axis.
The points are (1,4),(2,8),(3,12),(4,16) and(5,20).
Now, observing all the 4 graphs carefully we see that these points lie on the graph(d).

Fill in the Blanks
In questions 11 to 25, fill in the blanks to make the statements true.
Question. 11________.displays data that changes continuously over periods of time.
Solution. Line graph
We have already discussed in the previous question that a line graph displays data that changes continuously over periods of time.

Question. 12 The relation between dependent and independent variables is shown
through a________.
Solution. graph .
Various types of graph depicts the relation between two variables, one of them is independent and the other is dependent.
A graph shows how a change in the independent variable changes the dependent variable.

Question. 13 We need________coordinates for representing a point on the graph
sheet.
Solution. pair of (or two)
To represent the position of a point on the graph sheet we require two measures namely x-coordinate and y-coordinate.

Question. 14 A point in which the x-coordinate is zero and y-coordinate is non- zero
will lie on the________.
Solution. Y-axis
Since, the x-coordinate is zero, i.e. the distance of the point from Y-axis is 0, the point lies on the Y-axis at a certain distance from the origin, which is given by y-coordinate.

Question. 15 The horizontal and vertical lines in a graph are usually called________and
Solution. X-axis, Y-axis
To draw a graph we need pairs of points known as coordinates and to plot a point we require two mutually perpendicular axes, also known as the X-axis (horizontal line) and the Y-axis (vertical line).

Question. 16 The process of fixing a point with the help of the coordinates is known
as________of the point.
Solution. plotting
To locate the exact position of a point we need two coordinates viz, the x-coordinate and the y-coordinate and this process of finding the position or representing the numbers on a graph sheet with the help of the coordinates is known as plotting of the point.

Question. 17 The distance of any point from the y-axis is the________coordinate.
Solution. x
x-coordinate of a point is the distance of any point from the Y-axis.

Question. 18 All points with y-coordinate as zero lie on the________.
Solution. X-axis
Since, y-coordinate is zero, i.e. the distance of the point from X-axis is zero.
Hence, the points lies on the X-axis.

Question. 19 For the point (5,2), the distance from the X-axis is________units.
Solution. 2
We know that, the y-coordinate represents the distance of the point from the X-axis. ;Hence, the point (5,2) is at a distance of 2 units from the X-axis.

Question. 20 The x-coordinate of any point lying on the y-axis will be________.
Solution. zero
Since, the x-coordinate represents the distance of the point from Y-axis is zero, therefore the points lying on the Y-axis have x-coordinate as zero.

Question.21 The y-coordinate of the point (2,4) is________.
Solution. 4
In the ordered pair (2,4), i.e. coordinates of a point, the second number is called as the y-coordinate of the point.
Hence, the y-coordinate of (2,4) is 4

Question. 22 In the point (4,7), 4 denotes the________.
Solution. x-coordinate (abscissa)
First number (coordinate) of the ordered pair is called as the x-coordinate or abscissa. Hence, in the point (4,7),4 denotes the x-coordinate.

Question. 23 A point has 5 as its x-coordinate and 4 as its y-coordinate. Then, the
coordinates of the point are given by________.
Solution. (5,4)
To denote a point in 2-D, we use two numbers viz. the x-coordinate, the y-coordinate.
In the ordered pair, the x-coordinate is written in the first slot and the y-coordinate in the second slot separated by comma. Hence, the required point is (5,4).

Question. 24 In the coordinates of a point, the second number denotes the________.
Solution. y-coordinate y ordinate
As we have discussed in the above question, that the second number denotes the . y-coordinate, also known as the ordinate.

Question. 25 The point where the two axes intersect is called the________.
Solution. origin
The X-axis and y-axis intersect each other at the point representing the origin. Coordinates of origin are (0,0).

True/False
In questions 26 to 34, state whether the statements are True or False.
Question. 26 For fixing a point on the graph sheet we need two coordinates.
Solution. True
To plot a point on the graph sheet we require two numbers, known as coordinates viz. the x-coordinate and the y-coordinate.

Question. 27 A line graph can also be a whole unbroken line.
Solution. True
A fine graph, which represents the variation of a quantity with respect to the other, may be an unbroken line.

Question. 28 The distance of any point from the X-axis is called the x-coordinate.
Solution. False
The distance of any point from the X-axis is called the y-coordinate.

Question. 29 The distance of the point (3,5) from the Y-axis is 5.
Solution. False
We know that the x-coordinate of a point represents the distance of the point from Y-axis. Here x-coordinate is 3, so the distance of the point (3,5) from the Y-axis is 3.

Question. 30 The ordinate of a point is its distance from the Y-axis.
Solution. False
The ordinate of a point is nothing but ^coordinate of the point and the y-coordinate denotes the distance of a point from X-axis.

Question. 31 In the point (2,3), 3 denotes the y-coordinate.
Solution. True
In the ordered pair (2,3), the second number is called the y-coordinate or ordinate of the number.
Hence, 3 denotes the y-coordinate of the point (2,3).

Question. 32 The coordinates of the origin are (0,0).
Solution. True
Origin is the point, where two axes meet and its coordinates are (0,0).

Question. 33 The points (3,5) and (5,3) represent the same point.
Solution. False
Two ordered pairs are equal, if they have same numbers at corresponding slot, i.e. x-coordinates are equal and Y-coordinates are equal. Hence, (3,5) and (5,3) are different points.

Question. 34 The y-coordinate of any point lying on the X-axis will be zero.
Solution. True
The distance of the points which lie on X-axis, will be zero from the X-axis, i.e. y-coordinate is zero for the points lying on X-axis.

Question. 35 Match the coordinates given in Column A with the items mentioned in Column B.
Image may be NSFW.
Clik here to view.
ncert-exemplar-problems-class-8-mathematics-introduction-to-graphs-2-8

Solution.
(a) In the pair (0,5), the second number also known as ordinate represents the distance from X-axis, i.e. 5.
(a) In the pair (2,3), 2 the first number, also known as abscissa represents the distance from Y-axis that is 2.
(b) We have the coordinates (4,8). Clearly, ^coordinate is double of x-coordinate.
(c) We have the coordinate (3,7), where x-coordinate = 3 and y-coordinate = 7
Evidently, y-coordinate = 2 x x-coordinate + 1
(d) (0,0) are the coordinates of origin.
(e) In the point (5,0), the y-coordinate is zero.
Hence, (a) —> iv, (b) —> vi, (c) —> v, (d) —> i, (e) —> ii, (f) —> iii.

Question. 36 Match the ordinates of the points given in Column A with the items mentioned in Column B.
Image may be NSFW.
Clik here to view.
ncert-exemplar-problems-class-8-mathematics-introduction-to-graphs-2-82

Solution.
(a) Clearly, the ordinate of the point (7,0) is zero.
(b) In the point (11,11), the ordinate is equal to the abscissa.
(c) In the point (4,8), the ordinate is double of the abscissa.
(d) In the point (6,2), the abscissa, i.e. x-coordinate is triple of the ordinate, i.e. y-coordinate.
(e) The abscissa of the point (0,9) is zero.
(f) Clearly,the abscissa is double of the ordinate.
Hence, (a)—> ii, (b) —> iii, (c) —> i, (d) -> v, (e) —> vi, (f) —> iv.

Question. 37 From the given graph, choose the letters that indicate the location of the points given below.
Image may be NSFW.
Clik here to view.
ncert-exemplar-problems-class-8-mathematics-introduction-to-graphs-2-9

Solution.
On observing the graph, we see that the point F is on X-axis, so its Y-coordinate will be zero.
Also, it is at a distance of 2 units from origin.
... The coordinates of F are (2,0), similarly the coordinates of G are (4,0).
H is at a distance of 5 units from Y-axis and 1 unit from X-axis.
... The coordinates of Hare (5,1).
/ is at a distance of 6 units from Y-axis and 2 units from X-axis.
... The coordinates of / are (6,2).
The point D and A are on Y-axis at distances of 2 units and 4 units respectively from the origin.
Hence, the coordinates of D and A are (0,2) and (0,4), respectively.
B is at a distance of 1 unit from Y-axis and 5 units from X-axis.
... The coordinates of Bare (1,5).
C is at a distance of 2 units from Y-axis and 6 units from X-axis.
... The coordinates of C are (2,6).
E is at a distance of 3 units from Y-axis and X-axis both.
... The coordinates of E are (3,3).

Question. 38 Find the coordinates of all letters in the graph given below.
Image may be NSFW.
Clik here to view.
ncert-exemplar-problems-class-8-mathematics-introduction-to-graphs-2-10

Solution.
The point A is on the Y-axis at a distance of 7.5 units from the origin.
... The coordinates are (0,7.5).
The point B is at a distance of 4 units from Y-axis and 5 units from X-axis.
... The coordinates of B are (4,5).
The point C is at a distance of 7.5 units from Y-axis and 2.5 units from X-axis .
... The coordinates of C denotes (7.5, 2.5).
The point D lies on X-axis at a distance of 11 units from the origin.
... The coordinates of D are (11,0).
The point £ is at a distance of 14.5 units from Y-axis and 6.5 units from X-axis.
... The coordinates of £ are (14.5, 6.5).
The point F is at a distance of 18 units from Y-axis and 9.5 units from X-axis.
... The coordinates of £ are (18,9.5).

Question. 39 Plot the given points on a graph sheet
(a) (5,4) (b) (2,0) (c) (3,1) (d)(0,4) (e) (4,5)
Solution.
Image may be NSFW.
Clik here to view.
ncert-exemplar-problems-class-8-mathematics-introduction-to-graphs-2-11

Question. 40 Study the given map of a zoo and answer the following questions.
Image may be NSFW.
Clik here to view.
ncert-exemplar-problems-class-8-mathematics-introduction-to-graphs-2-12

(a) Give the location of lions in the zoo.
(b) (D,f) and (C,d) represent locations of which animals in the zoo?
(c) Where are the toilets located?
(d) Give the location of canteen?
Solution. 
(a) Lions are at a distance of A units from road Y and f units from road X.
... The location of lions is represented by the point (Af)
(b) Observing the map carefully, we see that (D, f) denotes the location of monkeys and (C,d) denotes the location of elephants.
(c) Toilets are located on the road Y at a distance of e units from the origin. Hence, the location of toilets is (0, e).
(d) Canteen is located at a distance of C units from road Y and c units from road X. Hence, the location of the canteen is (C,c).

Question. 41 Write the x-coordinate (abscissa) of each of the given points.
(a) (7,3) (b) (5,7) (c) (0,5)
Solution.
(a) The x-coordinate of the point (7,3) is 7
(b) The x-coordinate of the point (5,7) is 5.
(c) The x-ooordinate of the point (0,5) is 0,

Question. 42 Write the y-coordinate (ordinate) of each of the given points.
(a) (3,5) (b) (4,0) (c) (2,7)
Solution.
(a) The y-coordinate of the point (3,5) is 5.
(b) y-coordinate of the point (4,0) is 0.
(c) The ycoordinate of the point (2,7) is 7.

Question. 43 Plot the given points on a graph sheet and check if the points lie on a straight line. If not, name the shape they form when joined in the given order.
(a) (1,2), (2,4), (3,6), (4,8)
(b) (14), (1,2), (2,1), (2,2)
(c) (4,2), (2,4), (3,3), (5,4)
Solution.
Image may be NSFW.
Clik here to view.
ncert-exemplar-problems-class-8-mathematics-introduction-to-graphs-2-13

Image may be NSFW.
Clik here to view.
ncert-exemplar-problems-class-8-mathematics-introduction-to-graphs-2-14

Yes, the given points lie on a straight line.
No, the points do not form a straight line, they form a square.

Question. 44 If y-coordinate is 3 times x coordinate, form a table for it and draw a graph.
Solution.
Since, the ordinate is 3 times the abscissa, we get the following values.
Image may be NSFW.
Clik here to view.
ncert-exemplar-problems-class-8-mathematics-introduction-to-graphs-2-15

Question. 45 Make a line graph for the area of a square as per the given table.
Image may be NSFW.
Clik here to view.
ncert-exemplar-problems-class-8-mathematics-introduction-to-graphs-2-16

Solution.
Image may be NSFW.
Clik here to view.
ncert-exemplar-problems-class-8-mathematics-introduction-to-graphs-2-17

Question. 46 The cost of a notebook is Rs 10. Draw a graph after making a table showing cost of 2,3,4… notebooks. Use it to find
(a) the cost of 7 notebooks.
(b) the number of notebooks that can be purchased with Rs 50.
Solution.
Let x : number of notebooks
y: cost of a notebook
Image may be NSFW.
Clik here to view.
ncert-exemplar-problems-class-8-mathematics-introduction-to-graphs-2-18

(a) The cost of 7 notebooks is aqua) to the ordinate of the point (7,70), i.e. cost of 7 notebooks = Rs 70
(b) The number of notebooks that can be purchased with Rs 50 is equal to the abscissa of the point (5,50).
Hence, 5 notebooks can be purchased with Rs 50.

Question. 47 Explain the situations represented by the following distance-time graphs.
Image may be NSFW.
Clik here to view.
ncert-exemplar-problems-class-8-mathematics-introduction-to-graphs-2-19

Solution. Here X-axis represents time and y – axis represents distance.
(a) In the first graph, we observe that when time changes, distance also varies at the same rate.
When we move along time axis away from the origin, then the graph is strictly increasing.
Hence, the object is moving at a uniform speed.
(b) In the graph (b), we observe that initially graph increases steadily i.e. at uniform speed and after a certain period of time, it comes to rest position i.e. constant.
(c) In the graph (c), we see that the graph increases strictly with non-uniform speed and then slowly comes to the rest position.

Question. 48 Complete the given tables and draw a graph for each.
Image may be NSFW.
Clik here to view.
ncert-exemplar-problems-class-8-mathematics-introduction-to-graphs-2-20

Solution.
Image may be NSFW.
Clik here to view.
ncert-exemplar-problems-class-8-mathematics-introduction-to-graphs-2-21

Image may be NSFW.
Clik here to view.
ncert-exemplar-problems-class-8-mathematics-introduction-to-graphs-2-22

Question. 49 Study the given graphs (a) and (b) and complete the corresponding tables below
Image may be NSFW.
Clik here to view.
ncert-exemplar-problems-class-8-mathematics-introduction-to-graphs-2-23

Solution.
Image may be NSFW.
Clik here to view.
ncert-exemplar-problems-class-8-mathematics-introduction-to-graphs-2-24

Question. 50 Draw a graph for the radius and circumference of circle using a suitable scale.
[Hint Take radius = 7,14,21 units and so on.]
Form the graph,
(a) find the circumference of the circle when radius is 42 units.
(b) at what radius will the circumference of the circle be 220 units?
Solution.
Image may be NSFW.
Clik here to view.
ncert-exemplar-problems-class-8-mathematics-introduction-to-graphs-2-25

Image may be NSFW.
Clik here to view.
ncert-exemplar-problems-class-8-mathematics-introduction-to-graphs-2-26

Question. 51 The graph shows the maximum temperatures recorded for two consecutive
weeks of a town. Study the graph and answer the questions that follow.
Image may be NSFW.
Clik here to view.
ncert-exemplar-problems-class-8-mathematics-introduction-to-graphs-2-27

(a) What information is given by the two axes?
(b) In which week was the temperature higher on most of the days?
(c) On which day was the temperature same in both the weeks?
(d) On which day was the difference in temperatures the maximum for both the weeks?
(e) What were the temperatures for both the weeks on Thursday?
(f) On which day was the temperature 35°C for the first week?
(g) On which day was the temperature highest for the second week?
Solution.
(a) the X-axis represents days of a particular week and the X-axis represents the maximum temperature (in °C) recorded.
(b) Observing the graph, we see that in the first week temperature was higher on most of
the days.
(c) The temperature was same on Wednesday in both the weeks.
(d) The difference in temperatures was the maximum on Friday for both the weeks.
(e) The temperature for the first week on Thursday was 37°C and the temperature for the second week on the same day was 34°C.
(f) On Sunday, the temperature was 35° for the first week.
(g) On Wednesday, the temperature was highest for the second week.

Question. 52 The graph given below gives the actual and expected sales of cars of a company for 6 months. Study the graph and answer the questions that follow.
Image may be NSFW.
Clik here to view.
ncert-exemplar-problems-class-8-mathematics-introduction-to-graphs-2-28

(a) In which month was the actual sales same as the expected sales?
(b) For which month(s) was (were) the difference in actual and expected sales the maximum?
(c) For which month(s) was (were) the difference in actual and expected
sales the least?
(d) What was the total sales of carc in the months-January, February and March?
(e) What is the average sales of cars in the last three months?
(f) Find the ratio of sales in the first three months to the last three months.
Solution.Observing the graph carefully, we conclude that
(a) In April, the actual sales was same as the expected sales.
(b) In March, the difference in actual and expected sales was the maximum.
(c) In April, the difference in actual and expected sales was the least.
(d) The total sales of cars in the months January, February and March was (75 + 100+75) i.e. 250.
(e) The average sales of cars in the last three months is 125 i.e.125 + 100+ 150/3 = 125.
(f) The number of sales of car in the first three months = 250 and the number of sales of car in the last three months = 375
The required ratio is 250 : 375 i.e. 2:3,

Question. 53 The graph given below shows the marks obtained out of 10 by Sonia in two different tests. Study the graph and answer the questions that follow.
Image may be NSFW.
Clik here to view.
ncert-exemplar-problems-class-8-mathematics-introduction-to-graphs-2-29

(a) What information is represented by the axes?
(b) In which subject did she score the highest in Test I?
(c) In which subject did she score the least in Test II?
(d) In which subject did she score the same marks in both the Tests?
(e) What are the marks scored by her in English in Test II?
(f) In which test was the performance better?
(g) In which subject and which test did she score full marks?
Solution. Observing the graph carefully, we conclude that
(a) The X-axis represents subjects and the Y-axis represents the marks obtained by Sonia.
(b) In Maths, she scored the highest in Test I.
(c) In English and Hindi, she scored the least in Test II.
(d) In Hindi and Maths, she scored the same marks in both tests.
(e) She scored 6 marks in English in Test II.
(f) Same performance in both tests.
(g) Test I in Maths, she scored full marks i.e. 10 marks.

Question. 54 Find the coordinates of the vertices of the given figures.
Image may be NSFW.
Clik here to view.
ncert-exemplar-problems-class-8-mathematics-introduction-to-graphs-2-30

Solution.
Image may be NSFW.
Clik here to view.
ncert-exemplar-problems-class-8-mathematics-introduction-to-graphs-2-31

Question. 55 Study the graph given below of a person who started from his home and returned at the end of the day. Answer the questions that follow.
Image may be NSFW.
Clik here to view.
ncert-exemplar-problems-class-8-mathematics-introduction-to-graphs-2-32

(a) At what time did the person start from his home?
(b) How much distance did he travel in the first four hours of his journey?
(c) What was he doing from 3 PM to 5 PM?
(d) What was the total distance travelled by him throughout the day?
(e) Calculate the distance covered by him in the first 8 h of his journey.
(f) At what did he cover 16 km of his journey?
(g) Calculate the average speed of the man from A to B and B to C.
(h) At what time did he return home?
Solution. Observing the graph carefully, we conclude that
(a) At 10 AM, the person start from his home.
(b) In first 4 h (i.e. till 2PM), he travelled 16 km.
(c) He was taking rest from 3 PM to 5 PM.
(d) The total distance covered by the person throughout the day was 40 km, i.e. 20 km from A to Sand then 20 km from C to D.
(e) The distance covered by him in the first 8 h i.e. from 10 AM to 6 PM was 24 km,
(f) He covered 16 km of his journey at 2 PM,
(g) The total distance covered from A to S=20 km
and the time taken to travel from A to B = 5 h
...Average speed of the man from A to B =20/5 = 4 km/h
and average speed from Sto C = 0/2 = 0 km/h
(h) He returned home at 10 PM.

Question. 56 Plot a line graph for the variables p and q, where p is two times q i.e. the equation is p = 2q. Then, find
(a) the value of p when q = 3.
(b) the value of q when p = 8.
Solution.
Image may be NSFW.
Clik here to view.
ncert-exemplar-problems-class-8-mathematics-introduction-to-graphs-2-33

Image may be NSFW.
Clik here to view.
ncert-exemplar-problems-class-8-mathematics-introduction-to-graphs-2-34

Question. 57 Study the graph and answer the questions that follow.
Image may be NSFW.
Clik here to view.
ncert-exemplar-problems-class-8-mathematics-introduction-to-graphs-2-35

(a) What information does the graph give?
(b) On which day was the temperature the least?
(c) On which day was the temperature 31 °C?
(d) Which was the hottest day?
Solution.
(a) The information obtained from the given graph is that the maximum temperature is 34°C
and minimum temperature is 25°C in a week.
(b) On Sunday, the temperature was 25°C. So, it is least temperature in the week.
(c) On Saturday, the temperature was 31 °C.
(d) On Friday, the temperature was maximum i.e. 34°C. Hence, it is the hottest day of the week.

Question. 58 Study the distance-time graph given below for a car to travel to certain places and answer the questions that follow.
Image may be NSFW.
Clik here to view.
ncert-exemplar-problems-class-8-mathematics-introduction-to-graphs-2-36

(a) How far does the car travel in 2′-h?
(b) How much time does the car take to reach /??
(c) How long does the car take to cover 80 km?
(d) How far is Q from the starting point?
(e) When does the car reach the place S after starting?
Solution.
(a) From the given graph, the car travels 80 km in 2h.
(b) 5 h taken by car to reach ft.
(c) 2 h taken by car to cover 80 km. •
(d) G is 120 km far from the starting point.
(e) The car reaches the places after starting in 6 h.

Question. 59 Locate the points A (1, 2), B (4, 2) and C (1, 4) on a graph sheet taking suitable axes. Write the coordinates of the fourth point D to complete the rectangle ABCD.
Solution.
Image may be NSFW.
Clik here to view.
ncert-exemplar-problems-class-8-mathematics-introduction-to-graphs-2-37

Question. 60 Locate the points .A <1, 2), B (3, 4) and C (5, 2) on a graph sheet taking suitable axes. Write the coordinates of the fourth point D to complete the rhombus ABCD. Measure the diagonals of this rhombus and find whether they are equal or not.
Solution.
Image may be NSFW.
Clik here to view.
ncert-exemplar-problems-class-8-mathematics-introduction-to-graphs-2-38

Question. 61 Locate the points P (3,4), Q (1,0), R (0,4), S (4,1) on a graph sheet and write the coordinates of the point of intersection of line segments PQ and RS.
Observing the graph, we see that the line segments PQ and RS intersect at the point!
Solution.
Image may be NSFW.
Clik here to view.
ncert-exemplar-problems-class-8-mathematics-introduction-to-graphs-2-39

Question. 62 The graph given below compares the sales of ice-creams of two vendors for a week.
Image may be NSFW.
Clik here to view.
ncert-exemplar-problems-class-8-mathematics-introduction-to-graphs-2-40

Observe the graph and answer the following questions.
(a) Which vezndor has sold more ice-creams on Friday?
(b) For which day was the sales same for both the vendors?
(c) On which day did the sale of vendor A increase the most as compared to the previous day?
(d) On which day was the difference in sales the maximum?
(e) On which two days was the sales same for vendor B?
Solution. Observing the graph carefully, we conclude that
(a) Vendor A has sold more ice-creams on Friday.
(b) On Sunday , the sales was the same for both the vendors.
(c) On Sunday, the sale of vendor A increased the most as compared to Saturday.
(d) The difference in sales was the maximum on Thursday.
(e) On Tuesday and Wednesday, the sales was the same for vendor B.

Question. 63 The table given below shows the temperatures recorded on a day at different times.
Image may be NSFW.
Clik here to view.
ncert-exemplar-problems-class-8-mathematics-introduction-to-graphs-2-41

Observe the graph and answer the following questions.
(a) What is the temperature at 8 AM?
(b) At what time is the temperature 3°C?
(c) During which hour did the temperature fall?
(d) What is the change in temperature between 7 AM and 10 AM?
(e) During which hour was there a constant temperature?
Solution. Observing the given graph carefully, we have
(a) At 8 AM, the temperature is 7°C.
(b) At 6 AM, the temperature is 3°C.
(c) The temperature fall in the hour 5 AM to 6 AM.
(d) The change in temperature is 3°C between 7 AM and 10 AM.
(e) Between 8 AM to 9 AM, there was a constant temperature.

Question. 64 The following table gives the growth chart of a child.
Image may be NSFW.
Clik here to view.
ncert-exemplar-problems-class-8-mathematics-introduction-to-graphs-2-42

Draw a line graph for the table and answer the questions that follow.
(a) What is the height at the age 4 yr? ,
(b) How much taller was the child at the age of 10 yr than at the age of
6 yr?
(c) Between which two consecutive periods did the child grow more faster?
Solution.
Image may be NSFW.
Clik here to view.
ncert-exemplar-problems-class-8-mathematics-introduction-to-graphs-2-43

Question. 65 The following is the time-distance graph of Sneha’s walking.
Image may be NSFW.
Clik here to view.
ncert-exemplar-problems-class-8-mathematics-introduction-to-graphs-2-44

(a) When does Sneha make the least progress ? Explain your reasoning.
(b) Find her average speed in km/h.
Solution.
Image may be NSFW.
Clik here to view.
ncert-exemplar-problems-class-8-mathematics-introduction-to-graphs-2-45

Question. 66 Draw a parallelogram ABCD on a graph paper with the coordinates given in Table I. Use this table to complete Tables II and III to get the coordinates of E, F, G, H and J, K, L, M.
Image may be NSFW.
Clik here to view.
ncert-exemplar-problems-class-8-mathematics-introduction-to-graphs-2-46

Draw parallelograms EFGH and JKLM on the same graph paper. Plot the points (2, 4) and (4, 2) on a graph paper, then draw a line segment joining these two points.
Solution. Complete table is shown below
Image may be NSFW.
Clik here to view.
ncert-exemplar-problems-class-8-mathematics-introduction-to-graphs-2-47

Image may be NSFW.
Clik here to view.
ncert-exemplar-problems-class-8-mathematics-introduction-to-graphs-2-48

Question. 67 Extend the line segment on both sides to meet the coordinate axes. What are the coordinate of the points, where this line meets the X-axis and the y-axis?
Solution.
Let PQ is a line segment which is extened from both ends to meet the axes.
Image may be NSFW.
Clik here to view.
ncert-exemplar-problems-class-8-mathematics-introduction-to-graphs-2-49

The coordinates of the point on y-axis, where the line segment meet will be of form (0,y) whereas the
coordinates of the point of interaction on X-axis will be of type (x,0).

Question. 68 The following graph shows the change in temperature of a block of ice when heated. Use the graph to answer the following questions.
Image may be NSFW.
Clik here to view.
ncert-exemplar-problems-class-8-mathematics-introduction-to-graphs-2-50

(a) For how many seconds did the ice block have no change in temperature?
(b) For how long was there a change in temperature?
(c) After how many seconds of heating did the temperature become constant at 100°C?
(d) What was the temperature after 25 s?
(e) What will be the temperature after 1.5 min? Justify your answer.
Solution.
(a) In the first 20 s, the ice block have no change in temperature.
(b) There was a change in temperature from 20 s to 50 s, i.e. 50-20 = 30s.
(c) Observing the graph, we see that after 50 s of heating the temperature became constant.
(d) 20°C was the temperature after 25 s.
(e) Since, the temperature became constant at 100°C after 50 s heating, so the temperature will be 100°C even after 1.5 min.

Question. 69 The following graph shows the number of people present at a certain shop at different times. Observe the graph and answer the following questions.
Image may be NSFW.
Clik here to view.
ncert-exemplar-problems-class-8-mathematics-introduction-to-graphs-2-51

(a) What types of a graph is this?
(b) What information does the graph give?
(c) What is the busiest time of day at the shop?
(d) How many people enter the shop when it opens?
(e) About how many people are there in the shop at 1:30 PM?
Solution.
(a) This is a line graph.
(b) It represents the number of people, who visited the store at a particular time.
(c) The busiest time of day is 1 PM at a shop, as at this time maximum number of people i.e. 25 visited the shop.
(d) When it opens less than 5 people enter the shop.
(e) There are 20 people in the shop at 1:30 PM.

Question. 70 A man started his journey on his car from Location A and came back. The given
graph shows his position at different times during the whole journey.
Image may be NSFW.
Clik here to view.
ncert-exemplar-problems-class-8-mathematics-introduction-to-graphs-2-53

(a) At what time did he start and end his journey?
(b) What was the total duration of journey?
(c) Which journey, forward or return, was of longer duration?
(d) For how many hours did he not move?
(e) At what time did he have the faster speed?
Solution. Analysing the graph carefully, we observe that
(a) He started his journey at 5:30 AM and end at 6 PM.
(b) Total duration of journey was 12:30 h.
(c) His forward journey is of duration 8:30 h and return journey is of duration 4 h. Forward journey was of longer duration.
(d) He did not move from 6:30 AM to 9:30 AM and 10 AM to 1 PM.
So, he did not move for 6 h.
(e) He have the fastest speed at 1 PM. ”

Question. 71 The following graph shows the journey made by two cyclists, one from town A to B and the other from town B to A.
Image may be NSFW.
Clik here to view.
ncert-exemplar-problems-class-8-mathematics-introduction-to-graphs-2-54

(a) At what time did cyclist II rest? How long did the cyclist rest?
(b) Was cyclist II cycling faster or slower after the rest?
(c) At what time did the two cyclists meet?
(d) How far had cyclist II travelled when he met cyclist I?
(e) When cyclist II reached town A, how far was cyclist I from town 6?
Solution.
(a) On the basis of given graph, the cyclist II rest at 8 : 45 AM for 15 min.
(b) Cyclist II is cycling faster after rest as he has covered a distance of 20 km in 1 h.
(c) Both cyclists meet at 9:00 AM.
(d) The cyclist II had travelled 20 km, when he met cyclist I.
(e) When cyclist II reached town A, the cyclist I was 10 km for from town B.

Question. 72 Ajita starts off from home at 07.00 h with her father on a scooter that goes at a uniform speed of 30 km/h and drops her at her school after half an hour. She stays in the school till 13.30 h and takes an auto rickshaw to returrThome. The rickshaw has a uniform speed of 10 km/h. Draw the graph for the above situation and also determine the distance of Ajita’s school from her house.
Solution.
Image may be NSFW.
Clik here to view.
ncert-exemplar-problems-class-8-mathematics-introduction-to-graphs-2-55

Question. 73 Draw the line graph using suitable scale to show the annual gross profit of a company for a period of five years.
Image may be NSFW.
Clik here to view.
ncert-exemplar-problems-class-8-mathematics-introduction-to-graphs-2-56

Solution.
We have taken years on X-axis and gross profit on Y-axis. The line graph of an annual gross profit of a company for a period of five years are given below.
Image may be NSFW.
Clik here to view.
ncert-exemplar-problems-class-8-mathematics-introduction-to-graphs-2-57

Question. 74 The following chart gives the growth in height in terms of percentage of full height of boys and girls with their respective ages.
Image may be NSFW.
Clik here to view.
ncert-exemplar-problems-class-8-mathematics-introduction-to-graphs-2-58

Draw the line graph of above data on the same sheet and answer the following questions.
(a) In which year both the boys and the girls achieve their maximum height?
(b) Who grows faster at puberty (14 yr to 16 yr of age)?
Solution.
Image may be NSFW.
Clik here to view.
ncert-exemplar-problems-class-8-mathematics-introduction-to-graphs-2-59

(a) In 18 yr, the boys and in 17 yr, the girls achieve their maximum height. I
(b) Boys grows faster the girls during puberty.

Question. 75 The table shows the data collected for Dhruv’s walking on a road.
Image may be NSFW.
Clik here to view.
ncert-exemplar-problems-class-8-mathematics-introduction-to-graphs-2-60

(a) Plot a line graph for the given data using a suitable scale.
(b) In what time periods did Dhruv make the most progress?
Solution.(a)
Image may be NSFW.
Clik here to view.
ncert-exemplar-problems-class-8-mathematics-introduction-to-graphs-2-61

Question. 76 Observe the given graph carefully and complete the table given below.
Image may be NSFW.
Clik here to view.
ncert-exemplar-problems-class-8-mathematics-introduction-to-graphs-2-62

Solution.
Image may be NSFW.
Clik here to view.
ncert-exemplar-problems-class-8-mathematics-introduction-to-graphs-2-63

Question. 77 This graph shows the per cent of students who dropped out of school after completing high school. The point labelled A shown that, in 1996, about 4.7% of students dropped out.
Image may be NSFW.
Clik here to view.
ncert-exemplar-problems-class-8-mathematics-introduction-to-graphs-2-64

(a) In which year was the drop out the rate’highest? In which year was it the lowest?
(b) When did the per cent of students who dropped out of high school first fall below 5%?
(c) About what per cent of students dropped out of high school in 2007? About what per cent of students stayed in high school in 2008?
Solution. Observing the graph carefully, we have
(a) The drop out rate was the highest in the year 1990 and the least in 2000.
(b) In the year 1996, the per cent of students dropped out of high school first fall below 5%.
(c) About 4.7% students dropped out of high school in 2007.

Question. 78 Observe the toothpick pattern given below
Image may be NSFW.
Clik here to view.
ncert-exemplar-problems-class-8-mathematics-introduction-to-graphs-2-65

Solution.
Image may be NSFW.
Clik here to view.
ncert-exemplar-problems-class-8-mathematics-introduction-to-graphs-2-66

Image may be NSFW.
Clik here to view.
ncert-exemplar-problems-class-8-mathematics-introduction-to-graphs-2-67

Question. 79 Consider this in put/output table.
Image may be NSFW.
Clik here to view.
ncert-exemplar-problems-class-8-mathematics-introduction-to-graphs-2-68

Solution.
Image may be NSFW.
Clik here to view.
ncert-exemplar-problems-class-8-mathematics-introduction-to-graphs-2-69

Question. 80 This graph shows a map of an island just off the coast of a continent. The point labelled B represents a major city on the coast. The distance between grid lines represents 1 km.
Image may be NSFW.
Clik here to view.
ncert-exemplar-problems-class-8-mathematics-introduction-to-graphs-2-70

Point A represents a resorts that is located 5 km East and 3 km North of point B. The values 5 and 3 are the coordinates of point A. The coordinates can be given as the ordered pair (5,3), where 5 is the horizontal coordinate and 3 is the vertical coordinate.
(i) On a copy of the map, mark the point that is 3 km East and 5 km North of point and lebel it S. Is point S in the water or on the island? Is point 5 in,the same place as point A?
(ii)Mark the point that is 7 km east and 5 km north of point B and label it C. Then, mark the point that is 5 km east and 7 km north of point B and label it D. Are points C and D in the same place? Give the coordinates of points C and D.
(iii)Which points is in the water (2, 7) or (7, 2)? Mark the point which is in water otLyour map and Label it f.
(iv)Give the coordinates of two points on the island that are exactly 2 km apart from point A.
(v) Give the coordinates of the point that is halfway between points L and P.
(vi)List three points on the island with their x-coordinates greater than 8.
(vii)List three points on the island with a y-coordinate less than 4.
Solution.
Image may be NSFW.
Clik here to view.
ncert-exemplar-problems-class-8-mathematics-introduction-to-graphs-2-71

(i) The points is in the water.
No, it is not in the same place as point A
(ii)No, they are not in the same place. The coordinates of points C and D are (7, 5) and
(5, 7), respectively.
(iii)(2, 7) is in the water.
(iv)(7, 3), (5, 5)
(v) (8.5, 3)
(vi)(9,4), (10,4), (11,5)
(vii)(5, 3), (6, 2), (7, 2)
Note Answer for option (vi) and (vii) may vary from student to student

Question. 81 As part of his science project, Prithvi was supposed to record the temperature every hour one Saturday from 6 AM to midnight. At noon, he was taking lunch and forgot to record the temperature. At 8:00 PM, his favourite show came on and so forgot again. He recorded the data. So, collected on a graph sheet as shown below.
Image may be NSFW.
Clik here to view.
ncert-exemplar-problems-class-8-mathematics-introduction-to-graphs-2-72

(a) Why does it make sense to connect the points in the situation?
(b) Describe the overall trend, or pattern, in the way the temperature changes over the time period shown on the graph.
(c) Estimate the temperature at noon and 8 PM.
Solution.
(a) By connecting the points, it is easier to understand a change in the temperature.
(b) Initially the temperature was 8°C at 6 AM and started increasing strictly till 1 PM and after that it decreased to 8°C till 12 PM;
(c) At 12 PM 19°Candat8PM 10°C.

Question. 82 The graph given below compares the price (in ?) and weight of 6 bags (in kg) of sugar of different brands A, B, C, D, E, F.
Image may be NSFW.
Clik here to view.
ncert-exemplar-problems-class-8-mathematics-introduction-to-graphs-2-73

(a) Which brand(s) cost/costs more than brand 0?
(b) Bag of which brand of sugar
(c) Which brands weigh the same?
(d) Which brands are heavier than brand B?
(e) Which bag is the lightest?
(f) Which bags are of the same price?
Solution. On observing the graph carefully, we note that
(a) The brands E and F cost more than brand D.
(b) The bag of sugar of brand D is the heaviest.
(c) The weights of bag of brand S and F; brand E and C weighs same.
(d) Brands C, D, E are heavier than brand B.
(e) Bag of brand A is the lightest.
(f) Bags of brand A and C are of the same price.

Question. 83 The points on the graph below represent the height and weight of the donkey, dog, crocodile and ostrich shown in the drawing.
Image may be NSFW.
Clik here to view.
ncert-exemplar-problems-class-8-mathematics-introduction-to-graphs-2-74

(a) What are the two variables represented in the graph?
(b) Which point represents each animals? Explain.
Solution.
(a) Height and weight are the two variables in the graph.
(b) In the graph,we observe that the points A represents a crocodile as it has least height and greatest weight among all animals.
A-Crocodile ‘ [least height, greatest weight]
B – Donkey [height and weight more than dog]
C – Dog
D – Ostrich [greatest height]

Question. 84 The two graph below compare car A and car B. The left graph shows the relationship between age and value. The right graph shows the relationship between size and maximum speed.
Image may be NSFW.
Clik here to view.
ncert-exemplar-problems-class-8-mathematics-introduction-to-graphs-2-75

Use the graphs to determine whether each statement is true or false and explain your answer.
(a) The older car is less valuable.
(b) The faster car is larger.
(c) The larger car is older.
(d) The faster car is older.
(e) The more valuable car is slower.
Solution.
(a) False, the older car is 8 i.e. 8 valuable more than car A.
(b) True, in the second graph 8 is larger car having greater speed.
(c) True, larger car is 8 which is older than A
(d) True, as 8 is faster as well as older than A.
(e) False, as 8 is more valuable but not slower.

Question. 85 Sonal and Anmol made a sequence of the designs from square white tiles surrounding one square purple tile. The purple tiles come in many sizes. Three of the designs are shown below.
(a) Copy and complete the table
Image may be NSFW.
Clik here to view.
ncert-exemplar-problems-class-8-mathematics-introduction-to-graphs-2-83

(b) Draw a graph using the first five pairs of numbers in your table.
(c) Do the points lie on a line?
Solution.
(a) In side length 1 the number of white tile surrounding purple tile Is 4.
Similarly, in side length 2 the number of white tiles surrounding purple tile is 8.
Thus, we can arrange the following table which shows side length of purple corresponding to the number of white tiles in border.
Image may be NSFW.
Clik here to view.
ncert-exemplar-problems-class-8-mathematics-introduction-to-graphs-2-77

Image may be NSFW.
Clik here to view.
ncert-exemplar-problems-class-8-mathematics-introduction-to-graphs-2-78

Question. 86 Sonal and Anmol then made another sequence of the designs. Three of the designs are shown below.
Image may be NSFW.
Clik here to view.
ncert-exemplar-problems-class-8-mathematics-introduction-to-graphs-2-79

(b) Draw a graph of rows and number of white tiles. Draw another graph of the number of rows and the number of purple tiles. Put the number of rows on the horizontal axis.
(c) Which graph is linear?
Solution.
Image may be NSFW.
Clik here to view.
ncert-exemplar-problems-class-8-mathematics-introduction-to-graphs-2-80

Image may be NSFW.
Clik here to view.
ncert-exemplar-problems-class-8-mathematics-introduction-to-graphs-2-81

The post NCERT Exemplar Problems Class 8 Mathematics Introduction to Graphs appeared first on Learn CBSE.

Important Question for CBSE Class 9 Science Atoms and Molecules

Important  Question for CBSE Class 9 Science Chapter 3 Atoms and Molecules

IMPORTANT QUESTIONS

1 MARK QUESTIONS
Question. 1. Name the scientist who laid the foundation of chemical sciences. How ?
Answer. Antoine Laurent Lavoisier, by establishing two important laws of chemical combination.

Question. 2. Define law of conservation of mass.
Answer. It states that, ‘Mass is neither created nor destroyed in a chemical reaction.’ In other-words, the mass of the reactants must be equal to the mass of products.

Question. 3. Define law of constant proportion.
Answer. It states that, ‘In a pure chemical substance, the elements are always present in definite proportions by mass’.

Question. 4. Which organisation approves the names of elements all over the world ? Write the symbol of gold. [SA II-2014]
Answer. • International Union of Pure and Applied Chemistry (IUPAG). • Au.

Question. 5. Write the symbols of tungsten and iron.
Answer. (i) Tungsten (W) (ii) Iron (Fe).

Question. 6. Name the element which is used as the reference for atomic mass.
Answer. Carbon.

Question. 7. ‘Atoms of most elements are not able to exist independently’. Name two atoms which exist as independent atoms. [SA II-2012]
Answer. Noble gases such as argon (Ar), helium (He) exist as independent atoms.

Question. 8. What is the number of electrons in Mg atom and \({ Mg }^{ 2+ }\) ion ? [SA II-2014]
Answer. \(Mg:{ 12 }^{ e- }:{ Mg }^{ 2+ }:{ 10 }^{ e- }\)

Question. 9. Which of the following species is electrically neutral and why ? [SA II-2014]
Answer. \({ Na }^{ + },{ Cl }^{ – }{ K }^{ + };Na,{ Ca }^{ +2 }\)

Question. 10. Write atomicity of the following:
(i) Sulphur, (ii) Phosphorus [SA 11-2014]
Answer. (i) Polyatomic, (ii) Tetra atomic.

Question. 11. Define atomicity.
Answer. The number of atoms present in one molecule of an element or a compound is known as its atomicity.

Question. 12. What are polyatomic ions ? Give two examples.
Answer. A group of atoms having a charge is known as polyatomic ion.
Examples : \({ NH }_{ 4 }^{ 1+ },{ SO }_{ 4 }^{ 2- }\)

Question. 13. What is the atomicity of Argon ?
Answer. Mono atomic.

Question. 14. Give one relevant reason why scientists choose 1/16 of the mass of an atom of naturally occurring oxygen as the atomic mass unit.
Answer. Initially 1/16th of the mass of naturally occurring oxygen was taken as the atomic mass unit because this unit gave masses of most of the elements as whole numbers.

Question. 15. State the number of hydrogen atoms in 1 g of hydrogen.
Answer. One gram of hydrogen = One mole
=6.022 x \({ 10 }^{ 23 }\) atoms

Question. 16. What is molar mass ? What are its units ?
Answer. The mass of one mole of a substance is called its molar mass. Its unit is gram per mole (g \({ mol }^{ -1 }\)).

Question. 17. Define atomic mass unit.
Answer. One atomic mass unit is a mass unit equal to exactly one twelfth (1/12th) the mass of one atom of carbon -12.

Question. 18. The relative atomic mass of oxygen atom is 16. Explain its meaning.
Answer. The relative atomic mass of an atom is the average mass of the atom, as compared to 1/12th the mass of one carbon-12 atom.

Question. 19. Distinguish between molecular mass and molar mass.
Answer. The molecular mass of a substance is the sum of the atomic masses of all atoms in a molecule, whereas the mass of 1 mole of any substance is called its molar mass.

Question. 20. Which postulate of Dalton’s atomic theory is the result of the law of conservation of mass ? [SAII-2011]
Answer. Atoms are indivisible particles, which cannot be created or destroyed in a chemical reaction.

Question. 21.Which postulate of Dalton’s atomic theory can explain the law of definite proportions ? [SAII-2011]
Answer. Atoms combine in the ratio of small numbers to form compounds. In a compound, the relative number and kinds of atoms are constant.

Question. 22. Define the atomic mass unit. [SAII -2011]
Answer. The mass of 1/12 part of C-12 is equivalent to one atomic mass unit. Previous, it was denoted by symbol amu but now, these days it is denoted by symbol u.

Question. 23. Why is it not possible to see an atom with naked eyes ?
Answer. Because an atom is too small, i.e., the atomic radii of an atom is of the order \({ 10 }^{ -10 }\) m to \({ 10 }^{ -9 }\) m.

Question. 24. What is meant by the term chemical formula ?
Answer. The chemical formula of a compound is a symbolic representation of its composition and actual number of atoms in one molecule of a pure substance, may be an atom or a compound.

Question. 25. How many atoms are present in a (i) \({ H }_{ 2 }\)S molecule and (ii)\( { PO }_{ 4 }^{ 3- }\)ion ? [SAII – 2011]
Answer. (i) In \({ H }_{ 2 }\)S, 3 atoms are present (ii) In \( { PO }_{ 4 }^{ 3- }\), 5 atoms are present.

2 MARKS QUESTIONS
Question. 1. Name the scientists whose experimentation established laws of chemical combination. Name the laws also. [SAII-2014]
Answer. Experimentation done by Antoine Laurent Lavoisier and Joseph L Broust established two laws of chemical combination. These laws are :(i) Law of conservation of mass.
(ii) Law of constant proportions.

Question. 2. Give two drawbacks of Dalton’s atomic theory.
Answer. Drawbacks of Dalton’s Atomic Theory :
(i) According to modern theory, atom is not the ultimate indivisible particle of matter. Today, we know that atoms are divisible, i.e., they are themselves made-up of particles (protons, electrons, neutrons, etc.).
(ii) In case of isotopes of an element, the assumption that the atoms of the same element have same mass does not hold good.

Question. 3. Calculate molar mass of sulphuric acid.
Answer.
Image may be NSFW.
Clik here to view.
important-question-for-cbse-class-9-science-atoms-and-molecules-1

Question. 4. Give the electronic configuration of:
(i) Al atom and its ion
(ii) O atom and its ion
Answer.
Image may be NSFW.
Clik here to view.
important-question-for-cbse-class-9-science-atoms-and-molecules-2

Question. 5. Give one example each of (i) Monovalent cation (ii) Bivalent cation (iii) Monovalent anion (iv) Bivalent anion
Answer.
Image may be NSFW.
Clik here to view.
important-question-for-cbse-class-9-science-atoms-and-molecules-3

Question. 6. How would you differentiate between a molecule of element and a molecule of compound ? Write one example of each type. [SA II-2012]
Answer. Molecule of an element is made-up of only one kind of atoms, e.g., \({ O }_{ 2\quad },{ N }_{ 2 },{ F }_{ 2 },{ O }_{ 3 },{ P }_{ 4 }{ ,S }_{ 8 }\) etc. Molecule of a compound is made-up of two or more different kinds of atoms in a fixed ratio, e.g.,\( { H }_{ 2 }O,C{ S }_{ 2 },{ H }_{ 2 }S,N{ H }_{ 3 },C{ H }_{ 4 }\).

Question. 7. What is meant by a chemical formula ? Give examples. [SA II-2012]
Answer. A chemical formula of a compound shows its constituent elements and the number of atoms of each combining element.
e.g., Chemical formula of ammonia is \( N{ H }_{ 3 }\),water is \( { H }_{ 2 }O\),carbon dioxide is \(C{ O }_{ 2 }\)

Question. 8. What is meant by a molecule ? Give examples. [SA II-2012]
Answer. A molecule is the smallest particle of an element or a compound capable of independent existence under ordinary conditions. It shows all the properties of the substance. e.g., molecule of oxygen is \({ O }_{ 2 }\), ozone is \({ O }_{ 3 }\), phosphorus is \({ P }_{ 4 }\), sulphur is \({ S }_{ 8 }\), etc.

Question. 9. Define one mole, illustrate its relationship with Avogadro constant.
Answer. One mole of any species (atoms, molecules, ions or particles) is that quantity in number having a mass equal to its atomic or molecular mass in grams.
The number of particles (atoms, molecules or ions) present in 1 mole of any substance is fixed, with a value of 6.022 x \({ 10 }^{ 23 }\). This number is called Avogadro constant or Avogadro number.

Question. 10. Hydrogen and oxygen combine in the ratio of 1:8 by mass to form water. What mass of oxygen gas would be required to react completely with 3 g of hydrogen gas ? [SAII -2011]
Answer. 1 g of hydrogen reacts with oxygen = 8 g
3 g of hydrogen reacts with oxygen
= 8 x 3 g = 24 g

Question. 11. Write down the formula of : [SAII-2011]
(i) Sodium oxide (ii) Aluminium chloride
(iii) Sodium sulphide (iv) Magnesium hydroxide
Answer.
Image may be NSFW.
Clik here to view.
important-question-for-cbse-class-9-science-atoms-and-molecules-4

Question. 12. When 3.0 g of carbon is burnt in 8.00 g oxygen, 11.00 g of carbon dioxide is produced. What mass of carbon dioxide will be formed when 3.00 g of carbon is burnt in 50.00 g of oxygen ? Which law of chemical combination will govern your answer ? [SAII -2011]
Answer. 3 g of carbon produce carbon dioxide = 11 g.
The remaining oxygen 50 g – 8 g = 42 g does not take part in the reaction.
The law of definite proportion is governed by the above data.

Question. 13. What are polyatomic ions ? Give examples. [SAII -2011]
Answer. Polyatomic ions: Two or more different atoms unite to form a charged particle is called polyatomic ions.
Examples: \({{\left( P{{O}_{4}} \right)}^{3-}}\) Phosphate, \({{\left( N{{O}_{3}} \right)}^{-1}}\) Nitrate.

Question. 14. Give the names of the elements present in the following compounds:
(a) Quicklime (b) Hydrogen bromide
(c) Baking powder (d) Potassium sulphate.
Answer.
Image may be NSFW.
Clik here to view.
important-question-for-cbse-class-9-science-atoms-and-molecules-5

Question. 15. What is the mass of: [SAII -2011]
(a) 0.2 mole of oxygen atoms ?
(b) 0.5 mole of water molecules ?
Answer. (a) 1 mole of oxygen atoms = 1 x 16 – 16 g
0.2 mole of oxygen atoms – 16 g x 0.2 = 3.2 g
(b) 1 mole of water (\( { H }_{ 2 }O\)) molecules = 2 x 1 g + 1 x 16 g=18 g
0.5 mole of water (\( { H }_{ 2 }O\)) molecules – 18 g x 0.5 = 9.0 g

3 MARKS QUESTIONS
Question. 1. Write the chemical symbols of two elements:
(i) Which are formed from the first letter of the elements’ name ?
(ii) Whose names have been taken from the names of the elements in Latin ?
(iii) Which are formed from the first two letters of the elements’ name ?
Answer. (i) N (Nitrogen), F(Fluorine), I (Iodine), O (Oxygen) (any two)
(ii) Fe (Ferrum), Cu (Cuprum), Ag (Argentum), Au (Aurum) (any two)
(iii) Ca (Calcium), He (Helium), Al (Aluminium), Si (Silicon) (any two)

Question. 2. Write the correct symbols of the following elements which are written incorrectly. (i) FE (Iron) (ii) AL (Aluminium) (iii) CO (Cobalt) (iv) AG (silver) (v) NA (sodium) (vi) AR (Argon)
Answer. (i) Fe (ii) Al (iii) Co (iv) Ag (v) Na (vi) Ar

Question. 3. State two examples in each case and write their chemical formulae:
(a) Molecules having (me kind of atoms only.
(b) Molecules having two different kinds of atoms.
(c) Molecules having three different kinds of atoms.
Answer.
Image may be NSFW.
Clik here to view.
important-question-for-cbse-class-9-science-atoms-and-molecules-6

Question. 4. State what are ions. Write the formulae of two divalent cations and anions each.
Answer. Ions are charged particles of atoms or group of atoms Cations \({ Ba }^{ 2+ },{ Mg }^{ 2+ },{ Ca }^{ 2+ }\)
Anions : \({ O }^{ 2- },{ S }^{ 2- }\)

Question. 5. Classify the following compounds diatomic, triatomic and polyatomic molecules: [SAII-2014]
\(HCl,{ H }_{ 2 },{ H }_{ 2 }O,N{ H }_{ 3 },{ CH }_{ 3 }OH,PC{ l }_{ 5 }\)
Answer. Diatomic : \(HCl,{ H }_{ 2 }\)
Tri atomic : \({ H }_{ 2 }O\)
Polyatomic :\(N{ H }_{ 3 },{ CH }_{ 3 }OH,PC{ l }_{ 5 }\)

Question. 6. Write the names of the following compounds:
Image may be NSFW.
Clik here to view.
important-question-for-cbse-class-9-science-atoms-and-molecules-7

Answer. (a) Nickel sulphide, (b) Magnesium nitrate,
(c) Sodium sulphate, (d) Aluminium nitrate,
(e) Potassium phosphate, (f) Calcium nitride

Question. 7. Write the chemical names of the following compounds:
Image may be NSFW.
Clik here to view.
important-question-for-cbse-class-9-science-atoms-and-molecules-8

Answer. (a) Potassium sulphate, (b) Magnesium phosphate,
(c) Ammonium chloride, (d) Zinc sulphide,
(e) Sodium nitride, (f) Silver bromide.

Question. 8. In a reaction, 5.3 g of sodium carbonate reacted with 6 g of ethanoic acid. The products were 2.2 g of carbon dioxide, 0.9 g water and 8.2 g of sodium ethanoate. Show that these observations are in agreement with the law of conservation of mass, sodium carbonate + ethanoic acid —> sodium ethanoate + carbon dioxide + water.
Answer. Mass of reactants before experiment
= Mass of sodium carbonate + Mass of ethanoic acid
= 5.3 g + 6g = 11.3 g
Mass of products after experiment
= Mass of sodium ethanoate + Mass of carbon dioxide + Mass of water
=8.2 g + 2.2 g +0.9 g = 11.3 g
The mass of reactants is equal to the mass of products, therefore, it proves law of conservation of mass.

Question. 9. Write down the names of compounds represented by the following formulae: [SAII -2011]
Image may be NSFW.
Clik here to view.
important-question-for-cbse-class-9-science-atoms-and-molecules-9

Answer.
Image may be NSFW.
Clik here to view.
important-question-for-cbse-class-9-science-atoms-and-molecules-10

Image may be NSFW.
Clik here to view.
important-question-for-cbse-class-9-science-atoms-and-molecules-11

Question. 10. Calculate the formula unit masses of ZnO, \({ Na }_{ 2 }\)O, \({ K }_{ 2 }\)C\({ O }_{ 3 }\), given atomic masses of Zn = 65u, Na = 23u, K = 39 u, C =3 12 u, and O = 16 u. [SAII – 2015]
Answer.
Image may be NSFW.
Clik here to view.
important-question-for-cbse-class-9-science-atoms-and-molecules-12

Question. 11. If one mole of carbon atoms weighs 12 gram, what is the mass (in gram) of 1 atom of carbon? [SAII-2011]
Answer.
Image may be NSFW.
Clik here to view.
important-question-for-cbse-class-9-science-atoms-and-molecules-13

Question. 12. A 0.24 g sample of compound of oxygen and boron was found by analysis to contain 0.096 g of boron and 0.144 g of oxygen. Calculate the percentage composition of the compound by weight.
Answer.
Image may be NSFW.
Clik here to view.
important-question-for-cbse-class-9-science-atoms-and-molecules-14

The post Important Question for CBSE Class 9 Science Atoms and Molecules appeared first on Learn CBSE.

CBSE Sample Papers for Class 12 Economics Outside Delhi – 2009

CBSE Sample Papers for Class 12 Economics Outside Delhi – 2009

Time allowed : 3  hours                                                                                         Maximum marks 100

GENERAL INSTRUCTIONS
(i) All questions in both the sections are compulsory.
(ii) Marks for questions are indicated against each.
(iii) Questions No. 1-5 and 17-21 are very short-answer questions carrying 1 nick each. They are required to be answered in one sentence each.
(iv) Questions No. 6-10 and 22-26 are? short-answer questions carrying 3 marks each. Answers to them should normally not exceed 60 words each.
(v) Questions No. 11-13 and 27-29 are also short-answer questions carrying 4 marks each. Answers to them should normally not exceed 70 words each.
(vi) Questions No. 14-16 and 30-32 are long-answers questions carrying 6 marks each. Answers to them should normally not exceed 100 words each.
(vii) Answers should be brief and to the point and the above word limit should be adhered to as far as possible.

SET I

SECTION A

Question.l. Give the meaning of opportunity cost.
Answer. Opportunity cost is the value of benefit that is foregone by choosing one alternative rather than, the other.

Question.2. What is meant by inferior good in economics?
Answer. Inferior good is that good whose demand decreases with an increase in the income of the buyer.

Question.3. Define marginal cost.
Answer. Marginal cost is defined as addition to the total cost for producing one more unit of output

Question.4. Give one reason for a rightward shift in supply Curve.
Answer. Decrease in input prices can lead to a rightward shift in supply curve.

Question.5. Why is average total cos| greater than average variable cost?
Answer. Average total cost is the sum of average fixed cost and average variable cost, therefore, average total cost is greater than average variable cost (ATC = AFC + AVC).

Question.6. State the law of demand and show it with the help of a schedule.
Answer. According to the law of demand, “Other things being constant, quantity demanded of a commodity is inversely related to the price of the commodity.”
It can be explained with the help of a demand schedule given below:
The above schedule shows an increase in quantity demanded from 2 kg to 3 kg and then 4 kg when price reduces from Rs. 20/kg to Rs. 16/kg and then to Rs. 12/kg respectively.
Image may be NSFW.
Clik here to view.
cbse-sample-papers-for-class-12-economics-outside-delhi-2009-1

Question.7. ‘Explain the geometric method of measuring price elasticity of demand.
Answer. Geometric method (or point method) is used when elasticity is to be measured at different points on the straight line demand curve. According to this method, elasticity. can be measured using the following formula:
\({ E }_{ d }=\ frac { Lower Section of the demand curve }{ Upper Section of the demand curve } \)
This can also be explained by a diagram.
Image may be NSFW.
Clik here to view.
cbse-sample-papers-for-class-12-economics-outside-delhi-2009-3

AE is the straight line demand curve which is extended to both sides so as to touch X-axis and Y-axis. According to this method,
Image may be NSFW.
Clik here to view.
cbse-sample-papers-for-class-12-economics-outside-delhi-2009-2

Question.8. Why do problems related to allocation of resources in an economy arise? Explain.
Answer. Problems relating to allocation of resources arise because of the problem of choice. This happens because the resources are scarce.
There are three mam causes of this economic problem:
(0 Unlimited wants—Human wants are unlimited and differ in intensity. This fact enables a man to arrange his wants in order of preference and make a choice among different wants in order of urgency.
(ii) Limited resources—Resources, generally called as factors of production (land, labour, capital and enterprise) are scarce in relation to need for them. In view of resources being limited, there is great need for economising of resources.
(iii) Alternative uses of resources–This means that resources can be put to different uses. But the same resource cannot be used for more than one purpose. This makes a resource all the more scarce because when it is used for one purpose, it cannot be used for other purposes.
Or
Explain the problem of ‘for whom to produce.
Answer. The problem ‘for whom to produce’ is a central problem which refers to the problem of distribution of income among income earners. It is the problem of how goods and services produced in the economy be distributed. Goods and services are produced for those who have the capacity to buy them. Capacity to buy depends on income or purchasing power. Incomes are distributed in the form of wages, rent, interest and profits. Thus, the question ‘for whom to produce’ actually boils down to the distribution of income generated from production of output.

Question.9. Complete the following table:
Image may be NSFW.
Clik here to view.
cbse-sample-papers-for-class-12-economics-outside-delhi-2009-4

Answer.
Image may be NSFW.
Clik here to view.
cbse-sample-papers-for-class-12-economics-outside-delhi-2009-5

Question.10. Explain the effect of fall in prices of other goods on the supply of a given good.
Answer. Besides its own price, supply of a good also depends upon prices of other goods! Increase in the prices of other goods makes them more profitable in comparison to the given commodity. As a result, the firm shifts its limited resources from production of the given commodity to production of other goods, e.g., increase in the price of other good (wheat) will induce the farmer to use land for cultivation of wheat in place of the given commodity (rice).

Question.11. Explain two points of distinction between monopoly and monopolistic competition.
Answer.
Image may be NSFW.
Clik here to view.
cbse-sample-papers-for-class-12-economics-outside-delhi-2009-6

Explain any two main features of perfect competition.
Answer.
Two main features of perfect competition are:
(i) Large number of buyers and sellers. The number of sellers is so large that output by an industrial seller is an insignificant proportion of the total output of the industry; As such, change in the output of a single seller has negligible effect on the total output and hence no influence on the market price.
(ii) Homogeneous product. All sellers sell identical units of a product. Accordingly, buyers have no reason to prefer the product of one seller compared to that of the other. Also, no producer is in a position to charge a different price for (he product it produces.

Question.12. The price elasticity of supply of commodity Y is half the price elasticity of supply of commodity X. 16 per cent rise in the price of X results in a 40 per cent rise in its supply. If the price of Y falls hf 8 per cent, calculate the percentage fall in its supply.
Answer.
Image may be NSFW.
Clik here to view.
cbse-sample-papers-for-class-12-economics-outside-delhi-2009-7

Question.13. Given below is a cost and revenue schedule of a produce. At what level of output is the producer in equilibrium? Give reasons for your answer.
Image may be NSFW.
Clik here to view.
cbse-sample-papers-for-class-12-economics-outside-delhi-2009-8

Answer.
Image may be NSFW.
Clik here to view.
cbse-sample-papers-for-class-12-economics-outside-delhi-2009-9

Producer is in equilibrium at the 6th unit of output. Profit is maximum at both 5th and 6th unit of output. But it is only at the 6th unit that MR = MC = 10.
Therefore, producer is in equilibrium at 6th unit of output.

Question.14. With the help of a demand and supply schedule, explain the meaning of excess demand and its effect on price of a commodity.
Answer. Equilibrium price of a product is determined at the point where market demand equals market supply. This can be better explained with the help of a supply schedule.
Image may be NSFW.
Clik here to view.
cbse-sample-papers-for-class-12-economics-outside-delhi-2009-10

In the schedule given above, equilibrium price is Rs. 3 and equilibrium quantity is 300 units. At prices Rs. 1 and Rs. 2. demand is greater than supply, creating a situation of excess demand, i. e., a situation where demand is greater than price. As a result, the consumer will not be able to get the required quantity. Hence, the consumers will start offering a higher price to producers. Therefore, the price will rise and ultimately settle at Rs.3.
Thus equilibrium will be restored through expansion of supply and contraction of demand as a result of increase in price.
Or
Define equilibrium price of a commodity. How is it determined? Explain with the help of a schedule.
Answer. Equilibrium price of a good is the price at which the market supply of the good equals the market demand for that good. Market supply is the total supply by all the producers of a good taken together. Market demand represents the total demand by all the consumers of that good taken together.
This can be better explained with the help of the following schedule and Diagram 2 .
Image may be NSFW.
Clik here to view.
cbse-sample-papers-for-class-12-economics-outside-delhi-2009-11

Image may be NSFW.
Clik here to view.
cbse-sample-papers-for-class-12-economics-outside-delhi-2009-12
In the above table, equilibrium price is Rs. 3 and equilibrium quantity is 30 units. At any other price, market demand and market supply are not equal.
This equilibrium is also represented by Diagram 2. Equilibrium price OP and quantity OQ are determined at the intersection point of demand curve (DD) and supply curve (SS) i.e. at point E.

Question.15. Giving reasons, state whether the following statements are true or false:
(i) Average cost falls only when marginal cost falls.
(ii) The difference between average total cost and average variable cost is constant
(iii) When total revenue is maximum, marginal revenue is also maximum.
Answer. (i) False. AC can fall even when MC is rising. This is shown in the diagram (i) Eor MN level of output, AC is falling and MC is rising.
(ii) False. The difference between ATC and AVC is AFC and AFC declines with increase in output. Therefore, the difference between ATC and AVC must reduce as output increases. See diagram (ii).
(iii). False. Because MR is zero when TR is maximum. See diagram (iii).
Image may be NSFW.
Clik here to view.
cbse-sample-papers-for-class-12-economics-outside-delhi-2009-13

Question.16. Explain the effect of the following on the market demand of a commodity:
(i) Change in price of related goods
(ii) Change in the number of its buyers
Answer. (i) Change in price of related goods. Related goods are Y of two types:
Image may be NSFW.
Clik here to view.
cbse-sample-papers-for-class-12-economics-outside-delhi-2009-14

(a) Substitute goods. These are those goods which can be used in place of one another, e.g., tea and g p coffee. Increase in the price of tea will lead to an increase in the demand of coffee as coffee will now become cheaper in comparison to its substitute (tea).
This will lead to a rightward shift in demand curve signifying that more coffee is
(b) Complementary goods—:these are those goods which are used together to satisfy a given want, e.g. scooter and petrol. A fall in the price of a complementary good (here petrbl) causes an increase in the demand for the other good (here scooter) and shifts the demand curve for the good to the right.
(ii) Change in the number of buyers. Demand will increase with the increase in the number of buyers and vice versa. Other things remaining constant, mote buyers would mean more demand and less buyers would mean less demand for a commodity.

SECTION B
Question.17. Give meaning of aggregate supply.
Answers. Aggregate supply means the total value of final goods and services available for purchase by the economy during a given period.

Question.18. Why are taxes received by the government not capital receipts?
Answer. Taxes are not considered capital receipts because these neither create a liability nor do they lead to a reduction in the assets of the government.

Question.19. Give die meaning of excess demand in an economy.
Answer. It is a situation when aggregate demand is in excess of aggregate supply at the full employment level in the economy.

Question.20. What is meant by cash reserve ratio?
Answer. It refers to the percentage of deposits of the commercial banks which is required to be kept as cash reserve with the central bank.

Question.21. Define involuntary unemployment.
Answer. It is a situation in which a worker i§ able and willing to work at the current wage rate but does not get work.

Question.22. Complete the following fable:
Image may be NSFW.
Clik here to view.
cbse-sample-papers-for-class-12-economics-outside-delhi-2009-15

Answer.
Image may be NSFW.
Clik here to view.
cbse-sample-papers-for-class-12-economics-outside-delhi-2009-16

Question.23. Give the meai-ing of factor income to abroad and factor income from abroad. Also give an example of each.
Answer. Factor income to abroad is the factor income earned by non-residents who are temporarily residing in our country or are working within our domestic territory, e.g. salaries paid to residents of Japan working in Indian Embassy in Japan.
Factor income from abroad is the factor income earned by normal residents of our country who are temporarily residing abroad or are working outside our domestic territory, e.g. profits earned by an Indian bank from its branches abroad.
Or
Distinguish between domestic product and national product. When can domestic product be more than national product?
Answer.
Image may be NSFW.
Clik here to view.
cbse-sample-papers-for-class-12-economics-outside-delhi-2009-17

Domestic product can be more than national .product, when net factor income earned from abroad is negative.

Question.24. Distinguish between balance of trade account and balance of current account.
Answer.
Image may be NSFW.
Clik here to view.
cbse-sample-papers-for-class-12-economics-outside-delhi-2009-18

Question.25. Give three main functions of a commercial, bank. Explain any one of them.
Answer. The three main functions of a commercial bank are:
1. Accepting deposits
2. Advancing loans
3. Agency functions like purchase and sale of securities and shares, transfer of funds, etc. Advancing of loans: Extending loans is another important function of commercial banks. The deposits received by banks are not allowed to remain idle. So, after keeping certain cash reserves, the balance is given to needy borrowers and interest is charged from them. Loans and advances can be made in different terms like cash-credit, short-term loans, etc.

Question.26. Give the meaning of revenue deficit, fiscal deficit and primary deficit.
Answer. Revenue deficijt„ lt refers to the excess of the government’s revenue expenditure over its revenue receipts during the given fiscal year.
Image may be NSFW.
Clik here to view.
cbse-sample-papers-for-class-12-economics-outside-delhi-2009-19

Revenue Deficit Revenue Expenditure — Revenue Receipts Fiscal deficit. It refers to the excess of the government’s total expenditure over its total receipts (excluding borrowings) during the given fiscal year.
Image may be NSFW.
Clik here to view.
cbse-sample-papers-for-class-12-economics-outside-delhi-2009-20

Fiscal Deficit = Total Expenditure – Total Receipts excluding borrowings The extent of fiscal deficit is an indication of how far the government is spending beyond its means.
Primary Deficit. It refers to the difference between fiscal deficit of the current year and interest payments on the previous borrowings.
Image may be NSFW.
Clik here to view.
cbse-sample-papers-for-class-12-economics-outside-delhi-2009-21

Primary Deficit reflects the extent to which interest commitments on earlier loans have compelled the government to borrow in the current period.

Question.27. Describe the evolution of money.
Answer. (Out of syllabus, for 2012 examination and onwords)
Or
Explain any two functions of money.
Answer. The two functions of money are as follows:
1. Medium of exchange. This means that it can be used to make- payments for all transactions of goods and services. Money has the quality of general acceptability. This function has removed the difficulty of double coincidence of wants.
2. Measure of Value. Since money has general acceptability, it works as a common denomination in.which values of all goods and services are expressed. After knowing the values of various commodities in tends of a single unit called price, it becomes very easy to find out the exchange ratios between them.

Question.28. Explain any two objectives of a government budget.
Answer. The two objectives of a government budget are:
1. Reallocation of resources. Through the budgetary policy, government aims to reallocate resources in accordance with the economic (profit maximisation) and Social (public welfare) priorities of the country. For example, government discourages the production of harmful goods like cigarettes, etc. through heavy taxes and encourages the use of khadi products by providing subsidies.
2. Management of public enterprises. There is a large number of public sector industries which are established and managed for social welfare of the public. Budget is prepared with the objective of jnakiftg various provisions for managing such enterprises and providing them financial help.

Question.29. Explain two merits each of flexible foreign exchange rate and fixed foreign exchange rate.
Answer. Two merits of flexible exchange rate:
1. It eliminates the need for the government to hold large foreign exchange reserves.
2. It enhances efficiency and resource allocation, and raises the level of efficiency in the economy
Two merits of fixed exchange rate are:
1. It creates conditions for smooth flow of foreign capital between nations. A stable exchange rate promotes international trade.
2. As exchange rate is fixed by the government it eliminates the possibility of speculative transactions in foreign exchange.

Question.30. While estimating National Income, how will you treat the following? Give reasons for your answer.
(i) Imputed rent of self occupied houses.
(ii) Interest received on debentures.
(iii)Financial help received by flood victims.
Answer. (i) Imputed rent of self-occupied house is included in the estimation of National Income.
This is because people who live in their own houses, do not pay rent but enjoy housing services similar to those people who stay in rented houses.
(ii) Interest received on debentures is included in national income because it is a part of factor income.
(iii) Financial help received by flood victims is a transfer income and therefore will not be included in the estimation of national income.

Question.31. In an economy S = -50 + 0.5 Y is the saving function (where S = saving and Y = National Income) and investment expenditure is 7,000. Calculate:
(i) Equilibrium level of National Income.
(ii) Consumption expenditure at equilibrium level of national income.
Answer.
Image may be NSFW.
Clik here to view.
cbse-sample-papers-for-class-12-economics-outside-delhi-2009-22

(or)
Consumption function: C = 200 + 0.9 Y (where C = Consumption expenditure and Y = National Income).
Investment expenditure: I = 3000
Answer.
Image may be NSFW.
Clik here to view.
cbse-sample-papers-for-class-12-economics-outside-delhi-2009-23

Question.32. Frorit the fdll&Wihg data, calculate “National Income” by (a) income method and (b) expenditure method:
Image may be NSFW.
Clik here to view.
cbse-sample-papers-for-class-12-economics-outside-delhi-2009-24

Answer. (a) National Income by Income Method
= Compensation of employees + Interest + Rent + Profits – Net factor income to abroad = (vi) + (i) + (ii) + (v) – (vii)
= 1,000 + 150 + 250 + 640 – 30 = 2,040 – 30 = Rs. 2,010 crores
Answer. (b) National Income by Expenditure Method
= Private final consumption expenditure + .Government final consumption expenditure + Net domestic capital formation + Net exports – Net indirect.taxes – Net factor income to abroad
(iv) + (iii) + (xi) + (ix) – (viii) – (vii)
= 1,200 + 600 + 340 + (- 40) – 60 – 30 = 2140 – 130 = Rs. 2,010 crores

SET II

Note : Except for the following questions, atl the remaining questions have been asked in Set I.
SECTION A
Question.3. Define fixed costs.
Answer. Fixed costs are those costs which remain constant, i.e., which do not change with a change in the volume of output, e.g., salary of-naanager.

Question.6. Distinguish between demand by an individual and market demand with the help of a schedule.
Answer. Individual demand is the demand for a commodity by an individual consumer in the market at different prices, whereas market demand shows demand for a commodity by all the consumers in the market at different prices. Market demand is the horizontal summation of individual demands as under:
Image may be NSFW.
Clik here to view.
cbse-sample-papers-for-class-12-economics-outside-delhi-2009-25

The above schedule shows die individual demand of consumer A mid B along with the market demand, which is a horizontal summation of these two individual consumer demands, at different prices.

Question.9.Complete the following table:
Image may be NSFW.
Clik here to view.
cbse-sample-papers-for-class-12-economics-outside-delhi-2009-26

Answer.
Image may be NSFW.
Clik here to view.
cbse-sample-papers-for-class-12-economics-outside-delhi-2009-27

Question.13. The price of commodity X is Rs. 20 per unit and it remains constant. Given below is the cost schedule of one of its producers. Find out the level of output at which this producer is in equilibrium. Give reasons for your answer.
Image may be NSFW.
Clik here to view.
cbse-sample-papers-for-class-12-economics-outside-delhi-2009-28

Answer.
Image may be NSFW.
Clik here to view.
cbse-sample-papers-for-class-12-economics-outside-delhi-2009-29

At 5th and 6th units of output, the difference between total revenue and total cost, i.e., profit is the maximum. But MR = MC (20) only at the 6th unit of output. So, the producer will be in equilibrium at the 6th unit of output.

Question.15. Giving reasons, state Whether the following statements are true or false:
(i) When there are diminishing returns to a factor, marginal product and total product both always diminish.
(ii) When marginal revenue is positive and constant, average and total revenue will both increase at constant rate.
(iii) As output is increased^ the difference between average total cost and average variable cost falls and ultimately becomes zero.
Answer. (i)False, because when there are diminishing returns to a factor, then marginal product falls but total product increases at a diminishing rate,
(ii) False, because when Marginal Revenue is positive and constant, Total Revenue increase at constant rate but Average Revenue will not increase at all. Average Revenue will remain positive and constant just like Marginal Revenue.
(iii) False, because as output increases, the difference between average total cost and average variable cost (equal to average fixed cost) falls but remains positive. It can never be zero.
correct position of these statements is shown below in the diagrams:
Image may be NSFW.
Clik here to view.
cbse-sample-papers-for-class-12-economics-outside-delhi-2009-30

Question.22. Complete the following table:
Image may be NSFW.
Clik here to view.
cbse-sample-papers-for-class-12-economics-outside-delhi-2009-31

Answer.
Image may be NSFW.
Clik here to view.
cbse-sample-papers-for-class-12-economics-outside-delhi-2009-32

Question.29. How is foreign exchange rate determined in the market?
Answer. The equilibrium exchange rate is determined at a point when the demand for and supply of foreign currency is equal. As shown in the Diagram, demand curve DD and supply curve SS intersect at point E (equilibrium point) where exchange rate is OR. If the exchange rate rises to ORv it will lead to a situation of excess supply. As a result, exchange rate will fall till it again reaches the equilibrium level of OR. On the contrary, if the exchange rate falls to \({ OR }_{ 1 }\) it will lead to a situation of excess supply. As a result, exchange rate will fall till it again reaches the equilibrium level of OR. On the contrary, if the exchange rate falls to \({ OR }_{ 1 }\) , it will create a situation of excess demaqjd and will push , up the exchange rate till it reaches the equilibrium level of OR.
Image may be NSFW.
Clik here to view.
cbse-sample-papers-for-class-12-economics-outside-delhi-2009-33

Question.30. How will you treat the following while estimating National Income? Give reasons for your answer.
(i) Capital gain on sale of a house
(ii) Prize won in a lottery
(iii) Interest on public debt
Answer. (i) Capital gain on the sale of a house is not included in National lncome because it does not add to the flow of goods and services of the economy.
(ii) Prize won in a lottery is a transfer income and therefore riot included in National Income.
(iii) Interest on public debt is not included in National Income because interest on consumption5 loans is considered as a transfer payment.

SET III

Note: Except for the following questions, all the remaining questions have been asked in Set I and Set II.
SECTION A
Question.7. Explain any two factors that affect the price elasticity of demand of a commodity.
Answer. Two factors affecting price elasticity of demand are as follows:
(i) Nature-of the commodity. The demand for necessities is generally inelastic because they will be demanded at any price, whereas the demand for luxury goods is generally elastic because they can be dispensed with easily if they appear to be costly.
(ii) Availability of close substitutes. The demand for a commodity will be elastic if some other commodity can be used in its place, whereas commodities with no close substitutes have inelastic demand. For example, salt.

Question.9. Complete the fallowing table:
Image may be NSFW.
Clik here to view.
cbse-sample-papers-for-class-12-economics-outside-delhi-2009-34

Answer.
Image may be NSFW.
Clik here to view.
cbse-sample-papers-for-class-12-economics-outside-delhi-2009-35

Question.13. Explain the eruditions of producer’s equilibrium with the help of a total cost and total revenue schedule.
Answer. A producer will be at equilibrium at that level of output which gives him maximum profit. Total Profit is the excess of Total Revenue (TR) over Total Cost (TC). Equilibrium is at 4 units of output where profit (TR – TC) is maximised.
Image may be NSFW.
Clik here to view.
cbse-sample-papers-for-class-12-economics-outside-delhi-2009-36

Question.14. State the causes of an ‘increase’ in demand. Explain any two of them.
Answer. The main causes of increase in demand of a commodity are:
(i) Increase in income of the consumer in case of normal good.
(ii) Decrease- in income of the consumer in case of inferior good.
(iii) Rise in prices of substitute goods.
(iv) Fall in prices of complementary goods.
(v) Favourable change in tastes and preferences of the consumer
(i) Income of the consumer. In case of normal goods, when income increases, there is an increase in demand leading to a rightward shift in the demand curve. On the other hand, in case of inferior goods, demand increases only when income of the consumer decreases.
(v) Favourable change in the tastes of consumer. Unless the consumer has a taste for a good, he is not likely to buy it, howsoever cheap it may be. Therefore, the demand for a commodity, towards which the consumer is favourably inclined, will increase.

Question.22. Complete the following table:
Image may be NSFW.
Clik here to view.
cbse-sample-papers-for-class-12-economics-outside-delhi-2009-37

Answer.
Image may be NSFW.
Clik here to view.
cbse-sample-papers-for-class-12-economics-outside-delhi-2009-38

Question.25. Explain any two functions of a central bank.
Answer. Two functions of a central bank are as follows:
1. Bank of issue. This means that the Reserve Bank of India has the sole right to issue currency notes. The main reasons as to why the monopoly power of note issue is given to central banks is:
– It leads to uniformity in notes circulation.
– It gives the central bank direct control over the money supply.
– It ensures public faith in the currency system.
2. Banker to the government. Central bank is the banker, agent and financial advisor to die government. As a banker to the government, it carries out all banking business of the government. As an agent to the government, it buys and sells securities on behalf of the government. As a financial advisor, the central bank advises the government from time to time on economic, financial and monetary matters.

Question.27. What are the implications of revenue deficit? State two measures to reduce this deficit?
Answer. Revenue deficit refers to the excess of government revenue expenditure over its revenue receipts during the given fiscal year.
Revenue Deficit = Total Revenue Expenditure – Total Revenue Receipts Implications. Revenue deficit indicates that the government’s own revenue is insufficient to meet its regular and recurring expenditure on normal functioning of its departments and various services. It also implies that die government has to make up this deficit from capital receipts, i.e., through borrowings or disinvestments. It means revenue deficit either leads to increase in liabilities or reduces the assets.
Measure to reduce revenue deficit:
1. Reduce expenditure. Government should take steps to reduce and avoid unproductive . and unnecessary expenditure.
2. Increase revenue. Government should increase its receipts from various sources of tax and non-tax revenue

The post CBSE Sample Papers for Class 12 Economics Outside Delhi – 2009 appeared first on Learn CBSE.

Important Question for CBSE Class 9 Science Is Matter Around us Pure

Important Question for CBSE Class 9 Science Chapter 2 Is Matter Around us Pure

IMPORTANT QUESTIONS

1 MARK QUESTIONS
Question.1
Define atomic number. How it is denoted ?
Answer.
Number of protons of an atom determines its atomic number. It is denoted by ‘Z’.

Question.2
What are nucleons ?
Answer.
Protons and neutrons present in the nucleus of an atom are called nucleons.

Question.3
Which study led to the conclusion that atoms are not indivisible ?
Answer.
Study of static electricity and the condition under which electricity is conducted by different substances.

Question.4
Name the particles which determine the mass of an atom. [SAII – 2014]
Answer.Proton and neutron.

Question.5
Electron attributes negative charge, protons attribute positive charge. An atom has both but why there is no charge ? [SAII- 2014]
Answer.
The negative and positive charges of electrons and protons respectively are equal in magnitude. So, the atom as a whole is electrically neutral.

Question.6
Write the charge and mass of an electron. [SAII – 2014]
Answer.
Its mass is 1/2000 mes that of proton and it is negatively charged.

Question.7
What type of charge is present on the nucleus of an atom ?
Answer. Positive charge.

Question.8
Name one element, the nucleus of which does not have any neutron.
Answer. Hydrogen.

Question.9
Who discovered neutron ?
Answer. James Chadwick.

Question.10
What is the charge and mass of alpha particle ?
Answer.
Charge = + 2 units
Mass = 4 units

Question.11
Why did Rutherford select a gold foil for his experiment ? [SAII – 2014]
Answer.
Rutherford selected a gold foil for his experiment as he wanted a very thin layer and gold is highly malleable.

Question.12
An uncharged particle is found in the nucleus of an atom. Identify this uncharged particle.
[SAII -2014]
Answer. Neutron.

Question.13
Name the scientist who concluded That the size of nucleus is very small as compared to the size of atom.
Answer. Rutherford.

Question.14
Write the names of three elementary particles which constitute an atom.
Answer.
Electron, proton and neutron.

Question.15
Helium atom has two electrons in its valence shell but its valency is not two. Explain.
[SAII -2014]
Answer.
Since its outermost-shell is completely filled, its combining capacity or valency is zero.

Question.16
What do you understand by an octet of electrons in the valence shell ?
Answer.
When the outermost-shell of an atom of an element Has eight electrons, it is said to possess an octet.

Question.17
The sulphate of aluminium has a chemical formula AI2(S04)3. State the valency of aluminium.
[SAII-2014]
Answer.3.

Question.18
Write the symbols of two isotopes of uranium.
Answer.
Image may be NSFW.
Clik here to view.
important-question-for-cbse-class-9-science-is-matter-around-us-pure-1

Question.19
Write any two observations which support the fact that atoms are divisible.
Answer.
Discovery of electrons and protons.

Question.20
If an atom contains one electron and one proton, will it carry any charge or not ?
[SAII-2011]
Answer.
It will not carry any charge because the positive charge on the proton neutralise the negative charge on the electron.

Question.21
On the basis of Rutherford’s model of an atom, which subatomic particle is present in the nucleus of an atom?
Answer.
The positively charged ‘Protons’.

2 MARKS QUESTIONS
Question.1.
Describe briefly Thomson’s model of an atom. [SAII – 2014]
Answer.
Thomson’s model of an atom :
(i) An atom consists of a positively charged sphere and the electrons are embedded in it.
(ii) The negative and positive charges are equal in magnitude. So the atom as a whole is electrically neutral.

Question.2
(a) (i) Name the scientist who discovered neutrons.
(ii) State the charge and mass of a neutron.
(iii)Where is neutron located in an atom ?
(b) Helium atom has an atomic mass of 4 u and two protons in its nucleus. How many neutrons does it have? [SAII – 2014]
Answer.
(a). (i) J. Chadwick.
(ii) Neutron has no charge and its mass is equal to that of a proton.
(iii) It is located in the nucleus of an atom.
(b) Two.

Question.3
Mention two uses of isotopes in the field of medicine.
Answer.
(i) An isotope of uranium is used as a fuel in nuclear reactors.
(ii) An isotope of cobalt is used in the treatment of cancer.
(iii)An isotope of iodine is used in the treatment of goitre.

Question.4
Write two differences between isobars and isotopes.
Answer.
Image may be NSFW.
Clik here to view.
important-question-for-cbse-class-9-science-is-matter-around-us-pure-2 - Copy

Question.5
Give four characteristics of Isotopes.
Answer.
(i) All isotopes of an element consist of the same number of protons inside their nuclei. Hence, they have the same atomic number.
(ii) All isotopes of an element consist of different number of neutrons in their nuclei. Hence, they have different mass number.
(iii) All isotopes of an element give identical chemical reactions.
(iv) Isotopes of an element have same electronic configuration.

Question.6
Give drawbacks of Thomson’s model.
Answer.
Drawbacks of Thomson’s Model:
(i) It could not explain about the stability of an atom, i.e., how both positive and negative charges could remain so close together.
(ii) It could not explain the results of experiments (such as alpha ray scattering experiment) carried out by other scientists.

Question.7
Draw a sketch of Bohr’s model of an atom with three shells. [SAII – 2017]
Answer.
Image may be NSFW.
Clik here to view.
important-question-for-cbse-class-9-science-is-matter-around-us-pure-3 - Copy

Question.8
What do you think would be the observation if the a-particle scattering experiment is carried out using a foil of a metal other than gold ?
Answer.
If experiment is carried out by using a foil of some other metal, then results would not have been such as most of the oc-particles will be deflected back. Gold can be beaten to an extremely thin sheet as it is extremely malleable metal. With the use of extremely thin gold foil, Rutherford could predict that, “Most of the space inside the atom is empty and nucleus is positively charged occupying a small volume within the atom”.

Question.9
Name the three subatomic particles of an atom. [SAII -2011]
Answer.
(ii) Electrons [negatively charged particles] which revolve around the nucleus.
(ii) Protons [positively charged particles] which are present in the nucleus.
(iii) Neutrons [having no charge] which are present in the nucleus.

Question.10
Helium atom has an atomic mass of 4 u and two protons in its nucleus. How many neutrons it have ? [SAII – 2011]
Answer.
Mass number = Number of protons + Number of neutrons 4 = 2 + No. of neutrons
Number of neutrons = (4 – 2) = 2.

Question.11
Write the distribution of electrons in carbon and sodium atoms. [SAII -2011]
Answer.
Image may be NSFW.
Clik here to view.
important-question-for-cbse-class-9-science-is-matter-around-us-pure-4 - Copy

Question.12
If K and L shells of an atom are full, then what would be the total number of electrons in the atom? [SAII-2011]
Answer.
Number of electrons in full K-shell = 2 Number of electrons in full L-shell = 8
... Total number of electrons in an atom is 2 + 8 = 10 electrons.

Question.13
If number of electrons in an Mom is 8 and number of protons is also 8, then (i) what is the atomic number of the atom ? and (ii) what is the charge on the atom ? [SAII -2011]
Answer.
(i) Atomic number
= Number of Protons = 8
(ii) The charge of the atom is zero, as total number of positive charge is equal to total number of negative charge.
Number of Protons = Number of Electrons
8 = 8 ,

Question.14
If Z = 3, what would be the valency of the element ? Also, name the element [SAII – 2011]
Answer.
Atomic number Z = 3
Electronic configuration = 2(K), T(L)
Valence shell has 1 electron in the outermost shell, so valency of element is 1. The element is Lithium.

Question.15
Composition of the nuclei of two atomic species X and Y are given as under :
Image may be NSFW.
Clik here to view.
important-question-for-cbse-class-9-science-is-matter-around-us-pure-5 - Copy

Give the mass numbers of X and Y, What is the relation between the two species ?
Answer.
Mass number of X = No. of Protons + No. of Neutrons !
= 6 + 6 = 12 u
Mass number of Y = No. of Protons + No. of Neutrons
= 6 + 8 = 14 u
The species X and Y are isotopes, as they have same atomic number but different mass number.

3 MARKS QUESTIONS

Question.1
Elaborate the postulates put forward by E. Rutherford about the structure of atom based on the a-particle scattering experiment. [SAII – 2014]
Answer.
(i) Most of the space inside the atom is empty because most of the alpha particles passed through the gold foil without getting deflected.
(ii) Very few particles are deflected from their path, indicating that positive charge of the atom occupies very little space.
(iii) A very small fraction of particles was deflected by 180°, indicating that all tbe positive charge and mass of the gold atom were concentrated in a small volume within the atom.

Question.2
(i) What are canal rays ? Who discovered them ? What is the charge and mass of canal ray ?
(ii) How are the canal rays different from electrons in terms of charge and mass ?
[SAII -2012]
Answer.
(i) New radiations in a gas discharge tube which are positively charged are known as canal rays.
They were discovered by E. Goldstein. Charge on canal rays is positive and its mass is one unit.
(ii) Electrons are negatively charged particles, mass of which is approximately 1/2000 that of – canal rays.

Question.3
List three main features of Rutherford’s nucleus model of an atom. [SAII – 2012]
Answer.
On the basis of a-particle scattering experiment, Rutherford proposed a model of atom. According to him :
(i) There is positively charged centre in an atom called the nucleus which contains the whole mass of the atom.
(ii) The electrons revolve around the nucleus.
(iii) The size of the nucleus is very small as compared to the size of the atom.

Question.4
List three subatomic particles of an atom. Compare them on the basis of relative mass and charge in a tabular form.4 [SAII -2014]
Answer.
Image may be NSFW.
Clik here to view.
important-question-for-cbse-class-9-science-is-matter-around-us-pure-6 - Copy

Question.5
List the three observations made by Rutherford in the experiment of scattering of a-particles . by a gold foil.
Answer.
(i) Most of the fast moving a-particles passed straight through the gold foil.
(ii) Some of the a-particles were deflected by the foil by small angles. .
(iii) Surprisingly one out of every 12000 particles appeared to rebound.

Question.6
Write the maximum number of electrons which can be accommodated in K, L, M, N shells and give the rule on basis of which it is decided. [SAII -2011]
Answer.
Image may be NSFW.
Clik here to view.
important-question-for-cbse-class-9-science-is-matter-around-us-pure-7 - Copy

Question.7
(a) Which of the following electronic configurations are not possible ? Give reasons.
(i) X: 2, 8, 4 (ii) Y : 3, 8, 2 (iii) Z : 2, 8, 9
(b) Write electronic configurations of the following elements and predict their valencies :
Fluorine : 9, Aluminium : 13, Argon : 18. [SAII – 2014]
Answer.
Image may be NSFW.
Clik here to view.
important-question-for-cbse-class-9-science-is-matter-around-us-pure-8 - Copy

Question.8
Give drawbacks of Rutherford’s model.
Answer.
Drawbacks of Rutherford’s Model:
(i) It could not explain the source of energy required for movement of electrons.
(ii) Rutherford’s model could not explain as to why a moving charge (charged electron) does not lose energy and fali into the nucleus.
(iii) It could not explain about the emission of radiations of different frequencies by different atoms when heated.
(iv) It could not explain the stability of an atom when charged electrons are moving under the attractive force of positively charged nucleus.

Question.9
On the basis of Thomson’s model of an atom, explain how the atom is neutral as a whole.
[SAII – 2011]
Answer.
Image may be NSFW.
Clik here to view.
important-question-for-cbse-class-9-science-is-matter-around-us-pure-9

Thomson proposed that:
(i) An atom consists of a positively charged sphere and the electrons are embedded in it.
(ii) The negative and positive charges are equal in magnitude. So the atom as a whole is electrically neutral.

Question.10
How will you find the valency of chlorine, sulphur and magnesium ? [SAII -2011]
Answer.
Image may be NSFW.
Clik here to view.
important-question-for-cbse-class-9-science-is-matter-around-us-pure-10

Question.11
Valency of magnesium = Number of electrons in the valence shell = 2 Write the electronic configuration of any one pair of isotopes and isobars. [SAII,- 2011]
Answer.
Image may be NSFW.
Clik here to view.
important-question-for-cbse-class-9-science-is-matter-around-us-pure-11

Question.12
Compare all the proposed models of an atom given in this Chapter.
Answer.
Image may be NSFW.
Clik here to view.
important-question-for-cbse-class-9-science-is-matter-around-us-pure-12

Question.13
Image may be NSFW.
Clik here to view.
important-question-for-cbse-class-9-science-is-matter-around-us-pure-13

Answer.
Image may be NSFW.
Clik here to view.
important-question-for-cbse-class-9-science-is-matter-around-us-pure-14

Question.14
Image may be NSFW.
Clik here to view.
important-question-for-cbse-class-9-science-is-matter-around-us-pure-15

Answer.
Image may be NSFW.
Clik here to view.
important-question-for-cbse-class-9-science-is-matter-around-us-pure-16

5 MARKS QUESTIONS

Question.1
Write the postulates of Bohr’s model of atom.
Answer.
A Danish physicist, Neils Bohr proposed an atomic model in 1913. This model of atom is called Bohr’s model of atom.
Basic postulates of the Bohr’9 atomic model are :
(i) In an atom, the electrons revolve around the nucleus in certain definite circular orbits.
These circular orbits are also called energy shells or energy levels.
(ii) An electron revolving in a particular orbit has a fixed energy. While revolving in discrete orbits the electrons do not radiate energy.
(iii) The orbits or the energy shells are numbered as n = 1, 2, 3, 4, … etc. or K, L, M, N,… etc. shells starting from the nucleus. These integers 1, 2, 3, 4, … etc. are known as quantum numbers of the orbits.
(iV) An electron can lose or gain only certain definite energies.

Question. 2
Number of electrons, protons and neutrons in chemical species A, B, C and D is given below:
Image may be NSFW.
Clik here to view.
important-question-for-cbse-class-9-science-is-matter-around-us-pure-17

Now, answer the following questions :
(a) What is the mass number of A and B ?
(b) What is the atomic number of B ?
(c) Which two elements represent a pair of isotopes and why ?
(d) What is the valency of element C ?
Also, justify your answers. [SAII – 2014]
Answer.
(a) Mass number ofA=3 + 4 = 7 Mass number of B = 9 + 8 = 17
(b) Atomic number of B = Number of protons
= 9
(c) Elements C and D represent a pair of isotopes because their atomic numbers are the same.
(d) Electronic configuration of C (8) = 2, 6.
So its valency is 2.

Question.3
Read the information given in the table and answer the following questions :
Image may be NSFW.
Clik here to view.
important-question-for-cbse-class-9-science-is-matter-around-us-pure-18

(i) How many pairs of isotopes are there amongst the given atoms ?
(ii) Find a pair of isobars.
(iii) Find an element with valency 2.
(iV) Find an element with 5 valence electrons.
(v) Calculate the numtfer of different subatomic particles in atom C.
Answer.
(i) Two pairs : (a) Elements B and C (b) Elements D and E.
(ii) Elements A and C
Image may be NSFW.
Clik here to view.
important-question-for-cbse-class-9-science-is-matter-around-us-pure-19

(iii) Elements D and E are having valency 2.
(iv)Element A has 5 valence electrons.
(v) Number of protons : 6 Number of neutrons : 8 Number of electrons : 6

Question.4.
With the help of labelled diagram describe in brief the Rutherford’s alpha-particle scattering experiment. Write any three important conclusions drawn from the experiment.
[SAII – 2012, 2014]
Answer.
Rutherford took a very thin gold foil and bornbared it with a-particles.He observed that:
(i) Most of the fast moving a-particles passed straight through the gold foil.
(ii) Some of the alpha-particles were deflected by the foil by small angle.
(iii) Out of every 12000 particles, one appeared to rebound.
From the above observations he concluded :
(i) There is a positively charged centre in an atom called the nucleus. Nearly all mass of an
atom resides in the nucleus.
(ii) The electrons revolve around the nucleus in well defined orbits.
Image may be NSFW.
Clik here to view.
important-question-for-cbse-class-9-science-is-matter-around-us-pure-20

(iii) The size of the nucleus is very small as compared with the size of the atom.

Question.5 Complete the following table :
Image may be NSFW.
Clik here to view.
important-question-for-cbse-class-9-science-is-matter-around-us-pure-21

Answer.
Image may be NSFW.
Clik here to view.
important-question-for-cbse-class-9-science-is-matter-around-us-pure-22

APPLICATION BASED QUESTIONS

Question.1
State the major drawback in Rutherford’s model of an atom. Mention two features of Bohr’s model which helped compensate this drawback. [SAII – 2014]
Answer.
The major drawback of Rutherford’s model of an atom is that it does not explain the stability of an atom. Any particle in a circular orbit would undergo acceleration. During acceleration, charged particles would radiate energy. Thus, the revolving electron would lose energy and finally fall into the nucleus.
•Two features of Bohr’s model which helped compensate this drawback :
(i) Only certain special orbits known as discrete orbits of electrons are allowed inside the atom.
(ii) While revolving in these discrete orbits, the electrons do not radiate energy.

Question.2
Image may be NSFW.
Clik here to view.
important-question-for-cbse-class-9-science-is-matter-around-us-pure-23
.
Answer.
(a) 8.
(b) 16.
(c) Number of neutrons = Mass number – Atomic number
= 16 — 8 = 8.

Question.3
Define isotopes. Why do isotopes have same atomic number but different mass number ?
Explain with the help of an example. [SAII – 2014]
Answer.
Image may be NSFW.
Clik here to view.
important-question-for-cbse-class-9-science-is-matter-around-us-pure-30

Question.4
(a) What is the relationship between two elements X and Y whose atomic numbers are 18 and 20 respectively but their mass numbers remain same as 40 ?
(b) Are their chemical properties same or different? Explain.
(c) Which has more number of electrons Na or Na+ ? Why ? [SAII – 2014]
Answer.
(a) Elements X and Y are isobars.
(b) Chemical properties of an element depends upon its atomic number. Since both the
elements have different atomic numbers, their chemical properties will be different.
(c) Na has more number of electrons because Na+ has lost an electron.

Question.5
In the Gold foil experiment of Geiger and Marsden, that paved the way for Rutherford’s model of an atom, ∼ 1.00% of the a-particles were found to deflect at angles > 50°. If one mole of a-particles were bombarded on the gold foil, compute the Humber of a-particles that would deflect at angles less than 50°.
Answer.
Image may be NSFW.
Clik here to view.
important-question-for-cbse-class-9-science-is-matter-around-us-pure-24

Question.6
If K and L shells of an atom are full, then what would be the total number of electrons in the atom ? What is the valency of this element ? Name the element. [SAII – 2012]
Answer.
The maximum number of electrons that can occupy K and L shells of an atom are 2 and 8 respectively. Therefore, if K and L shells of an atom are full then the total number of electrons in the atom would be (2 + 8) = 10 electrons. So the valency of this element is zero. The element is neon (Ne).

Higher Order Thinking Skills (HOTS) Questions

Question.1
One electron is present in the outermost shell of the atom of an element X. What would be the nature and value of charge on the ion formed if this electron is removed from the outermost shell ?
Answer. +1.

Question.2
Find put the valency of the atom represented by the fig. (a) and (b).
Image may be NSFW.
Clik here to view.
important-question-for-cbse-class-9-science-is-matter-around-us-pure-25

Answer.(a) 0 (b) 0

Question.3
Write down the electron distribution of chlorine atom. How many electrons are there in the L-shell ? (Atomic number of chlorine is 17).
Answer. 2, 8, 7. The L-shell has eight electrons.

Question.4
In the atom of an ejement X, 6 electrons are present in the outermost shell. If it acquires noble gas configuration by accepting requisite number of electrons, then what would be the charge on the ion so formed ?
Answer. -2.

Question. 5
What information do you get from the fig. about the atomic number, mass number and valency of atoms X, Y and Z ? Give your answer in a tabular form.
Answer.
Image may be NSFW.
Clik here to view.
important-question-for-cbse-class-9-science-is-matter-around-us-pure-26

Question. 6
The ratio of the radii of hydrogen atom and its nucleus is ~  105. Assuming the atom and the nucleus to be spherical, (i) What will be the ratio of their sizes ? (ii) If atom is represented by planet earth ‘ Re‘ = 6.4 x  106m, estimate the size of the nucleus.
Answer.
Image may be NSFW.
Clik here to view.
important-question-for-cbse-class-9-science-is-matter-around-us-pure-27

Question.7
In what way is the Rutherford’s atomic model different from that of Thomson’s atomic model ?
Answer.
Rutherford proposed a model in which electrons revolve around the nucleus in well-defined orbits. There is a positively charged centre in an atom called the nucleus. He also proposed :that the size of the nucelus is very small as compared to the size of the atom and nearly all the mass of an atom is centred in the nucleiis. Whereas Thomson proposed the model of an atom to be similar to a Christmas pudding. The electrons are studded like currants in a positively charged sphere like Christmas pudding and the mass of the atom was supposed to be uniformly distributed.

Question.8
In response to a question, a student stated that in an atom, the number of protons is greater than the number of neutrons, which in turn is greater than the number of electrons. Do you agree with the statement ? Justify your answer.
Answer.
No, the statement is incorrect. In an atom, the number of protons and electrons is always equal.

Reasoning Questions

Question.1
Give reason for the following :
(а) Ions are more stable than atoms.
(b) Noble gases show low reactivity.
(c) An atom is electrically neutral. [SAII -2014]
Answer.
(a) Ions are more stable than atoms because except the atoms of inert gases, the outermost shell of the atoms is incomplete. In order to complete octet, they lose or gain electrons.
(b) The outermost shell of the atoms of noble gases are complete. So they show little chemical
activity. . . .
(c) Because number of protons (+ve charge) inside the nucleus are equal to the number of electrons (-ve charge) outside the nucleus.

Question.2
Why atomic nurtiber is more fundamental attribute of an element ?
Atomic number is more fundamental attribute of element. In fact, elements are defined by the
number of protons they possess. Chemical properties of an element change with the change in
atomic number.
Answer.

Question.3
Give reasons:
(а) Mass number of an atom excludes the mass of an electron.
(b) Nucleus of art atom is charged.
(c) Alpha-particle scattering experiment was possible by using gold foil only and not by foil bf any other mfetal. 4 [SAII – 2014]
Answer.
(a) Mass number of an atom excludes the mass of an electron because electrons have negligible mass in comparison to protons and neutrons.
(b) Nucleus of an atom consists orprotons and neutrons. Protons are positively charged particles. So the nucleus of an atom is charged.
(c) Because an extremely thin film was required for the experiment and it was only possible by using gold, as gold is a highly malleable metal.

Question.4
(a) An ion M2+ contains 10 electrons and 12 neutrons. What is the atomic number and mass number of the element M ?
(b) Is it possible in an atom to have 12 protons and 13 electrons ? Explain. [SAII -2014]
(c) Why helium gas is inert ?
Answer.
(a) Atomic number = 12,
Mass number = 24.
(b) No, it is not possible. An atom is electrically neutral. The number of positively charged particles (protons) is always equal to the number of negatively charged particles (electrons).
(c) HeliUm atom has completely filled outermost-shell. Thus, it is inert.

Question.5
State reason for the following statements:
(i) Some elements possess fractional atomic mass.
(ii) Isotopes of an element have similar chemical properties.
(iii) Noble gases are inert. [SAII – 2012]
Answer.
(i) Some elements possess fractional atomic mass because they occur in nature in different isotopic forms. So accordingly their average mass is calculated.
(ii) Isotopes of an element have similar chemical properties because they have the same atomic number and valence electrons.
(iii) The outermost-shell of the atoms of noble gases are completely filled up. So they do not show affinity towards chemical reactions.

Question.6
Will 35cI and 37cI have different valencies ? Justify your answer.
Answer.
No,  35cI and 37cI are isotopes of an element.

Question.7
Why did Rutherford select a gold foil in his a-ray scattering experiment ?
Answer.
Because gold has high malleability.

Question.8
Is it possible for the atom of an element to have one electron, one proton and no neutron. If so, name the element.
Answer.
Image may be NSFW.
Clik here to view.
important-question-for-cbse-class-9-science-is-matter-around-us-pure-29

Question.9
Why do Helium, Neon and Argon have a zero valency ?
Answer.
Helium has two electrons in its only energy shell while Argon and Neon have eight electrons in their valence shells. As these have maximum number of electrons in their valence shells, they do not have any tendency to combine with other elements. Hence, they have a valency equal to zero.

Question.10
Give reasons for the following :
(а) The valency of Na is 1 and not 7.
(b) When an atom loses or gains electrons it becomes stable.
Answer.
(a) it is easy for Na atom to lose one electron instead of gaining seven electrons. So valency of Na is one and not seven
(b) For stability of an atom it must have either 2 or 8 electrons in the outermost orbit. So by losing or gaining electrons it reaches to the electronic configuration of the nearest noble gas.

Important Topics/Areas/Questions which, are
frequently asked in the examination

Question.1
Image may be NSFW.
Clik here to view.
important-question-for-cbse-class-9-science-is-matter-around-us-pure-28

(a) Find the number of electrons present in A and B.
(b) Find the number of nucleons present in A and B.
(c) Explain the special*termaused to represent A and B. [SAII – 2014]
Answer.
(a) The elements A and B have 20 and 18 electrons respectively.
(b) Both have 40 nucleons. ‘
(c) Isobars.

Question. 2
Write the conclusions drawn by Rutherford for the following observing during his a-scattering experiment:
(a) Most of the alpha-particles passed straight through the gold foil.
(b) Some alpha-particles getting deflected from their path.
(c) Very small fraction of alpha-particles getting deflected by 180°. [SAII -2014]
Answer.
(a) Most of the space inside the atom is empty.
(b) It indicates that the positive charge of the atom occupies a very little space.
(c) All the positive charge and mass of the gold atom were concentrated in a very small volume within the atom.

Question.3
Define the following terms:
(a) Electronic configuration
(b) Valence shell
(c) Valency
Answer.
(a) The distribution of electrons amongst different orbits of an atom is known as electronic configuration.
(b) The outermost-shell of an atom is called its valence shell.
(c) The combining capacity of an atom is called its valency.

Question. 4
Image may be NSFW.
Clik here to view.
important-question-for-cbse-class-9-science-is-matter-around-us-pure-31

Answer.
(a) (i) 16, (ii) 32, (iii) Electronic configuration : 2, 8, 6.
(b) Element Z, having atomic number 16 is chemically more reactive than element X of atomic number 18. It is because the outermost-shell of the atom of element Z has six electrons only and has to complete its octet whereas the outermost-shell of the atom of element X is completely filled up /;e., its octet is complete and thus it shows little chemical activity.

Question.5
What kind of elements have a tendency to lose electrons ? What are they commonly called ?
Answer.
The elements having 1, 2 or 3 valence electrons have the tendency to lose electrons. They are commonly called metals.

Question.6
What kind of elements have a tendency to gain electrons ? What are they commonly called ?
Answer.
The elements having 5, 6 or 7 valence electrons have the tendency to gain electrons. They are commonly called as non-metals.

The post Important Question for CBSE Class 9 Science Is Matter Around us Pure appeared first on Learn CBSE.

CBSE Sample Papers for Class 12 Economics Outside Delhi – 2016

CBSE Sample Papers for Class 12 Economics Outside Delhi – 2016

Time allowed : 3  hours                                                                                         Maximum marks 100

GENERAL INSTRUCTIONS
(i) All questions in both the sections are compulsory.
(ii) Marks for questions are indicated against each.
(iii) Questions No. 1-5 and 17-21 are very short-answer questions carrying 1 nick each. They are required to be answered in one sentence each.
(iv) Questions No. 6-10 and 22-26 are? short-answer questions carrying 3 marks each. Answers to them should normally not exceed 60 words each.
(v) Questions No. 11-13 and 27-29 are also short-answer questions carrying 4 marks each. Answers to them should normally not exceed 70 words each.
(vi) Questions No. 14-16 and 30-32 are long-answers questions carrying 6 marks each. Answers to them should normally not exceed 100 words each.
(vii) Answers should be brief and to the point and the above word limit should be adhered to as far as possible.

SET I

SECTION A
Question.1. What is the relation between Average Variable Cost and Average Total Cost, if Total Fixed Cost is zero?
Answer. If Total Fixed Cost is zero then this implies that Average Fixed Cost is also zero. Then in this case Average Variable Cost will be the same as Average Total Cost i.e., AVC = ATC.

Question.2. A firm is able to sell any quantity of a good at a given price. The firm’s marginal . revenue will be: (choose the correct alternative)
(a) Greater than Average Revenue (b) Less than Average Revenue
(c) Equal to Average Revenue (d) Zero
Answer. (c) Equal to Average Revenue.

Question.3. When does ‘change in demand’ take place?
Answer. Change in demand takes place due to change in the factors other than own price of the gold. These factors include change in the income of consumer, change in the price of substitute goods, favorable change in tastes and preferences etc.

Question.4. Differentiated products is a characteristic of: (choose the correct alternative)
(a) Monopolistic competition only
(b) Oligopoly only
(c) Both monopolistic competition and oligopoly
(d) Monopoly
Answer. (c) Both monopolistic competition and oligopoly.

Question.5. Demand curve of a firm is perfectly elastic under: (choose the correct alternative) 1
(a) Perfect competition (b) Monopoly
(c) Monopolistic competition (d) Oligopoly
Answer. (a) Perfect competition

Question.6. A consumer consumes only two goods X and Y. Marginal utilities of X and Y are 3 and 4 respectively. Prices of X and Y are Rs4 per unit each. Is consumer in equilibrium? What will be further reaction of the consumer? Give reasons.
Answer.
Image may be NSFW.
Clik here to view.
cbse-sample-papers-for-class-12-economics-outside-delhi-2016-1

Since the equilibrium condition is not being satisfied, therefore the consumer is not in equilibrium. The given situation represents the following inequality:
\(\frac { { MU }_{ x } }{ P_{ x } } <\frac { { MU }_{ y } }{ { P }_{ y } }\)
Since the consumer is getting more marginal utility per rupee from good Y than from good X, therefore, this will induce the consumer to buy less of X and more of Y. When less of X is purchased/ consumed, \({ MU }_{ x }\) will rise. When more of Y is consumed, \({ MU }_{ y }\) will fall.
The switch over from X to Y will continue till \( \frac { { MU }_{ x } }{ P_{ x } } =\frac { { MU }_{ y } }{ { P }_{ y } }\) and equilibrium is attained.

Question.7. What will be the effect of 10 percent rise in price of a good on its demand if price elasticity of demand is (a) Zero, (b) -1, (c) -2.
Answer. (a) Zero elasticity or perfectly inelastic demand refers to a situation when change in price causes no change in quantity demanded. This means that a 10% rise in price of a good will have no effect on its quantity demanded.
(b) -1. In case of unitary elastic demand, the percentage change in quantity demanded is
equal to percentage change in price. This means that a 10% rise in price of a good will decrease the demand of the good by 10%.
(c) -2. Elasticity greater than one refers to highly elastic demand. This happens when the percentage change in quantity demanded is more than the percentage change in price. Therefore, a 10% rise in price will lead to a decline in demand of the good by 20%.
Image may be NSFW.
Clik here to view.
cbse-sample-papers-for-class-12-economics-outside-delhi-2016-2

Question.8. What is Minimum Price Ceiling? Explain its implications.
Answer. See Q. 8, 2015 (I Outside Delhi).
Or
If the prevailing market price is above the equilibrium price, explain its chain of effects.
Answer. If the prevailing market price is more than the equilibrium price, it leads to a situation of excess supply.
Image may be NSFW.
Clik here to view.
cbse-sample-papers-for-class-12-economics-outside-delhi-2016-3

In this situation, if market price is OP1(more than the equilibrium price OP), then it creates excess supply in the market equal to Q1Q2. Excess supply of Q1Q2 will lead to competition amongst sellers as each seller would want to sell his product.
This means that sellers would be ready to charge a lower price to sell their output. With decrease in price, market supply will fall (contraction) and market demand will rise (extension). The price will continue to fall till excess supply is wiped out and equilibrium is attained (as shown by arrows).

Question.9. Define demand. Name the factors affecting market demand.
Answer. Demand for a good is defined as the quantity of a good that a consumer or consumers taken together are willing to buy at a particular price during a particular period of time.
Factors affecting market demand:
(i) Own price of the commodity. Generally, other things remaining constant, demand for a good rises with fall in price and falls with rise in price of that good.
(ii) Population. Higher the number of consumers or total population in a market, higher will be the market demand for a commodity and vice versa.
(iii) Income of the consumers:
(a) Normal goods. Generally,’ increase in the money income of consumers increases the demand for a normal good and a fall in income reduces the demand for it.
(b) Inferior goods. In case of inferior goods, an increase in income reduces the demand for these goods and vice versa.
(iv) Market price of other goods. Quantity demanded of a good also depends upon the market price of other goods (i.e., related goods).
(a) Substitute goods. A rise in price of a substitute good, increases the demand for the given good, and fall in price of substitute good leads to a fall in demand for the given good. Example, tea and coffee.
(b) Complementary goods. Rise in price of a complementary good leads to a fall in demand for the given good and vice versa. Example, tea and milk.
(v) Season and weather. Seasonal and weather conditions also affect consumer’s demand. For example, demand for woollen clothes increases during winters.

Question.10. Define Fixed Cost. Give an example. Explain with reason the behaviour of Average Fixed Cost as output is increased.
Answer. Fixed costs refer to those costs that a firm incurs to employ fixed inputs or factors. For example, cost incurred on plant and equipment, land and building etc.
Average Fixed Cost (AFC) refers to the per unit fixed cost of production. It is derived by dividing Total Fixed Cost by Quantity of output produced.
\( AFC=\frac { TFC }{ Output }\)
AFC falls continuously with rise in output because constant TFC is divided by increasing output. AFC curve is downward sloping and a rectangular hyperbola.
Or
Define Marginal Product. State the behaviour of marginal product when only one input is increased and other inputs ^re held constant.
Answer. Marginal Product is defined as the change in output per unit of change in input when all other inputs are held constant. It is the change in total output brought about by employing one additional unit of variable input.
According to the law of variable proportions, as more and more units of variable inputs are employed with fixed factors, marginal product (MP) first increases, then falls but remains positive and finally becomes negative.
The three phases of the law are:
Phase I: Increasing Returns—MP rises.
Phase II: Diminishing Returns—MP falls but remains positive.
Phase III: Negative Returns—MP becomes negative.

Question.11. When price of a commodity falls from Rs12 per unit to Rs9 per unit, the producer supplies 75 per cent less output. Calculate price elasticity of supply.
Answer.
Image may be NSFW.
Clik here to view.
cbse-sample-papers-for-class-12-economics-outside-delhi-2016-4

Image may be NSFW.
Clik here to view.
cbse-sample-papers-for-class-12-economics-outside-delhi-2016-5

Question.12. Why do central problems of an economy arise? Explain the central problem of “for whom to produce”?
Answer. The mam reasons for existence of central or economic problems are:
(i) Scarcity of resources. Resources (i.e., land, labour, etc.) are limited in relation to , their demand and economy cannot produce all what people want.
(ii) Unlimited human wants. Human wants are never ending and cannot be satisfied due to limited resources. Also human wants differ in priorities.
(iii) Alternate uses of resources. Resources are scarce and can be put to various alternate uses. This makes choice among resources more important. Therefore, economy has to make choice between the alternative uses of the given resources.
For whom to produce. See Q. 6, 2014 (I Delhi).

Question.13. Explain three properties of indifference curves.
Answer. (i) An 1C slopes downwards from left to right. An indifference curve always slopes downwards to tire right. In other words, it has a negative slope. To consume more of one good, the consumer must give up some quantity of the other good so that total utility remains the same.
Image may be NSFW.
Clik here to view.
cbse-sample-papers-for-class-12-economics-outside-delhi-2016-6

(ii) Higher indifference curve —denotes higher level of satisfaction. This is because higher IC represents more of adeast one commodity, which means more utility as more is preferred to less.
Image may be NSFW.
Clik here to view.
cbse-sample-papers-for-class-12-economics-outside-delhi-2016-7

(iii) Two indifference curves never intersect each other because at the point of intersection, the level of satisfaction is the same but at other points it is not the same which is not possible. Therefore, it is wrong to suggest that they intersect as shown in Diagram (iii).
Image may be NSFW.
Clik here to view.
cbse-sample-papers-for-class-12-economics-outside-delhi-2016-8

(iv) An IC is convex to the origin. This is because Marginal Rate of Substitution (MRS) declines as more of one good is consumed.

Question.14. Examine the effect of (a) fall in the own price of good X and (b) rise in tax rate on good X, on rile supply curve. Use diagrams.
Answer. (a) Fall in the own price of good X. Fall in price from OP to OPj leads to decrease in quantity supplied from OQ to OQ1 (known as contraction in supply). This results in a downward movement along the same supply curve from point A. to point B.
Image may be NSFW.
Clik here to view.
cbse-sample-papers-for-class-12-economics-outside-delhi-2016-9

(b)Rise in tax rate on good X. Taxation and subsidy policy of the government also impact market supply of a commodity. Increase in taxation increases the cost of the good, thereby decreasing the supply of the commodity on which the tax has-been levied.
An increase in the tax rate on good X will lead to decline in the supply of X from OQ to OQj resulting in a leftward shift in the supply curve from SS to S1S1. Price remains unchanged.
Image may be NSFW.
Clik here to view.
cbse-sample-papers-for-class-12-economics-outside-delhi-2016-10

Note: For BLIND CANDIDATES in lieu of Q. No. 14.
Examine the effect of (a) fall in the own price of good X and (b) rise in tax rate on good X on supply of a good. Use schedule.
Answer. (a) Fall in the own price of good X:
Image may be NSFW.
Clik here to view.
cbse-sample-papers-for-class-12-economics-outside-delhi-2016-11

When price of good X falls from Rs15 to Rs10, its quantity supplied also decreases from 150 to 100 units.
(b) Rise in tax rate on good X:
Image may be NSFW.
Clik here to view.
cbse-sample-papers-for-class-12-economics-outside-delhi-2016-12

Due to increase in tax rate on good X, the supply decreases from 100 to 70 units at the same price of Rs10 per unit.

Question.15. Explain the implications of the following in a perfectly competitive market:
(a) Large number of sellers
(b) Homogenous products.
Answer. (a) Large number of sellers. The number of sellers in a perfectly competitive market is so large that output by an individual seller is an insignificant proportion of the total output of the industry. As such, any change in the output plan of a single producer, assuming no change in output by other producers, has negligible effect on total output and hence no influence on the market price.
(b) Homogenous products. Under perfect competition, firms sell homogenous products, i.e., identical in all respects like size, weight, design, etc. The implication of this feature is that die buyers do not distinguish between the products of different firms. So, they are not ready to pay a different price for the product of any particular firm. Therefore, any attempt by a firm to sell its product at a higher price will fail. This ensures a uniform price for the products of all the firms in the industry.
Or
Explain the implications of the following in an oligopoly market:
(a) Barriers to entry of new firms
(b) A few or a few big sellers
Answer. (a) Barriers to entry of new firms. There are barriers to the entry of new firms in oligopoly industries. These barriers are created largely through patent rights, requirement of huge capital etc. Because of these barriers, the existing firms are not much worried about the entry of new firms in the market. They continue to earn abnormal profits even in the long run.
(b) A few or a-few big sellers. Oligopoly, as an industry, is composed of a few firms or a few large firms controlling the bulk of its output. Since there are a few large firms, each big firm is likely to have a market share big enough to have some influence on the price of the product, the firm produces. Accordingly, the firms are able to generate abnormal profits.

SECTION B
Question.16. Define flows.
Answer. Variables whose magnitude is measured over a period of time are called flow variables. For example, income, expenditure, etc.

Question.17. National income is the sum of factor incomes accruing to: (choose the correct alternative)
(a) Nationals (b) Economic territory
(c) Residents (d) Both residents and non-residents
Answer. (c) Residents

Question.18. What are Revenue Receipts in a government budget?
Answer. The receipts which neither create any liabilities nor lead to any reduction in assets are called revenue receipts.

Question.19. Primary deficit equals: (choose die correct alternative)
(a) Borrowings
(b) Interest payments
(c) Borrowings less interest payments
(d) Borrowings and interest payments both
Answer. (c) Borrowings less interest payments.

Question.20. Foreign exchange transactions which are independent of other transactions in the Balance of Payments Account are called: (choose the correct alternative)
(a) Current transactions (b) Capital transactions
(c) Autonomous transactions (d) Accommodating transactions
Answer. (c) Autonomous transactions.

Question.21. Assuming real income to be Rs 200 crore and price index to be 135, calculate nominal income.
Answer. Real Income = Rs 200 crore, Price index = 135
Image may be NSFW.
Clik here to view.
cbse-sample-papers-for-class-12-economics-outside-delhi-2016-13

Question.22. What is Aggregate Demand? State its components.
Answer. Aggregate Demand refers to the total value of final goods and services that all sectors of the economy taken together are planning to buy at a given level of income during a period of time. Aggregate demand consists of the following:
(i) Private Final consumption expenditure (C)
(it) Private Investment expenditure (I)
(iii) Government expenditure (G), including government consumption and investment expenditure.
(iv) Net Exports (X – M) i.e., Exports – Imports.
Or
Explain how controlling money supply is helpful in reducing excess demand.
Answer. Excess demand refers to the situation when aggregate demand is in excess of aggregate supply i.e., (AD > AS) corresponding to full employment in the economy.
The central bank controls the supply of money in the economy by regulating the creation of credit by commercial banks. Credit creation by commercial banks in times of excess demand can be controlled by
— increasing the bank rate/ repo rate;
— by selling government securities in the open market to absorb liquidity from the system;
— by increasing CRR and SLR; and
— by raising the margin requirement.
Less money supply i.e., stock of money with people leaves less purchasing power in their hands. Therefore, people demand less goods and services.

Question.23. An economy is in equilibrium. Calculate Marginal Propensity to Consume:
National income =1,000
Autonomous consumption expenditure = 200
Investment expenditure = 100
Answer.
Image may be NSFW.
Clik here to view.
cbse-sample-papers-for-class-12-economics-outside-delhi-2016-14

Question.24. Sale of petrol and diesel cars is rising particularly in big cities. Analyse its impact on gross domestic product and welfare.
Answer. Final sale of cars raises GDP, because final sales are final products. Cars provide convenience in transportation and add to the welfare of people. At the same time, rise in sale of petrol and diesel cars particularly in big cities will lead to air pollution and noise pollution which will adversely affect the health of citizens (including children). This is a negative externality occuring because of the smoke and harmful gases being emitted by the vehicles. This leads to respiratory problems and other health issues which will impact the GDP of the nation adversely and reduce the welfare of the people.

Question.25. Explain the ‘medium of exchange’ function of money. How has it solved the related problem created by barter?
Answer. Medium of exchange. Money acts as a medium of exchange for the sale and purchase of goods and services. In the absence of money under the barter system, goods were exchanged for goods. This required double coincidence of wants. Introduction of money has separated the acts of sale and purchase. It allows freedom of choice as every person can buy goods of his choice and also from people offering him the best bargain.
Or
Explain the ‘standard of deferred payment’ function of money. How has it solved the related problem created by barter?
Answer. Standard of deferred payment. Deferred payments refer to those payments which are to be made at some time in the future. In a modem economy, a large number of transactions involve future payments which can easily be stated in terms of money. This is possible because value of money remains more or less constant and it possesses the quality of general acceptability.
In the barter system, it was difficult to make such future payments in terms of goods and services. There was always the risk of the commodity to be repaid increasing or decreasing in value. There could also be a disagreement regarding the quality of goods or services to be repaid.
This function of money has facilitated borrowing and lending activities and promoted trading. It has also led to the creation of financial institutions.

Question.26. Explain how ‘Repo Rate’ can be helpful in controlling credit creation.
Answer. The rate of interest at which the RBI (Central bank) gives short term loans to commercial banks is called Repo Rate.
The Central Bank make use of Repo Rate to control the supply of money and credit creation. A rise in Repo Rate would make borrowings by commercial banks costly. This increase forces these banks to raise the interest rates on lending to the general public. As borrowings from banks become costly, it leads to a decline in demand for borrowings from the banks which decreases credit creation in the economy. Chi the other hand, a fall in Repo Rate encourages banks to keep small proportion of their deposits as reserves since borrowing from Central Bank is now less costly than before. This increases credit creation in the economy.

Question.27. What is the difference between revenue expenditure and capital expenditure? Explain how taxes and government expenditure can be used to influence distribution of income in the society.
Answer. Revenue Expenditure and Capital Expenditure. See Q.-32(b), 2014 (Comptt. I Delhi).
Redistribution of income and wealth. Through its tax and expenditure policy, the ’government attempts to reduce inequalities of income. The government influences the personal disposable income of households by making transfer payments and collecting taxes. In this way, it influences the income distribution in the society. Inequalities of income and wealth can also be reduced through progressive income taxation in which higher the income, higher is the tax rate. Government can impose higher taxes on incomes of the rich and on the luxury goods and services consumed by them. The money so collected can be spent on welfare projects for the poor such as providing free healthcare, free education, subsidies, minimum wages etc. These expenditures by the government will raise the disposable income of the poor.
Or
What is the difference between direct tax and indirect tax? Explain the role of government budget in influencing allocation of resources.
Answer. Direct tax and Indirect tax. See Q. 32(a), 2014 (Comptt. I Delhi).
Allocation of Resources. Government budget is a reflection of certain goals the government wants to achieve. There are many economic activities which are not undertaken by the private sector either due to lack of profits or due to huge investment expenditure involved. Government can undertake these activities on its own. There are many other activities like water supply, sanitation etc. which are necessarily undertaken by government in public interest. In addition, government can encourage the private sector through tax concessions, subsidies etc. to undertake certain production in public interest. By doing so, government helps in influencing allocation of resources.

Question.28. Given saving curve, derive consumption curve and state the steps in doing so. Use diagram.
Answer. See Q.32(Or), 2014 (I Outside Delhi).
Note: For BLIND CANDIDATES in lieu of Q. No. 14 .
Explain Consumption Function. Derive saving function from consumption function.
Answer.Consumption function
Image may be NSFW.
Clik here to view.
cbse-sample-papers-for-class-12-economics-outside-delhi-2016-15

Question.29. Indian investors lend abroad. Answer the following questions:
(a) In which sub-account and on which side of the Balance of Payments Account such lending is recorded? Give reasons.
(b) Explain the impact of this lending on market exchange rate.
Answer. (a) Indian investors lending abroad will be recorded in the capital account of the Balance of Payments account on the debit side because it leads to creation of foreign exchange assets. It is recorded on the debit side because it leads to outflow of foreign exchange.
(b) Lending abroad by Indian investors would lead to decrease in the supply of foreign will depreciate.

Question.30. Find Gross National Product at Market Price and Private Income:
Image may be NSFW.
Clik here to view.
cbse-sample-papers-for-class-12-economics-outside-delhi-2016-16

Image may be NSFW.
Clik here to view.
cbse-sample-papers-for-class-12-economics-outside-delhi-2016-17

Answer.
Image may be NSFW.
Clik here to view.
cbse-sample-papers-for-class-12-economics-outside-delhi-2016-18

SET II

Note: Except for the following questions, all the remaining questions have been asked in Set-I.
SECTION A
Question.4. When does “change in quantity demanded” take place?
Answer. Change in quantity demanded takes place due to change in price of the commodity itself.

Question.5. What happens to the difference between Average Total Cost and Average Variable Cost as production is increased?
Answer. The difference between Average Total Cost (AC) and Average Variable Cost (AVQ is represented by Average Fixed Cost (AFC) which decreases as production is increased.

Question.7. A consumer consumes only two goods X and Y. Marginal utilities of X and Y are 4 and 3 respectively. Price of X and price of Y is 13 per unit, Is consumer in equilibrium? What will be further reaction of the consumer? Give reasons.
Answer.
Image may be NSFW.
Clik here to view.
cbse-sample-papers-for-class-12-economics-outside-delhi-2016-19

Question.9. When price of a good rises from Rs 10 to Rs 12 per unit, the producer supplies 10 percent more. Calculate price elasticity of supply.
Answer.
Image may be NSFW.
Clik here to view.
cbse-sample-papers-for-class-12-economics-outside-delhi-2016-20

Image may be NSFW.
Clik here to view.
cbse-sample-papers-for-class-12-economics-outside-delhi-2016-21

Question.10. Define Utility. Explain the Law of Diminishing Marginal Utility.
Answer. Utility is the amount of satisfaction obtained from consuming goods and services. The law of “diminishing marginal utility” states “as more and more units of a commodity are consumed, marginal utility derived from each successive unit goes on falling”. Ultimately, it may become zero or even negative.
This is because as a consumer consumes more and more units of a commodity, his intensity of want for that commodity goes on falling and a point is reached when he wants no more units of it (as shown in the schedule below).
Image may be NSFW.
Clik here to view.
cbse-sample-papers-for-class-12-economics-outside-delhi-2016-22

SECTION B
Question.19. Define Fiscal Deficit.
Answer. Fiscal deficit is the excess of Government total expenditure over Total receipts excluding Borrowings of the government.

Question.21. An economy is in equilibrium. Find investment expenditure: 3
National income = 1,200 ,
Autonomous consumption expenditure = 150 Marginal Propensity to consume =0.8
Answer.
Image may be NSFW.
Clik here to view.
cbse-sample-papers-for-class-12-economics-outside-delhi-2016-23

Question.22. If nominal income is Rs 500 and price index is 125, calculate real income.
Answer.
Image may be NSFW.
Clik here to view.
cbse-sample-papers-for-class-12-economics-outside-delhi-2016-24

Question.24. Explain the role of Cash Reserve Ratio in controlling credit creation.
Answer. Cash Reserve Ratio (CRR) is the fraction of net total demand and time deposits that commercial banks must keep as cash reserves with the Central Bank, If the Central Bank increases the CRR, the banks are required to hold a larger fraction of their deposits with . the Central Bank. As a result, the lending capacity of the banks decreases which in turn decreases borrowings. This leads to a decline in the credit creation capacity of commercial banks. On the other hand, a credit and money supply will increase.

Question.27. Calculate Net National Product at Market Price and Private income.
Image may be NSFW.
Clik here to view.
cbse-sample-papers-for-class-12-economics-outside-delhi-2016-25

Answer.
Image may be NSFW.
Clik here to view.
cbse-sample-papers-for-class-12-economics-outside-delhi-2016-26

SET III

SECTION A
Note: Except for the following questions, all the remaining questions have been asked in Set I and Set II.
Question.3. What happens to the difference between Total Cost and Total Variable Cost as output is increased?
Answer. The difference between Total Cost and Total Variable Cost remains the same as output is increased because this difference is signified by Total Fixed Cost which remains the same, ’ irrespective of the level of output.

Question.5. When does ‘shift in supply curve’ take place?
Answer. Shift in supply curve takes place as a result of change in factors affecting supply other than the own price of the commodity.

Question.8. A consumer consumes only two goods X and Y. The marginal utilities of X and of Y is 3 each. Prices of X and Y are Rs 2 and Rs 1 respectively. Is consumer in equilibrium? What will be further reaction of the consumer? Give reasons.
Answer.
Image may be NSFW.
Clik here to view.
cbse-sample-papers-for-class-12-economics-outside-delhi-2016-27

Question.10. When price of a good rises from Rs 8 per unit to Rs 10 per unit, producer supplies 40 units more. Price elasticity of supply is 2. What is the quantity supplied before price change? Calculate.
Answer.
Image may be NSFW.
Clik here to view.
cbse-sample-papers-for-class-12-economics-outside-delhi-2016-28

The quantity supplied before the price change is 80 units.

Question.11. Distinguish between individual’s demand and market demand. Name the factors affecting demand for a good by an individual.
Answer. Individual demand for a good is the quantity of Brat good which a consumer is willing to buy at a given price, during a period of time. –
Market demand refers to the total quantity demanded of a commodity by all the consumers at a given pri«e during a period of time. It is derived by adding up the demands of all individual consumers at that price.
TIte factors affecting demand of a good by an individual are:
(i) Price of the commodity itself.
(ii) Prices of related goods—Substitute goods and Complementary goods.
(iii) Income of the consumer—In case of normal and inferior goods.
(iv) Tastes and preferences of the consumer.

SECTION B
Question.20. What are Capital Receipts in a government budget?
Answer. Capital Receipts in a government budget are the receipts that either create a liability (e.g., fresh borrowings) or reduce assets (e.g., disinvestment of PSUs).

Question.22. An economy is in equilibrium. Find investment expenditure:
National Income = 1000
Autonomous Consumption =100 Marginal Propensity to consume = 0.8
Answer.
Image may be NSFW.
Clik here to view.
cbse-sample-papers-for-class-12-economics-outside-delhi-2016-29

Image may be NSFW.
Clik here to view.
cbse-sample-papers-for-class-12-economics-outside-delhi-2016-30

Question.23. If real income is Rs 400 and price index is 105, calculate nominal income.
Answer.
Image may be NSFW.
Clik here to view.
cbse-sample-papers-for-class-12-economics-outside-delhi-2016-31

Question.25. Explain the role of Reverse Repo Rate in controlling credit creation.
Answer. When the commercial banks have surplus funds they can deposit the same with Central Bank of interest paid by the Central Bank on such deposits is called Reverse Repo Rate . When Reverse Repo Rate is increased, it encourages*the commercial banks to deposit more of their funds with the Central Bank. This has a negative effect on the lending capability of commercial banks as it reduces the liquidity of the banks. Thus, this controls credit creation by commercial banks.
Whereas, lowering Reverse Repo Rate has the opposite effect as it increases the credit creation capacity of commercial banks.

Question.29. Calculate National Income and Personal Disposable Income:
Image may be NSFW.
Clik here to view.
cbse-sample-papers-for-class-12-economics-outside-delhi-2016-32

Answer.
Image may be NSFW.
Clik here to view.
cbse-sample-papers-for-class-12-economics-outside-delhi-2016-33

The post CBSE Sample Papers for Class 12 Economics Outside Delhi – 2016 appeared first on Learn CBSE.

CBSE Sample Papers for Class 12 Economics Delhi 2010

CBSE Sample Papers for Class 12 Economics Delhi 2010

Time allowed : 3  hours                                                                                         Maximum marks 100

GENERAL INSTRUCTIONS
(i) All questions in both the sections are compulsory.
(ii) Marks for questions are indicated against each.
(iii) Questions No. 1-5 and 17-21 are very short-answer questions carrying 1 nick each. They are required to be answered in one sentence each.
(iv) Questions No. 6-10 and 22-26 are? short-answer questions carrying 3 marks each. Answers to them should normally not exceed 60 words each.
(v) Questions No. 11-13 and 27-29 are also short-answer questions carrying 4 marks each. Answers to them should normally not exceed 70 words each.
(vi) Questions No. 14-16 and 30-32 are long-answers questions carrying 6 marks each. Answers to them should normally not exceed 100 words each.
(vii) Answers should be brief and to the point and the above word limit should be adhered to as far as possible.

SET I

SECTION A

Question.l. Define an indifference curve.
Answer. Indifference curve refers to the graphical representation of different combinations of two goods, each combination offering the same level of satisfaction to a consumer.

Question.2. Name the characteristic which makes monopolistic competition different from perfect competition.
Answer. Product differentiation in monopolistic competition makes it different from perfect competition where there is homogeneity in product.

Question.3. Why is demand for water inelastic?
Answer. Demand for water is inelastic because it is essential to life and thus its demand will not change with the change in its price.

Question.4. State one feature of oligopoly.
Answer. In oligopoly the firms are mutually dependent to a great deal for taking price and output decisions.

Question.5. In which market form demand-curve of a firm is perfectly elastic?
Answer. In perfect competition, demand curve of a firm is perfectly elastic.

Question.6. Distinguish between ‘increase in demand’ and ‘increase in quantity demanded’ of a commodity.
Answer.
Image may be NSFW.
Clik here to view.
cbse-sample-papers-class-12-economics-delhi-2010-1

Question.7. Explain the law of diminishing marginal utility with the help of a utility schedule.
Answer. Law of diminishing marginal utility states that as we consume more and more units of a , commodity, the utility derived from each successive unit goes on decreasing. This law holds true under certain assumptions, i.e. a reasonable quantity of the commodity is consumed and that consumption is a continuous process. This law can be explained with the help of the following utility schedule.
Image may be NSFW.
Clik here to view.
cbse-sample-papers-class-12-economics-delhi-2010-2

Image may be NSFW.
Clik here to view.
cbse-sample-papers-class-12-economics-delhi-2010-3

The above table shows that as the consumer consumes more units of ice cream, the marginal utility obtained from each successive unit goes on declining. Accordingly, MU curve slopes downwards from left to right.
Or
Goods X and Y are substitutes. Explain the effect of fall in price of Y on demand for X.
Substitute goods are those goods which can be used in place of one another for satisfaction of a particular want like tea and coffee. Demand for a given commodity varies directly with the price of a substitute good, e.g. when price of Y falls, consumer switches over from X to Y and hence demand for X falls.
As shown in the diagram, when price of Y falls, it leads to a leftward shift in the demand curve of X indicating decrease in demand. The quantity demanded of X falls from OQ to OQ1
Image may be NSFW.
Clik here to view.
cbse-sample-papers-class-12-economics-delhi-2010-4

Question.8. At a price of 5 per unit of commodity A, total revenue is Rs. 800. When its price rises by 20 per cent, total revenue increases, by Rs. 400. Calculate its price elasticity of supply.
Answer.
Image may be NSFW.
Clik here to view.
cbse-sample-papers-class-12-economics-delhi-2010-5

Question.9. Explain the implications of freedom of entry and exit of firms under perfect competititon.
Answer. In a perfectly competititve market, there are no obstacles in the way of new firms joining the industry and existing firms leaving the industry. This ensures that in the long run there are neither above-normal profits nor losses by any firm. In case of extra normal profits, new firms will join the industry raising the market supply. The market price will fall and the extra normal profits will be wiped out. In case of losses, the existing firms start leaving the industry reducing the total supply. This raises the price till all the losses are wiped out .

Question.10. Given below is the cost schedule of a firm. Its average fixed cost is Rs. 20 when it produces 3 units.
Image may be NSFW.
Clik here to view.
cbse-sample-papers-class-12-economics-delhi-2010-6

Calculate its marginal cost and average total cost at each given level of output.
Answer.
Image may be NSFW.
Clik here to view.
cbse-sample-papers-class-12-economics-delhi-2010-7

Question.11. Explain the problem of ‘what to produce’.
Answer. This problem signifies that what goods should be produced and in what quantities. This problem arises when due to scarcity of resources we cannot produce each and every thing – that we want. Therefore, a decision has to be taken as to what goods should be produced and in what quantities. In short, we have to see whether we should produce consumer goods or producer goods or defence goods or all the goods in some quantity combination.
So, on the basis of importance of various goods, an economy has to decide which goods should be produced and in what quantities. This is a problem of allocation of resources for different goods.
Or
Explain any two main features of a centrally planned economy.
Answer. Centrally planned economy refers to an economy in which means of production are owned, controlled and operated by the Government.
Its two main features are as follows:
(1) The Government organises all economic activities and distributes various goods among the consumers on the basis of its own decisions. In this economy, die basic problems are solved by a Central Planning Authority, generally known as the Planning Commission.
(2) The basic aim, while solving the central problems, is to work for social welfare of the people.

Question.12. When the price of a commodity falls by Rs. 2 per unit, its quantity demanded increases by 10 units. Its price elasticity of demand is
(-) 1. Calculate its quantity demanded at the price before change which was Rs. 10 per unit.
Answer.
Image may be NSFW.
Clik here to view.
cbse-sample-papers-class-12-economics-delhi-2010-8

Question.13. Explain the effect of increase in income of buyers of a ‘normal’ commodity on its equilibrium price.
Answer. In case of a normal commodity, an increase in die income of the consumer signifies an increase in its demand.
As shown in the diagram, point E is the equilibrium point with OP and OQ being the equilibrium price and equilibrium quantity respectively. When demand increases due to an increase in the income of the buyer of a normal good, the new equilibrium will be attained at point Ea. The equilibrium price will increase from OP to OP1 and the equilibrium quantity °- will increase from OQ to OQ1
Image may be NSFW.
Clik here to view.
cbse-sample-papers-class-12-economics-delhi-2010-10

Question.14. Using indifference curves approach, explain the conditions of consumer’s equilibrium.
Answer. Consumer’s equilibrium refers to the optimum combination of the two goods which a consumer can afford (given his income and price of two commodities) and this combination gives him maximum satisfaction he possibly can get. According to indifference curves approach, consumer’s equilibrium is established at a point where budget line is tangent to the indifference curve. At this point the slope of
Image may be NSFW.
Clik here to view.
cbse-sample-papers-class-12-economics-delhi-2010-9

Conditions for consumer’s equilibrium are:
1. Budget line should be tangent to the indifference curve i.e.
\( { MRS }_{ xy }=\frac { { P }_{ x } }{ { P }_{ y } } \)
i.e. Slope of Indifference curve Slope of Budget line
2. MRS is diminishing or Indifference curve is convex to the point of origin.
At equilibrium, marginal rate of substitution be equal to the ratio of prices of two goods. However, if \( { MRS }_{ xy }>\frac { { P }_{ x } }{ { P }_{ y } } \), then it means that to obtain one extra unit of X, the consumer is willing to sacrifice more than he has to sacrifice actually. The consumer gains so he goes on obtaining more and more units of X until marginal utility of X declines.
Therefore the consumer is willing to sacrifice less and less of Y each time he obtains one extra unit of X. As a result \( { MRS }_{ xy } \) falls and ultimately becomes equal to Px/Py at some combination of X and Y. At this combination the consumer is in equilibrium
\( { MRS }_{ xy }<\frac { { P }_{ x } }{ { P }_{ y } } \)
If the consumer attempts to obtain more units of X beyond the equilibrium level, \({ MRS }_{ xy } \)will become less than Px/Py and he will start losing. So he will not try to obtain more of X.

Question.15. State whether the following statements are true or false. Give reasons for your answer.
(a) When total revenue is constant, average revenue will also be constant.
(b) Average variable cost can fall even when marginal cost is rising,
(c) When marginal product falls, average product will also fall.
Answer. (a) False. Because when TR is constant AR will fall, as shown in the following table:
Image may be NSFW.
Clik here to view.
cbse-sample-papers-class-12-economics-delhi-2010-12

(b) True. Because near the point of intersection AVC falls and MC rises, as shown in the diagram (a)
Image may be NSFW.
Clik here to view.
cbse-sample-papers-class-12-economics-delhi-2010-13

Image may be NSFW.
Clik here to view.
cbse-sample-papers-class-12-economics-delhi-2010-14

(c) True. Because if marginal product (MP) falls, total product (TP) increase at decreasing rate and finally average product (AP) falls. See diagram (b).

Question.16. Explain the law of variable proportions with the help of total product and marginal – product curves.
Answer. See Q. 15, 2008 (I Delhi)
Or
Explain producer’s equilibrium with the help of a marginal cost and marginal revenue schedule.
Answer. Producer’s equilibrium is established at a point where marginal cost and marginal revenue are equal and marginal cost should be rising. Any deviation from this position will either increase the loss or reduce the profit. Therfore, as long as the cost of an additional unit produced, i.e. marginal cost is less than the additional revenue obtained from its sale, i.e. marginal revenue, the firm goes on increasing its output. But it stops at that point where MC and MR are equal. If the fjrm goes beyond this point, MC will exceed MR and the firm will incur loss. Therefore, the producer’s equilibrium is always at a point, where MC and MR are equal.
According to cost-revenue schedule^equilibrium output will be 4 units, because at this level MC and MR are equal and MC is rising.
Image may be NSFW.
Clik here to view.
cbse-sample-papers-class-12-economics-delhi-2010-15

Note: MC = MR at 2 units of output also, but there the MC is falling with increase in output. Hence equilibrium is not at the level of output equal to 2 units but at the level of output equal to 4 units.

SECTION B
Question.17. State the components of money supply.
Answer. Components of money supply are currency held by the public and demand deposits.

Question.18. Give the meaning of ex-ante savings.
Answer. Ex-ante savings mean planned savings.

Question.19. How is primary deficit calculated?
Answer. Primary deficit = Fiscal deficit – Interest payments

Question.20. Give the meaning of deflationary gap.
Answer. The deficiency in aggregate demand in comparison to aggregate supply at the full employment level is deflationary gap.

Question.21. State two sources nffupply of foreign exchange.
Answer. Two sources of supply of foreign exchange:
(i) Payments available from the supply of goods and services to foreign countries.
(ii) Investment by foreign companies in the domestic market.

Question.22. Explain how distribution of gross domestic product is its limitation as a measure of economic welfare.
Answer. It is not only the quantum of GDP which affects economic welfare but also the pattern of the distribution of GDP. If GDP is distributed in a manner which increases income-inequalities, it will adversely affect economic welfare. This is so because the utility of income is more for the poor than for the rich.
Or
Explain the basis of classifying goods into intermediate and final goods. Give suitable examples.
Answer. Intermediate goods refer to those goods and services which are purchased during the year by one
production unit from other production units and completely used up, or resold during the same year. Example : Tyres used by car manufacturers.
Final goods are goods and services purchased, or own-produced for the purpose of consumption and investment. Example : Bread purchased by household, machine purchased by a firm.

Question.23. Explain the ‘lender of last resort’ function of the Central Bank.
Answer. When commercial banks exhaust all resources to supplement their funds at the time of liquidity crisis, they turn to the central bank as a last resort.
As the lender of last resort, central bank guarantees solvency and provides financial support to commercial banks
(i) by rediscounting their eligible securities and bills of exchange and
(ii) by providing loans against their securities. This saves the banks from failure and the banking system from a breakdown.

Question.24. How can Government budget be helpful in altering distribution of income in an economy? Explain.
Answer. Government budget can be ah effective instrument of changing the pattern of distribution of income with a view to help weaker section of society and promoting social justice. It does so by taxing the rich and providing subsidies to the low income group people. It also tries to incur public expenditure on such schemes which help the poor the most such as providing health care and education.

Q.25. Explain the meaning of deficit in balance of payments.
Ans. The balance of payments (BOP) will be in deficit when autonomous receipts are less than autonomous payments. The deficit in the BOP is restored by way of borrowing from IMF (International Monetary Fund), which is an accommodating item, so that BOP is always balanced.

Question.26. Distinguish between devaluation and depreciation of domestic currency.
Answer. When Government decides to lower down the value of its currency in terms of a foreign currency, it is called devaluation. In this case Government reduces the external value of its currency. As such our currency becomes cheaper in terms of file foreign currency.
A raise in exchange rate due to increase in the demand for foreign currency leads to appreciation of foreign currency (say, US dollar) and depreciation of domestic currency (say, Indian rupee). In this situation more rupees will have to be paid for buying goods worth one dollar from United States.

Question.27. Explain the proems of money creation by Commercial Banks.
Answer. Commercial banks accept deposits from the public known as primary deposits and keeping a percentage of these deposits in reserve as loans (known as Legal Reserve Ratio), the banks ‘ advance the balance of the deposits to other customers. While advancing them loans banks deposit the amount in their accounts and again keeping a percentage of the deposits (Legal Reserve Ratio), further advance the balance amount to others. This process continues and is known as credit creation by banks. In this way banks create credit many times more than the primary deposits.
The phenomenon of credit creation can be explained by taking a hypothetical example, where primary deposit is Rs. 1,000 and legal reserve ratio (LRR) is 20%. Say, Mr A. deposits Rs. 1,000 with the Bank. Keeping Rs. 200 (20%) in reserve, the balance Rs. 800 is advanced as loan to B, keeping Rs. 160 (20%) in reserve, the balance Rs. 640 is advanced as loan to C. This process cpntinues and finally total deposits created are five times more. So money multiplier is 5. This process becomes clftar, looking at the schedule given below:
Image may be NSFW.
Clik here to view.
cbse-sample-papers-class-12-economics-delhi-2010-16

(or)
How do changes in bank rate affect money creation by Commercial Banks? Explain.
Answer. Bank rate is the rate of interest which central bank charges from commercial banks for giving them credit. Thus, when bank rate increases, rate of interest also increases and the demand for credit goes down and vice-versa. As such, by increasing bank rate, volume of credit can be reduced and by lowering bank rate volume of credit can be increased.

Question.28. State whether the following statements are true or false. Give reasons for your answer.
(a) When marginal propensity to consume is greater than marginal propensity to save, the value of investment multiplier will be greater than 5.
(b) The value of marginal propensity to save can never be negative.
Answer.
Image may be NSFW.
Clik here to view.
cbse-sample-papers-class-12-economics-delhi-2010-17

Question.29. Distinguish between:
(a) Capital receipts and revenue receipts.
(b) Direct tax and indirect tax.
Answer. (a) Capital receipts are those which are available to the Government for incurring liabilities such as takinjpfoans. On the other hand, reveittte receipts are those for which Government is under no liability to pay them back Such as tax receipts etc.
(b) Direct taxes are those whose real burden is borne by those upon whom they are levied. Income-tax is a direct tax. Its burden cannot be shifted. On the other hand, indirect taxes are those whose real burden is not felt in real frame by those upon whom these are levied. Excise duty and sales tax are the examples of indirect taxes. These taxes are levied upon traders and manufacturers but their burden is passed on to the users of these goods, who use the goods which are taxed.

Question.30. How will you treat the following while estimating National Incoine Of India?
(a) Dividend received by an Indian from his investment in shares Of a foreign Company.
(b) Money received by a family in India from relatives working abfoad.
(c) Interest received on loans given to a friend for purchasing a car.
Answer. (a) Dividend is a part of profit and comes as factor income from abroad and is a part of domestic income, therefore it will be included in the estimation of National Income.
(b) It is not included in the National Income, because it is not a result of flow of goods and services.
(c) Loan given for the consumption purpose is not included in the estimation of National Income because it does not relate to flow of goods and services.

Question.31. From the following data, calculate (a) Gross Domestic Product at Factor Cost and (b) Factor Income to Abroad.
Image may be NSFW.
Clik here to view.
cbse-sample-papers-class-12-economics-delhi-2010-18

Answer.
Image may be NSFW.
Clik here to view.
cbse-sample-papers-class-12-economics-delhi-2010-19

Question.32. In an economy 75 per cent of the increase in income is spent on consumption. Investment is increased by crores. Calculate:
(a) Total increase in income
(b) Total increase in consumption expenditure
Answer.
Image may be NSFW.
Clik here to view.
cbse-sample-papers-class-12-economics-delhi-2010-20

SET II

Note : Except for the following questions, all the remaining questions .
SECTION A
Question.1. Define an indifference map.
Image may be NSFW.
Clik here to view.
cbse-sample-papers-class-12-economics-delhi-2010-21

Answer. It is a set of indifference curves representing different levels of satisfaction, as shown in the diagram.

Question.6. Distinguish between ‘decrease in demand’ and ‘decrease in quantity demanded’.
Answer. When demand decreases due to factors other than price, such a fall in demand is said to be decrease in demand. In this case, as shown in diagram (i) below, a leftward shift takes place in the demand curve.
When demand falls due to increase in the price of a commodity, there is an upward movement on the demand curve. This situation is shown in diagram (ii) below.
Image may be NSFW.
Clik here to view.
cbse-sample-papers-class-12-economics-delhi-2010-22

Question.8. Price of commodity A is Rs. 10 per unit and total revenue at this price is Rs. 1,600. When its price rises by 20 per cent, total revenue increases by Rs. 800. Calculate its price elasticity of supply.
Answer.
Image may be NSFW.
Clik here to view.
cbse-sample-papers-class-12-economics-delhi-2010-23

Question.11. When the price of commodity falls by Rs. 2 per unit, its quantity demanded increases by 10 units. Its price elasticity of demand is (-) 1. Calculate its quantity demanded at the puce before change which was Rs. 10 per unit
Answer.
Image may be NSFW.
Clik here to view.
cbse-sample-papers-class-12-economics-delhi-2010-24

Question.12. Explain the effect of increase in income of buyers of a ‘normal’ commodity on its equilibrium price.
Answer. An increase in income would result in an increase in the demand for ‘normal’ commodity. So the demand curve will shift to the right. Given the supply curve as SS in the diagram new equilibrium will be \({ E }_{ 1 }\) and the equilibrium price will now be higher at \({ P }_{ 1 }\) and quantity will also be larger.
Image may be NSFW.
Clik here to view.
cbse-sample-papers-class-12-economics-delhi-2010-25

Question.15. State whether the following statements are true or false. Give reasons for your answer
(i) When there are diminishing returns to a factor, total product first increases and then starts falling,
(ii) When marginal revenue falls to zero, average revenue becomes maximum.
(iii) The difference between total cost and total variable cost falls with increase in output.
Answer. (i) True. In this case first total product increases at diminishing rate and later starts declining; see diagram(i).
(ii) False. When MR. is zero, total revenue is maximum and not the AR; see diagram (ii).
(iii) False. The difference between TC and TVC is total fixed cost (TFC) which remains . constant; see diagram (iii).
Image may be NSFW.
Clik here to view.
cbse-sample-papers-class-12-economics-delhi-2010-26

SECTION B
Question.21. Howis primary deficit calculated?
Answer. Primary deficit is the difference of fiscal defidt and interest payments. In short, if we subtract interest payments from fiscal defidt, we get primary defidt.
Primary Defidt = Fiscal Defidt – Interest Payments

Question.22. Distinguish betweertreal and nominal gross domestic product.
Answer. Real gross domestic product may be defined as the gross money value of final goods and services produced within the domestic territory of a country. Gross money value is calculated at constant prices and therefore real gross domestic produd indicates real changes. On the other hand, nominal gross domestic product is the money value of final goods and services produced within the domestic territory of a country. Gross money Value in the case of nominal GDP is calculated at current prices. Therefore the only difference between real GDP and nominal GDP is that while real GDP is calculated using constant prices, nominal GDP is calculated using current prices.
Or
Distinguish between consumer goods and capital goods. Which of these are final goods?
Answer.
Image may be NSFW.
Clik here to view.
cbse-sample-papers-class-12-economics-delhi-2010-27

Note: Goods cannot be classified absolutely as consumer goods or capital goods because it depends upon the use a good is meant for. For example, a washing machine used for a household is a consumer good but a washing machine used in a factory is a capital good.

Question.23. Explain the ‘banker to the government’ function of the central bank.
Answer. The Central bank acts as Government’s banker, agent and advisor.
The RBI conducts the banking accounts of government departments and performs the same banking functions as a commercial bank performs for its customers.
RBI manages public debt and as agent of die government undertakes payment of interest on debt and other such services.
It gives expert advice to government on economic policy matters and money market.
Question.29. Giving reasons classify the following into direct and indirect tax:
(i) Welath tax (ii) Value added tax
Answer. (i) Wealth tax is a direct tax because its real burden is borne by those upon whom it is levied. In short, its burden cannot be transferred to others as happens in the case of indirect taxes.
(ii) Value’added tax is an indirect tax because it is generally paid by producers and sellers and they transfer its burden to consumers. In short, the burden of this tax falls on the shoulde of the consumers.

Question.31. From the following data, calculate (a) Gross Domestic Product at Factor Cost and (b) Factor Income to Abroad.
Image may be NSFW.
Clik here to view.
cbse-sample-papers-class-12-economics-delhi-2010-28

Answer.
Image may be NSFW.
Clik here to view.
cbse-sample-papers-class-12-economics-delhi-2010-29

Calculate Net National Product at Factor Cost and Gross National Disposable Income from the following:
Image may be NSFW.
Clik here to view.
cbse-sample-papers-class-12-economics-delhi-2010-30

Image may be NSFW.
Clik here to view.
cbse-sample-papers-class-12-economics-delhi-2010-31

Answer.
Image may be NSFW.
Clik here to view.
cbse-sample-papers-class-12-economics-delhi-2010-32

SET III

Note: Except for the following questions, all the remaining questions have been asked in Set I and Set II.
SECTION A
Question.1. What is law of demand?
Answer. Law of demand explains the relationship between the price and quantity demanded of a commodity. This law states that price and quantity demanded are inversely related, other things being constant. According to the Law of Demand, demand for a commodity rises with a fall in its price and falls with a rise in its price, other factors remaining constant.

Question.6. Explain any two factors that affect price elasticity of demand.
Answer. Two factors affecting price elasticity of demand:
(1) Price level. Generally the demand for very costly and cheap goods is inelastic. Highly priced commodities, such as diamonds, have a low price elasticity since a change in their prices has very little effect on their consumers. Cheap commodities like pencils also have low price elasticity as the demand for these does not change when price changes.
(2) Availability of substitutes. The demand for a commodity will be very elastic if some other commodities can be used in its place. When a commodity has a large number of close substitutes, demand for it is usually very elastic-
For example, gas, kerosene oil, coal, etc. will be used more as fuel if the price of wood increases. On the other hand, the demand of commodities is inelastic which have no substitutes at all, such as salt. u- y

Question.8. Total revenue at a price of Rs. 4 per unit of a commodity is Rs. 480. Total revenue increases by Rs. 240 when its price rises by 25 per Cent. Calculate its price elasticity of supply.
Answer.
Image may be NSFW.
Clik here to view.
cbse-sample-papers-class-12-economics-delhi-2010-33

Question.9. Explain the implication of ‘homogeneous products’ feature of perfect competition.
Answer. An important characteristic of perfect competition is that he products are homogeneous. As a result of this feature all sellers of the commodity charge the same price. If a seller charges’ a price less than the market price, all the buyers will buy things from him. With the result, the other sellers will also reduce their price. On the other hand, if a seller charges a price higher than the market price no buyer will buy his product. As such, in all situations and cases, market price fixed by supply and demand of the industry is accepted by all the sellers.

Question.14. State and explain the characteristics of indifference curves.
Answer. (i) An indifference curve always slopes downwards to the right. In other words, it has a negative slope. To consume more of one good, the consumer must give up some quantity of the other good so that total utility remains the same.
Image may be NSFW.
Clik here to view.
cbse-sample-papers-class-12-economics-delhi-2010-34

(ii) Higher,, indiference curve denotes higher level of satisfaction. This is becuase higher IC represents more of atleast one commodity, which means more utility as more is preferred to less.
(iii) Two indifference curves never intersect each other because at the point of intersection, the level of satisfaction is the same hut at other points it is not the same. It is not possible. Therefore, it is wrong to suggest they intersect as shown in Diagram 3.

SECTION B
Question.18. Give the meaning of aggregate demand.
Ans. Aggregate demand refers to the total expenditure which is incurred on consumption and investment goods.

Question.25. Distinguish between autonomous and accommodating transactions of balance of payments account.
Ans. Autonomous transactions refer to those international economic transactions which are taken with the motive of profit. These transactions are not dependent on or linked with the country’s balance of payments position.
Accommodating transactions refer to those transactions which are taken up by the Government in order to keep the balance of payments balanced.
For example, to meet its deficit, Government borrows money from IMF.

Question.26. Giving two examples, explain the relation between the rise in price of a foreign currency and its demand.
Ans. There is an inverse relationship between price of a foreign currency and its demand. Therefore if there is an increase in the price of a foreign currency its demand will fall. This we can explain with the help of an example.
For example,
(i) If value of the American dollar increases in terms of Indian rupees, then we will have to pay more for buying the same amount of goods from America. As a result of this, imports will fall meaning thereby that the demand for US dollar will fall.
(ii) If price of the foreign currency, say US dollar, in terms of rupee rises, then demand for US services by Indians, such as banking, insurance, etc., will fall resulting in fall in the demand for US dollar.

Question.28. Giving reasons, state whether the following statements are true or false:
(i) Average propensity to save is always greater than zero.
(ii) Value of investment multiplier varies between zero and infinity.
Answer. (i) The statement is noj; true. APS can be negative when APC is more than income and APC is greater thanl. This happens when consumption is more than income.
(iii) The statement is true. If MPC is 0, then the value of multiplier will be 1, and, if MPG is 1 then value of multiplier will be \(\infty\)

The post CBSE Sample Papers for Class 12 Economics Delhi 2010 appeared first on Learn CBSE.

CBSE Sample Papers for Class 12 Economics Examination & on wards – 2016

CBSE Sample Papers for Class 12 Economics Examination & on wards – 2016

Time allowed : 3  hours                                                                                         Maximum marks 100

GENERAL INSTRUCTIONS
(i) All questions in both the sections are compulsory.
(ii) Marks for questions are indicated against each.
(iii) Questions No. 1-5 and 17-21 are very short-answer questions carrying 1 nick each. They are required to be answered in one sentence each.
(iv) Questions No. 6-10 and 22-26 are? short-answer questions carrying 3 marks each. Answers to them should normally not exceed 60 words each.
(v) Questions No. 11-13 and 27-29 are also short-answer questions carrying 4 marks each. Answers to them should normally not exceed 70 words each.
(vi) Questions No. 14-16 and 30-32 are long-answers questions carrying 6 marks each. Answers to them should normally not exceed 100 words each.
(vii) Answers should be brief and to the point and the above word limit should be adhered to as far as possible.

SECTION A: Microeconomics

Question.1. If it is given that the total variable cost for producing 15 units of output is Rs 3,000 and for 16 units is Rs 3,500, find the value of Marginal Cost.
Answer.
Image may be NSFW.
Clik here to view.
cbse-sample-papers-for-class-12-economics-examination-on-wards-2016-1

Question.2. Ceteris Paribus, if the government provides subsidies on electricity bills, what would be the likely change in the market demand of desert coolers?
Answer. Keeping other factors constant, demand for desert coolers will increase if government provides subsidies on electricity bills.

Question.3. Which of the following can be referred to as ‘point of satiety’?
(i) Marginal Utility is negative (iii) Total Utility is rising
(it) Marginal Utility is zero (iv) Total Utility is falling
Answer. (ii) Marginal Utility is zero.

Question.4. Which of the following is an assumption of Production Possibility Frontier?
(i)Resources are not fully employed.
(ii) Resources are not equally efficient for production of the two goods.
(iii) Resources are not efficiently employed.
(iv) Resources available are not fixed.
Answer. (ii) Resources are not equally efficient for the production of the two goods.

Question.5. State any two central problems under ‘problem of allocation of resources’.
Answer. (i) What to produce and in what quantity? To allocate resources in a manner which gives maximum aggregate satisfaction.
(ii) How to produce? To combine factors of production in a manner which gives maximum output at minimum cost using least possible scarce resources.
(iii) For whom to produce? To ensure that the urgent wants of each productive factor are
fulfilled to the maximum possible extent. (any two)

Question.6. ‘Supply curve is the rising portion of Marginal Cost curve over and above the minimum of Average Variable Cost curve’. Do you agree? Support your answer with valid reason.
Answer. Yes, we do agree with the given statement. Supply curve indicates the relationship between price and quantity supplied. In the short run, supply can be changed by changing only the variable factors. In the short run, the firm only needs to cover its variable costs and it will supply the commodity till price is either greater or equal to average variable cost. Therefore, the given statement that the supply curve is the rising portion of marginal cost curve over and above the minimum of average cost curve is true. This is because no rational producer/ seller would like to supply his output to the market if he is unable to recover his per unit variable cost as it would lead to losses between the range of minimum ?of marginal cost (additions to variable cost) and minimum of average variable cost.

Question.7. Explain ‘black marketing’ as a direct consequence of price ceiling.
Answer. Black marketing may be termed as a direct consequence of price ceiling, as it implies a situation whereby the commodity under the government’s control policy is illegally sold at a higher price than the one fixed by the government. It may primarily arise due to the presence of consumers who may be willing to pay higher price for the commodity than to go without it. Price ceiling creates shortage of a good in the market. This may lead to black marketing.
Or
Explain the concept of ‘buffer stock’ as a tool of price floor.
Answer. Buffer stock is an important tool in the hands of the government to ensure price floor/ minimum support price. If in case the market price is lower than what the government feels should be given to the farmers/ producers, it would purchase the commodity at a higher price from the farmers/producers so as to maintain stock of the commodity with itself to be released in case of shortage of the commodity in future.

Question.8. Explain any two sources of restricted entry under monopoly.
Answer. Sources of restricted entry under monopoly, may be:

  1. Government license. Some monopolies are created by law in public interest. Most of the state monopolies in the public utility sector, e.g., post, telegraph and telephone services, state roadways etc. are public monopolies. By not granting licenses to new firms, the government aims to assure that only one firm operates in the market.
  2. Patents, trademarks & copyrights. Another source of monopoly is the patent rights of the firm for a product or for a production process. Patent rights are granted by the government-to -a firm to produce a commodity of specified quality or to use a specified technique of production.
  3. Ownership of scarce resource. Monopoly also arises due to sole ownership or control of certain essential raw materials needed in a particular industry. Some firms acquire monopoly power from their legally granted control over certain scarce and key raw materials that are essential for the production of certain other goods. E.g., bauxite, graphite, etc. (any two)

Question.9. Comment upon the degree of elasticity of demand for Good X, in the following given situations, if the price of the commodity rises from Rs5 per unit to Rs7 per unit and the quantity demanded falls from 20 units to 16 units:
(i) Using the total household expenditure method,
(ii) Using proportionate method.
Answer.
Image may be NSFW.
Clik here to view.
cbse-sample-papers-for-class-12-economics-examination-on-wards-2016-2

Question.10. ‘Higher indifference curve represents higher level of satisfaction to the consumer’. Explain the statement, also state the underlying assumption related to this property of indifference curve.
Answer. Higher indifference curve represents higher level of satisfaction. In other words, any combination that lies on a higher indifference curve, i.e., away from the origin represents a higher level of satisfaction.
In the diagram given , Point A on 1C1 represents the combination (OX1, OY1) and Point B on IC2 represents the combination (OX2, OY1).
Since, Combination B > Combination A OX2, OY1 > OX1 , OY1
Therefore, the consumer gets more satisfaction at IC2 as it represents more of one commodity while the amount of the other remains the same.
Image may be NSFW.
Clik here to view.
cbse-sample-papers-for-class-12-economics-examination-on-wards-2016-3

The underlying assumption here is the assumption of monotonic preference which represents that a consumer will prefer a combination which contains more of at least one and no less of the other.
Or
A consumer consumes two goods X and Y. Explain what will happen if \(\frac { { MU }_{ X } }{ { P }_{ X } } \) is greater than \(\frac { { MU }_{ Y } }{ { P }_{ XY} } \)?
Answer. If \(\frac { { MU }_{ X } }{ { P }_{ X } } \) is greater than \(\frac { { MU }_{ Y } }{ { P }_{ Y } } \), then it means the satisfaction that the consumer derives from spending a rupee on Good X is greater than the satisfaction derived from spending a rupee on Good Y.
The consumer will reallocate his income by substituting Good X for Good Y. As the consumption of Good X increases, the marginal utility derived from it goes on diminishing (Law of diminishing marginal utility) and reverse proposition occurs for Good Y. This process will continue till \(\frac { { MU }_{ X } }{ { P }_{ X } } \) becomes equal to \(\frac { { MU }_{ Y } }{ { P }_{ Y } } \)

Question.11. Define Marginal Opportunity Cost. Explain the concept with a hypothetical numerical example.
Answer. Marginal Opportunity Cost (MOC) of a given commodity along a PPC is defined as the amount of sacrifice of a commodity so as to gain one additional unit of the other commodity. MOC can also be termed as Marginal Rate of Transformation i.e., the ratio of number of units of a good sacrificed to produce an additional unit of the other good.
Image may be NSFW.
Clik here to view.
cbse-sample-papers-for-class-12-economics-examination-on-wards-2016-4

As shown in the schedule, if the economy uses all its resources to produce only guns, then a maximum of 21 units of guns and no butter can be produced. On the other hand, if all resources are used for butter, then a maximum of 6 units can be produced. In between there are various possibilities with different combinations of the two commodities.

Question.12. (a) What is meant by price rigidity, under oligopoly?
(b) Elaborate the implication of the conditions of equilibrium of a firm.
Answer. (a) Price rigidity is the price of the product fixed after deliberations and negotiations by the oligopolistic firms, to which they generally stick, with a view to avoid any sort of price, war. Firms use other methods like advertising, better services to customers, etc. to compete with each other.
(b) The equilibrium of a firm is that level of output where its profits are maximized and at that level of output, its marginal cost equals marginal revenue. Conditions of firm’s equilibrium:
(i) Marginal Revenue must be equal to Marginal Cost.
(ii) Marginal Cost must be rising after the MC = MR output level.
The conditions imply that a firm would keep on producing and maximise its profits till the slope of rising Marginal Cost Curve is equal to the slope of Marginal Revenue Curve.
In the diagram, E represents the point where the Slope of MC and MR curve are equal.
Implication of the conditions lies in the fact that beyond the equilibrium point, MC would become greater than MR, i.e., for each additional unit sold beyond output OQ, the cost of producing that unit will be more than the revenue generated by the unit. Before this output level (OQ), since MC is less than MR, it is possible to add to profits by producing more.
Image may be NSFW.
Clik here to view.
cbse-sample-papers-for-class-12-economics-examination-on-wards-2016-5

Question.13. (a) Distinguish between stock and supply.
(b) Complete the following schedule:
Image may be NSFW.
Clik here to view.
cbse-sample-papers-for-class-12-economics-examination-on-wards-2016-6

Answer. (a) Supply refers to that quantity of a commodity which a seller is willing to sell at different prices during a given period of time. It is that part of stock which is offered for sale at any time.
Image may be NSFW.
Clik here to view.
cbse-sample-papers-for-class-12-economics-examination-on-wards-2016-7

Question.14. Suppose the demand and supply curves of a Commodity-X is given by the following . two equations simultaneously:
\({ Q }_{ d }\)= 200 – p \({ Q }_{ s }\)= 50 + 2p
(i) Find the equilibrium price and equilibrium quantity.
(ii) Suppose that the price of a factor of production producing the commodity has changed, resulting in the new supply curve given by the equation:
\({ Q }_{ s }\)= 80 + 2p
Analyse the new equilibrium price and new equilibrium quantity as against the original equilibrium price and equilibrium quantity.
Answer.
Image may be NSFW.
Clik here to view.
cbse-sample-papers-for-class-12-economics-examination-on-wards-2016-8

Question.15. Show diagrammatically the conditions for consumer’s equilibrium, in Hicksian analysis of demand.
Answer.
Image may be NSFW.
Clik here to view.
cbse-sample-papers-for-class-12-economics-examination-on-wards-2016-9

SECTION B: Macroeconomics

Question.16. If an economy is to control recession like most of the Euro-Zone nations, which of the following can be appropriate:
(i) Reducing Repo Rate (ii) Reducing CRR
(iii) Both (i) and (it) (iv) None of (i) and (ii)
Answer. (iii) Both (i) and (ii)

Question. 17. Which of the following agency is responsible for issuing *1 currency note in India? 1 (i) Reserve Bank of India (ii) Ministry of Commerce
(iii) Ministry of Finance (iv) Niti Aayog
Answer. (iii) Ministry of Finance

Question. 18. Flow of goods & services and factors of production across different sectors in a barter economy is known as:
(i) Circular flow (ii) Real Flow
(iii) Monetary Flow (iv) Capital Flow .
Answer. (ii) Real Flow

Question. 19. The government budget of a hypothetical economy presents the following information. Which of the following value represents Budgetary Deficit? (all fig. in Rs crores)
A. Revenue Expenditure = 25,000
B. Capital Receipts = 30,000
C. Capital Expenditure = 35,000
D. Revenue Receipts = 20,000
E. Interest Payments = 10,000
F. Borrowings = 20,000
(i) Rs 12,000 (ii) Rs 10,000
(iii) Rs 20,000 (iv) None of the above
Answer. (iv) None of the above .
Budgetary Deficit = Revenue Expenditure + Capital Expenditure – (Revenue Receipts + Capital Receipts)
= 25,000 + 35,000 – (20,000 + 30,000) = Rs 5,000 crores

Question. 20. Which of the following statement is true?
(i) Loans from IMF is a Revenue Receipt,
(ii) Higher revenue deficit necessarily leads to higher fiscal deficit.
(iii) Borrowing by a government represents a situation of fiscal deficit.
(iv) Revenue deficit is the excess of capital receipts over the revenue receipts.
Answer. (iii) Borrowing by a government represents a situation of fiscal deficit.

Question .21. ‘Devaluation and Depreciation of currency are one and the same thing’. Do you agree? How do they affect the exports of a country?
Answer. Devaluation is the fall in the value of domestic currency in relation to foreign currency as planned by the government in a situation when exchange rate is not determined by the forces of demand and supply but is fixed by the government of different countries. Depreciation is the fall in the value of domestic currency in relation to foreign currency in a situation when exchange rate is determined by the forces of demand and supply in the international money market.
As a general phenomena, any depreciation/devaluation of currency may result into increase in exports of the goods and services from the country since more can now be purchased from India with the same amount of dollars. Also, it would increase the global competitiveness of the goods.

Question.22. If in an economy Saving function is given by S = (-)50 + 0.2 Y and Y = Rs 2,000 crores; consumption expenditure for the economy would be Rs 1,650 crores and the auto-nomous investment is Rs 50 crores and the marginal propensity to consume is 0.8. True or False?
Justify your answer with proper calculations.
Answer.
Image may be NSFW.
Clik here to view.
cbse-sample-papers-for-class-12-economics-examination-on-wards-2016-10

Or
“Economists are generally concerned about the rising Marginal Propensity to Save (MPS) in an economy”. Explain why?
Answer. Since the sum of MPC and’MPS is unity, any increase in Marginal Propensity to Save (MPS) would directly lead to a decrease in Marginal Propensity to Consume (MPC). This would mean that lesser proportion of the additional income is now being consumed. Consumption expenditure being a major factor of Aggregate Demand/Expenditure. This may further lead to fall in equilibrium level of income in the economy as reduction in aggregate demand would lead to reduction in aggregate supply.

Question.23. Explain how the economy achieves equilibrium level of income using Savings- Investment (S-I) approach.
Answer. As per the S-I approach, equilibrium is achieved where ex-ante Savings are equal to ex- ante Investments. Savings and investments indicate leakages and injections respectively, thus at equilibrium the leakages and injections are equal to each other. If there is any deviation from the equilibrium level of income, i.e., if planned saving is not equal to planned investment, than a process of readjustment will start which will bring the economy back to the equilibrium level.

Question.24. Suppose in an imaginary economy GDP at Market Price in a particular fiscal year was Rs 4,000 crores, National Income ‘was Rs 2,500 crores, Net Factor Income paid by the economy to Rest of the World was Rs 400 crores and the value of Net Indirect Taxes is Rs 450 crores. Estimate the value of consumption of fixed capital for the economy from the given data.
Answer.
Image may be NSFW.
Clik here to view.
cbse-sample-papers-for-class-12-economics-examination-on-wards-2016-11

Question.25. What is meant by ‘official reserve transactions’? Discuss their importance in Balance of Payments.
Answer. Transactions by a Central Bank that cause changes in its official reserves are called official reserve transactions. These are usually purchases or sales of its own currency in the exchange market in exchange for foreign currencies or other foreign currency denominated assets. These accommodating transactions are meant to correct the disequilibrium between autonomous transactions, i.e., autonomous receipts and autonomous payments which may occur as deficit/surplus in Balance of Payments.

Question.26. State the various components of the Expenditiue Method that are used to calculate national income.
Answer. Components of Expenditure method:
(i) Private Final Consumption Expenditure (ii) Government Final Consumption Expenditure (iii) Investment Expenditure
(iv) Net Exports
Or
Discuss any two differences between GDP at constant prices and GDP at current prices.
Answer. Two main differences between GDP at current prices and at constant prices are:

  1. GDP at current prices is measured at current year’s prices whereas GDP at constant prices is measured at base year’s prices.
  2. GDP at current prices may increase even if there is no flow of goods and services whereas GDP at constant prices will only increase when there is an increase in the flow of goods and services.

Question.27, “Governments across nations are too much worried about the term fiscal deficit”. Do you think that fiscal deficit is necessarily inflationary in nature? Support your answer with valid reasons.
Answer. The term fiscal deficit is the difference between the government’s total expenditure and its total receipts (excluding borrowings). It indicates how far the government is spending beyond its means and represents the total borrowings of the government from all sources during the current year.
Fiscal Deficit = Total expenditure .Total receipts excluding borrowings Such borrowings are generally financed by issuing new currency called as deficit financing which may lead to inflation in the economy due to more money supply. However, if the borrowings are for infrastructural developmental purposes this may lead to capacity building and may not be inflationary. If such borrowings lead to increase in productivity and capital formation, then it may not generate inflationary trends in the economy.

Question. 28. Derive a straight line saving curve using the following consumption function: C = 20 + 0.6Y .
Presuming the income levels to be Rs 100,Rs 200 and Rs 300 crores. Also calculate that level of income where consumption is equal to income.
Answer.
Image may be NSFW.
Clik here to view.
cbse-sample-papers-for-class-12-economics-examination-on-wards-2016-12

Image may be NSFW.
Clik here to view.
cbse-sample-papers-for-class-12-economics-examination-on-wards-2016-13

For Visually Impaired candidates:
Calculate savings from the following saving function: C = 20 + 0.6Y Presuming the income levels to be Rs 100, Rs 200 and Rs 300 crores. Also calculate that level of income where consumption is equal to income.
Answer. See. Q. 28 above.

Question.29. (a) What is meant by Repo Rate? How does the Central Bank use this measure to control inflationary conditions in an economy?
(b) What is meant by Margin Requirement? How does the Central Bank use this measure to control deflationary conditions in an economy?
Answer. (a) Repo rate is the rate of interest at which the central bank lends money to commercial banks for a short-term. The central bank fixes the Repo Rate and it plays the role of 1 an indicator of the lending rate and deposit rate fixation by the banks. Under inflationary conditions central bank increases the Repo Rate. It makes borrowings by commercial banks costly and they also increase their lending rates. As a result, borrowings decrease, reducing the level of aggregate demand in the economy.
(b) Margin requirement refers to the difference between market value of the security offered for loans and the amount of loans offered by the commercial banks.The central bank fixes the margin requirements and under deflationary conditions central bank reduces the margin requirements so that the capacity to borrow is increased which will raise the level of aggregate demand in the economy to restore it at the full employment level.

Question.30. Compute (a) Domestic Income and (b) Net National Disposable Income.
Image may be NSFW.
Clik here to view.
cbse-sample-papers-for-class-12-economics-examination-on-wards-2016-14

Image may be NSFW.
Clik here to view.
cbse-sample-papers-for-class-12-economics-examination-on-wards-2016-15

Answer. Domestic Income
= (xi) + (ii) + (iv) – (vi) + (vii) – (x) – (viii)
= Private final consumption expenditure + Government final consumption expenditure + Gross domestic fixed capital formation – Net decrease in inventories + Net Exports – Current replacement cost – Net Indirect Taxes . = 2,200 + 2,500 + 1,190 – 100 + (-420) – 145 – 470
= Rs 4,755 crores .
Net National Disposable Income
= National Income + Net Indirect Taxes + Net Current Transfers from abroad
= Domestic Income – (v) + (viii) + (ix)
= Domestic Income – Net factor income to abroad + Net Indirect Taxes + Net Current transfers from abroad
= 4,755 – 125 +’470 + 350 = Rs 5,450 crores
Or
Explain any four limitations of using GDP as a measure/index of welfare of a country.
Answer. Four limitations of using GDP as a measure/index of welfare of a country are: ,

  1. Distribution of GDP. If the distribution of GDP is not uniform then even though GDP rises the welfare of the people may not rise because the rise in GDP may be concentrated in the hands of very few individuals or firms.
  2. Composition of GDP. GDP only shows the total goods and services produced and does not exhibit the structure of the product. If the increase in GDP is mainly due to increase in production of war equipments, then such an increase cannot be associated with any improvement in economic welfare.
  3. Non-Monetary Exchanges. Many activities in an economy are not evaluated in monetary terms and therefore are not included in estimates of GDP. This is a case of underestimation of GDP, e.g., services of a housewife.
  4. Externalities. It refers to the-benefits (or harms) a firm or an individual causes to another for which it is not paid (or penalized). Positive externalities though increase economic welfare are not included in GDP. Similarly, negative externalities, though decrease the actual welfare are not considered in GDP. Hence, GDP is not a true indicator of economic welfare.

The post CBSE Sample Papers for Class 12 Economics Examination & on wards – 2016 appeared first on Learn CBSE.


CBSE Sample Papers for Class 12 Economics Delhi – 2015

CBSE Sample Papers for Class 12 Economics Delhi – 2015

Time allowed : 3  hours                                                                                         Maximum marks 100

GENERAL INSTRUCTIONS
(i) All questions in both the sections are compulsory.
(ii) Marks for questions are indicated against each.
(iii) Questions No. 1-5 and 17-21 are very short-answer questions carrying 1 nick each. They are required to be answered in one sentence each.
(iv) Questions No. 6-10 and 22-26 are? short-answer questions carrying 3 marks each. Answers to them should normally not exceed 60 words each.
(v) Questions No. 11-13 and 27-29 are also short-answer questions carrying 4 marks each. Answers to them should normally not exceed 70 words each.
(vi) Questions No. 14-16 and 30-32 are long-answers questions carrying 6 marks each. Answers to them should normally not exceed 100 words each.
(vii) Answers should be brief and to the point and the above word limit should be adhered to as far as possible.

SET I

SECTION A
Question.1. Give equation of Budget Line.
Answer.
Image may be NSFW.
Clik here to view.
cbse-sample-papers-for-class-12-economics-delhi-2015-1

Question.2. When income of the consumer falls the impact on price-demand curve of an inferior good is: (choose the correct alternative)
(a) Shifts to the right.
(b) Shifts to the left.
(c) There is upward movement along the curve.
(d) There is downward movement along the curve.
Answer. (a) Shifts to the right

Question.3. If Marginal Rate of Substitution is constant throughout, the Indifference curve will be: (choose the correct alternative)
(a) Parallel to the x-axis. (b) Downward sloping concave.
(c) Downward sloping convex. (d) Downward sloping straight line.
Answer. (d) Downward sloping straight line.

Question.4. Giving reason comment on the shape of Production Possibilities Curve based on the following schedule:
Image may be NSFW.
Clik here to view.
cbse-sample-papers-for-class-12-economics-delhi-2015-2

Answer. The shape of PPC would be downward sloping and concave to the origin because of increasing marginal rate of transformation.
Marginal Rate of Transformation (MRT) is the ratio of number of units of a commodity sacrificed to gain an additional unit of another commodity.
Image may be NSFW.
Clik here to view.
cbse-sample-papers-for-class-12-economics-delhi-2015-3

Increasing MRT means that more and more of commodity Y will have to be sacrificed for a unit increase in the production of commodity X. This happens because the resources are not equally efficient in production of all commodities.

Question.5. What will be the impact of recently launched ‘Clean Indian Mission’ (Swachh Bharat Mission) on the Production Possibilities curve of the economy and why?
Answer. The ‘Clean India Mission’ has been recently launched by the government. A cleanliness drive would ensure a cleaner, healthier and hygienic environment which reduces chances of people falling ill, which in turn will reduce forced absenteeism from work thereby increasing efficiency and productivity levels along with better utilization of available resources. Rise in this potential shifts PPC to the right signifying an increase in its productive capacity.
Or
What will likely be the impact of large scale outflow of foreign capital on Production Possibilities curve of the economy and why?
Answer. Large scale outflow of foreign capital would lead to reduction in the resources available to an economy. Decline in resources would further lead to a decline in the country’s overall capacity .to produce. This would then result in a downward shift in the production possibilities curve because the economy can now produce less of both commodities keeping in mind the reduction in available resources.

Question.6. The measure of price elasticity of demand of a normal good carries minus sign while price elasticity of supply carries plus sign. Explain why?
Answer. The minus sign of the measure of price elasticity of demand of a normal good is indicative of the inverse relationship between price of a commodity and its quantity demanded. If price increases the quantity demanded of a commodity decreases and vice versa which is , why the co-efficient of price elasticity of demand is negative.
On the other hand, measure of price elasticity of supply carries plus sign due to the direct relationship between price of a commodity and its supply. If price of the commodity increases its supply would also increase and vice versa. This means that since both change in supply and change in price will be in the same direction, therefore the resultant answer . would automatically carry a positive sign.

Question.7. There are large number of buyers in a perfectly competitive market. Explain the significance of this feature.
Answer. In a perfectly competitive market, there are very large number of buyers and sellers. This means that the number of buyers is so large that any individual buyer is not in a position to influence the market price on its own by purchasing more or less. It is because the individual buyer’s share in total purchase in the market is insignificant.

Question.8. Explain the effects of ‘maximum price ceiling’ on the market of a good. Use diagram.
Answer. Maximum price ceiling refers to fixing the maximum price of a commodity by the government at a level lower than the equilibrium price.
In the diagram, demand curve and supply curve intersect each other at point E where the equilibrium price is OP and OP1 is the price ceiling. The equilibrium price OP being too high, many people cannot afford the commodity at this price. Wheat being an essential commodity, government fixes the maximum price at OP1 at which the quantity demanded exceeds the quantity supplied By \({ Q }_{ S }{ Q }_{ D}\) This creates a shortage in the market equal to MN(\({ Q }_{ S }{ Q }_{ D}\) ) which may ultimately lead to hoarding and black marketing.
To prevent this situation and to ensure that the basic demand of wheat consumers is met, the government has enforced the system of rationing through the. public distribution system. Through the system of rationing, the government tries to sell a minimum quota of essential commodities such as wheat below the equilibrium price or at cheaper price to the poor community.
Image may be NSFW.
Clik here to view.
cbse-sample-papers-for-class-12-economics-delhi-2015-4

For the blind candidates only in lieu of Q. No. 8.
What is price ceiling? Explain the effects of maximum price ceiling.
Answer. Price ceiling means putting the upper limit by the government on the price that can be charged by the producers of a good from the buyers. Maximum price ceiling is lower than equilibrium price, pleading to a rise in demand and fall in supply. This creates shortage of the good in the market which may further lead to black marketing and hoarding.

Question.9. A consumer spends Rs 1000 on a good priced at Rs 8 per unit. When price by 25 per cent, the consumer continues to spend Rs 1000 on the good. Calculate price elasticity of demand by percentage method.
Answer. 
Image may be NSFW.
Clik here to view.
cbse-sample-papers-for-class-12-economics-delhi-2015-5

Question.10. Define cost. State the relation between marginal cost and average variable cost.
Answer. Cost in economics refers to the shin of actual money expenditure on inputs and the imputed expenditure in the form of inputs supplied by the owners including normal profit.
Relation between MC and AVC:
(i) Both AVC and MC are derived from TVC and are U-shaped due to the law of variable proportions.
(ii) When MC is less than AVC , AVC falls.
(in) When MC is equal to AVC, AVC is constant and at its minimum point.
(iv) When MC is more than AVC, AVC rises with increase in output.
(v) Thereafter, both AVC and MC rise, but MC increases at a faster rate as compared to AVC ”
Or
Define revenue. State the relation between marginal revenue and average revenue.
Answer. Revenue in economics refers to the amount received by a firm from the sale of a given quantity of a commodity in the market or the market value of output produced.
Relationship between AR and MR:
(i) When price remains constant,
MR and AR curve is a horizontal straight line parallel to X axis as MR = AR = Price.
(ii) When price-falls with rise in output,
Both MR and AR fall with increase in output but MR falls at double rate than AR.
General Relationship between AR arid MR:
(i) AR increases as long as MR is higher than AR (MR>AR, AR increases)
(ii) When MR=AR, AR is maximum.
(iii) When MR<AR, AR falls.

Question.11. A consumer consumes only two goods X and Y both priced at Rs 3 per unit. If the consumer chooses a combination of these two goods with Marginal Rate of Substitution equal to 3, is the consumer in equilibrium? Give reasons. What will a rational consumer do in this situation? Explain.
Answer.
Image may be NSFW.
Clik here to view.
cbse-sample-papers-for-class-12-economics-delhi-2015-6

implying that the consumer is willing to sacrifice more units of Y as compared to what is required in the market. Therefore, a rational consumer will be induced to buy more units of X leading to a fall in MRS due to the Law of Diminishing Marginal Utility. This will continue till MRS becomes equal to the ratio of prices and equilibrium is established.
Or
A consumer consumes only two goods X and Y whose prices are Rs 4 and Rs 5 per unit respectively. If the consumer chooses a combination of the two goods with marginal utility of X equal to 5 and (hat of Y equal to 4, is the consumer in equilibrium? Give reasons. What will a rational consumer do in this situation? Use utility analysis.
Answer.
Image may be NSFW.
Clik here to view.
cbse-sample-papers-for-class-12-economics-delhi-2015-7

Question.12. State %e different phases of changes in Total Product and Marginal Product in the Law of Variable Proportions. Also show the same in a single diagram.
Answer. Law of variable proportions states that as we increase quantity of only one input keeping other inputs fixed, Total Product (TP) initially increases at an increasing rate (MP also increases and reaches its maximum point), then at a decreasing rate (MP falls but remains positive) and finally at a negative rate i.e., it falls (MP becomes negative).
This behavior of output can be shown through a diagram.
As shown in the diagram, the relation between input and output is discussed in three phases:
• Phase I: Increasing returns to a factor—TP increases at an increasing rate and MP also rises. This happens because as the variable factor increases, due to specialization its efficiency increases leading to better utilization of fixed factors.
• Phase II: TP increases at a decreasing rate and MP is declining but remains positive.This happens because with increase in the quantity
of variable input, a pressure is felt on the fixed factors. The amount of fixed input looks smaller than actually required to engage the increasing variable input reducing the efficiency of the & variable factor.
• Phase III: TP starts falling and MP becomes negative—This happens because the quantity of the variable input becomes too big in relation to the quantity of fixed inputs.
Image may be NSFW.
Clik here to view.
cbse-sample-papers-for-class-12-economics-delhi-2015-8

For the blind candidates in lieu of Q. No. 12 only.
State, on the basis of a numerical example, different phases of changes in Total Product and Marginal Product in the Law of Variable Proportions.
Answer. Law of Variable Proportions, also known as law of returns to a factor shows the nature of rate of change in output due to a change in variable factors. It shows how total product (TP) initially increases at an increasing rate, then at a decreasing rate and finally at a negative/rate due to change in only one variable factor of production.
This behaviour of output is shown through the following schedule.
Image may be NSFW.
Clik here to view.
cbse-sample-papers-for-class-12-economics-delhi-2015-9

Phase I: Increasing returns to a factor. As shown in the schedule, on unit of labour produces 10 units whereas 2 units of labour produce 30 units implying that initially TP increases at an increasing rate and MP rises till it reaches its maximum point.
Phase II: Diminishing returns to a factor. After the 2nd labour, every additional unit of labour that is . increased increases TP but the rate of increases is decreasing leading to decline in MP. This phase ends with,TP becoming maximum and MP becoming zero.
Phase III: Negative returns to a factor. After the 5th unit, when the 6th unit of labour is employed, then TP starts falling as MP becomes negative due to conflict between fixed and variable factors and decline in efficiency of the variable factor. This phase ends with TP becoming maximum and MP becoming zero.

Question.13. Why is the equality between marginal cost and marginal revenue necessary for a firm to be in equilibrium? Is it sufficient To ensure equilibrium? Explain.
Answer. Producers equilibrium refers to that price and output combination which brings maximum profit to the producer and profit declines as more is produced.
In order to reach equilibrium the first condition which has to be satisfied is that MC has to be equal to MR (MC = MR).
To maximise profits, a firm compares its MR with its MC. Profits will increase as long as MR exceeds MC and profits will fall if- MR is less than MC. So, equilibrium is not achieved when MC < MR as it is still possible to add to profits by producing more. A producer is also not in equilibrium when MC > MR because then benefit is less than cost. This means that the firm will only be in equilibrium when MC = MR.
However MC = MR, though a necessary condition, is not sufficient enough to ensure equilibrium. It is because MC =MR may occur at more than one level of output.
However, out of these, only that output level is the equilibrium output when MC becomes greater than MR after the equilibrium. It is because if MC is greater than MR, then producing beyond MC = MR output level will reduce profits.
So, the first condition must be supplemented with the second condition to attain producer’s equilibrium.
.•.The two necessary conditions, under. MR-MC approach to attain producer’s equilibrium are:
(i) MC = MR
(ii) MC is greater than MR after MR = MC output level.

Question.14. Market for a good is in equilibrium. The demand for the good ‘increases’. Explain the chain of effects of this change.
Answer. Market demand curve DD and market supply curve SS intersect each other at point E, which is the market equilibrium where OQ is the equilibrium quantity. and OP is the equilibrium price.
An increase in demand (assuming no change in supply and price) leads to a rightward shift in demand curve from DD to D1D1 as shown in the diagram.
When demand increases to D1D1, it creates excess demand at the old equilibrium price of OP. This leads to competition amongst buyers which pushes up the price. As price increases demand decreases (Law of demand) and supply increases (Law of supply) as indicated by the arrows. These changes continue till new equilibrium is established at point E1 where the new equilibrium price rises from OP to OP1 and equilibrium quantity rises from OQ to OQ1.
Image may be NSFW.
Clik here to view.
cbse-sample-papers-for-class-12-economics-delhi-2015-10

SECTION B
Question.15. What is ‘aggregate supply’ in macroeconomics?
Answer. Aggregate Supply refers to the money value of total quantity of final goods and services that all the producers are willing to supply in an economy in a given time period.

Question.16. The value of multiplier is: (choose the correct alternative)
(a) 1/MPC (b) 1/MPS (c) 1/1-MPS (d) 1/MPC-l
Answer. (b) 1/MPS

Question.17. Borrowing in government budget is: (choose the correct alternative)
(a) Revenue deficit (b) Fiscal deficit
(c) Primary deficit (d) Deficit in taxes
Answer. (b) Fiscal deficit

Question.18. The non-tax revenue in the following is: (choose the correct alternative)
(a) Export duty (b) Import duty (c) Dividends (d) Excise
Answer. (c) Dividends

Question.19. Other things remaining unchanged, when in a country the price ‘of foreign currency rises, national income is: (choose the correct alternative)
(a) Likely to rise (b) Likely to fall
(c) Likely to rise and fall both (d) Not affected
Answer. (a) Likely to rise

Question.20. If Real GDP is Rs 200 and Price Index (with base = 100) is 110, calculate Nominal GDP.
Answer.
Image may be NSFW.
Clik here to view.
cbse-sample-papers-for-class-12-economics-delhi-2015-11

Question.21. Name the broad categories of transaction? recorded in the ‘capital account’ of the Balance of Payments Accounts.
Answer. Capital account of Balance of Payments Account records all those transactions between the residents of a country and the rest of the world, which cause a change in the assets or liabilities of the residents of the country or its government.
Capital account is concerned with financial transfers. So it does not have any direct effect on income, output and employment of the country.
The categories of transactions recorded in the capital account are:
(i) Borrowings and lending to and from abroad. By private individuals, institutions, government, etc.
(ii) Investments to and from abroad. It includes investments from abroad (credits) in shares of Indian companies, real estate, etc. It also includes investment by Indian residents in shares of foreign companies, real estate abroad, etc.
(iii) Changes (Increase and Decrease) in foreign exchange reserves. Treated as a financial asset of the government, a withdrawal from this reserve leads to a decrease in foreign financial asset (recorded on credit side) and vice versa.
Or
Name the broad categories of transactions recorded in the ‘current account’ of the – Balance of Payments Accounts.
Answer. Categories of transactions recorded in the current account of the Balance of Payments Accounts are as follows:
(i) Export and Import of goods (visible). Balance of these visible exports and imports is known as balance of trade.
(ii) Export and Import of services (invisible trade), like shipping, banking, insurance, etc.
(iii) Unilateral transfers. It includes gifts, donations, personal remittances etc.
(iv) Income receipts and payments to and from abroad. It includes investment income in the form of interest, rent and profits.

Question.22. Where will sale of machinery to abroad be recorded in the Balance of Payments Accounts? Give reason.
Answer. Sale of machinery abroad will be considered as export of visible item of trade and will be a part of country visible trade. Visible trade, i.e., export and import of goods is recorded in Current account of Balance of Payments Accounts (BOP).
Since receipt from exports is .shown on the positive side (credit item), therefore Sale, of machinery abroad brings in foreign exchange and will be shown as credit item in the current account of BOP.

Question.23. Explain the ‘bank of issue’ function of the central bank.
Answer. Central Bank has the sole authority for issue of currency in the country. In India, RBI has the sole right of issuing paper currency notes (except one rupee notes and coins, which are issued by the Ministry of Finance). The reason for this is that it brings about uniformity in note circulation and gives the Central Bank direct control over the money supply. It promotes efficiency in the financial system.
Or
Explain’Government’s Bank function of central bank.
Answer. A central bank conducts the banking accounts of government departments. It performs the same banking functions for the government as a commercial bank performs for its customers. It accepts their deposits and undertakes inter-bank transfers. It also gives loads to the government. As an agent, it manages the government’s public debt. It undertakes payment of interest on this debt and other related services to public debt. It accepts the payment of taxes from the public on behalf of the government and makes payment for the cheques issued by the government.
It also gives advise to the government regarding money market, capital market, government loans and on economic policy- matters.

Question.24. Government of India has recently launched ‘Jan Dhan Yojna’ aimed at every household in the country to have at least one bank account. Explain how deposits made under the plan are going to affect national income of the country.
Answer. The ‘Jan Dhan Yojna’ recently launched by the government aims at having at least one bank account per household. Bank accounts or demand deposits are a part of the money supply of the country.
— Opening more bank accounts means more bank deposits. More the demand deposits more would be the money supply.
— Also increase in the amount of the demand deposits, legal reserve ratio remaining the same, increases a bank’s capacity for money creation or credit creation as new deposits in banks lead to creation of more deposits by banks.
— Increase in the deposits with commercial banks would mean an increase in their lending capacity.
— More lending by banks means more investment and capital formation in the economy. — As a result, aggregate demand rises leading to an increase in the national income of the country.

Question.25. An economy is in equilibrium. Calculate national income from the following:
Autonomous consumption = 100
Marginal propensity to save = 0.2
Investment expenditure = 200
Answer.
Image may be NSFW.
Clik here to view.
cbse-sample-papers-for-class-12-economics-delhi-2015-12

Question.26. Giving reason explain how should the following be treated in estimation of national income:
(i) Expenditure by a firm on payment of fees to a chartered accountant (ii) Payment of corporate tax by a firm.
(iii) Purchase of refrigerator by a firm for own use
Answer. (i) It will not be included as expenditure by a firm on payment of fees to a chartered accountant is intermediate expenditure and hence will not be considered while calculating national income.
(ii) Payment of corporate tax by a firm is a transfer payment and thus not included in estimation of national income. However, corporate tax is included while estimating national income because it is a part of profit of a joint stock company which is a factor income.
(iii) It will be included in national income because money spent by firm on purchase of refrigerator for own use will be considered as investment expenditure and is thus included.

Question.27. Explain the concept of Inflationary Gap. Explain the role of Repo Rate in reducing this gap.
Answer. Concept of inflationary gap. If equilibrium level of income is determined after the stage of full employment, it is a situation of excess demand. Excess demand refers to the situation when aggregate demand (AD) is more than the aggregate supply (AS) corresponding to full employment level of output in the economy. Excess demand gives rise to inflationary gap.
Inflationary gap refers to the gap by which actual aggregate demand exceeds the aggregate demand required to establish full employment equilibrium. As a result of excess demand, prices increase because corresponding to increase in demand, output cannot increase.
Repo rate is the rate of interest charged by Central Bank on loans given to commercial banks. During actuation of excess demand leading to inflation, the Central Bank raises the repo rate which discourages commercial banks in borrowing from the Central Bank. Increase in repo rate forces commercial banks to increase their own lending rates making – credit costlier. As a result, the demand for loans falls discouraging investment. Also reduction in availability of credit helps in correcting the situation of excess demand and reducing inflationary gap.
Or
Explain the concept of Deflationary Gap and the role of ‘Open Market Operations’ in reducing this gap.
Answer. Concept of deflationary gap. If equilibrium level of income is determined before the stage of full employment, it is a situation of deficient demand. This situation arises when planned aggregate expenditure falls short of aggregate supply at the full employment level. It gives rise to deflationary gap.
In such a situation due to inadequacy of demand, production will be less and therefore involuntary unemployment will be created. This means that all the resources will not be fully employed. Therefore the deficiency in aggregate demand at full employment level is the measure of deficient demand. It also means that during deficient demand, equilibrium is determined at a level less than full employment equilibrium.
Role of Open Market Operations. Open market operations refer to sale and purchase of government securities in the open market by the Central Bank. During deficient demand, the central bank starts purchasing securities from the open market. Those who sell receive payments by cheques from the Central Bank. The money flows out from Central Bank into the commercial banks. This raises lending capacity of commercial banks. As banks lend more, spending rises which increases the money supply in the economy which further increases the level of aggregate demand in the economy thereby correcting the situation of deficient demand and reducing deflationary gap.

Question.28. Explain the role the government can play through the budget in influencing allocation of resources.
Answer. An important objective of government budget is to allocate resources for accomplishing socio-economic objectives of the government. There are a number of projects and activities for which resources are allocated. Through budgetary policy, government aims to influence allocation of resources in the following ways:
(i) Tax concessions or subsidies. To encourage investment, government can give tax concessions, subsidies etc. to the producers. The government discourages the production of harmful consumption goods (like liquor, cigarettes, etc.) through heavy taxes and encourages the production of ‘khadi products’ and other products useful for the masses by providing subsidies and recessions.
(ii) Directly producing goods and services. If private sector does not take interest due to lack of profits, government can directly undertake the production of certain goods and services.
(iii) Resources may be allocated for increasing investment, reducing income inequalities (by imposing taxes on the rich) and providing better healthcare and education facilities.

Question.29. Calculate National Income and Personal Disposable Income:
Image may be NSFW.
Clik here to view.
cbse-sample-papers-for-class-12-economics-delhi-2015-13

Answer.
Image may be NSFW.
Clik here to view.
cbse-sample-papers-for-class-12-economics-delhi-2015-14

SET II

Note: Except for the following questions, all the remaining questions have been asked in Set-I.
SECTION A
Question.2. Give equation of Budget Set.
Answer.
Image may be NSFW.
Clik here to view.
cbse-sample-papers-for-class-12-economics-delhi-2015-15

Question.5. Giving reason comment on the shape of Production Possibilities Curve based on the following schedule:
Image may be NSFW.
Clik here to view.
cbse-sample-papers-for-class-12-economics-delhi-2015-16

Answer.
Image may be NSFW.
Clik here to view.
cbse-sample-papers-for-class-12-economics-delhi-2015-17

Since the marginal rate of transformation, i.e., MRT is constant,
PPC will be a downward sloping straight line. This means that the same amount of a commodity (Good Y) is sacrificed to gain an additional unit of another commodity (Good X)- It is possible only when we assume that all the resources are equally efficient in . production of all goods.

Question.10. A consumer spends Rs 60 on a good priced at Rs 5 per unit. When price falls by 20 per cent, the consumer continues to spend Rs 60 on the good. Calculate price elasticity of demand by percentage method.
Answer.
Image may be NSFW.
Clik here to view.
cbse-sample-papers-for-class-12-economics-delhi-2015-18

Question.11. Market for a good is in equilibrium. The demand for the good ‘decreases’. Explain the chain of effects of this change.
Answer. See Q. 14, 2014 (II Delhi).

SECTION B
Question.21. If the Nominal GDP is 71200 and Price Index (with base = 100) is 120, calculate Real GDP.
Answer.
Image may be NSFW.
Clik here to view.
cbse-sample-papers-for-class-12-economics-delhi-2015-19

Question.23. An economy is in equilibrium. Find ‘autonomous consumption’ from the following:
National Income = 1000
Marginal propensity to consume = 0.8
Investment expenditure = 100
Answer. Y = 1000 MPC = 0.8 I=100
Image may be NSFW.
Clik here to view.
cbse-sample-papers-for-class-12-economics-delhi-2015-20

Question.29. Calculate ‘Gross National Product at Market Price’ and ‘Net National Disposable Income’:
Image may be NSFW.
Clik here to view.
cbse-sample-papers-for-class-12-economics-delhi-2015-21

Answer.
Image may be NSFW.
Clik here to view.
cbse-sample-papers-for-class-12-economics-delhi-2015-22

SET III

Note: Except for the following questions, all the remaining questions have been asked in Set-I and Set-II.
SECTION A
Question.3. Define Budget Set.
Answer. Budget set refers to the set of possible combinations of only two goods the consumer consumes which he can afford from his income and given prices.

Question.5. Explain the feature ‘interdependence of firms’ in an oligopoly market.
Answer. Firms under oligopoly are interdependent. Interdependence means that actions of one firm affect the actions of other firms. A firm considers the action and reaction of the rival firms while determining its price and output levels. A change in output or price by one firm evokes reactions from other firms operating in the market.

Question.8. Giving reason comment on the shape of Production Possibilities Curve based on the following schedule:
Image may be NSFW.
Clik here to view.
cbse-sample-papers-for-class-12-economics-delhi-2015-23

Answer.
Image may be NSFW.
Clik here to view.
cbse-sample-papers-for-class-12-economics-delhi-2015-24

Since the above schedule is representing increasing MRT, i.e., more and more of Y have to be sacrificed for a unit increase in production of X, therefore, the PPC in the case will be a downward sloping curve which is concave to the origin.

Question.9. A consumer spends ?100 on a good priced at Rs 4 per unit. When price falls by 50 per cent, the consumer continues to spend Rs 100 on the good. Calculate price elasticity of demand by percentage method.
Answer.
Image may be NSFW.
Clik here to view.
cbse-sample-papers-for-class-12-economics-delhi-2015-25

Question.12. Market for a good is in equilibrium. Supply of the good ‘increases’. Explain the chain of effects of this change.
Answer. Market equilibrium is determined when the quantity demanded of a commodity becomes equal to the quantity supplied. The price determined corresponding to market equilibrium is known as equilibrium price and the corresponding quantity is known as equilibrium quantity.
As shown in the diagram, original equilibrium is determined at point E where die original demand and supply curves intersect each other where OQ is the equilibrium quantity and OP is the equilibrium price.With increase in supply, demand remaining unchanged the supply curve shifts from SS to S1S1. When supply increases to S1S1, it creates a situation of excess supply at the old equilibrium price of OP. This leads to competition among sellers, which reduces the price. Decrease in price leads to rise in demand and fall in supply. These changes continue till the new equilibrium is established at point E1.
Equilibrium price falls from OP to OP1 and equilibrium quantity rises from OQ to OQ1.
Image may be NSFW.
Clik here to view.
cbse-sample-papers-for-class-12-economics-delhi-2015-26

SECTION B
Question.22. If the Real GDP is Rs 300 and Nominal GDP is Rs 330, calculate Price Index (base = 100).
Answer.
Image may be NSFW.
Clik here to view.
cbse-sample-papers-for-class-12-economics-delhi-2015-27

Question.24. An economy is in equilibrium. Find Marginal Propensity to Consume from the following:
National income = 2,000
Autonomous consumption = 400
Investment expenditure = 200
Answer.
Image may be NSFW.
Clik here to view.
cbse-sample-papers-for-class-12-economics-delhi-2015-28

Question.29. Calculate ‘Net Domestic Product at Factor Cost’ and ‘Gross National Disposable Income’:
Image may be NSFW.
Clik here to view.
cbse-sample-papers-for-class-12-economics-delhi-2015-29

Answer.
Image may be NSFW.
Clik here to view.
cbse-sample-papers-for-class-12-economics-delhi-2015-30

The post CBSE Sample Papers for Class 12 Economics Delhi – 2015 appeared first on Learn CBSE.

Important Question for CBSE Class 9 Science Diversity in Living Organisms

Important Question for CBSE Class 9 Science Chapter 7 Diversity in Living Organisms

IMPORTANT QUESTIONS

1 MARK QUESTIONS
Question. 1 Name one basic characteristic for classifying organisms.
Answer. Cell structure.

Question. 2 What was the modification introduced by Woese in kingdom Monera ? [SAII -2012]
Answer.
Carl Woese modified the Kingdom Monera by dividing it into Archae-bacteria and Eubacteria.

Question. 3 Define the term henpaphrodite. Give one example [SAII – 2014]
Answer.
A hermaphrodite is an organism that has both male and female genitalia.
Example: Earthworm.

Question. 4 What do you understand by diploblasty ? Give one example of an organism which is diploblastic. [SAII -2014]
Answer.
Diploblasty is a condition of the blastula in which there are two primary germ layers — the ectoderm and the endoderm.
Example: Jellyfish.

Question.5 A pore bearing organism like creature ‘A’ belongs to a phylum ‘B’ of kingdom Animalia. Identify ‘A’and’B’ [SAII- 2014]
Answer. A — Sponges, B — Porifera.

Question.6 Homo sapiens is the scientific name of human beings. What do these two terms imply ?
[SAII – 2014]
Answer.
Homo sapiens : Homo is the human”genus and sapiens is the only surviving species of the genus Homo.

Question.7 Rewrite the scientific names correctly (i) panthera tigris (ii) periplaneta Americana. [SAII – 2014]
Answer.
(i) Panthera tigris (ii) Periplaneta americana.

Question.8 Name two egg laying mammals.
Answer. (i) Platypus (ii) Echidna.

Question.9 What is the primary characteristic on which the first division of organisms is made ?
Answer.
The primary characteristics on which the first division of organisms is made in : Do the cells
occur singly or are they grouped together and do they live as an indivisible group ?

Question.10 Which division among plants has the simplest organisms ?
Answer.
The division ‘Thallophyta’ among plants has the simplest organisms as they do not have well- differentiated body design.

2-MARKS QUESTIONS
Question.1
In which two categories, the organisms are divided on the basis of evolution ? How are the two categories different from each other ? [SAII – 2014]
Answer.
On the basis of evolution, the organisms are divided into two categories :
(a) ‘Primitive’ or ‘lower’ organisms
(b) ‘Advanced’ or ‘higher’ organisms
Advanced organisms results from the gradual development of more complex organisms jfrom the simpler ones /.e., primitive organisms.

Question.2 Why is it difficult to classify bacteria ? Give two reasons. [SAII -2014]
Answer.
It is difficult to classify bacteria because :
(i) These organisms do not have a well defined nucleus or organelles and do not have multicellular body designs.
(ii) Mode of nutrition of these organisms can be either autotrophic or heterotrophic.

Question.3 What are species ? State in terms of (i) hierarchy of classifications and (ii) in relation to reproduction. [SAII -2014]
Answer.
(i) Hierarchy of classification: Species is the lowermost category in the hierarchy of classification of groups of organisms.
(ii) In relation to reproduction : A species includes all organisms that are similar enough to breed and perpetuate.

Question.4 What are Cryptogams ? Why are they called so ?
Answer.
Thallophytes, Bryophytes and Pteridophytes are classified as Cryptogams as they have hidden reproductive organs.

Question.5 We start classification of plants on the basis of differentiation of plant body.
(i) Which division lacks a well differentiated body design ?
(ii). Where are such plants pr&iominantly found ? [SAII -2014]
Answer.
(i) Thalldphyta
(ii)These plants are predominantly found in aquatic habitat.

Question.6 Differentiate between Bryophyta and Pteridophyta. Give one example of each.
Answer.
Image may be NSFW.
Clik here to view.
important-question-for-cbse-class-9-science-diversity-in-living-organisms-1

Question.7 Prawn, housefly and spider belong to one group.
(i) Name the group.
(ii)Justify the name given to this group.
Answer.
(i) Arthropoda.
(ii) They have jointed legs. The word ‘arthropod’ means jointed legs.

Question.8 What is meant by warm-blooded and cold-blooded animals ? Explain. [SAII – 2014]
Answer.
Warm-blooded animals have a relatively higher blood temperature and can maintain the constant  internal body temperature through metabolic processes. Mammals and birds are warm-blooded
animals.
Cold-blooded animals like reptiles, amphibians and fish continuously change their body temperature
with change in the temperature of the environment.

Question.9 List two distinguishing features between annelids and arthropods.
Answer.
(i)Arthropods have jointed legs that annelids lacks.
(ii) Arthropods have their coelomic cavity filled with blood that is absent in annelids,

Question. 10 Write the type of body cavity and symmetry possessed by nematodes. [SAII-2014]
Answer.
Body cavity: Pseudocoelom.
Symmetry: Bilateral.

Question.11 Give two features of Pisces.
Answer.
(i) Their skin are covered with scales/plates.
(ii) They are cold-blooded animals.

Question.12 On what basis are plants and animals put into different categories ?
Answer.
Plants and animals are put into different categories on the basis of their ability to make their f own food.
Being able to produce one’s own food versus having to get food from outside would make very different body designs necessary.

Question.13 Will advanced organisms be the same as complex organisms ? Why ?
Answer.
Yes, advanced organisms be the same as complex organisms because there is a possibility that complexity in design will increase over evolutionary time. Thus, it may not be wrong to say that ‘older’ (‘primitive’ or ‘lower’) organisms are simpler while ‘younger’ (‘advanced’ or ‘higher’) organisms are more complex.

Question.14 In the hierarchy of classification, which grouping will have the smallest number of organisms with a maximum of characteristics in common and which will have the largest number of organisms? [SAII – 2011]
Answer.
The grouping having the smallest number of organisms with a maximum of characteristics in ; common is the ‘species’. While the grouping having the largest number of organisms with a maximum of characteristics in common is the ‘kingdom’.

Question.15 Explain the basis for grouping organisms into five kingdoms.[SAII – 2011]
Answer.
The basis for grouping organisms into five kingdoms depends on their:
(a) Cell structure.
(b) Mode and source of nutrition.
(c) Body organisation.

3 MARKS QUESTIONS
Question.1 Classify the following organisms into their respective kingdoms as per Whittaker’s five kingdom classification:
Amoeba, Euglena, Birds, Herbs, Cats, Lactobacillus.
Answer.
Amoeba — Protista Euglena — Protista
Birds — Animalia Herbs — Plantae
Cats — Animalia Lactobacillus — Monera

Question.2 Make a table to differentiate between Monera and Fungi on the following basis:
(а) Prokaryotic/Eukaryotic
(b) Cell wall
Answer.
Image may be NSFW.
Clik here to view.
important-question-for-cbse-class-9-science-diversity-in-living-organisms-2

Question. 3 Study the figure and Answer the following questions:
Image may be NSFW.
Clik here to view.
important-question-for-cbse-class-9-science-diversity-in-living-organisms-3

Answer.
Image may be NSFW.
Clik here to view.
important-question-for-cbse-class-9-science-diversity-in-living-organisms-4

Question.4 Give three differences between Monocot and Dicot plant. [SAII – 2014]
Answer.
Image may be NSFW.
Clik here to view.
important-question-for-cbse-class-9-science-diversity-in-living-organisms-5

Question.5 What is meant by characteristic of an organism ? Mention two characteristics of class reptilia.
[SAII- 2014]
Answer.
Characteristics of an organism are the features that help us in knowing, identifying and classifying the organism.
The two characteristics of class Reptilia are as follows :
(i) These are cold-blooded animals.
(ii) They respire through lungs.

Question.6 State any two characteristics of mammalia. Name two egg laying mammals. [SAII – 2014]
Answer.
Characteristics of mammalia :
(i) Mammals are warm-blooded animals having hairs on body.
(ii) They have four chambered heart.
(iii) They have mammary glands for the production of milk to nourish their young onesv
(iv) Skin has sweat or oil glands.
Egg laying mammals: Platypus, Echidna.

Question.7 Explain the following terms:
(i) Bilateral symmetry, (n) Triploblastic animals and (iii) Open circulatory system.[SAII-2014]
Answer.
(i) Bilateral symmetry : It means that the left and right halves of the body have the same design. It is fourid in Platyhelminthes.
(ii) Triploblastic animals : In triploblastic animals, there are three layers of cells from which differentiated tissues can be made.
(iii) Open circulatory system : In open circulatory system, blood does not flow in well defined blood vessels. Blood is pumped by a heart into the body cavities, where tissues are surrounded by the blood.

Question.8 Tabulate the following features and compare Reptilia, Aves and Mammalia:
(a) Exoskeleton (b) Number of chambers in heart [SAII – 2014]
Answer.
Image may be NSFW.
Clik here to view.
important-question-for-cbse-class-9-science-diversity-in-living-organisms-6

Question.9 Write appropriate terms for the following:
(a) Animals that are able to maintain a certain body temperature over a wide range of temperature in the environment.
(b) Plants which bear naked seed.
(c) Animals which have pseudocoelom.
Answer.
(a) Warm-blooded animal, (b) Gym nosperms, (c) Nematoda.

Question.10 Give three examples of the range of variations that you see in life forms around you.
Answer.
The following range of variations can be seen in different life forms which occurs on Earth.
(i) Variation in size: Some microscopic bacteria are few micrometre in size whereas blue whale
and redwood trees of California are approximately of 30 metres and 100 metres respectively.
(ii)Variation in lifespan : Some pine trees live for thousands of years while insects like mosquitoes die within a few days.
(iii) Variation in body colours: Some life forms such as worms are colourless or even transparent whereas many of the birds, insects, flowers etc., are brightly coloured.

Question.11 Which do you think is a more basic characteristic for classifying organisms :
(a) the place where they live ?
(b) the kind of cells they are made of with?
Answer.
The more basic characteristic for classifying organisms is the kind of cells they are made of because:
(a) An eukaryotic cell has membrane-bound organelles, including a nucleus, which allow cellular processes to be carried out efficiently in isolation from each other.
(b) Further, nucleated cells would have the capacity to participate in makinga multicellular organism because they can take up specialised functions. Therefore, this is a basic characteristic of classification.

Question.12 Which organisms are called primitive and how are they different from the so-called advanced organisms ? [SAII – 2011]
Answer.
Image may be NSFW.
Clik here to view.
important-question-for-cbse-class-9-science-diversity-in-living-organisms-7

Question.13 How are pteridophytes different from the phanerogams? [SAII-2011]
Answer.
Image may be NSFW.
Clik here to view.
important-question-for-cbse-class-9-science-diversity-in-living-organisms-8

Question.14 How do gymnosperms and angiosperms differ from each other ? [SAII -2011]
Answer.
Image may be NSFW.
Clik here to view.
important-question-for-cbse-class-9-science-diversity-in-living-organisms-9

Question.15
How would you choose between two characteristics to be used for developing a hierarchy in classification ?
Answer.
The characteristics which are taken into account for developing a hierarchy in classification are :
(i) Whether cell has membrane-bound organelles present or not. Because such cells would have different biochemical pathways having an effect on every aspect of cell design.
(ii) Whether the cells occur singly or are they grouped together. Because this makes a very basic distinction in the body designs of organisms.
(iii) Whether organisms produce their own food (plants) or acquire it(animals).
In this way, we can build-up a whole hierarchy of mutually related characteristics to be used for classification.

Question.16 How are the criteria for deciding divisions in plants different from the criteria for deciding the subgroups among animals ?
Answer.
The characteristics of body design used for classification of plants will be very different from those important for classifying animals. This is because the basic designs are different, based on the need to make their own food (plants) or acquire it (animals). Therefore, these design features (having a skeleton) are to be used to make subgroups, rather than making broad groups.

5 MARKS QUESTIONS
Question.1 X, Y and Z are living organisms.
(a) Identify the group to which they belong on the basis of following features. .
(i) X — Microscopic, unicellular, prokaryotic.
(ii) Y — Microscopic, unicellular, eukaryotic, and shows locomotion with the help of pseudopodia/flagella.
(iii) Z — Multicellular, filamentous, eukaryotic, autotrophic and aquatic.
(b) Which amongst the above is most advanced ?
(c) Name one organism each belonging to the groups of X, Y and Z. [SAII-2014]
Answer.
(a) (i) Monera, (ii) Protista, (iii) Plantae (Thallophyta)
(b) Fungi are most advanced organisms as compared to Protista and Monera.
(c) X — Bacteria
Y — Amoeba
Z —Spirogyra.

Question.2 Give the appropriate term for each of the following:
(а) Complex sugar that makes the fungal cell wall.
(b) Plants which bear naked seeds.
(c) Blue-green algae.
(d) Basic unit of classification.
(e) Group of unicellular eukaryotic organism. [SAII – 2014]
Answer.
(a) Chitin
(b) Gymnosperms
(c) Cyanobacteria
(d) Cell structure
(e) Protista.

Question.3 (a) Why are angiosperms so called ? In which structures do the seeds develop ? Why are cotyledons considered an important part of seed ?
(b) Differentiate between the two categories of angiosperms giving an example of each. [SAII – 2014]
Answer.
(a) Angiosperms are so called because these plants have covered seeds.
Seeds develop within ovary which later modify into fruit.
Cotyledons are considered an important part of seed as these are seed leaves. In many instances, they emerge and become green when the seed germinates.
(b) The angiosperms are divided into two categories depending upon the number of cotyledons present in the seeds:
(i) Monocot — Plants having single cotyledon, e.g., Paphiopedilum, maize.
(ii) Dicot — Plants having two cotyledons, e.g., Ipomoea, sunflower.

Question.4
(a) Identify the organism shown in the picture and write the common name and scientific name of the organism.
(b) Name its phylum and kingdom.
(c) Which organ of digestive system normally harbours this organism ?
Image may be NSFW.
Clik here to view.
important-question-for-cbse-class-9-science-diversity-in-living-organisms-10

Answer.
(a) It is Ascaris.Common name is Roundworm.Scientific name is Ascaris lumbricoides.
(b) Phylum — Nematoda and Kingdom-Animalia.
(c) Small intestine (an organ of digestive system) normally harbours this organism.

Question.5
(a) Identify three features possessed by all chordates.
(b) Name the classes of vertebrates which have the following characteristics:
(i) Animals that have streamlined body and gills for breathing.
(ii) Animals that are found both on land and in water.
(iii) Animals that have mammary glands for the production of milk to nourish their young ones.
Answer.
(a) Features of chordates:
• They have notochord. • They have a dorsal hollow nerve chord.
• They are triploblastic. • They have paired gill pouches.
• They are coelomate.
(b) (i) Pisces, (ii) Amphibians, (iii) Mammals.

Question.6 What is the criterion for classification of organisms as belonging to kingdom Monera or Protista? [SAII – 2011]
Answer.
Criterion for classification of organisms belonging to kingdom Monera:
(i) These organisms do not have a defined nucleus or organelles, nor do any of them show multicellular body designs.
(ii) Some of them have cell walls while some do not have.
The mode of nutrition of organisms in this group can be either by synthesizing their own food (autotrophic) or getting it from the environment (heterotrophic).
(iv) This group includes bacteria, blue-green algae or cyanobacteria and mycoplasma.
Criterion for classification of organisms belonging to kingdom Protista:
(i) This group includes many kinds of unicellular eukaryotic organisms.
(ii) Some of these organisms use appendages, such as hair-like cilia or whip-like flagella for moving around.
(iii) Their mode of nutrition can be autotrophic or heterotrophic.
(iv) This group includes unicellular algae, diatoms and protozoans.

Question.7 How do poriferan animals differ from coelenterate animals ? [SAII -2011]
Answer.
Image may be NSFW.
Clik here to view.
important-question-for-cbse-class-9-science-diversity-in-living-organisms-11

Question.8 How do annelid animals differ from arthropods ? [SAII -2014]
Answer.
Image may be NSFW.
Clik here to view.
important-question-for-cbse-class-9-science-diversity-in-living-organisms-12

Question.9 What are the differences between amphibians and reptiles ? [SAII – 2011]
Answer.
Image may be NSFW.
Clik here to view.
important-question-for-cbse-class-9-science-diversity-in-living-organisms-13

Question.10 What are the differences between animals belonging to the Aves group and those in the Mammalia
group ? [SAII- 2011]
Answer.
Image may be NSFW.
Clik here to view.
important-question-for-cbse-class-9-science-diversity-in-living-organisms-14

Question.11 What are the advantages of classifying organisms ?
Answer.
The importance of classification is :
(a) It provides us an information regarding the diversity of plants and animals.
(b) It provides insight into the origin of organism and interrelationship between them.
(c) It makes the study of wide variety of organisms.
(d) It helps in understanding the evolution of organisms. –
(e) Various fields of applied biology like agriculture, environmental biology etc., also depends t upon correct identification and classification of pest, disease, vector etc.
(f) It serves as a base for the development of other biological sciences like biogeography and
ecology.
The science of classification thus contributes to a larger extent in advancing knowledge in
most of the other disciplines of biology.

Question.12 What are the major divisions in the plantae ? What is the basis for these divisions ?
[SAII – 2014]
Answer.
The major divisions in the kingdom plantae are :
(i) Thallophyta (ii) Bryophyta
(iii) Pteridophyta (iv) Gymnosperms
(v) Angiosperms.
The basis of these divisions are :
(a) Whether the plant body has well differentiated, distinct components.
(b) Whether the differentiated plant body has special tissues for the transport of water and other substances within it.
(c) The ability to bear seeds.
(d) Whether the seeds are enclosed within fruits.

Question.13 Explain how animalflTrvertebrata are classified into further subgroups.
Answer.
The following characteristics are considered while classifying Vertebrata into further subgroups – (classes) :
Image may be NSFW.
Clik here to view.
important-question-for-cbse-class-9-science-diversity-in-living-organisms-15

APPLICATION BASED QUESTIONS

Question.1 Distinguish between the following:
(a) Fungi and Plantae on the basis of nutrition.
(b) Gymnosperms and Angiosperms on the basis of fruits. [SAII-2014]
Answer.
(a) Fungi shows heterotrophic mode of nutrition /.e., they cannot prepare their food while Plantae
are autotrophs i.e., they prepare their own food by the process of photosynthesis.
(b) Gymnosperms bear naked seeds while Angiosperms have seeds which develop inside an organ which later modifies into a fruit.

Question.2 A plant specimen was found without differentiated roots. ‘
(a) Which plant structure helps in attaching this plant to the substratum ?
(b) To which group you will keep this plant ?
(c) Which plant could it be ?
Answer.
(a) Thallus, (b) Thallophyta, (c) Algae.

Question.3 Give a comparative account of the following:
(a) Ulothrix and Funaria
(b) Marchantia and Marselia
(c) Fern and Pinus
(d) Cycas and rose
(e) Wheat and gram
Answer.
(a) Ulothrix is Thallophytes and Funaria is Bryophytes.
(b) Marchantia is Bryophytes and Marsilea is Pteridophytes.
(c) Fern and Pinus both the plants belongs to group Pteridophyta.
(d) Cycas is gymnospermic plant whereas rose is angiospermic plant.
(e) Wheat is monocot plant whereas gram is dicot plant.

Question.4 Protochordates possesses a notochord in larval forms. Appraise the use of this feature in these animals. :
Answer.
The notochord is a long rod-like support structure (chord = string) that runs along the back of the animal separating the nervous tissue from the gut. It provides a place for muscles to attach for ease of movement.

Question.5 Write two examples of each of the following:
(a) Animals that do not loeomote.
(b) Flightless birds.
(c) Molluscs without shell.
Answer.
(a) Shrimps and squids.
(b) Kiwi and ostrich.
(c) Octopus and squids.

Question.6 Associate the following features with groups in which they first appeared. ..
(a) Vascular tissues
(b) Notochord
(c) Seeds inside fruits.
Answer.
(a) Vascular tissues —Pteridophyta
(b) Notochord — Protochordata
(c) Seeds inside fruits — Angiosperms

Question.7 Give the scientific and common name of an organism that possesses the following:
(а) Organisms that are found both on land and water.
(b) Organisms that lay eggs on land.
(c) Organisms having skeleton made of cartilage.
(d) Organisms having hair on their body.
(e) Organisms that are cold-blooded.
Answer.
(a) Frog (Ranatigrina)         (b) Platypus (Ornithorhynchus anatinus)
(c) Shark (Selachimorpha) (d) Human beings (Homo sapiens)
(e) Rohu (Labio rohita).

Question.8 Match items of column (A) with items of column (B)
(A)                                                                          (B)
(a) Pore bearing animals                               (A) Arthropoda
(b) Diploblastic                                                  (B) Coelenterata
(c) Metameric segmentation                        (C) Porifera
(d) Jointed legs                                                  (D) Echinodermata
(e) Soft bodied animals                                   (E) Mollusca
(f) Spiny skinned animals                              (F) Annelida
Answer.
(a) C (b) B (c) F (d) A (e) E

Question.9 Match items of column (A) with items of column (B).
(A)                                                                     (B)
(а) Naked seed                                              (A) Angiosperms
(b) Covered seed                                          (B) Gymnosperms
(c) Flagella                                                     (C) Bryophytes
(d) Marchantia                                             (D) Euglena
(e) Marsilea                                                   (E) Thallophyta
(f) Cladophora                                              (F) Pteridophyta
(g) Pencillium                                              (G) Fungi
Answer.
(a) B (b) A (c) D (d) C (e) F (f) E (g) G

Question.10 Classify the following organisms based on the absence/ presence of true coelom (i.e., acoelomate, pseudocoelomate and coelomate)
Spongilla, Sea anemone, Planaria, Liver fluke, Wuchereria, Ascaris, Nereis, Earthworm, Scorpion, Birds, Fishes, Horse.
Answer.
Spongilla – Acoelomate Sea anemone – Acoelomate Planaria – Acoelomate
Liver fluke – Acoelomate
Wuchereria – Pseudocoelomate
Ascaris – Pseudocoelomate
Nereis – Coelomate
Scorpion – Coelomate
Earthworm – Coelomate
Birds,’ fishes and horse – Coelomate

Question.11 Endoskeleton of fishes are made-up of cartilage and bone; classify the following fishes as
cartilaginous or bony:
Torpedo, Sting ray, Dogfish, Rohu, Anglerfish, Exocoetus.
Answer.
Torpedo-Cartilaginous,Stingray-Cartilaginous, Dogfish-Cartilaginous, Rohu-Bony, Anglerfish- Cartilaginous, Exocoetus-bony.

Question.12 Classify the following based on number of chambers in their heart.Rohu, Scoliodon, Frog, Salamander, Flying lizard, King Cobra, Crocodile, Ostrich, Pigeon, Bat, Whale.
Answer.
Rohu, Scoliodon 2 chambered.
Frog, Salamander, Flying lizard, King Cobra-3 chambered.
Crocodile, Ostrich, Pigeon, Bat, Whale-4 chambered.

Question.13 Give examples for the following:
(a) Bilateral, dorsiventral Symmetry is found in………………………….
(b) Worms causing disease elephantiasis is………………………….
(c) Open circulatory system is found in ………………………….where coelomic cavity is filled with blood
(d)………………………….are known to have pseudocoelom.
Answer.
(a) Liver fluke (b) Filarial worm
(c) Arthropods (d) Nematodes

Question.14 Label a, b, c and d in given fig. Give the function of (b).
Image may be NSFW.
Clik here to view.
important-question-for-cbse-class-9-science-diversity-in-living-organisms-16

Answer.
(a) Dorsal fin (b) Caudal fin
(c) Pelvic fin (d) Pectoral fin
Function of CaudaHm(b)-Caudal fin helps in streamlined movement in water.

Question.15 Classify Rohu, Scoliodon, Flying lizard, King Kobra, Frog, Salamander, Ostrich, Pigeon, Bat, – Crocodile and Whale into the cold-blooded/warm blooded animals.
Answer.
Cold-blooded-Rohu, Scoliodon, Frog, Salamander, Flying Lizard, King Cobra, Crocodile.
Warm-blooded-Ostrich, Pigeon, Bat, Whale. v

Question.16 Why do we keep both snake and turtle in the same class ?
Answer.
Because both are:
(1) cold-blooded
(2) have scales
(3) breathe through lungs
(4) have three chambered fjeart and
(5) they lay eggs with tough covering.

Higher Order Thinking Skills (HOTS) Question.

Question. 1 List any two similarities and two differences between Amoeba and Paramecium.
Answer.
Similarity between Amoeba and Paramecium
(a) Both are unicellular and eukaryotic organisms.
(b) Both the organisms have food vacuole.
Difference between Amoeba and Paramecium
(a) Locomotary organ of Amoeba is pseudopodia where Paramecium moves with the help of . cilia.
(b) Amoeba has only one nucleus whereas Paramecium have two nuclei.

Question.2 You are provided with the seeds of gram, wheat, rice, pumpkin, maize and pea. Classify them whether they are monocot or dicot.
Answer.
Gram-dicot Wheat-monocot
Rice-monocot Pumpkin-dicot
Maize-monocot Pea-dicOt

Question.3 Define the terms and give one example of each:
(a) Bilateral symmetry (b) Coelom (c) Triploblastic
Answer.
(a) The left and right halves of the body have the same design, e.g. Liver fluke.
Image may be NSFW.
Clik here to view.
important-question-for-cbse-class-9-science-diversity-in-living-organisms-17

(b) Coelom is the internal body cavity between visceral organs and body wall in which well developed organs can be accommodated, e.g., butterfly.
(c) Animals having three layers of cells from which differentiated tissue can be made are called triploblastic, e.g., starfish.

Question.4 You are given Leech, Nereis, Scolopendra, Prawn and Scorpion; and all have segmented body organisation. Will you classify them in one group ? If no, give the important characters based on which you will separate these organisms into different groups. ‘
Answer.
All organisms given in the question do not belong to same group. Leech and Nereis belong to phylum annelida because they have metamerically segmented body, i.e., body is divided into many segments internally by septa. Body segments are lined up one after the other from head to tail. But Scolopendra, prawn and scorpion belong to phylum arthropoda as these have jointed legs and open circulating system.

Question.5 Which organism is more complex and evolved among Bacteria, Mushroom and Mango tree ? Give reasons.
Answer.
Mango tree is more complex and evolved because, it is eukaryotic, autotrophic, terrestrial sporophyte with covered seed. The bacteria is unicellular prokaryote and fungi are the heterotrophic, simple thallophyte with no tissue systems –

Question.6 Differentiate between flying lizard and bird. Draw their diagram.
Answer.
Flying lizard belongs to group reptiles and characterised as cold-blooded, body covered with scales and having three chambered heart, while Birds belong to group aves and have characteristics of being warm-blooded, having feather covered body, fore limbs modified as wings and having four chambered heart.
Image may be NSFW.
Clik here to view.
important-question-for-cbse-class-9-science-diversity-in-living-organisms-18

Reasoning Questions

Question.1 State reasons for each of the following:
(i) Echidna and Platypus lay eggs but are considered as mammals.
(ii) Forelimbs of birds are modified.
(iii) Crocodiles have four-chambered heart but are still reptiles.
Answer.
(i) Echidna and Platypus lay eggs but are considered as mammals because they both are semi- aquatic as they can stay both in water as well as out of it. ~
(ii) Forelimbs of birds are modified into wings to fly.
(iii) Crocodiles have four-chambered heart but are still reptiles as they lay eggs with tough coverings and do not need to lay their eggs in water, unlike amphibians.

Question.2 State reason for the following:
(a) Tapeworm does not have digestive trait.
(b) Frogs use both skin and lungs for breathing.
(c) Birds have pneumatic/hollow bones.
Answer.
(a) Tapeworm does not have digestive trait as tapeworm are parasites and their body wall absorbs the digested food from the host animal iri which they live.
(b) Frogs use both skin and lungs for breathing as skin is effective for breathing when they are in water whereas lungs are used when the frog is on land or swimming at the surface. ”
(c) Birds have pneumatic/hollow bones to lighten their weight and make it easier for them to fly.

Question.3 Why do we keep both snake and turtle in the same class ?
Answer.
This is because both snake and turtle:
(i) are cold-blooded
(ii) have scales
(iii) breathe through lungs, and
(iv) do not need to lay their eggs in water.

Question.4 Give reasons:
(a) Animals of phylum platyhelminthes are called flatworms.
(b) Bryophytes are called amphibians of the plant kingdom.
(c) Fungi are called saprophytes.
(d) Bacteria and tapeworms are very different in their body design.
(e) Plants like Pinus and Deodar are called gymnosperms.
Answer.
(a) Animals of phylum platyhelminthes are called flatworms because the body is flattened dorsiventrally, from top to bottom.
(b) Bryophytes are called amphibians of the plant kingdom as they are found on both land as
well as in water.
(c) Fungi are called saprophytes as they use the decaying organic material as food.
(d) Bacteria and tapeworms are very different in their body design due to change in course of
evolution.
(e) Plants like Pinus and Deodar are called gymnosperms as the plants of this group bear naked seeds and are usually perennial, evergreen and woody.

Question.5 Give reasons for the following:
(i) From phylum platyhelminthes onwards, animals are categorised as ‘triploblastic.
(ii) The presence of ’coelom’ in an animal’s body is considered as advantageous. [SAII-20 74]
Answer.
(i) There are three layers of cells from which differentiated tissues can be made. This allows outside and inside body linings as well as some organs to be made. There is thus some degree of tissue fqrmation.
(ii) Coelom is a true internal body cavity in which well-developed organs can be accommodated.

Important Topics/Areas/Questions which are
frequently asked in the examination

Question.1 Which organisms are called primitive ? How are they different from the advanced organisms ? [SAII-2014]
Answer.
Primitive organisms: Organisms which have simpler ancient body designs that have not changed much over evolutionary time are called primitive organisms.
Advanced organisms : Organisms which have complex body designs that have changed over evolutionary time are called advanced organisms.
Question.2 Identify the group comprising specialised tissue for the conduction of water in the plant body. Name one such plant. [SAII-2014]
Answer.
The group pteridophyta shows specialised tissue for the conduction of water in the plant body. Marsilea is one such plant that belongs to this group.

Question.3 Name the type of flowers that can be seen in a: (i) Monocot plant, (ii) Dicot plant.
Answer.
(i) Monocot plant: Trimerous flower having one cotyledon in the seed.
(ii)Dicot plant: Pentamerqips flower having two cotyledons in the seed.

Question.4 State which were the first terrestrial plants ? Mention two characteristic features of these plants.
Answer.
(a) Bryophyta are the first terrestrial plants. These are called the “amphibians of the plant kingdom.
(b) The two characteristics features of these plants are :
(i) The plant body is commonly differentiated into stem and leaf like structures.
(ii) There is no specialised tissue for the conduction of water and other substances from one part of the plant body to another.

Question.5 Differentiate between maize plant and mustard plant in terms of:
(a) root system, (b) leaf venation, (c) number of cotyledons
Answer.
Image may be NSFW.
Clik here to view.
important-question-for-cbse-class-9-science-diversity-in-living-organisms-19

Question.6 What are Phanerogams ? Name its two groups along with the criteria of division. [SAII-2013]
Answer.
The plants with well differentiated reproductive tissues that ultimately makes seeds are called as phanerogams. Seeds are the result of the reproductive process. They consist of the embryo along with the stored food, which serves for the initial growth of the embryo during germination.
The phanerogams are further divided into two groups, based on whether the seeds are naked or enclosed within fruits – gymnosperms and angiosperms.

Question.7 Name the phylum to which the following organisms belong.
(a) Earthworm (6) Planaria (c) Sycon
(d) Starfish (e) Housefly (f) Octopus
Answer.
(a) Earthworm-Annelida
(b) Planaria – Platyhelminthes
(c) Sycon – Porifera
(d) Starfish – Echinodermata
(e) Housefly – Arthropoda
(f) Octopus – Mollusca

Question.8 Write three characteristics each of the following groups of animals: [SAll-2012, 2014]
(a) Platyhelminthes (b) Nematoda
Answer.
(a) Platyhelminthes:
(i) The body is bilaterally symmetrical.
(ii) They are triploblastic.
(iii) Body is flattened dorsiventrally.
(b) Nematoda:
(i) Body is bilaterally symmetrical.
(ii)They are triploblastic.
(iii)Body is cylindrical.

Question.9 Name the following:
(a) An egg laying mammal.
(b) A reptile with four chambered heart.
(c) Scientist who proposed the five kingdom classification.
(d) A mammal which can fly.
(e) An aquatic mammal.
(f) Group of animals having fins for locomotion.
Answer.
(a) Platypus, (b) Crocodile, (c) Robert Whittaker, (d) Bat, (e) Whale, (f) Pisces.

Question.10 Out of the various phyla under animal kingdom, choose the one that fits the given description. Also give an example in each case:
(a) Have dorsiventrally flattened body.
(b) Water driven tube system is present for locomotion.
(c) Mostly with shells, some are without shell.
(d) Holes are present all over the body.
(e) Longest phylum due to a variety of adaptation. [SAII-2013, 2014]
Answer.
(a) Platyhelminthes (Planaria),
(b) Echinodermata (Starfish),
(c) Mollusca (Pila)
(d) Porifera (Sycon),
(e) Arthropoda (Spiders).

Question.11
(a) What do you understand by nomenclature ?
(b) Who introduced this system ?
(c) Give any four conventions while writing scientific names.
Answer.
(a) Nomenclature is the system of scientific naming of living beings so that they can be uniformly identified all over the world.
(b) The system of scientific naming was introduced by Carolus Linnaeus.
(c) Conventions:
(i) The name of genus begins with a capital letter.
(ii) The name of species begins with a small letter.
(iii) When printed, the scientific name is typed in italics.
(iV) When written by hand, the genus name and species names have to be underlined

The post Important Question for CBSE Class 9 Science Diversity in Living Organisms appeared first on Learn CBSE.

CBSE Sample Papers for Class 12 Economics Outside Delhi – 2015

CBSE Sample Papers for Class 12 Economics Outside Delhi – 2015

Time allowed : 3  hours                                                                                         Maximum marks 100

GENERAL INSTRUCTIONS
(i) All questions in both the sections are compulsory.
(ii) Marks for questions are indicated against each.
(iii) Questions No. 1-5 and 17-21 are very short-answer questions carrying 1 nick each. They are required to be answered in one sentence each.
(iv) Questions No. 6-10 and 22-26 are? short-answer questions carrying 3 marks each. Answers to them should normally not exceed 60 words each.
(v) Questions No. 11-13 and 27-29 are also short-answer questions carrying 4 marks each. Answers to them should normally not exceed 70 words each.
(vi) Questions No. 14-16 and 30-32 are long-answers questions carrying 6 marks each. Answers to them should normally not exceed 100 words each.
(vii) Answers should be brief and to the point and the above word limit should be adhered to as far as possible.

SET I

SECTION A
Question.1. Define indifference curve.
Answer. Indifference curve refers to the graphical representation of various alternative combinations of bundles of two goods among which the consumer is indifferent.

Question.2. If due to fall in the price of good X, demand for good Y rises, the two goods are: (choose the correct alternative)
(a) Substitutes (b) Complements (c) Not related (d) Competitive
Answer. (b) Complements

Question. 3. If Marginal Rate of Substitution is increasing throughout, the Indifference Curve will be: (choose the correct alternative)
(a) Downward sloping convex (b) Downward sloping concave
(c) Downward sloping straight line (d) Upward sloping convex
Answer. (b) Downward sloping concave

Question. 4. Giving reason comment on the shape of Production Possibilities Curve based on the following schedule:
Image may be NSFW.
Clik here to view.
cbse-sample-papers-for-class-12-economics-outside-delhi-2015-1

Answer. Marginal rate of transformation (MRT) is the ratio of number of units of a commodity sacrificed to gain an additional unit of another commodity.
\(MRT=\frac { \Delta \quad in\quad units\quad sacrificed }{ \Delta \quad in\quad units\quad gained } \)
It represents the slope of PPF. As can be seen from the given schedule, MRT is increasing, i.e., more and more units of Y have to be sacrificed for a unit increase in production of X.The PPC would be concave to the origin.
Image may be NSFW.
Clik here to view.
cbse-sample-papers-for-class-12-economics-outside-delhi-2015-2

Question.5. What is likely to be the impact of “Make in India” appeal to the foreign investors by the Prime Minister of India, on the production possibilities frontier of India? Explain.
Answer. The Prime Minister of India has appealed to the people the world over to come and make in India and sell their products in any country of the world. This initiative is expected to focus on job creation and skill enhancement in various sectors.
The Prime Minister has appealed to the foreign investors to make investments in various sectors and boost the ‘Make in India’ initiative with the much needed infrastructure and rapid urbanisation. The inflow of foreign resources will increase the country’s available resources and boost the country’s developmental process thereby shifting the Production Possibility Frontier (PPF) upwards to the right signifying increase in its productive capacity.
Or
What is likely to be the impact of efforts towards reducing unemployment on the production potential of the economy? Explain.
Answer. Reducing unemployment has no effect on the production potential of the economy because production potential is determined assuming full employment. If the government makes efforts towards reducing unemployment by starting employment generating schemes, since the amount of resources has not changed and only the existing resources are being optimally utilized, therefore the economy would move from below potential production towards or on optimum production potential. This means that the PPF would not be affected but that the economy would move from a production combination inside the PPF to a production combination nearer or on the PPF.

Question.6. Explain the significance of ‘minus sign’ attached to the measure of price elasticity of demand in case of a normal good, as compared to the ‘plus sign’ attached to the measure of price elasticity of supply.
Answer. See Q. 6, 2015 (I Delhi).

Question.7. In a perfectly competitive market the buyers treat products of all the firms as homogeneous. Explain the significance of this feature.
Answer. Homogeneous product means identical in all respects like shape, size, quality etc. The buyers in a perfectly competitive market, treat the products of all firm as homogeneous, they do not differentiate between products of different firms.They are willing to pay only the same price for the products of all the firms. It also implies that no individual firm is in a position to charge a higher price for its product. This ensures uniform price in the market.
Image may be NSFW.
Clik here to view.
cbse-sample-papers-for-class-12-economics-outside-delhi-2015-3

Question.8. What are the effects of ‘price-floor’ (minimum price ceiling) on the market of a good?Use diagram.
Answer. Price floor refers to the minimum price (above the equilibrium price), fixed by the government, which the producers must be paid for their produce. When the government feels that the price fixed by the forces of demand and supply is not remunerative from the producer’s point of view, then it fixes a price (know as price floor) which is more than the equilibrium price, e.g., agricultural price support programs and minimum wage legislation.
The effect of price floor can be better understood with the help of the given diagram.As shown in the diagram, equilibrium is determined at point E and equilibrium price at OP.Suppose to protect the producer’s interest, government declares OP2 as the minimum price (price floor) which is more than the equilibrium price. At this price producers are willing to supply OQs while consumers demand only OQD, there is surplus in the market equal to MN. This excess supply may be purchased by the government either to create buffer stocks or for exports or producers may try to illegally sell below the minimum price.
Note: The following question is for the blind candidates only in lieu of Q. No. 8,
Explain the effects of ‘price-floor’ (minimum price ceiling) on the market of a good.
Answer. When government imposes a lower limit on a price that may be charged by the producers of a good or service, it is called price floor.
Since this price is above the equilibrium price, at this price producers-are willing to supply more but the buyers are willing to buy less. This creates surplus in the market. Due to this producers may adopt illegal ways and sell the product or service at a lower price.

Question.9. A consumer spends Rs 1000 on a good priced at Rs 10 per unit. When its price falls by 20 percent, the consumer spends Rs 800 on the good. Calculate the price elasticity of demand by the Percentage method.
Answer.
Image may be NSFW.
Clik here to view.
cbse-sample-papers-for-class-12-economics-outside-delhi-2015-4

Question.10. What is the behavior of (a) Average Fixed Cost and (b) Average Variable Cost as more and more units of a good are produced?
Answer.
Image may be NSFW.
Clik here to view.
cbse-sample-papers-for-class-12-economics-outside-delhi-2015-5

Or
Define Average Revenue . Show that Average Revenue and Price are same.
Answer.
Image may be NSFW.
Clik here to view.
cbse-sample-papers-for-class-12-economics-outside-delhi-2015-6

Question.11. A consumer consumes only two goods X and Y, both priced at Rs 2 per unit. If the consumer chooses a combination of these two goods with Marginal Rate of Substitution equal to 2, is the consumer in equilibrium? Why or why not? What will a rational consumer do in this situation? Explain.
Answer.
Image may be NSFW.
Clik here to view.
cbse-sample-papers-for-class-12-economics-outside-delhi-2015-7

Or
A consumer consumes only two goods X and Y whose prices are Rs 5 and Rs 4 respectively. If the consumer chooses a combination of the two goods with marginal utility of X equal to 4 and that of Y equal to 5, is the consumer in equilibrium? Why or why not? What will a rational consumer do in this situation? Use utility analysis.
Answer. In utility analysis, the two necessary conditions to attain consumer’s equilibrium in case of two commodities are:
Image may be NSFW.
Clik here to view.
cbse-sample-papers-for-class-12-economics-outside-delhi-2015-8

Question.12. What are the different phases in the Law of Variable Proportions in terms of marginal product? Give reason behind each phase. Use diagram.
Answer. Law of Variable Proportions states that as we increase quantity of only one input keeping other inputs fixed, MP initially increases then falls and becomes zero and finally becomes negative. The behavior of output can be explained with the help of the given diagram. Phase I. Increasing returns to a factor. In the first phase, variable input is too small as compared to the fixed factor so every additional variable factor adds more and more to the total product leading to increase in MP. This is due to better utilization of fixed factor and improved efficiency of variable factor due to specialization. This phase continues till point Q on the diagram.
Image may be NSFW.
Clik here to view.
cbse-sample-papers-for-class-12-economics-outside-delhi-2015-9

Phase II. Diminishing returns to a factor. In the second phase, every additional variable factor adds lesser and lesser amount of output leading to decrease in MP. This happens because after making optimum use of fixed factor, the marginal return or productivity of variable factor begins to diminish. Also there is a limit to the extent of which one factor of production can be substituted for another. This phase continues till point L on the diagram. MP starts falling but remains positive.
Phase III. Negative returns to a factor. In this phase, the employment of additional variable factor causes TP to decline and MP becomes negative. This happens because the amount of variable factor becomes too large in comparison to the fixed factor. Excess of variable factor leads to poor co-ordination between fixed and variable factors. As a result total output falls instead of rising and marginal product becomes negative. Also with increase in variable factor, the advantages of specialization and division of labour start diminishing resulting in inefficiencies of variable factor.
Note: The following question is for the blind candidates in lieu of Q. No. 12 only.
Explain with the help of a numerical example different phases in the Law of Variable Proportions.
Answer. See Q. 12, 2015 (I Delhi).

Question.13. Explain why will a producer not be in equilibrium if the conditions of equilibrium are not met.
Answer. Producer’s equilibrium refers to that price and output combination which brings maximum profit to the producer and profit declines as more is produced. According to MR-MC approach, producer’s equilibrium will be achieved when, (i) MC = MR and (ii) MC > MR after MC = MR output level. Both the conditions are needed to achieve equilibrium.
Image may be NSFW.
Clik here to view.
cbse-sample-papers-for-class-12-economics-outside-delhi-2015-10

(i) MC = MR. Every producer aims to maximise the total profits. For this, a firm compares its MR with its MC. Profits will increase as long as MR exceeds MC. So, equilibrium cannot be achieved when MC < MR as it is still possible to earn profits by increasing production. Producer would also not like to produce when MC>MR because this implies that benefit is less than cost, i.e., the firm will be in equilibrium only when MC=MR.
(ii) MC > MR after MC = MR output level. This condition also has to be fulfilled because MC=MR may occur at more than one output level. But equilibrium output level will be the one after which MC becomes greater than MR because then it will not be possible to earn any profit.
As shown in the diagram, producer’s equilibrium will be determined at OQ level of output corresponding to point E because it is only at this point that both the conditions of equilibrium are met.

Question.14. Market for a good is in equilibrium. The supply of good “decreases”. Explain the chain of effects of this change.
Answer. See Q. 15, 2014 (III Delhi).

SECTION B
Question.15. What is ‘aggregate demand’ in macroeconomics?
Answer. Aggregate demand refers to the total value of final goods and services which all the sectors of an economy are planning to buy at a given level of income during a period of one accounting year.

Question.16. If MFC = 1, the value of multiplier is: (choose the correct alternative)
(a) 0 (b) 1 (c) Between 0 and 1 (d) Infinity
Answer. (d) Infinity

Question.17. Primary deficit in a government budget is: (choose the correct alternative)
(a) Revenue expenditure – Revenue receipts (b) Total expenditure – Total receipts
(c) Revenue deficit – Interest payments (d) Fiscal deficit – Interest payments
Answer. (d) Fiscal deficit – Interest payments

Question.18. Direct tax is called direct because it is collected directly from: (choose the correct alternative) 
(a) The producers on goods produced (b) The sellers on good sold
(c) The buyers of goods (d) The income earners
Answer. (d) The income earners

Question.19. Other things remaining the same, when in a country the market price of foreign currency falls, national income is likely: (choose the correct alternative)
(a) to rise (&) to fall .
(c) to rise or to fall (d) to remain unaffected
Answer. (b) to fall

Question.20. If the Real GDP is Rs 400 and Nominal GDP is Rs 450, calculate the Price Index (base = 100).
Answer.
Image may be NSFW.
Clik here to view.
cbse-sample-papers-for-class-12-economics-outside-delhi-2015-11

Question.21. What are fixed and flexible exchange rates?
Answer. Fixed exchange rate refers to a system in which exchange rate for a currency is fixed by the government/central bank and is not influenced by the demand and supply of foreign exchange. The basic purpose of adopting this system is to ensure stability in foreign trade and capital movements.
Flexible exchange rate refers to a system in which exchange rate is determined by forces of demand and supply of different currencies in the foreign exchange market. There is no official (government) intervention in the foreign exchange market. It is also known as floating exchange rate.
Or
Explain the meaning of Managed Floating Exchange Rate.
Answer. Managed floating exchange rate refers to a system in which foreign exchange rate is determined by market forces and central bank influences the exchange rate through intervention in the foreign exchange market. It is a hybrid of fixed exchange rate and flexible exchange rate system. In this system, central bank intervenes in the foreign exchange market to restrict the fluctuations in the exchange rate within certain limits. When foreign exchange rate is too high, the central bank starts selling foreign currency from its reserves. When it is too low central bank starts buying foreign currency. The aim is to keep the exchange rate close to the desired target values. It is also known as ‘dirty floating’.

Question.22. Where is ‘borrowings from abroad’ recorded in the Balance of Payments Accounts? Give reasons.
Answer. “Borrowings from abroad” will be recorded in the ‘capital account’ of BOP account because it increases international liability of the country.
Capital account of BOP account records international transactions which affect foreign assets and foreign liabilities of domestic country with rest of the world. Transactions relating to borrowings from abroad by private sector, government etc. are recorded on the credit side of capital account. It is recorded on the credit side because it leads to inflow of – foreign exchange in the country.

Question.23. Explain the “Bankers’ Bank function” of the central bank.
Answer. Being the apex bank, the Central Bank acts as the banker to other banks, i.e., it has the same relationship with commercial banks as the latter maintains with the general public.
As the banker to banks, the Central Bank functions in three capacities:
(i) Custodian of Cash reserves. Commercial banks are required to keep a certain percentage of their cash reserves (known as cash reserve ratio or CRR) with the Central Bank.
(ii) Lender of the last resort When commercial banks fail to meet their financial requirements from other sources, they approach the Central Bank to give loans and advances as lender of the last resort.
(iii) Clearing House. As Central Bank holds the cash reserves of all the commercial banks, it becomes easier and more convenient for it to act as their clearing house. It provides cheque clearing and remittance facilities to commercial banks.
Or
Explain the “Bank of Issue function” of the central bank.
Answer. See Q. 23, 2015 (I Delhi).

Question.24. Currency is issued by the central bank, yet we say that commercial banks create money. Explain. How is this money creation by commercial banks likely to affect the national income? Explain.
Answer. Money creation or credit creation is one of the most important activities of commercial banks. Through the process of money creation, commercial banks are able to create credit, which is in far excess of the initial deposits. There are two components of money supply: currency with the public and demand deposits with commercial banks. Currency is issued by the central bank whereas demand : deposits are created by commercial banks. Banks use the money in demand deposits to give loans. When a bank gives loan to somebody, it simply creates a bank deposit equal to the loan amount in the name of the borrower. This new deposit created by the bank becomes a part of the money supply. This is how banks create money. Commercial banks lend money mainly to investors thereby increasing investment. Increase in money supply in the economy would increase the aggregate demand leading to increase in the national income.

Question.25. An economy is in equilibrium. Calculate the Investment Expenditure from the following:
National Income = 800
Marginal Propensity to Save = 0.3
Autonomous Consumption = 100
Answer.
Image may be NSFW.
Clik here to view.
cbse-sample-papers-for-class-12-economics-outside-delhi-2015-12

Question.26. Giving reason explain how the following should be treated in estimation of national income:
(i) Payment of interest by a firm to a bank
(ii) Payment of interest by a bank to an individual
(iv) Payment of interest by an individual to a bank
Answer. (i) Payment of interest by a firm to a bank will be included in national income as such interest is paid on loan taken by a firm for productive purpose and is a factor payment by a producer.
(ii) Payment of interest by a bank to an individual is considered as a factor payment because bank borrows for carrying out banking services and therefore it will be included in national income.
(iii) Payment of interest by an individual to a bank will not be included in national income because it is non-factor receipt as the loan has been used for consumption purpose and not for production.

Question.27. What is ‘deficient demand’? Explain the role of ‘Bank Rate’ in removing it.
Answer. If equilibrium level of income is determined before the stage of full employment, it is a situation of deficient demand. This situation arises when aggregate demand falls short of aggregate supply at the full employment level. It gives rise to deflationary gap. It causes fall in price level.
In such a situation due to inadequacy of demand, production will be less and therefore involuntary unemployment will be created. This means that all the resources will not be fully employed. Therefore the deficiency in aggregate demand at full employment level is the measure of deficient demand. It also means that during deficient demand, equilibrium is determined at a level less than full employment equilibrium.
Role of Bank Rate. Bank Rate is one of the methods which can be used for correcting the situation of deficient demand. Bank rate is the rate of interest which a Central Bank charges from the commercial banks for giving them credit. During deficient demand, the central bank reduces the bank rate in order to expand, credit. When Central Bank lowers bank rate, commercial banks also lower their lending rates. Since borrowing becomes cheaper, it induces people to borrow more funds. It ultimately leads to increase in the aggregate demand and thus helps in reducing deficient demand.
Or
What is ‘excess demand’? Explain the role of ‘Reverse Repo Rate’ in removing it.
Answer. If equilibrium level of income is determined after the stage of full employment, it is a situation of excess demand. Excess demand refers to the situation when aggregate demand is more than the aggregate supply corresponding to full employment level of output in the economy. Excess demand gives rise to inflationary gap.
Inflationary gap refers to the gap by which actual aggregate demand exceeds the aggregate demand required to establish full employment equilibrium. As a result of . excess demand, prices increase because corresponding to increase in demand, output cannot increase.
Role of Reverse Repo Rate. Reverse Repo Rate (RRR) is the interest rate at which the commercial banks can deposit their funds with the Central Bank. Central bank can reduce excess demand by raising the RRR. Raising the Reverse Repo Rate (RRR) gives incentive to the commercial banks to park their funds with the central bank. This reduces liquidity with the commercial banks and adversely affects credit creation. Borrowing by public from banks fall causing aggregate demand to fall reducing inflationary gap.

Question.28. Explain how the government can use the budgetary policy in reducing inequalities in incomes.
Answer. Government budget is a reflection of certain goals the government wants to achieve. These goals are directly related to governments economic, social and political policies. Reducing inequalities in income and wealth is one such objective which the government strives to achieve through fiscal policy measures such as tax and expenditure policy. First, government can impose higher taxes on the income of the rich and on the goods . and services consumed by the rich. It will reduce the disposable income of the rich. The money so collected can be spent on die poor in the form of free education, free medical facilities, cheaper housing etc. in order to raise their disposable income. The government must spend reducing inequalities in the distribution of income.

Question.29. Calculate the ‘National Income’ and ‘Private Income’:
Image may be NSFW.
Clik here to view.
cbse-sample-papers-for-class-12-economics-outside-delhi-2015-13

Answer.
Image may be NSFW.
Clik here to view.
cbse-sample-papers-for-class-12-economics-outside-delhi-2015-14

SET II

Note: Except for the following questions, all the remaining questions have been asked in Set-1.
SECTION A
Question.2. Define budget line.
Answer. Budget line is a graphical representation of all possible combinations of two goods which can be purchased with given income and prices, such that the cost of each of these combinations is equal to the money income of the consumer.

Question.5. Giving reason comment on the shape of Production Possibilities Curve based on the following schedule:
Image may be NSFW.
Clik here to view.
cbse-sample-papers-for-class-12-economics-outside-delhi-2015-15

Answer. MRT being constant PPC will be a downward sloping straight line implying that same amount of a commodity is sacrificed to gain an additional unit of another commodity.
Image may be NSFW.
Clik here to view.
cbse-sample-papers-for-class-12-economics-outside-delhi-2015-16

Question.8. Explain the implication of non-price competition in an oligopoly market.
Answer. Non-price competition means competition between firms on the basis of methods other than price. Under oligopoly, firms are in a position to influence the prices. However, they try to avoid price competition for fear of price-war. They follow the policy of price rigidity. Price rigidity refers to a situation in which price tends to stay fixed irrespective of changes in demand supply Conditions. Firms use other methods like advertising, better services to customers etc. to compete with each other.

Question.10. A consumer spends Rs 100 on a good priced at Rs 4 per unit. When its price falls by 25 percent, the consumer spends Rs 75 on the good. Calculate the price elasticity of demand by the Percentage method.
Answer.
Image may be NSFW.
Clik here to view.
cbse-sample-papers-for-class-12-economics-outside-delhi-2015-17

Question.11. Market for a good is in equilibrium. The supply of the good “increases”. Explain the chain of effects of this change.
Answer. See Q. 12, 2015 (III Delhi).

SECTION B
Question.21. If the Real GDP is ?500 and Price Index (base = 100) is 125, calculate the Nominal GDP.
Answer.
Image may be NSFW.
Clik here to view.
cbse-sample-papers-for-class-12-economics-outside-delhi-2015-18

Question.23. An economy is in equilibrium. Calculate the Marginal Propensity to Save from the following:
National Income = 1000
Autonomous Consumption = 100
Investment = 120
Answer.
Image may be NSFW.
Clik here to view.
cbse-sample-papers-for-class-12-economics-outside-delhi-2015-19

Question.29. Calculate “Net National Product at Market Price’ and ‘Personal Income’:
Image may be NSFW.
Clik here to view.
cbse-sample-papers-for-class-12-economics-outside-delhi-2015-20

Answer.
Image may be NSFW.
Clik here to view.
cbse-sample-papers-for-class-12-economics-outside-delhi-2015-21

SET III

Note: Except for the following questions, all the remaining questions have been asked in Set-l and Set-II.
SECTION A
Question.3. Define Indifference Map.
Answer. Indifference map refers to the family of indifference curves that represent consumer preferences over all the bundles of the two goods.

Question.5. Distinguish between cooperative and non-cooperative oligopoly.
Answer. If the firms co-operate with each other in determining price or output or both, it is called co-operative oligopoly. It is also called as collusive oligopoly.
If firms in an oligopoly market compete with each other, it is called non-cooperative oligopoly. It is also called as non-collusive oligopoly.

Question.8. Giving reason comment on the shape of Production Possibilities Curve based on the following tables
Image may be NSFW.
Clik here to view.
cbse-sample-papers-for-class-12-economics-outside-delhi-2015-22

Answer.
Image may be NSFW.
Clik here to view.
cbse-sample-papers-for-class-12-economics-outside-delhi-2015-23

Since MRT or the ratio between the units of goods sacrificed (Y) to units of commodity gained (X) is increasing . PPC-will he downward sloping concave to the origin implying that more and more of Y will have to be sacrificed for a unit increase in production of X.

Question.9. A consumer spends Rs 400 on a good priced at Rs 8 per unit. When its price rises by 25 per cent, the consumer spend Rs 500 on the good. Calculate the price elasticity of demand by the Percentage method.
Answer.
Image may be NSFW.
Clik here to view.
cbse-sample-papers-for-class-12-economics-outside-delhi-2015-24

Question.12. Market for a good is in equilibrium. Demand for the good “increases”. Explain the chain of effects of this change.
Answer. See Q. 14, 2015 (I Delhi).

SECTION B
Question.22. If the Nominal GDP is Rs 600 and Price Index (base = 100) is 120, calculate the Real GDP.
Answer.
Image may be NSFW.
Clik here to view.
cbse-sample-papers-for-class-12-economics-outside-delhi-2015-25

Question.24. An economy is in equilibrium. Calculate the National Income from the following :
Autonomous Consumption = 120
Marginal Propensity to Save = 0.2
Investment Expenditure = 150
Answer.
Image may be NSFW.
Clik here to view.
cbse-sample-papers-for-class-12-economics-outside-delhi-2015-26

Question.29. Calculate ‘Net Domestic Product at Market Price’ and ‘Gross National Disposable Income’:
Image may be NSFW.
Clik here to view.
cbse-sample-papers-for-class-12-economics-outside-delhi-2015-27

Answer.
Image may be NSFW.
Clik here to view.
cbse-sample-papers-for-class-12-economics-outside-delhi-2015-28

The post CBSE Sample Papers for Class 12 Economics Outside Delhi – 2015 appeared first on Learn CBSE.

CBSE Sample Papers for Class 12 Economics Outside Delhi 2010

CBSE Sample Papers for Class 12 Economics Outside Delhi 2010

Time allowed : 3  hours                                                                                         Maximum marks 100

GENERAL INSTRUCTIONS
(i) All questions in both the sections are compulsory.
(ii) Marks for questions are indicated against each.
(iii) Questions No. 1-5 and 17-21 are very short-answer questions carrying 1 nick each. They are required to be answered in one sentence each.
(iv) Questions No. 6-10 and 22-26 are? short-answer questions carrying 3 marks each. Answers to them should normally not exceed 60 words each.
(v) Questions No. 11-13 and 27-29 are also short-answer questions carrying 4 marks each. Answers to them should normally not exceed 70 words each.
(vi) Questions No. 14-16 and 30-32 are long-answers questions carrying 6 marks each. Answers to them should normally not exceed 100 words each.
(vii) Answers should be brief and to the point and the above word limit should be adhered to as far as possible.

SET I

SECTION A

Question.1. Define a budget line.
Answer. A budget line graphically represents various bundles of two goods (i.e., all possible combinations of two goods) which a consumer can buy by spending his entire income at the prevailing prices.

Question.2. What is meant by inferior good in economics?
Answer. Inferior goods are those whose demand falls with the rise in the level of income.

Question.3. In which market form can a firm not influence the price of the product?
Answer. Under perfect competitition, a firm cannot influence the price of the commodity. In short, a competitive firm is a price taker.

Question.4. Define monopoly.
Answer. Monopoly may be defined as a market situation, where there is a single seller of a commodity and there are no close substitutes of that commodity.

Question.5. What can you say about the number of buyers and sellers under monopolistic competition?
Answer. Under monopolistic competition, there are a large number of small sellers selling closely related products and the number of buyers is very large.

Question.6. Explain the effect of the following on the price elasticity of demand of a Commodity:
(i) Number of substitutes
(ii) Nature of the commodity
Answer. See Q. 7, 2009

Question.7. Explain any two causes of ‘increase’ in demand of a commodity.
Answer. Two causes of increase in demand of a commodity:
(i) Increase in income of the consumer:
As the income of a consumer increases, it leads to an : increase in the demand for normal goods like pens,
shoes, clothes etc. This leads to a righward shift in the demand curve.
(ii) Increase in Price of substitute good:
An increase in the price of a substitute causes an increase in demand for the particular good and thereby shifts demand curve to the right.
Image may be NSFW.
Clik here to view.
cbse-sample-papers-class-12-economics-delhi-2010-1

Or
Explain the inverse relationship between price and quantity demanded of a commodity.
Answer. The law of diminishing marginal utility states that as we consume more and more units of a commodity, the utility derived from each successive unit goes on decreasing. This law holds true under certain assumptions, i.e. a reasonable quantity of the commodity is consumed and that consumption is a continuous process. This law can be explained with the help of a utility schedule.
Image may be NSFW.
Clik here to view.
cbse-sample-papers-class-12-economics-delhi-2010-2

Image may be NSFW.
Clik here to view.
cbse-sample-papers-class-12-economics-delhi-2010-3

Answer. According to the law of demand, price and quantity demanded are inversely related, other things remaining the same. As Such, when price increases, demand for a good falls and when there is a decline in price of that good, the demand for it increases. As such, usually a demand curve slopes downwards.
The inverse relationship between price and demand can also be explained with the help of a demand schedule given below:
Image may be NSFW.
Clik here to view.
cbse-sample-papers-class-12-economics-delhi-2010-4

Image may be NSFW.
Clik here to view.
cbse-sample-papers-class-12-economics-delhi-2010-5

As is evident from the Schedule, when price increases, demand falls and vice-versa. This is shown by the demand cuiVe DD in the adjacent diagram.

Question.8. A firm’s average fixed cost, when it produces 2 units, is Rs. 30. Its average total cost schedule is given below. Calculate its marshal cost and average variable cost at each level of output.
Image may be NSFW.
Clik here to view.
cbse-sample-papers-class-12-economics-delhi-2010-6

Answer.
Image may be NSFW.
Clik here to view.
cbse-sample-papers-class-12-economics-delhi-2010-7

Question.9. Total revenue is Rs. 400 when the price of the commodity is Rs.2 per unit When price rises to Rs. 3 per unit, the quantity supplied is 300 units. Calculate the price elasticity of supply?
Answer.
Image may be NSFW.
Clik here to view.
cbse-sample-papers-class-12-economics-delhi-2010-8

Image may be NSFW.
Clik here to view.
cbse-sample-papers-class-12-economics-delhi-2010-9

Question.10. Why is tile number of firms small in an oligopoly market? Explain. 3
Answer. Oligopoly is a market situation where the number of sellers is very small (more titan two) and each seller sells a substantial portion of the output of the industry. The number of sellers is so small that each seller knows the attitude of the rival firms.
Though the seller is free to pursue independent price policy, but in practice he does not change the price, because of the fear of retaliation by other sellers.
This is so because if he increases his price, others may not increase and his sale will fall. On the other hand, if he lowers the price others may also lower their prices and he may not get the benefit of that. Therefore price tends to be rigid under oligopoly.

Question.11. Explain the problem of ‘how to produce’.
Answer. The problem of ‘how to produce’ relates to the choice of technique by an economy for producing a commodity. Therefore, in view of the fact that resources are scarce, a cost effective technique is chosenr Therefore the decision as to how goods should be produced, depends on the price of the factors. If labour is abundant and cheap, labour intensive technology of production would lead to cost effective production. In case labour is scarce mid costly, capital intensive technology would be used For Example, in a country like India, labour-intensive technique is adopted and in a country like Japan, capital-intensive technique is adopted.
Or
Distinguish between microeconomics and macroeconomics. Give examples.
Answer. Microeconomics studies the problems of individual economic units, such as a firm and industry, a consumer etc. As such the problems of consumer’s equilibrium, producer’s equilibrium, price determination, factor determination etc. are the part of the scope of Microeconomics. Its main tools are demand and supply of a particular commodity. Macroeconomics studies the economic problems relating to an economy, such as National Income, total savings, employment determination, etc. International trade and banking also form part bf Macroeconomics. Its mam tools are aggregate demand and aggregate supply of an economy.

Question.12. When price of a commodity falls by Rs. 1 per unit, its quantity demanded rises by 3 units. Its price elasticity of demand is (-) 2. Calculate its quantity demanded if the price before the change was Rs. 10 per unit.
Answer.
Image may be NSFW.
Clik here to view.
cbse-sample-papers-class-12-economics-delhi-2010-10

Question.13. How does the equilibrium price of a ‘normal’ commodity change when income of its buyers falls? Explain the chain of effects.
Answer. In case of a normal commodity, a fall in the income of the consumer signifies a fall in its demand. As shown in the diagram, point E is the equilibrium point with OP and OQ, being the equilibrium price and equilibrium quantity respectively. When demand falls due to a fall in the income of a buyer, a new equilibrium will be attained at point E1 Equilibrium price will fall from OP to OP1 and the equilibrium quantity from OQ to OQ1
Image may be NSFW.
Clik here to view.
cbse-sample-papers-class-12-economics-delhi-2010-11

Question.14. State whether the following statements are true or false. Give reasons for yonr answer .
(i) When marginal revenue is constant and not equal to zero, then total revenue will also be constant.
(ii) As soon as marginal cost starts rising, average variable cost also starts rising.
(iii) Total product always increases whether there is increasing returns or diminishing returns to a factor.
Answer. (i) False. When marginal revenue is constant total revenue will be increasing.
(ii) False. MC cuts AVC from its lowest point but near the point of intersection AVC is falling and MC is increasing.
Image may be NSFW.
Clik here to view.
cbse-sample-papers-class-12-economics-delhi-2010-12

(iii) True. TP always increases whether there is increasing retruns to a factor. Initially TP increases at increasing rate then it increases at diminishing rate.

Question.15. What are the conditions of consumer’s equilibrium under the indifference curve approach? What changes will take place if the conditions are not fulfilled to reach equilibrium?
Answer. See Q. 14, 2010 (I Delhi)

Question.16. From the following schedule find out the level of output at which the producer is in equilibrium, using marginal cost and marginal revenue approach. Give reasons for your answer.
Image may be NSFW.
Clik here to view.
cbse-sample-papers-class-12-economics-delhi-2010-13

Answer.
Image may be NSFW.
Clik here to view.
cbse-sample-papers-class-12-economics-delhi-2010-14

The equilibrium ouput will be 3, because at this level MR and MC are equal. The equilibrium will be Rs. 6. At any other level of ouput either MR > MC or MR < MC.
Or
Explain the law of returns to a factor with the help of total product and marginal product schedule.
Answer.See Q. 15, 2008 (I Delhi).

SECTION B
Question.17. Give the meaning of money.
Answer. Anything which is accepted as a medium of exchange is called money.

Question.18. What is meant by revenue deficit?
Answer. Revenue deficit is the excess of the government’s revenue expenditure over its revenue receipts.

Question.19. What is ex-ante aggregate demand?
Answer.Ex-ante aggregate demand is the planned (ex-ante) expenditure which people are willing to incur on consumer and investment demand.

Question.20.Give the meaning of inflationary gap.
Answer.The excess of aggregate demand over aggregate supply at the full employment level is inflationary gap.

Question.21.State two sources of demand for foreign exchange.
Answer.The two sources of demand for foreign exchange are:
• for making payments for the imported goods and
• for making payments for unilateral transfers such as sending gifts.

question.22.Distinguish between real and nominal gross domestic product
Answer.See Q. 22, 2010 (II Delhi).
Or
Giving reasons, classify the following into intermediate and final goods:
(i) Machines purchased by a dealer of machines.
(ii) A car purchased by a household.
Answer.(i) Machines purchased by a dealer of machines is an intermediate good because these are for re-sale.
(ii) A car purchased by a household is a final good because expenditure incurred on this item is final expenditure and it is not for re-sale.

Question.23.Explain the ‘banker to the government’ function of the Central Bank.
Answer.See Q. 23, 2010 (II Delhi)

Question.24.Explain the allocation function of a Government Budget.
Answer.An important function of a Government Budget is to allocate the resources on different items of expenditure. There are various items of expenditure for which money is allocated, for example, development projects, welfare activities like health care, education, etc. and many other items meant for the health of the country. On the basis of priorities and significance of the items Government allocates resources through its budget.

Question.25.Distinguish between autonomous and accommodating transactions of balance of payments account
Answer.See Q. 25, 2010, (III Delhi)

Question.26.Giving two examples, explain why there is a rise in demand for a foreign currency when its price falls.
Answer.See Q. 26, 2010 (III Delhi).

Question.27.How does a commercial bank create money?
Answer.See Q. 27, 2010 (I Delhi).
(or)
Explain how do open market operations’ by the central bank affect money creation by commercial banks.
Answer.An important tool in the hands of central bank for affecting money creation by commercial banks is open market operations. Open market operations mean the sale and purchase of government securities by central bank from the public and commercial banks in the open market. Through sale and purchase of securities, liquidity position of commercial banks is affected and this finally results in the capacity of commercial banks to create credit.
When central bank sells securities to the commercial banks, the cash reserves of the commercial banks will reduce as there is flow of money out of commercial banks. This will decrease the credit creation and money supply in the economy.
When central bank buys securities from the commercial banks, the cash reserves with banks increase. This will increase the credit creation and money supply in the economy.

Question.28. Giving reasons, state whether the following statements are true or false:
(i) When marginal propensity to consume is zero, the value of investment multiplier
will also be zero.
(ii) Value of average propensity to save can never be less than zero.
Answer. (i) The statement is false because when MPC is 0, the value of investment multiplier will be 1 and not 0.
(ii) The statement is false because the value of average propensity to save can be less than 0, when consumption is more than income. ,

Question.29. Distinguish between:
(a) Capital expenditure and Revenue expenditure
(b) Fiscal deficit and Primary deficit.
Answer. (a) An expenditure which does not result into creation of assets or reduction of liability is called revenue expenditure, such as payment of salaries to the employees. An expenditure which results into creation of assets or reduction in liabilities is called a capital expenditure, such as construction of a dam or bridge, etc.
(b) Fiscal deficit is equal to the excess of total expenditure (revenue expenditure and capital expenditure.) over total receipts (revenue receipts and capital receipts), excluding borrowings. Fiscal deficit indicates borrowing requirements of the Government during the budget year.
Primary deficit equals Fiscal deficit minus interest payments. In short,
Primary deficit = Fiscal deficit – Interest payments Primary deficit shows actual amount of borrowings of the government.

Question.30. How Primary deficit shows actual amount of borrowings of the government will you treat the following while estimating National Income of India? Give reasons for your answer.
(i) Dividend received by a foreigner from investment in shares of an Indian company.
(ii) Profits earned by a branch of an Indian bank in Canada.
(iii) Scholarship given to Indian students studying in India by a foreign company.
Answer. (i) It is factor income to abroad and, therefore, it should be subtracted from National Income.
(ii) Profits earned by aqjndian bank is part of income earned from abroad and, therefore, it will be included in National Income.
(iii) It is a transfer payment and hence not included in National Income.
Or
Explain the problem of double counting in estimating National Income with the help of an example. Also explain two alternative ways of avoiding the problem.
Answer. While estimating national product, the value of a commodity should be counted only once. If the value of a commodity is counted more than once, this is known as the problem of double counting, such as counting the value of sugarcane arid sugar both. Here, sugarcane used as raw material in the production of sugar has been counted twice. In the given example, we should only count the value of sugar which is a final product. Sugarcane is a raw material or an intermediate good. Therefore, while calculating the national product, value of intermediate goods should not be counted. In this way, by finding the value of final goods, the problem of double counting is solved..
The problem of double counting can be solved with the help of value added method also. This can be explained with the help of die example of production of sugar taken above. Let us say that the value of sugarcane is Rs. 1,000 and that of sugar Rs. 1,500. Here, in the first production stage, where sugarcane is produced, value added is Rs. 1,000 (sale price – cost of production) and during the second and final production stage where sugar is manufactured, value added is Rs. 500 (Rs. 1,500 – Rs. 1,000). Therefore total value added is Rs. 1,500
(Rs. 1,000 + 1500) which is equal to the value of final product i.e., sugar.
Image may be NSFW.
Clik here to view.
cbse-sample-papers-class-12-economics-delhi-2010-15

Thus the problem of double counting can be avoided either by finding value added or by finding the value of final goods.

Question.31. In an economy the equilibrium level of income is Rs. 12,000 crows. Hie ratio of marginal propensity to consume and marginal propensity to save is 3 :1. Calculate the additional investment needed to reach a new equilibrium level of income of Rs. 20,000 crores.
Answer.
Image may be NSFW.
Clik here to view.
cbse-sample-papers-class-12-economics-delhi-2010-16

Question.32. Calculate (a) Gross domestic product at market price and (6) Factor income from abroad from the following data:
Image may be NSFW.
Clik here to view.
cbse-sample-papers-class-12-economics-delhi-2010-17

Answer.
Image may be NSFW.
Clik here to view.
cbse-sample-papers-class-12-economics-delhi-2010-18

Image may be NSFW.
Clik here to view.
cbse-sample-papers-class-12-economics-delhi-2010-19

SET II

Note : Except for the following questions, all the remaining questions have been asked in Set I.
SECTION A
Question.2. What is meant by normal good in economics?
Answer. Those goods whose demand rises with the increase in the income of the consumer are normal goods.

Question.8. Given below is the cost schedule of a firm. Its total fixed cost is Rs. 120. Calculate the marginal cost and average variable cost at each level of output.
Image may be NSFW.
Clik here to view.
cbse-sample-papers-class-12-economics-delhi-2010-20

Answer.
Image may be NSFW.
Clik here to view.
cbse-sample-papers-class-12-economics-delhi-2010-21

Question.10. Giving reason, distinguish between the behaviour of demand curves of firms under perfect competition and monopolistic competition.
Answer. Under perfect competition demand curve is perfectly elastic because products are homogeneous. In short, under perfect competition a firm can sell as much as it likes on the prevailing price because of which, as shown in diagram (i), its demand curve is a straight, horizontal line.
Under monopoly, a firm can sell more only when it lowers down its price. With the result its demand curve has a negative slope, i.e. its demand is less elastic as shown in diagram (ii).
Image may be NSFW.
Clik here to view.
cbse-sample-papers-class-12-economics-delhi-2010-22

Question.12. The price elasticity of demand of a commodity is (-) 1.5. When its price falls by Rs. 1 per unit, its quantity demanded rises by 3 units. If the quantity demanded before the price change was 30 units, what was the price at this demand? Calculate.
Answer.
Image may be NSFW.
Clik here to view.
cbse-sample-papers-class-12-economics-delhi-2010-23

Image may be NSFW.
Clik here to view.
cbse-sample-papers-class-12-economics-delhi-2010-24

SECTION B
Question.18. Define tax.
Answer. A tax is a compulsory contribution which a person has to pay to the Government to defray the expenses of the Government.

Question.19. Give the meaning of marginal propensity to save.
Answer. Marginal propensity to save is the ratio of change in saving to change in income.
Image may be NSFW.
Clik here to view.
cbse-sample-papers-class-12-economics-delhi-2010-25

 

Question.23. Explain the ‘lender of the last resort’ function of the central bank.
Answer. When commercial banks exhaust all resources to supplement their funds at the time of liquidity crisis, they turn to the central bank as a last resort.
As the lender oflast resort, central bank guarantees solvency and provides financial support to commercial banks (i) by rediscounting their eligible securities and bills of exchange and (ii) by providing loans against their securities. This saves the banks from failure and the banking system from a breakdown.

Question.26. Distinguish between fixed and flexible foreign exchange rate.
Answer. Fixed exchange rate is that rate which is fixed by the Government and does not change with the change in demand and supply of foreign exchange. On the other hand, flexible foreign exchange rate is that which is determined by the forces of demand arid supply of a given country in the foreign exchange market. These days, in most of the countries, foreign exchange rates are flexible

Question.32. From the following data, calculate (a) Gross domestic product at market price, and (b)Factor Income to abroad
Image may be NSFW.
Clik here to view.
cbse-sample-papers-class-12-economics-delhi-2010-26

Answer.
Image may be NSFW.
Clik here to view.
cbse-sample-papers-class-12-economics-delhi-2010-27

Image may be NSFW.
Clik here to view.
cbse-sample-papers-class-12-economics-delhi-2010-28

SET III

Note: Except far the following questions, all the remaining questions have been asked in Set I and Set II.
SECTION A
Question.2. When is the demand* for a good said to be perfectly inelastic?
Answer. Demand for a good will be perfectly inelastic when it does not change with the change in price. A perfectly inelastic curve is a straight vertical line.

Question.8. From tjte following cost schedule of a firm, calculate marginal cost and average variable cost at each level of output.
Image may be NSFW.
Clik here to view.
cbse-sample-papers-class-12-economics-delhi-2010-29

Answer.
Image may be NSFW.
Clik here to view.
cbse-sample-papers-class-12-economics-delhi-2010-30

Question.12. Quantity demanded of a commodity rises by 6 units when its price falls by Rs. 1 per unit Its elasticity of demand is (-) 1. If the pnce before the change was Rs. 20 per unit, calculate quantity demanded at this price.
Answer.
Image may be NSFW.
Clik here to view.
cbse-sample-papers-class-12-economics-delhi-2010-31

SECTION B
Question.18. Give two examples of direct tax.
Answer. Wealth tax and Income tax are direct taxes.

Question.19. When is there the equilibrium level of National Income?
Answer. At the equilibrium level of National Income aggregate demand is equal to aggregate supply.

Question.23. Explain the meaning of cash reserve ratio and statutory liquidity ratio.
Answer. Cash Reserve Ratio: It is that fraction of the deposits which commercial banks are required under law to keep as cash reserves with the central bank. CRR is a powerful instrument to control credit and lending capacity of the banks.
Statutory Liquidity Ratio is a part of the deposits which commercial banks have to keep with themselves. Banks are required to keep a fixed percentage of its assets in cash, gold or other securities. SLR is raised to reduce the ability of die banks to give credit.

Question.28. Giving reasons, state whether the following statements ate trueorfalse:
(i) If tihe rath) of marginal propensity to consume and marginal propensity to save is 4:1, the value of investment multiplier will be 4.
(ii) Sum of marginal propensity to save and marginal propensity to consume is always equal to 1.
Answer.
Image may be NSFW.
Clik here to view.
cbse-sample-papers-class-12-economics-delhi-2010-35

Question.32. From the following data calculate (a) Gross domestic product at market price and (b) Factor income from abroad.
Image may be NSFW.
Clik here to view.
cbse-sample-papers-class-12-economics-delhi-2010-36

Answer.
Image may be NSFW.
Clik here to view.
cbse-sample-papers-class-12-economics-delhi-2010-37

The post CBSE Sample Papers for Class 12 Economics Outside Delhi 2010 appeared first on Learn CBSE.

CBSE Sample Papers for Class 12 Economics Compartment Delhi -2015

CBSE Sample Papers for Class 12 Economics Compartment Delhi -2015

Time allowed : 3  hours                                                                                         Maximum marks 100

GENERAL INSTRUCTIONS
(i) All questions in both the sections are compulsory.
(ii) Marks for questions are indicated against each.
(iii) Questions No. 1-5 and 17-21 are very short-answer questions carrying 1 nick each. They are required to be answered in one sentence each.
(iv) Questions No. 6-10 and 22-26 are? short-answer questions carrying 3 marks each. Answers to them should normally not exceed 60 words each.
(v) Questions No. 11-13 and 27-29 are also short-answer questions carrying 4 marks each. Answers to them should normally not exceed 70 words each.
(vi) Questions No. 14-16 and 30-32 are long-answers questions carrying 6 marks each. Answers to them should normally not exceed 100 words each.
(vii) Answers should be brief and to the point and the above word limit should be adhered to as far as possible.

SET I

SECTION A
Question.1. The demand curve of a good shifts from DD’ to dd’.
This shift can be caused by: (Choose the correct alternative)
(a) fall in the price of the goods
(b) rise in the price of the goods.
(c) rise in the price of substitute goods.
(d) rise in the price of complementary goods.
Image may be NSFW.
Clik here to view.
cbse-sample-papers-for-class-12-economics-compartment-delhi-2015-1

Answer. (c) rise in the price of substitute goods.

Question.2.When 5 units of a goods are sold, total revenue is Rs 100. When 6 units are sold, marginal revenue is Rs 8. At what price are 6 units sold? (Choose the correct alternative) .
(a) Rs 28 per unit (b) Rs 20 per unit
(c) Rs 18 per unit (d) Rs 12 per unit
Answer. (c) 718 per unit.

Question.3. Suggest any one economic measure by which the government can promote consumption of ‘Khadi’.
Answer. Consumption of Khadi can be promoted providing subsidies on it which Would rMuce its cost and increase its demand.

Question.4. Do rich countries also face central problems? Give reasons for your answer.
Answer. As resources & of an economy are scarce and they have alternative uses, every economy is compelled to make choices in the alternative uses of available resources. The very fact that every’ economy has to face the problem of choice makes it the central problem of the choice makes it the central problem of the economy.
Scarcity of resources is a universal problem faced by all economies no matter how rich a country is. The resources at command are always scarce to satisfy all of one’s needs irrespective of an individual or an economy’s fortunes or prosperity.

Question.5. Explain the effects of floods in Jammu and Kashmir on its production possibilities frontier.
Answer. Production Possibility Curve (PPC) represents various combinations of two goods which can be produced using the given resources and technology. Widespread floods in Jammu and Kashmir have led to destruction of resources, reducing their production capacities and productivity. This will result in a leftward shift in the Production Possibility Frontier.

Question.6. Why does the government of India fix ‘support price’ for some crops? Explain.
Answer. When government fixes price of a product at a level higher than equilibrium price, it is Called ‘support price’ (or Price Floor) It is the minimum price the producers must be paid for their products. Support price is fixed for some crops to safeguard the interest of producers. The main aim of fixing support price is to insulate and protect the farmers from fluctuations in their income which is caused by price variations in the free market.

Question.7. Why is a firm under perfect competition a ‘price-taker’ and not a ‘price-maker’? Explain.
Answer. The equilibrium price is determined with the help of the forces of demand and supply of industry in perfect competition. In perfect competition, the number of buyers and sellers is so large that none of them can influence the prevailing price in the market. Here, the . share of each seller in the total market’Supply is so small that no single seller on its own can influence the price. Hence it has no option but to sell the product at the price given by the industry. It is because of this the each firm is said to be a price-taker in perfect competition and not a price-maker.
Or
Explain ‘non-price competition’ feature of oligopoly.
Answer. Non-Price competition in oligopoly. Since there are only a few firms in oligopoly which are mutually dependent, they Are afraid of competing with each other by lowering their price. It may* start a price war and the firm who starts the price war may ultimately lose. Avoiding price war, the firms use other ways of competition like advertising customer care, free gifts etc. Such a competition is called non-price competition.

Question.8. State Whether the following statements are true or false. Give reasons for your answer.
(a) When total product is constant average product will fall.
(b) Average cost will rise only when marginal cost rises.
Answer. (a) True Average product is calculated as follows : \(AP=\frac { Total\quad Product }{ Units\quad of\quad variable\quad input } \)
Therefore, when total product is Constant, with increase inuahStsi of variable factor, average product will fall.
(b) True, because as has been stated in the relationship between average cost and marginal cost – Average cost will-rise when marginal cost, is more than average cost.

Question.9. Show that price and demand of a commodity are inversely related? Use utility analysis.
Answer. In case Of a single commodity a consumer buys a good only up to the point where marginal utility and price are equal, i.e… .’,MU = Price.Now, suppose price falls, the situation now changes to MU Price.Since marginal utility is greater than price, this induces the customer to buy more of the good . This shows that when price of a good falls, its demand increases. Similarly, if price-rises,, then MU < Price. Therefore, the consumers will buy less quantity of the good. This establishes the inverse relationship between price of a good and its demand.
Or
Explain any two factors that affect price elasticity of demand.
Answer. See Q. 12(0), 2013 (II Delhi).

Question.10. Define ‘market demand’ for a good. State the factors that affect it.
Answer. Market demand refers to the total quantity demanded of a commodity by all consumers at a given price during a period of time.
Following factors affect the market demand of a commodity:

  1. Own price of the commodity.
  2. Price of substitute goods.
  3.  Income of the consumer.
  4. Tastes and preferences of consumers.
  5. Population size/number of buyers
  6. Distribution of income.
  7. Price of complementary goods.

Question.11. Explain the conditions of producer’s equilibrium.
Answer. Producer’s equilibrium refers to the level of output of a commodity. , which gives maximum profit to the producer of that commodity.
The two conditions of Producer’s equilibrium under MC = MR approach are:
(i) MC = MR (Marginal Cost = Marginal Revenue).
(ii) MC > MR after the “MC = MR” Output level.
Explanation:
(i) MC = MR. MR is the addition to total, revenue (TR) from sale of an additional unit of output and MC is addition to total cost (TC) for producing one more unit of output. Every producer aims to maximise its profit. Profits will increases as long as MR exceeds MC and will be maximum when MR = MC.
Equilibrium will not be achieved when MC < MR as it is still possible to increase profits by producing more. The producer will not be in equilibrium When MC > MR because then cost will be greater than benefit implying losses. Therefore, the firm will be in equilibrium only when MC=MR.
(ii) MC is greater than MR after “MC = MR” output level: – MC = MR is necessary but not sufficient condition to ‘equilibrium. This is because this condition may be achieved at more than one,output level. Therefore, only that output level is the equilibrium output when MC,becomes greater than MR after the equilibrium. This is because producing beyond that level would reduce profits. So, the first condition must be supplemented with the Second coneRtioru to attain producers equilibrium.
Or
What is the relationship between:
(a)Marginal revenue and Average revenue; (b) Total revenue and Marginal revenue
Answer. (a) Relationship between Marginal Revenue (MR) andAverage Revenue(AR):
(i) When price remains constant—In perfect competition, since price remains the same, the revenue from every additional unit (MR) is equal to AR. As a result
both AR and MR curves coincide in a horizontal straight line parallel to X-axis.
(ii) When price falls with rise in output—
• This means that AR falls with increase in sales.
• MR (i.e., revenue from every additional unit) will fee less than AR.
• Therefore, both AR and MR curves slope downwards from left to right.
• However, fall in MR is double than that in AR.
(b) Relationship between Total Revenue (TR) and Marginal Revenue (MR):
(i) When price no mains constant—As a result,MR curve is a horizontal straight line parallel to X-axis. As MR is constant, TR increases at a constant rate and is a positively sloped straight line starting from the origin.
(ii)When price falls with rise in output—
• As long as MR is positive, TR increases.
• When MR is zero, TR is maximum.
• When MR becomes negative, TR starts falling.

Question.12.Explain the law of variable proportions with the help of marginal product and total product curves.
Answer.The law of variable proportions explains, the relationship between input and output in die short period. K die short period, some factors are fixed and for increasing the output additional units of variable factors are employed. This law states that “as more and more units of variable inputs are employed, first output increases at increasing rate; after that it increases at decreasing rate and ultimately it fads.” Alternatively, marginal product first increases then it decreases and finally it becomes negative.
This can Tie better understood with The help of a schedule and diagram:
Image may be NSFW.
Clik here to view.
cbse-sample-papers-for-class-12-economics-compartment-delhi-2015-2

As can he seen from the schedule and diagram, when more and more units of the variable factor are employed then initially
– TP increases at an increasing rate, MP also increases
-TP increases at decreasing rate, MP falls but remains positive; and
-TP ultimately falls and MP becomes negative.
Image may be NSFW.
Clik here to view.
cbse-sample-papers-for-class-12-economics-compartment-delhi-2015-3

This relation can be discussed in the following three phases:
PHASE I
Increasing returns to a factor. In this phase, every additional variable factor adds more and more to the total output implying better utilization of the fixed factor and increase in efficiency of the variable factor. This raises the MP of the variable factor.
PHASE II
Diminishing returns to a factor. Now with increase in the quantity of the variable input, the amount of fixed input looks smaller than actually required to engage the increasing variable input. This reduces MP of the variable factor which however remains positive.
This phase ends at the 5th unit of variable input when MP become 2010.This phase starts at Point S and ends at Point P in the diagrams
PHASE III
Negative returns to a factor. In this-phase the quantity of the variable input becomes too big in relation to the quantity of fixed inputs. This reduces the efficiency of the variable input so much that MP becomes negative and TP starts from 6th unit on wards in the schedule and after point P in the diagram.
Note: The following Question is for the Blind Candidates only in lieu of Q.NO 12 Explain the law of variable proportions with the help of schedule.
Answer. As explained in the above question except diagram.

Question.13. Define consumer’s equilibrium. Explain its conditions under indifference curve- analysis.
Answer. Consumers equilibrium means maximum satisfaction level of the consumer at the given level of his income and prices of goods and services in the market.
Conditions of Consumer’s Equilibriums Let the two goods consumed X and Y, whose prices are \({ P }_{ x }\) and \({ P }_{ y }\)respectively. The two conditions of Consumers equilibrium under indifference curve analysis are:
(i) Marginal rate of substitution =Ratio of prices of die two goods
\(MR{ S }_{ xy }=\frac { { P }_{ x } }{ { P }_{ y } } \)
(ii)\(MR{ S }_{ xy }\) falls as more of good X is consumed in place of good Y. \(MR{ S }_{ xy }\) is the number of units of good Y that the consumer is willing to sacrifice, to obtain one extra unit of good X. \( \frac { { P }_{ x } }{ { P }_{ y } } \) is the ratio of prices that prevail in the market. It gives the actual number of units of good Y the consumer is required to sacrifice to obtain one extra unit of good X in the market.
Suppose \(MR{ S }_{ xy }>\frac { { P }_{ x } }{ { P }_{ y } } \)

  • This means that to obtain extra unit of good X the consumer is writing to sentence more units of good Y than what is required to sacrifice in the market because He is getting more marginal utility from per unit consumption of X
  • As he goes on consuming more units to X, \({ MU }_{ x }\) decreases.
  • Therefore the consumer is willing to sacrifice less and less of good Y each time he obtains one extra unit of good X, In other Words \(MR{ S }_{ xy }\) declines.
  • This process continues till \(MR{ S }_{ xy }=\frac { { P }_{ x } }{ { P }_{ y } } \)
    Suppose \(MR{ S }_{ xy }<\frac { { P }_{ x } }{ { P }_{ y } } \)
  •  It means to obtain one extra unit of good X> the consumer is willing to sacrifice less units of good Y than what is required to sacrifice in the market because he is getting less marginal utility from per unit consumption of X.
  • Therefore, he buys less and less of X.
  •  As consumption of good X decreases, its marginal utility increases.
  •  Thus \(MR{ S }_{ xy } \) increases. This process continues till \(MR{ S }_{ xy }=\frac { { P }_{ x } }{ { P }_{ y } } \).

Question.14. The market for commodity A is in equilibrium. The price of its inputs rises. Explain its chain of Effects on equilibrium price, quantity demanded and supplied with the help of a diagram.
Answer. Market for commodity A is in equilibrium at point E where the market demand curve DD and the market supply curve SS intersect so that OP and OQ are the equilibrium price and quantity respectively.Increase in the price of its inputs will increase the cost of production of the firms producing this commodity.Price remaining unchanged, increase in cost leads to fall in demand profits. So the producers are induced to supply ” less quantity of the goods at the same price. Therefore, total market supply will decrease to S1S1.
Image may be NSFW.
Clik here to view.
cbse-sample-papers-for-class-12-economics-compartment-delhi-2015-4

It will create excess.demand in the market equal to AE at the equilibrium price OP. This leads to competition between consumers leading to rise in price. Rise in price leads to fall in demand (contraction) and rise in supply (expansion). These changes continue till the market is in equilibrium again, but at a higher price-OP1 where the equilibrium quantity exchanged is less than before by Q1Q.
Note: The following Question is for the Blind Candidates only in lieu of Q. No. 14.
Give die meaning of ‘excess demand’ of a commodity. Explain its chain of effects on equilibrium price, quantity demanded and quantity supplied.
Answer. Excess of the quantity demanded of a good or service, at a given price, over its supply at that price is known as excess demand in Rest of the answer: Same as above.

SECTION B
Question.15. Which one of the following is a combination of direct taxes? (Choose the correct alternative)
(a) Excise duty and Wealth tax (b) Service tax and Income tax
(c) Excise duty and Service tax (d) Wealth tax and Income tax
Answer.(d) Wealth tax and income tax.

Question.16. State the components of money supply.
Answer. Components of money supply are—(i) Currency with the public; (ii) Demand deposits.

Question.17.Which of the To Showing statements is true?
(a)Fiscal deficit is die difference between total expenditure and total receipts.
(b) Primary deficit is the difference between total receipt and interest payments.
(c) Fiscal deficit is the sum of primary deficit and interest payment
Answer. (c) Fiscal deficit is the sum of primary deficit and interest payment

Question.18. Which of the following is not a flow? (Choose-the correct alternative)
(a) Capital (b) Income (c) Investment (d) Depreciation
Answer. (a) Capital

Question.19. Which of the following is hot a function of money? (Choose the Correct alternative)
(a) Medium of exchange (b) Price stability
(c) Store of value (d) Unit of account
Answer. (b) Price stability

Question.20. What is the relationship between:
(a) Average propensity to consume and average propensity to save.
(b) Marginal propensity to consume and investment multiplier
Answer.
Image may be NSFW.
Clik here to view.
cbse-sample-papers-for-class-12-economics-compartment-delhi-2015-5

Give the meaning of:
(i) involuntary unemployment, and
(ii) inflationary gap.
Ans. (i) Involuntary unemployment. It means’ that part of the labour force of the country which is able and willing to work at the prevailing wage rate’but cannot get jobs because of the economy’s inability to generate employment opportunities. They are . unemployed against their wish.
(ii) inflationary gap. When aggregate demand is greater than aggregate supply (i.e. AD > AS) at the full employment level of income, the gap between the two is called inflationary gap.

Question.21. Describe any three sources of demand for foreign exchange.
Answer. Demand for foreign exchange is caused due to: –

  1. Imports. Governments need foreign exchange to make payments for imports of goods and services.
  2. Purchase of financial assets abroad. Foreign exchange is also demanded by those who wish to make investments in foreign countries.
  3. Sending unilateral transfers. Foreign exchange is needed, for making transfer payments to other countries in the form of gifts or, remittances.

Question.22. S = -100 + 0.2Y is the saving function in an economy. Investment expenditure is 5,000 Calculate the equilibrium level of income.
Answer.
Image may be NSFW.
Clik here to view.
cbse-sample-papers-for-class-12-economics-compartment-delhi-2015-6

Question.23. From the following data, calculate Net Value Added at factor cost.
Image may be NSFW.
Clik here to view.
cbse-sample-papers-for-class-12-economics-compartment-delhi-2015-7

Answer.
Image may be NSFW.
Clik here to view.
cbse-sample-papers-for-class-12-economics-compartment-delhi-2015-8

Question.24. Distinguish between intermediate goods and final goods. Give an example of each.
Answer.
Image may be NSFW.
Clik here to view.
cbse-sample-papers-for-class-12-economics-compartment-delhi-2015-9

Or
Explain the circular flow of income.
Answer. See (Comptt. I Delhi).

Question.25. Distinguish- between autonomous and accommodating transactions of balance of payments account. Explain the significance of this distinction.
Answer.
Image may be NSFW.
Clik here to view.
cbse-sample-papers-for-class-12-economics-compartment-delhi-2015-10

Significance of this distinction. Autonomous transactions are the reason for the imbalance in BOP (BOP surplus or BOP deficit) which is restored through Accommodating transactions undertaken by the Central Bank of our country. Therefore, the magnitude of accommodating transactions is determined by the autonomous transactions.

Question.26. Explain the need for reduction in inequalities of income and wealth. Explain any two budgetary measures by which it can be done.
Answer. The distribution of income and wealth is highly unequal in under developing countries like India. Caught in the vicious circle of poverty, the poor with fewer Skills and resources at their disposal are unable to bridge this economic divide. Without help from the government the poor will become poorer over time arid their standard of living will worsen. The economy will also suffer as the capacity of a large part of the workforce will remain underutilized. The government can bridge this economic divide with the help of its revenue and expenditure policy:
— The government can influence distribution of income by increasing the tax on incomes of the rich and on the goods consumed by the rich people. It can also reduce the taxes on lower income groups. This will bridge the difference between the disposable incomes of the higher income and lower income groups.
— The amount collected through taxes can be used by the government for spending on welfare of the poor people. It can provide them with transfer payments, subsides and other free services like education and healthcare. This will have twin effects. First, it will increase their disposable income and second, it will increase their standard of living and thus increase their welfare.

Question.27. Describe any two main functions of a Central Bank.
Answer. The Central Bank is the apex institution of a country’s monetary system. Following are the two main function performed by it:
(i) Bank of issue. See Q. 23, 2015 (I Delhi).
(ii) Bankers Bank and Supervisor. See Q. See Q. 23, 2015 (I Outside Delhi).
Or
How do changes in Bank Rate affect the money supply in an economy? Explain.
Answer. Bank Rate is the rate of interest at which the Central Bank lends money to the commercial banks in emergency, acting as “lender of the last resort”. The purpose of change in bank r rate is to change the cost of borrowings from the Central Bank.
(i) An increase in the bank rate increases the cost of borrowings from the Central Bank. Therefore, if the Central Bank increases the bank rate, the commercial banks also increase the rates at which they lend to the public and business firms. It makes borrowings by the people costly. This will discourage them to take loans. This also reduces the ability of commercial banks to create credit. Thus volume of credit and money supply will decrease in the economy.
(ii) A decrease in the bank rate will reduce the cost of borrowings of commercial banks from the Central Bank. The commercial banks will, further reduce their lending rates increasing the volume of credit and money supply in the economy. The decrease in bank rate induces the people to borrow more from the bank and this will have a ! positive effect on the money supply.

Question.28. Explain the changes that take place when aggregate demand and aggregate supply are not equal.
Answer. See Q. 32, 2014 (I Delhi).

Question.29. Calculate (a) national income and (b) gross national disposable income.
Image may be NSFW.
Clik here to view.
cbse-sample-papers-for-class-12-economics-compartment-delhi-2015-11

Answer.
Image may be NSFW.
Clik here to view.
cbse-sample-papers-for-class-12-economics-compartment-delhi-2015-12

SET II

Note: Except for the following questions, all the remaining questions have been asked in Set-1.
SECTION A
Question.4. Explain the problem of%For whom to produce’.
Answer. See Q. 6, 2014 (I Delhi).

Question.7. Why is average revenue curve of a firm parallel to the X-axis in a perfectly competitive market? Explain.
Answer. In perfect competition, since price remains constant, more quantity can be sold at-the same price. This implies that total revenue (TR) will increase at a constant rate and average revenue (AR) will also be constant throughout and will be equal to marginal revenue (MR). As a result, AR and MR curves will coincide in a horizontal straight line parallel to the X-axis.
Or
Explain the significance of ‘large number of buyers’ feature of a perfectly competitive market.
Answer. See Q. 7,2015 (I Delhi).

Question.8. State whether the following statements are true or false. Give reasons for your answer:
(i) When total revenue is constant, average revenue falls.
(ii) When marginal product falls, average product will also fall.
Answer. (i) True. Average Revenue (AR) is calculated by dividing Total Revenue (TR) with tire total output. We know, TR =\( \frac { AR }{ Q } \). As total revenue is constant, therefore with increase in output, average revenue will fall.
(ii) Fake. When marginal product (MP) falls due to diminishing returns to a factor, average product (AP) may rise so long as MP is greater than AP.

Question.14. The market for commodity X is in equilibrium. The prices of its inputs fall. Explain with the help of a diagram its chain of effects on equilibrium price, quantity demanded and quantity supplied.
Answer. Market for good X is in equilibrium at point E where the market demand curve DD and the market supply curve SS intersect at the point E so that OP and OQ are the equilibrium price and quantity respectively. Fall in the prices of inputs of commodity X will decrease the cost of production of firms producing this commodity. Price remaining the same, decrease in cost will lead to increase in profits. So, the producers will be induced to increase supply of good X at the same price which will increase the total market supply shifting the supply curve rightwards to S1S1. This will create excess supply-4n the market (equal to EA) at the equilibrium price OP. Competition amongst the sellers will reduce the price. Decrease in price will increase demand (expansion) and reduce supply (contraction). These changes continue till the market is in equilibrium (E1) again at a lower equilibrium price (OP1) and higher equilibrium quantity (OQ1).
Image may be NSFW.
Clik here to view.
cbse-sample-papers-for-class-12-economics-compartment-delhi-2015-13

Note: The following Question is for the Blind Candidates only in lieu of Q. No. 14.
Explain the chain of effects of excess supply of a commodity on its equilibrium price, demand and supply.
Answer. Market for a commodity is in equilibrium at point E where the market demand curve DD and the market supply curve SS intersect at the point E so that OP and OQ are the equilibrium price and quantity respectively.
Excess supply means market supply exceeds market demand at a price, which is greater than the equilibrium price.
Image may be NSFW.
Clik here to view.
cbse-sample-papers-for-class-12-economics-compartment-delhi-2015-14

— Excess supply leads to competition between sellers, which will lead to fall in price.
— Fall in price leads to rise in demand (expansion of demand) and fall in supply (contraction of supply) as indicated by the arrows.
— Price will continue to fall till excess supply is wiped out. This happens because price will decrease to a level where market demand becomes equal to market supply (OQ) and equilibrium price of OP is attained.

SECTION B
Question.20. What is the relationship between:
(a) Marginal propensity to save and marginal*propensityTo consume,
(b) Marginal propensity .to save and investment multiplier.
Answer.
Image may be NSFW.
Clik here to view.
cbse-sample-papers-for-class-12-economics-compartment-delhi-2015-15

Image may be NSFW.
Clik here to view.
cbse-sample-papers-for-class-12-economics-compartment-delhi-2015-16

Give the meaning of:
(a) Autonomous consumption, and
(b) Full employment
Answer. (a) Autonomous consumption. It is the consumption expenditure at zero level of income.
It happens because people need to fulfill their basic wants to sustain themselves, even if income is zero. Autonomous investment is also called as independent investment because it does not relate to the level of income; the factors affecting it are other than ‘ the income.
(b) Full employment. It refers to a situation in which all those people, who are willing and able to work at the existing wage rate, get work without any undue difficulty. In other words, it is the situation where the number of job seekers are nil.

Question.21.Describe any three sources of supply of foreign exchange.
Answer. Sources of supply of foreign exchange:

  1. Supply of foreign exchange comes through exports of goods and services.
  2. Foreign exchange flows in through gifts and other remittances from abroad.
  3. Foreign investments in the home country also increases the supply of foreign exchange.

Question.22. Calculate the equilibrium level of income in the economy.
C = 500 + (0.9)Y
Investment expenditure = 3000
Answer.
Image may be NSFW.
Clik here to view.
cbse-sample-papers-for-class-12-economics-compartment-delhi-2015-17

Question.29. Calculate (a) net national product at factor cost and (b) net national disposable income:
Image may be NSFW.
Clik here to view.
cbse-sample-papers-for-class-12-economics-compartment-delhi-2015-18

Answer.
Image may be NSFW.
Clik here to view.
cbse-sample-papers-for-class-12-economics-compartment-delhi-2015-19

Image may be NSFW.
Clik here to view.
cbse-sample-papers-for-class-12-economics-compartment-delhi-2015-20

SET III

Note : Except for the following questions, all the remaining questions have been asked in Set I.
SECTION A
Question.4. Why do central problems arise? Explain.
Answer. Central problems arise because of scarcity of resources, i.e., limitation of supply in relation to demand.
The three main reasons for existence of central problems are:

  1.  Scarcity of resources. Resources are limited in relation to their demand and economy cannot produce all what people want.
  2. Unlimited human wants. Human wants also differ in priorities.
  3. Alternate uses. Resources can be put to various uses which is why the problem of choice arises.

Question.7. Distinguish between perfect and imperfect oligopoly.
Answer. Perfect oligopoly. If the firms produce homogeneous products, then it is called perfect oligopoly, Example, cement, steel producing industries, etc.
Imperfect oligopoly. If the firms produce differentiated products, then it is called imperfect oligopoly, Example, cars, soft drinks, etc.
Or
Why is average revenue curve negatively sloped under monopolistic competition? Explain.
Answer. Monopolistic competition is characterized by product differentiation which implies presence of close substitutes. So in order to increase its sales, a firm will have to lower its price. Therefore, AR falls as more is produced and sold. This makes the AR curve downward sloping throughout.

Question.8. Explain the relationship between marginal cost and average variable cost.
Answer. Relationship between MC and average variable cost (AVC):
Image may be NSFW.
Clik here to view.
cbse-sample-papers-for-class-12-economics-compartment-delhi-2015-21

  1. When AVC is falling, MC is less than AVC.
  2. When AVC is minimum, MC is equal to AVC.
  3. When AVC is rising, MC is more than AVC.

Question.14. X is a normal good for its consumers. Their income increases. Explain its chain of effects on equilibrium price, demand and supply of X. (use diagram)
Answer. Normal goods are those goods whose demand increases with an increase in the income of the consumer. Being a normal good, demand for good X will increase as consumer’s income increases. This will shift the demand curve of good X towards right from DD to D1D1. When demand increases, it creates a situation of excess demand (EA) at the old equilibrium price of OP. This leads to competition amongst buyers which increases the price. Increase in price leads to rise in supply and fall in demand. These changes continue till new equilibrium is established at point E1 where equilibrium price rises from OP to OP1 and equilibrium quantity rises from OQ to OQ1.
Image may be NSFW.
Clik here to view.
cbse-sample-papers-for-class-12-economics-compartment-delhi-2015-22

Note: The following Question is for the Blind Candidates only in lieu of Q. No. 14.
Explain the chain of effects of “decrease” in supply of a commodity on its equilibrium price, demand and supply.
Answer. Initially, market of a good (say X) is in equilibrium at point E where the market demand curve DD and the market supply curve SS intersect each other. Where OP and OQ are the equilibrium price and quantity respectively.
Image may be NSFW.
Clik here to view.
cbse-sample-papers-for-class-12-economics-compartment-delhi-2015-23

Now, there is decrease in supply of commodity X, which means less quantity supplied at the same price.
• Due to decrease in supply, the supply curve shifts to the left to S1S1. This creates excess demand (equal to EA) at the given equilibrium price OP.
• The excess demand leads to competition between consumers causing price to rise.
• Rise in price leads to fall in demand (contraction of demand) and rise in supply (extension of supply) as indicated by the arrows.
• These changes continue till the market reaches new equilibrium at point E1 with a higher equilibrium price at QP1 but lower equilibrium quantity at OQ1

SECTION B
Question.20. Give the meaning of investment multiplier and aggregate supply.
Answer. Investment multiplier. It explains how many times the income increases as a result of an increase in investment. Investment Multiplier (k) is the ratio of increase in national income (\(\Delta Y\)) due to an increase in investment (\(\Delta I\)).
\(\quad \therefore \quad K=\frac { \Delta Y }{ \Delta I } \quad \)
Aggregate Supply (AS). AS refers to the money value of goods and services that all the producers are willing to supply in an economy in a given time period. Also the value of total output is distributed amongst factors of production in the form of rent, wages, interest and profit. The sum total of these factor incomes at domestic and national level is called National Income.
Aggregate Supply = National Income
Or
Give the meaning of aggregate demand arid full employment.
Answer. Aggregate Demand (AD). AD refers to the total value of final goods and services which all the sectors of an economy are planning to buy at a given level of income during a period of time. It also refers to the planned aggregate expenditure in the economy.
Full employment Full employment refers to a situation in which all those people who are willing and able to work at the existing wage rate, get work without any undue difficulty. In other words, where the number of job seekers is nill.

Question.21. Explain the effect of rise in price of foreign currency on exports.
Answer. Rise in price of foreign currency implies that the domestic currency has depreciated. It means that one unit of foreign currency is worth more rupees than earlier. So one unit of foreign currency can now buy more goods and services from India, which will increase our exports.

Question.22. Calculate equilibrium level of income:
(a) Autonomous consumption = 200
(b) Marginal propensity to consume = 0.9
(c) Investment expenditure = 1000
Answer.
Image may be NSFW.
Clik here to view.
cbse-sample-papers-for-class-12-economics-compartment-delhi-2015-24

Question.29. Calculate (a) national income and (b) net national disposable income.
Image may be NSFW.
Clik here to view.
cbse-sample-papers-for-class-12-economics-compartment-delhi-2015-25

Answer.
Image may be NSFW.
Clik here to view.
cbse-sample-papers-for-class-12-economics-compartment-delhi-2015-26

The post CBSE Sample Papers for Class 12 Economics Compartment Delhi -2015 appeared first on Learn CBSE.

Viewing all 9991 articles
Browse latest View live


<script src="https://jsc.adskeeper.com/r/s/rssing.com.1596347.js" async> </script>